НахоТдСниС ΠΊΠΎΠΎΡ€Π΄ΠΈΠ½Π°Ρ‚ Π²Π΅ΠΊΡ‚ΠΎΡ€Π°: НахоТдСниС ΠΊΠΎΠΎΡ€Π΄ΠΈΠ½Π°Ρ‚ Π²Π΅ΠΊΡ‚ΠΎΡ€Π° Ρ‡Π΅Ρ€Π΅Π· ΠΊΠΎΠΎΡ€Π΄ΠΈΠ½Π°Ρ‚Ρ‹ Ρ‚ΠΎΡ‡Π΅ΠΊ. Как Π½Π°ΠΉΡ‚ΠΈ Π²Π΅ΠΊΡ‚ΠΎΡ€ ΠΏΠΎ Π΄Π²ΡƒΠΌ Ρ‚ΠΎΡ‡ΠΊΠ°ΠΌ

Π‘ΠΎΠ΄Π΅Ρ€ΠΆΠ°Π½ΠΈΠ΅

ΠžΠΏΡ€Π΅Π΄Π΅Π»Π΅Π½ΠΈΠ΅ ΠΊΠΎΠΎΡ€Π΄ΠΈΠ½Π°Ρ‚ Π²Π΅ΠΊΡ‚ΠΎΡ€Π° Π·Π°Π΄Π°Π½Π½ΠΎΠ³ΠΎ ΠΊΠΎΠΎΡ€Π΄ΠΈΠ½Π°Ρ‚Π°ΠΌΠΈ Π΅Π³ΠΎ Π½Π°Ρ‡Π°Π»ΡŒΠ½ΠΎΠΉ ΠΈ ΠΊΠΎΠ½Π΅Ρ‡Π½ΠΎΠΉ Ρ‚ΠΎΡ‡ΠΊΠΈ.

ΠžΠΏΡ€Π΅Π΄Π΅Π»Π΅Π½ΠΈΠ΅ ΠΊΠΎΠΎΡ€Π΄ΠΈΠ½Π°Ρ‚ Π²Π΅ΠΊΡ‚ΠΎΡ€Π° Π·Π°Π΄Π°Π½Π½ΠΎΠ³ΠΎ ΠΊΠΎΠΎΡ€Π΄ΠΈΠ½Π°Ρ‚Π°ΠΌΠΈ Π΅Π³ΠΎ Π½Π°Ρ‡Π°Π»ΡŒΠ½ΠΎΠΉ ΠΈ ΠΊΠΎΠ½Π΅Ρ‡Π½ΠΎΠΉ Ρ‚ΠΎΡ‡ΠΊΠΈ.

Навигация ΠΏΠΎ страницС:

  • ОсновноС ΡΠΎΠΎΡ‚Π½ΠΎΡˆΠ΅Π½ΠΈΠ΅
  • Π€ΠΎΡ€ΠΌΡƒΠ»Ρ‹ для ΠΎΠΏΡ€Π΅Π΄Π΅Π»Π΅Π½ΠΈΠ΅ ΠΊΠΎΠΎΡ€Π΄ΠΈΠ½Π°Ρ‚ Π²Π΅ΠΊΡ‚ΠΎΡ€Π° Π·Π°Π΄Π°Π½Π½ΠΎΠ³ΠΎ ΠΊΠΎΠΎΡ€Π΄ΠΈΠ½Π°Ρ‚Π°ΠΌΠΈ Π΅Π³ΠΎ Π½Π°Ρ‡Π°Π»ΡŒΠ½ΠΎΠΉ ΠΈ ΠΊΠΎΠ½Π΅Ρ‡Π½ΠΎΠΉ Ρ‚ΠΎΡ‡ΠΊΠΈ
    • для плоских Π·Π°Π΄Π°Ρ‡
    • для пространствСнных Π·Π°Π΄Π°Ρ‡
    • для n -ΠΌΠ΅Ρ€Π½ΠΎΠ³ΠΎ пространства
  • ΠŸΡ€ΠΈΠΌΠ΅Ρ€Ρ‹ Π·Π°Π΄Π°Ρ‡
    • плоская Π·Π°Π΄Π°Ρ‡Π°
    • пространствСнных Π·Π°Π΄Π°Ρ‡Π°
    • Π·Π°Π΄Π°Ρ‡Π° Π² n -ΠΌΠ΅Ρ€Π½Ρ‹ΠΌ пространствС

Π‘ΠΌΠΎΡ‚Ρ€ΠΈΡ‚Π΅ Ρ‚Π°ΠΊΠΆΠ΅ ΠΎΠ½Π»Π°ΠΉΠ½ ΠΊΠ°Π»ΡŒΠΊΡƒΠ»ΡΡ‚ΠΎΡ€ для опрСдСлСния ΠΊΠΎΠΎΡ€Π΄ΠΈΠ½Π°Ρ‚ Π²Π΅ΠΊΡ‚ΠΎΡ€Π° ΠΏΠΎ Π΄Π²ΡƒΠΌ Ρ‚ΠΎΡ‡ΠΊΠ°ΠΌ.

ОсновноС ΡΠΎΠΎΡ‚Π½ΠΎΡˆΠ΅Π½ΠΈΠ΅.Π§Ρ‚ΠΎΠ±Ρ‹ Π½Π°ΠΉΡ‚ΠΈ ΠΊΠΎΠΎΡ€Π΄ΠΈΠ½Π°Ρ‚Ρ‹ Π²Π΅ΠΊΡ‚ΠΎΡ€Π° AB, зная ΠΊΠΎΠΎΡ€Π΄ΠΈΠ½Π°Ρ‚Ρ‹ Π΅Π³ΠΎ Π½Π°Ρ‡Π°Π»ΡŒΠ½ΠΎΠΉ Ρ‚ΠΎΡ‡Π΅ΠΊ А ΠΈ ΠΊΠΎΠ½Π΅Ρ‡Π½ΠΎΠΉ Ρ‚ΠΎΡ‡ΠΊΠΈ Π’, Π½Π΅ΠΎΠ±Ρ…ΠΎΠ΄ΠΈΠΌΠΎ ΠΈΠ· ΠΊΠΎΠΎΡ€Π΄ΠΈΠ½Π°Ρ‚ ΠΊΠΎΠ½Π΅Ρ‡Π½ΠΎΠΉ Ρ‚ΠΎΡ‡ΠΊΠΈ Π²Ρ‹Ρ‡Π΅ΡΡ‚ΡŒ ΡΠΎΠΎΡ‚Π²Π΅Ρ‚ΡΡ‚Π²ΡƒΡŽΡ‰ΠΈΠ΅ ΠΊΠΎΠΎΡ€Π΄ΠΈΠ½Π°Ρ‚Ρ‹ Π½Π°Ρ‡Π°Π»ΡŒΠ½ΠΎΠΉ Ρ‚ΠΎΡ‡ΠΊΠΈ.


Π€ΠΎΡ€ΠΌΡƒΠ»Ρ‹ опрСдСлСния ΠΊΠΎΠΎΡ€Π΄ΠΈΠ½Π°Ρ‚ Π²Π΅ΠΊΡ‚ΠΎΡ€Π° Π·Π°Π΄Π°Π½Π½ΠΎΠ³ΠΎ ΠΊΠΎΠΎΡ€Π΄ΠΈΠ½Π°Ρ‚Π°ΠΌΠΈ Π΅Π³ΠΎ Π½Π°Ρ‡Π°Π»ΡŒΠ½ΠΎΠΉ ΠΈ ΠΊΠΎΠ½Π΅Ρ‡Π½ΠΎΠΉ Ρ‚ΠΎΡ‡ΠΊΠΈ

Π€ΠΎΡ€ΠΌΡƒΠ»Π° опрСдСлСния ΠΊΠΎΠΎΡ€Π΄ΠΈΠ½Π°Ρ‚ Π²Π΅ΠΊΡ‚ΠΎΡ€Π° для плоских Π·Π°Π΄Π°Ρ‡

Π’ случаС плоской Π·Π°Π΄Π°Ρ‡ΠΈ Π²Π΅ΠΊΡ‚ΠΎΡ€ AB Π·Π°Π΄Π°Π½Π½Ρ‹ΠΉ ΠΊΠΎΠΎΡ€Π΄ΠΈΠ½Π°Ρ‚Π°ΠΌΠΈ Ρ‚ΠΎΡ‡Π΅ΠΊ A(AxΒ ;Β Ay) ΠΈ B(BxΒ ;Β By) ΠΌΠΎΠΆΠ½ΠΎ Π½Π°ΠΉΡ‚ΠΈ воспользовавшись ΡΠ»Π΅Π΄ΡƒΡŽΡ‰Π΅ΠΉ Ρ„ΠΎΡ€ΠΌΡƒΠ»ΠΎΠΉ

AB = {Bx — AxΒ ;Β By — Ay}


Π€ΠΎΡ€ΠΌΡƒΠ»Π° опрСдСлСния ΠΊΠΎΠΎΡ€Π΄ΠΈΠ½Π°Ρ‚ Π²Π΅ΠΊΡ‚ΠΎΡ€Π° для пространствСнных Π·Π°Π΄Π°Ρ‡

Π’ случаС пространствСнной Π·Π°Π΄Π°Ρ‡ΠΈ Π²Π΅ΠΊΡ‚ΠΎΡ€ AB Π·Π°Π΄Π°Π½Π½Ρ‹ΠΉ ΠΊΠΎΠΎΡ€Π΄ΠΈΠ½Π°Ρ‚Π°ΠΌΠΈ Ρ‚ΠΎΡ‡Π΅ΠΊ A(AxΒ ;Β AyΒ ;Β Az) ΠΈ B(BxΒ ;Β ByΒ ;Β Bz) ΠΌΠΎΠΆΠ½ΠΎ Π½Π°ΠΉΡ‚ΠΈ воспользовавшись ΡΠ»Π΅Π΄ΡƒΡŽΡ‰Π΅ΠΉ Ρ„ΠΎΡ€ΠΌΡƒΠ»ΠΎΠΉ

AB = {Bx — AxΒ ;Β By — AyΒ ;Β Bz — Az}


Π€ΠΎΡ€ΠΌΡƒΠ»Π° опрСдСлСния ΠΊΠΎΠΎΡ€Π΄ΠΈΠ½Π°Ρ‚ Π²Π΅ΠΊΡ‚ΠΎΡ€Π° для n -ΠΌΠ΅Ρ€Π½ΠΎΠ³ΠΎ пространства

Π’ случаС n-ΠΌΠ΅Ρ€Π½ΠΎΠ³ΠΎ пространства Π²Π΅ΠΊΡ‚ΠΎΡ€ AB Π·Π°Π΄Π°Π½Π½Ρ‹ΠΉ ΠΊΠΎΠΎΡ€Π΄ΠΈΠ½Π°Ρ‚Π°ΠΌΠΈ Ρ‚ΠΎΡ‡Π΅ΠΊ A(A1Β ;Β A2Β ;Β . ..Β ;Β An) ΠΈ B(B1Β ;Β B2Β ;Β …Β ;Β Bn) ΠΌΠΎΠΆΠ½ΠΎ Π½Π°ΠΉΡ‚ΠΈ воспользовавшись ΡΠ»Π΅Π΄ΡƒΡŽΡ‰Π΅ΠΉ Ρ„ΠΎΡ€ΠΌΡƒΠ»ΠΎΠΉ

AB = {B1 — A1Β ;Β B2 — A2Β ;Β …Β ;Β Bn — An}


ΠŸΡ€ΠΈΠΌΠ΅Ρ€Ρ‹ Π·Π°Π΄Π°Ρ‡ связанных с ΠΎΠΏΡ€Π΅Π΄Π΅Π»Π΅Π½ΠΈΠ΅ΠΌ ΠΊΠΎΠΎΡ€Π΄ΠΈΠ½Π°Ρ‚ Π²Π΅ΠΊΡ‚ΠΎΡ€Π° ΠΏΠΎ Π΄Π²ΡƒΠΌ Ρ‚ΠΎΡ‡ΠΊΠ°ΠΌ


ΠŸΡ€ΠΈΠΌΠ΅Ρ€Ρ‹ для плоских Π·Π°Π΄Π°Ρ‡

ΠŸΡ€ΠΈΠΌΠ΅Ρ€ 1. Найти ΠΊΠΎΠΎΡ€Π΄ΠΈΠ½Π°Ρ‚Ρ‹ Π²Π΅ΠΊΡ‚ΠΎΡ€Π° AB, Ссли A(1; 4), B(3; 1).

РСшСниС: AB = {3 — 1; 1 — 4} = {2; -3}.

ΠŸΡ€ΠΈΠΌΠ΅Ρ€ 2. Найти ΠΊΠΎΠΎΡ€Π΄ΠΈΠ½Π°Ρ‚Ρ‹ Ρ‚ΠΎΡ‡ΠΊΠΈ B Π²Π΅ΠΊΡ‚ΠΎΡ€Π° AB = {5; 1}, Ссли ΠΊΠΎΠΎΡ€Π΄ΠΈΠ½Π°Ρ‚Ρ‹ Ρ‚ΠΎΡ‡ΠΊΠΈ A(3; -4).

РСшСниС:

ABx = Bx — AxΒ Β Β =>Β Β Β Bx = ABx + AxΒ Β Β =>Β Β Β Bx = 5 + 3 = 8

ABy = By — AyΒ Β Β =>Β Β Β By = ABy + AyΒ Β Β =>Β Β Β By = 1 + (-4) = -3

ΠžΡ‚Π²Π΅Ρ‚: B(8; -3).

ΠŸΡ€ΠΈΠΌΠ΅Ρ€ 3. Найти ΠΊΠΎΠΎΡ€Π΄ΠΈΠ½Π°Ρ‚Ρ‹ Ρ‚ΠΎΡ‡ΠΊΠΈ A Π²Π΅ΠΊΡ‚ΠΎΡ€Π° AB = {5; 1}, Ссли ΠΊΠΎΠΎΡ€Π΄ΠΈΠ½Π°Ρ‚Ρ‹ Ρ‚ΠΎΡ‡ΠΊΠΈ B(3; -4).

РСшСниС:

ABx = Bx — AxΒ Β Β =>Β Β Β Ax = Bx — ABxΒ Β Β =>Β Β Β Ax = 3 — 5 = -2
ABy = By — AyΒ Β Β =>Β Β Β Ay = By — AByΒ Β Β =>Β Β Β Ay = -4 — 1 = -5

ΠžΡ‚Π²Π΅Ρ‚: A(-2; -5).


ΠŸΡ€ΠΈΠΌΠ΅Ρ€Ρ‹ для пространствСнных Π·Π°Π΄Π°Ρ‡

ΠŸΡ€ΠΈΠΌΠ΅Ρ€ 4. Найти ΠΊΠΎΠΎΡ€Π΄ΠΈΠ½Π°Ρ‚Ρ‹ Π²Π΅ΠΊΡ‚ΠΎΡ€Π° AB, Ссли A(1; 4; 5), B(3; 1; 1).

РСшСниС: AB = {3 — 1; 1 — 4; 1 — 5} = {2; -3; -4}.

ΠŸΡ€ΠΈΠΌΠ΅Ρ€ 5. Найти ΠΊΠΎΠΎΡ€Π΄ΠΈΠ½Π°Ρ‚Ρ‹ Ρ‚ΠΎΡ‡ΠΊΠΈ B Π²Π΅ΠΊΡ‚ΠΎΡ€Π° AB = {5; 1; 2}, Ссли ΠΊΠΎΠΎΡ€Π΄ΠΈΠ½Π°Ρ‚Ρ‹ Ρ‚ΠΎΡ‡ΠΊΠΈ A(3; -4; 3).

РСшСниС:

ABx = Bx — AxΒ Β Β =>Β Β Β Bx = ABx + AxΒ Β Β =>Β Β Β Bx = 5 + 3 = 8
ABy = By — AyΒ Β Β =>Β Β Β By = ABy + AyΒ Β Β =>Β Β Β By = 1 + (-4) = -3
ABz = Bz — AzΒ Β Β =>Β Β Β Bz = ABz + AzΒ Β Β =>Β Β Β Bz = 2 + 3 = 5

ΠžΡ‚Π²Π΅Ρ‚: B(8; -3; 5).

ΠŸΡ€ΠΈΠΌΠ΅Ρ€ 6. Найти ΠΊΠΎΠΎΡ€Π΄ΠΈΠ½Π°Ρ‚Ρ‹ Ρ‚ΠΎΡ‡ΠΊΠΈ A Π²Π΅ΠΊΡ‚ΠΎΡ€Π° AB = {5; 1; 4}, Ссли ΠΊΠΎΠΎΡ€Π΄ΠΈΠ½Π°Ρ‚Ρ‹ Ρ‚ΠΎΡ‡ΠΊΠΈ B(3; -4; 1).

РСшСниС:

ABx = B

x — AxΒ Β Β =>Β Β Β Ax = Bx — ABxΒ Β Β =>Β Β Β Ax = 3 — 5 = -2
ABy = By — AyΒ Β Β =>Β Β Β Ay = By — AByΒ Β Β =>Β Β Β Ay = -4 — 1 = -5
ABz = Bz — AzΒ Β Β =>Β Β Β Az = Bz — ABzΒ Β Β =>Β Β Β Az = 1 — 4 = -3

ΠžΡ‚Π²Π΅Ρ‚: A(-2; -5; -3).


ΠŸΡ€ΠΈΠΌΠ΅Ρ€Ρ‹ для n -ΠΌΠ΅Ρ€Π½ΠΎΠ³ΠΎ пространства

ΠŸΡ€ΠΈΠΌΠ΅Ρ€ 7. Найти ΠΊΠΎΠΎΡ€Π΄ΠΈΠ½Π°Ρ‚Ρ‹ Π²Π΅ΠΊΡ‚ΠΎΡ€Π° AB, Ссли A(1; 4; 5; 5; -3), B(3; 0; 1; -2; 5).

РСшСниС: AB = {3 — 1; 0 — 4; 1 — 5; -2 — 5; 5 — (-3)} = {2; -4; -4; -7; 8}.

ΠŸΡ€ΠΈΠΌΠ΅Ρ€ 8. Найти ΠΊΠΎΠΎΡ€Π΄ΠΈΠ½Π°Ρ‚Ρ‹ Ρ‚ΠΎΡ‡ΠΊΠΈ B Π²Π΅ΠΊΡ‚ΠΎΡ€Π° AB = {5; 1; 2; 1}, Ссли ΠΊΠΎΠΎΡ€Π΄ΠΈΠ½Π°Ρ‚Ρ‹ Ρ‚ΠΎΡ‡ΠΊΠΈ A(3; -4; 3; 2).

РСшСниС:

AB1 = B

1 — A1Β Β Β =>Β Β Β B1 = AB1 + A1Β Β Β =>Β Β Β B1 = 5 + 3 = 8
AB2 = B2 — A2Β Β Β =>Β Β Β B2 = AB2 + A2Β Β Β =>Β Β Β B2 = 1 + (-4) = -3
AB3 = B3 — A3Β Β Β =>Β Β Β B3 = AB3 + A3Β Β Β =>Β Β Β B3 = 2 + 3 = 5
AB4 = B4 — A4Β Β Β =>Β Β Β B4 = AB4 + A4Β Β Β =>Β Β Β B4 = 1 + 2 = 3

ΠžΡ‚Π²Π΅Ρ‚: B(8; -3; 5; 3).

ΠŸΡ€ΠΈΠΌΠ΅Ρ€ 9. Найти ΠΊΠΎΠΎΡ€Π΄ΠΈΠ½Π°Ρ‚Ρ‹ Ρ‚ΠΎΡ‡ΠΊΠΈ A Π²Π΅ΠΊΡ‚ΠΎΡ€Π° AB = {5; 1; 4; 5}, Ссли ΠΊΠΎΠΎΡ€Π΄ΠΈΠ½Π°Ρ‚Ρ‹ Ρ‚ΠΎΡ‡ΠΊΠΈ B(3; -4; 1; 8).

РСшСниС:

AB1 = B1 — A1Β Β Β =>Β Β Β A1 = B1 — AB1Β Β Β =>Β Β Β A1 = 3 — 5 = -2
AB2 = B2

— A2Β Β Β =>Β Β Β A2 = B2 — AB2Β Β Β =>Β Β Β A2 = -4 — 1 = -5
AB3 = B3 — A3Β Β Β =>Β Β Β A3 = B3 — AB3Β Β Β =>Β Β Β A3 = 1 — 4 = -3
AB4 = B4 — A4Β Β Β =>Β Β Β A4 = B4 — AB4Β Β Β =>Β Β Β A4 = 8 — 5 = 3

ΠžΡ‚Π²Π΅Ρ‚: A(-2; -5; -3; 3).

Π’Π΅ΠΊΡ‚ΠΎΡ€Π° Π’Π΅ΠΊΡ‚ΠΎΡ€: ΠΎΠΏΡ€Π΅Π΄Π΅Π»Π΅Π½ΠΈΠ΅ ΠΈ основныС понятия ΠžΠΏΡ€Π΅Π΄Π΅Π»Π΅Π½ΠΈΠ΅ ΠΊΠΎΠΎΡ€Π΄ΠΈΠ½Π°Ρ‚ Π²Π΅ΠΊΡ‚ΠΎΡ€Π° Π·Π°Π΄Π°Π½Π½ΠΎΠ³ΠΎ ΠΊΠΎΠΎΡ€Π΄ΠΈΠ½Π°Ρ‚Π°ΠΌΠΈ Π΅Π³ΠΎ Π½Π°Ρ‡Π°Π»ΡŒΠ½ΠΎΠΉ ΠΈ ΠΊΠΎΠ½Π΅Ρ‡Π½ΠΎΠΉ Ρ‚ΠΎΡ‡ΠΊΠΈ ΠœΠΎΠ΄ΡƒΠ»ΡŒ Π²Π΅ΠΊΡ‚ΠΎΡ€Π°. Π”Π»ΠΈΠ½Π° Π²Π΅ΠΊΡ‚ΠΎΡ€Π° ΠΠ°ΠΏΡ€Π°Π²Π»ΡΡŽΡ‰ΠΈΠ΅ косинусы Π²Π΅ΠΊΡ‚ΠΎΡ€Π° РавСнство Π²Π΅ΠΊΡ‚ΠΎΡ€ΠΎΠ² ΠžΡ€Ρ‚ΠΎΠ³ΠΎΠ½Π°Π»ΡŒΠ½ΠΎΡΡ‚ΡŒ Π²Π΅ΠΊΡ‚ΠΎΡ€ΠΎΠ² ΠšΠΎΠ»Π»ΠΈΠ½Π΅Π°Ρ€Π½ΠΎΡΡ‚ΡŒ Π²Π΅ΠΊΡ‚ΠΎΡ€ΠΎΠ² ΠšΠΎΠΌΠΏΠ»Π°Π½Π°Ρ€Π½ΠΎΡΡ‚ΡŒ Π²Π΅ΠΊΡ‚ΠΎΡ€ΠΎΠ² Π£Π³ΠΎΠ» ΠΌΠ΅ΠΆΠ΄Ρƒ Π²Π΅ΠΊΡ‚ΠΎΡ€Π°ΠΌΠΈ ΠŸΡ€ΠΎΠ΅ΠΊΡ†ΠΈΡ Π²Π΅ΠΊΡ‚ΠΎΡ€Π° Π‘Π»ΠΎΠΆΠ΅Π½ΠΈΠ΅ ΠΈ Π²Ρ‹Ρ‡ΠΈΡ‚Π°Π½ΠΈΠ΅ Π²Π΅ΠΊΡ‚ΠΎΡ€ΠΎΠ² Π£ΠΌΠ½ΠΎΠΆΠ΅Π½ΠΈΠ΅ Π²Π΅ΠΊΡ‚ΠΎΡ€Π° Π½Π° число БкалярноС ΠΏΡ€ΠΎΠΈΠ·Π²Π΅Π΄Π΅Π½ΠΈΠ΅ Π²Π΅ΠΊΡ‚ΠΎΡ€ΠΎΠ² Π’Π΅ΠΊΡ‚ΠΎΡ€Π½ΠΎΠ΅ ΠΏΡ€ΠΎΠΈΠ·Π²Π΅Π΄Π΅Π½ΠΈΠ΅ Π²Π΅ΠΊΡ‚ΠΎΡ€ΠΎΠ² БмСшанноС ΠΏΡ€ΠΎΠΈΠ·Π²Π΅Π΄Π΅Π½ΠΈΠ΅ Π²Π΅ΠΊΡ‚ΠΎΡ€ΠΎΠ² Π›ΠΈΠ½Π΅ΠΉΠ½ΠΎ зависимыС ΠΈ Π»ΠΈΠ½Π΅ΠΉΠ½ΠΎ нСзависимыС Π²Π΅ΠΊΡ‚ΠΎΡ€Π° Π Π°Π·Π»ΠΎΠΆΠ΅Π½ΠΈΠ΅ Π²Π΅ΠΊΡ‚ΠΎΡ€Π° ΠΏΠΎ базису

Онлайн ΠΊΠ°Π»ΡŒΠΊΡƒΠ»ΡΡ‚ΠΎΡ€Ρ‹ с Π²Π΅ΠΊΡ‚ΠΎΡ€Π°ΠΌΠΈ

Онлайн упраТнСния с Π²Π΅ΠΊΡ‚ΠΎΡ€Π°ΠΌΠΈ Π½Π° плоскости

Онлайн упраТнСния с Π²Π΅ΠΊΡ‚ΠΎΡ€Π°ΠΌΠΈ Π² пространствС

Как Π½Π°ΠΉΡ‚ΠΈ ΠΊΠΎΠΎΡ€Π΄ΠΈΠ½Π°Ρ‚Ρ‹ Π²Π΅ΠΊΡ‚ΠΎΡ€Π°

ΠŸΡ€Π΅Π΄Π²Π°Ρ€ΠΈΡ‚Π΅Π»ΡŒΠ½Ρ‹Π΅ свСдСния

Π—Π΄Π΅ΡΡŒ ΠΌΡ‹ ограничимся Π΄Π²ΡƒΠΌΠ΅Ρ€Π½Ρ‹ΠΌ случаСм. Π’Π²Π΅Π΄Π΅Π½ΠΈΠ΅ понятия для Ρ‚Ρ€Π΅Ρ…ΠΌΠ΅Ρ€Π½ΠΎΠ³ΠΎ случая проводится Π°Π½Π°Π»ΠΎΠ³ΠΈΡ‡Π½ΠΎ. Для Ρ‚ΠΎΠ³ΠΎ, Ρ‡Ρ‚ΠΎΠ±Ρ‹ ввСсти понятиС ΠΊΠΎΠΎΡ€Π΄ΠΈΠ½Π°Ρ‚ Π²Π΅ΠΊΡ‚ΠΎΡ€Π° сначала Π²Π²Π΅Π΄Π΅ΠΌ ΠΈ Π΄ΠΎΠΊΠ°ΠΆΠ΅ΠΌ ΡΠ»Π΅Π΄ΡƒΡŽΡ‰ΠΈΠ΅ Π»Π΅ΠΌΠΌΡƒ ΠΈ Ρ‚Π΅ΠΎΡ€Π΅ΠΌΡƒ.

Π›Π΅ΠΌΠΌΠ° 1: Если Π²Π΅ΠΊΡ‚ΠΎΡ€Ρ‹ $\overrightarrow{a}$ ΠΈ $\overrightarrow{b}$ ΠΊΠΎΠ»Π»ΠΈΠ½Π΅Π°Ρ€Π½Ρ‹, ΠΈ Π²Π΅ΠΊΡ‚ΠΎΡ€ $\overrightarrow{a}$ Π½Π΅ являСтся Π½ΡƒΠ»Π΅Π²Ρ‹ΠΌ, Ρ‚ΠΎ сущСствуСт Π΄Π΅ΠΉΡΡ‚Π²ΠΈΡ‚Π΅Π»ΡŒΠ½ΠΎΠ΅ число $k$, Ρ‚Π°ΠΊΠΎΠ΅ Ρ‡Ρ‚ΠΎ выполняСтся равСнство$\overrightarrow{b}=k\overrightarrow{a}$

Π”ΠΎΠΊΠ°Π·Π°Ρ‚Π΅Π»ΡŒΡΡ‚Π²ΠΎ.

Π’ΠΎΠ·ΠΌΠΎΠΆΠ½Ρ‹ Π΄Π²Π° случая:

  1. $\overrightarrow{a}\uparrow \uparrow \overrightarrow{b}$

    ΠžΠ±ΠΎΠ·Π½Π°Ρ‡ΠΈΠΌ число $k$ ΡΠ»Π΅Π΄ΡƒΡŽΡ‰ΠΈΠΌ ΠΎΠ±Ρ€Π°Π·ΠΎΠΌ: $k=\frac{|\overrightarrow{b}|}{|\overrightarrow{a}|}$. Π’Π°ΠΊ ΠΊΠ°ΠΊ Π²Π΅ΠΊΡ‚ΠΎΡ€Ρ‹ $\overrightarrow{a}$ ΠΈ $\overrightarrow{b}$ сонаправлСны, Π° $k\ge 0$, Ρ‚ΠΎ Π²Π΅ΠΊΡ‚ΠΎΡ€Ρ‹ $k\overrightarrow{a}$ ΠΈ $\overrightarrow{b}$ сонаправлСны. Π”Π°Π»Π΅Π΅, ΠΈΠΌΠ΅Π΅ΠΌ, Ρ‡Ρ‚ΠΎ

    \[\left|k\overrightarrow{a}\right|=\left|k\right|\left|\overrightarrow{a}\right|=\frac{|\overrightarrow{b}|}{|\overrightarrow{a}|}\left|\overrightarrow{a}\right|=|\overrightarrow{b}|\]

    Из этого всСго слСдуСт, Ρ‡Ρ‚ΠΎ $\overrightarrow{b}=k\overrightarrow{a}$.

  2. $\overrightarrow{a}\uparrow \downarrow \overrightarrow{b}$

    ΠžΠ±ΠΎΠ·Π½Π°Ρ‡ΠΈΠΌ число $k$ ΡΠ»Π΅Π΄ΡƒΡŽΡ‰ΠΈΠΌ ΠΎΠ±Ρ€Π°Π·ΠΎΠΌ: $k=-\frac{|\overrightarrow{b}|}{|\overrightarrow{a}|}$. Π’Π°ΠΊ ΠΊΠ°ΠΊ Π²Π΅ΠΊΡ‚ΠΎΡ€Ρ‹ $\overrightarrow{a}$ ΠΈ $\overrightarrow{b}$ ΠΏΡ€ΠΎΡ‚ΠΈΠ²ΠΎΠΏΠΎΠ»ΠΎΠΆΠ½ΠΎ Π½Π°ΠΏΡ€Π°Π²Π»Π΅Π½Π½Ρ‹Π΅, Π° $k \[\left|k\overrightarrow{a}\right|=\left|k\right|\left|\overrightarrow{a}\right|=\frac{|\overrightarrow{b}|}{|\overrightarrow{a}|}\left|\overrightarrow{a}\right|=|\overrightarrow{b}|\]

    Из этого всСго слСдуСт, Ρ‡Ρ‚ΠΎ $\overrightarrow{b}=k\overrightarrow{a}$.

Π›Π΅ΠΌΠΌΠ° Π΄ΠΎΠΊΠ°Π·Π°Π½Π°.

Π’Π΅ΠΎΡ€Π΅ΠΌΠ° 1

Π›ΡŽΠ±ΠΎΠΉ Π²Π΅ΠΊΡ‚ΠΎΡ€ ΠΌΠΎΠΆΠ½ΠΎ Ρ€Π°Π·Π»ΠΎΠΆΠΈΡ‚ΡŒ ΠΏΠΎ Π΄Π²ΡƒΠΌ Π½Π΅ΠΊΠΎΠ»Π»ΠΈΠ½Π΅Π°Ρ€Π½Ρ‹ΠΌ Π²Π΅ΠΊΡ‚ΠΎΡ€Π°ΠΌ, ΠΏΡ€ΠΈΡ‡Π΅ΠΌ коэффициСнты разлоТСния ΠΎΠΏΡ€Π΅Π΄Π΅Π»ΡΡŽΡ‚ΡΡ СдинствСнным ΠΎΠ±Ρ€Π°Π·ΠΎΠΌ:

\[\overrightarrow{c}=m\overrightarrow{a}+n\overrightarrow{b}\]

Π”ΠΎΠΊΠ°Π·Π°Ρ‚Π΅Π»ΡŒΡΡ‚Π²ΠΎ.

БущСствованиС: Π”ΠΎΠΊΠ°ΠΆΠ΅ΠΌ, Ρ‡Ρ‚ΠΎ Ρ‚Π°ΠΊΠΎΠ΅ Ρ€Π°Π·Π»ΠΎΠΆΠ΅Π½ΠΈΠ΅ ΠΈΠΌΠ΅Π΅Ρ‚ мСсто. Π—Π΄Π΅ΡΡŒ Π²ΠΎΠ·ΠΌΠΎΠΆΠ½Ρ‹ Π΄Π²Π° случая:

  1. Π’Π΅ΠΊΡ‚ΠΎΡ€ $\overrightarrow{c}$ ΠΊΠΎΠ»Π»ΠΈΠ½Π΅Π°Ρ€Π΅Π½ (ΠΊ ΠΏΡ€ΠΈΠΌΠ΅Ρ€Ρƒ) Π²Π΅ΠΊΡ‚ΠΎΡ€Ρƒ $\overrightarrow{b}$.

    По Π»Π΅ΠΌΠΌΠ΅ 1, Π±ΡƒΠ΄Π΅ΠΌ ΠΈΠΌΠ΅Ρ‚ΡŒ

    \[\overrightarrow{c}=n\overrightarrow{b}\]

    Π—Π½Π°Ρ‡ΠΈΡ‚, Ссли число $m=0$, Ρ‚ΠΎ ΠΏΠΎΠ»ΡƒΡ‡ΠΈΠΌ

    \[\overrightarrow{c}=m\overrightarrow{a}+n\overrightarrow{b}\]
  2. Π’Π΅ΠΊΡ‚ΠΎΡ€ $\overrightarrow{c}$ Π½Π΅ ΠΊΠΎΠ»Π»ΠΈΠ½Π΅Π°Ρ€Π΅Π½ Π²Π΅ΠΊΡ‚ΠΎΡ€Π°ΠΌ $\overrightarrow{a}$ ΠΈ $\overrightarrow{b}$.

    Π’ΠΎΠ·ΡŒΠΌΠ΅ΠΌ ΠΏΡ€ΠΎΠΈΠ·Π²ΠΎΠ»ΡŒΠ½ΡƒΡŽ Ρ‚ΠΎΡ‡ΠΊΡƒ $O$ ΠΈ ΠΎΡ‚Π»ΠΎΠΆΠΈΠΌ ΠΎΡ‚ Π½Π΅Π΅ Π²Π΅ΠΊΡ‚ΠΎΡ€Ρ‹ $\overrightarrow{OB}=\overrightarrow{b},\ \overrightarrow{OA}=\overrightarrow{a}$ ΠΈ $\overrightarrow{OC}=\overrightarrow{c}$. ΠŸΡƒΡΡ‚ΡŒ ΠŸΡ€ΠΎΠ²Π΅Π΄Π΅ΠΌ ΠΏΡ€ΡΠΌΡƒΡŽ $CD||OB$ (рис. 1)

    Рисунок 1. Π˜Π»Π»ΡŽΡΡ‚Ρ€Π°Ρ†ΠΈΡ Ρ‚Π΅ΠΎΡ€Π΅ΠΌΡ‹ 1

    По ΠΏΡ€Π°Π²ΠΈΠ»Ρƒ Ρ‚Ρ€Π΅ΡƒΠ³ΠΎΠ»ΡŒΠ½ΠΈΠΊΠ° для слоТСния Π²Π΅ΠΊΡ‚ΠΎΡ€ΠΎΠ², ΠΏΠΎΠ»ΡƒΡ‡ΠΈΠΌ

    \[\overrightarrow{c}=\overrightarrow{OD}+\overrightarrow{DC}\]

    По ΠΏΠΎΡΡ‚Ρ€ΠΎΠ΅Π½ΠΈΡŽ, ΠΏΠΎΠ»ΡƒΡ‡Π°Π΅ΠΌ Ρ‡Ρ‚ΠΎ Π²Π΅ΠΊΡ‚ΠΎΡ€Ρ‹ $\overrightarrow{OD}||\overrightarrow{a}$ ΠΈ $\overrightarrow{DC}||\overrightarrow{b}$, ΡΠ»Π΅Π΄ΠΎΠ²Π°Ρ‚Π΅Π»ΡŒΠ½ΠΎ, ΠΏΠΎ Π»Π΅ΠΌΠΌΠ΅ 1, ΠΈΠΌΠ΅Π΅ΠΌ

    \[\overrightarrow{OD}=m\overrightarrow{a},\ \overrightarrow{DC}=n\overrightarrow{b}\]

    Π—Π½Π°Ρ‡ΠΈΡ‚

    \[\overrightarrow{c}=m\overrightarrow{a}+n\overrightarrow{b}\]

Π•Π΄ΠΈΠ½ΡΡ‚Π²Π΅Π½Π½ΠΎΡΡ‚ΡŒ: ΠŸΡ€Π΅Π΄ΠΏΠΎΠ»ΠΎΠΆΠΈΠΌ ΠΏΡ€ΠΎΡ‚ΠΈΠ²Π½ΠΎΠ΅, Ρ‡Ρ‚ΠΎ ΠΏΠΎΠΌΠΈΠΌΠΎ разлоТСния$\overrightarrow{c}=m\overrightarrow{a}+n\overrightarrow{b}$ сущСствуСт Ρ€Π°Π·Π»ΠΎΠΆΠ΅Π½ΠΈΠ΅ $\overrightarrow{c}=m’\overrightarrow{a}+n’\overrightarrow{b}$. Π’Ρ‹Ρ‡Ρ‚Π΅ΠΌ эти Π΄Π²Π° равСнства ΠΈΠ· Π΄Ρ€ΡƒΠ³ Π΄Ρ€ΡƒΠ³Π°:

ΠŸΠΎΠ»ΡƒΡ‡Π°Π΅ΠΌ систСму:

Рисунок 2.

Π‘Π»Π΅Π΄ΠΎΠ²Π°Ρ‚Π΅Π»ΡŒΠ½ΠΎ, Ρ€Π°Π·Π»ΠΎΠΆΠ΅Π½ΠΈΠ΅ СдинствСнно.

Π’Π΅ΠΎΡ€Π΅ΠΌΠ° Π΄ΠΎΠΊΠ°Π·Π°Π½Π°.

ΠšΠΎΠΎΡ€Π΄ΠΈΠ½Π°Ρ‚Ρ‹ Π²Π΅ΠΊΡ‚ΠΎΡ€Π°

Рассмотрим Π΄Π°Π»Π΅Π΅ систСму ΠΊΠΎΠΎΡ€Π΄ΠΈΠ½Π°Ρ‚. ΠžΡ‚ Π½Π°Ρ‡Π°Π»Π° ΠΊΠΎΠΎΡ€Π΄ΠΈΠ½Π°Ρ‚ $O$ Π² Π½Π°ΠΏΡ€Π°Π²Π»Π΅Π½ΠΈΠΈ оси $Ox$ ΠΎΡ‚Π»ΠΎΠΆΠΈΠΌ Π²Π΅ΠΊΡ‚ΠΎΡ€ $\overrightarrow{i}$, Π° Π² Π½Π°ΠΏΡ€Π°Π²Π»Π΅Π½ΠΈΠΈ оси $Oy$ ΠΎΡ‚Π»ΠΎΠΆΠΈΠΌ Π²Π΅ΠΊΡ‚ΠΎΡ€ $\overrightarrow{j}$, Π΄Π»ΠΈΠ½Ρ‹ ΠΊΠΎΡ‚ΠΎΡ€Ρ‹Ρ… Ρ€Π°Π²Π½Ρ‹ Π΅Π΄ΠΈΠ½ΠΈΡ†Π΅.

ΠžΠΏΡ€Π΅Π΄Π΅Π»Π΅Π½ΠΈΠ΅ 1

Π’Π΅ΠΊΡ‚ΠΎΡ€Ρ‹ $\overrightarrow{i}$ ΠΈ $\overrightarrow{j}$ Π½Π°Π·Ρ‹Π²Π°ΡŽΡ‚ΡΡ ΠΊΠΎΠΎΡ€Π΄ΠΈΠ½Π°Ρ‚Π½Ρ‹ΠΌΠΈ Π²Π΅ΠΊΡ‚ΠΎΡ€Π°ΠΌΠΈ.

Π’Π°ΠΊ ΠΊΠ°ΠΊ Π²Π΅ΠΊΡ‚ΠΎΡ€Ρ‹ $\overrightarrow{i}$ ΠΈ $\overrightarrow{j}$ Π½Π΅ ΠΊΠΎΠ»Π»ΠΈΠ½Π΅Π°Ρ€Π½Ρ‹ Ρ‚ΠΎ, ΠΏΠΎ Ρ‚Π΅ΠΎΡ€Π΅ΠΌΠ΅ 1, любой Π²Π΅ΠΊΡ‚ΠΎΡ€ ΠΌΠΎΠΆΠ½ΠΎ Ρ€Π°Π·Π»ΠΎΠΆΠΈΡ‚ΡŒ Π² Π²ΠΈΠ΄Π΅ $\overrightarrow{c}=m\overrightarrow{i}+n\overrightarrow{j}$.

ΠžΠΏΡ€Π΅Π΄Π΅Π»Π΅Π½ΠΈΠ΅ 2

ΠšΠΎΡΡ„Ρ„ΠΈΡ†ΠΈΠ΅Π½Ρ‚Ρ‹ разлоТСния Π²Π΅ΠΊΡ‚ΠΎΡ€Π° $\overrightarrow{c}=m\overrightarrow{i}+n\overrightarrow{j}$ Π½Π°Π·Ρ‹Π²Π°ΡŽΡ‚ΡΡ ΠΊΠΎΠΎΡ€Π΄ΠΈΠ½Π°Ρ‚Π°ΠΌΠΈ Π΄Π°Π½Π½ΠΎΠ³ΠΎ Π²Π΅ΠΊΡ‚ΠΎΡ€Π° Π² Π΄Π°Π½Π½ΠΎΠΉ систСмС ΠΊΠΎΠΎΡ€Π΄ΠΈΠ½Π°Ρ‚, Ρ‚ΠΎ Π΅ΡΡ‚ΡŒ

\[\overrightarrow{c}=\{m,\ n\}\]

Π›ΠΈΠ½Π΅ΠΉΠ½Ρ‹Π΅ ΠΎΠΏΠ΅Ρ€Π°Ρ†ΠΈΠΈ Π½Π°Π΄ Π²Π΅ΠΊΡ‚ΠΎΡ€Π°ΠΌΠΈ

Π’Π΅ΠΎΡ€Π΅ΠΌΠ° 2

Π’Π΅ΠΎΡ€Π΅ΠΌΠ° ΠΎ суммС Π²Π΅ΠΊΡ‚ΠΎΡ€ΠΎΠ²: ΠšΠΎΠΎΡ€Π΄ΠΈΠ½Π°Ρ‚Ρ‹ суммы Π²Π΅ΠΊΡ‚ΠΎΡ€ΠΎΠ² Ρ€Π°Π²Π½Ρ‹ суммС ΡΠΎΠΎΡ‚Π²Π΅Ρ‚ΡΡ‚Π²ΡƒΡŽΡ‰ΠΈΡ… ΠΊΠΎΠΎΡ€Π΄ΠΈΠ½Π°Ρ‚ этих Π²Π΅ΠΊΡ‚ΠΎΡ€ΠΎΠ².

Π”ΠΎΠΊΠ°Π·Π°Ρ‚Π΅Π»ΡŒΡΡ‚Π²ΠΎ.

Π”ΠΎΠΊΠ°ΠΆΠ΅ΠΌ Ρ‚Π΅ΠΎΡ€Π΅ΠΌΡƒ для Π΄Π²ΡƒΡ… Π²Π΅ΠΊΡ‚ΠΎΡ€ΠΎΠ². Π’Π΅ΠΎΡ€Π΅ΠΌΠ° для большСго количСства Π²Π΅ΠΊΡ‚ΠΎΡ€ΠΎΠ² доказываСтся Π°Π½Π°Π»ΠΎΠ³ΠΈΡ‡Π½ΠΎ. ΠŸΡƒΡΡ‚ΡŒ $\overrightarrow{a}=\left\{x_1,\ y_1\right\}$, $\overrightarrow{b}=\{x_2,\ y_2\}$, Ρ‚ΠΎΠ³Π΄Π°

Π‘Π»Π΅Π΄ΠΎΠ²Π°Ρ‚Π΅Π»ΡŒΠ½ΠΎ

Π’Π΅ΠΎΡ€Π΅ΠΌΠ° Π΄ΠΎΠΊΠ°Π·Π°Π½Π°.

Π’Π΅ΠΎΡ€Π΅ΠΌΠ° 3

Π’Π΅ΠΎΡ€Π΅ΠΌΠ° ΠΎ разности Π²Π΅ΠΊΡ‚ΠΎΡ€ΠΎΠ²: ΠšΠΎΠΎΡ€Π΄ΠΈΠ½Π°Ρ‚Ρ‹ разности Π²Π΅ΠΊΡ‚ΠΎΡ€ΠΎΠ² Ρ€Π°Π²Π½Ρ‹ разности ΡΠΎΠΎΡ‚Π²Π΅Ρ‚ΡΡ‚Π²ΡƒΡŽΡ‰ΠΈΡ… ΠΊΠΎΠΎΡ€Π΄ΠΈΠ½Π°Ρ‚ этих Π²Π΅ΠΊΡ‚ΠΎΡ€ΠΎΠ².

Π”ΠΎΠΊΠ°Π·Π°Ρ‚Π΅Π»ΡŒΡΡ‚Π²ΠΎ.

Π”ΠΎΠΊΠ°ΠΆΠ΅ΠΌ Ρ‚Π΅ΠΎΡ€Π΅ΠΌΡƒ для Π΄Π²ΡƒΡ… Π²Π΅ΠΊΡ‚ΠΎΡ€ΠΎΠ². Π’Π΅ΠΎΡ€Π΅ΠΌΠ° для большСго количСства Π²Π΅ΠΊΡ‚ΠΎΡ€ΠΎΠ² доказываСтся Π°Π½Π°Π»ΠΎΠ³ΠΈΡ‡Π½ΠΎ. ΠŸΡƒΡΡ‚ΡŒ $\overrightarrow{a}=\left\{x_1,\ y_1\right\}$, $\overrightarrow{b}=\{x_2,\ y_2\}$, Ρ‚ΠΎΠ³Π΄Π°

Π‘Π»Π΅Π΄ΠΎΠ²Π°Ρ‚Π΅Π»ΡŒΠ½ΠΎ

Π’Π΅ΠΎΡ€Π΅ΠΌΠ° Π΄ΠΎΠΊΠ°Π·Π°Π½Π°.

Π’Π΅ΠΎΡ€Π΅ΠΌΠ° 4

Π’Π΅ΠΎΡ€Π΅ΠΌΠ° ΠΎ ΠΏΡ€ΠΎΠΈΠ·Π²Π΅Π΄Π΅Π½ΠΈΠΈ Π²Π΅ΠΊΡ‚ΠΎΡ€Π° Π½Π° число: ΠšΠΎΠΎΡ€Π΄ΠΈΠ½Π°Ρ‚Ρ‹ произвСдСния Π²Π΅ΠΊΡ‚ΠΎΡ€Π° Π½Π° число Ρ€Π°Π²Π½Ρ‹ ΠΏΡ€ΠΎΠΈΠ·Π²Π΅Π΄Π΅Π½ΠΈΡŽ ΡΠΎΠΎΡ‚Π²Π΅Ρ‚ΡΡ‚Π²ΡƒΡŽΡ‰ΠΈΡ… ΠΊΠΎΠΎΡ€Π΄ΠΈΠ½Π°Ρ‚ это число.

Π”ΠΎΠΊΠ°Π·Π°Ρ‚Π΅Π»ΡŒΡΡ‚Π²ΠΎ.

ΠŸΡƒΡΡ‚ΡŒ $\overrightarrow{a}=\left\{x,\ y\right\}$, Ρ‚ΠΎΠ³Π΄Π° $\overrightarrow{a}=x\overrightarrow{i}+\ y\overrightarrow{j}. $

Π‘Π»Π΅Π΄ΠΎΠ²Π°Ρ‚Π΅Π»ΡŒΠ½ΠΎ

Π’Π΅ΠΎΡ€Π΅ΠΌΠ° Π΄ΠΎΠΊΠ°Π·Π°Π½Π°.

ΠŸΡ€ΠΈΠΌΠ΅Ρ€ Π·Π°Π΄Π°Ρ‡ΠΈ Π½Π° Π½Π°Ρ…ΠΎΠΆΠ΄Π΅Π½ΠΈΠ΅ ΠΊΠΎΠΎΡ€Π΄ΠΈΠ½Π°Ρ‚ Π²Π΅ΠΊΡ‚ΠΎΡ€Π°

ΠŸΡ€ΠΈΠΌΠ΅Ρ€ 1

ΠŸΡƒΡΡ‚ΡŒ $\overrightarrow{a}=\left\{3,\ 4\right\}$, $\overrightarrow{b}=\{2,\ -1\}$. Найти $\overrightarrow{a}+\overrightarrow{b}$, $\overrightarrow{a}-\overrightarrow{b}$ ΠΈ $3\overrightarrow{a}$.

РСшСниС.

\[\overrightarrow{a}+\overrightarrow{b}=\left\{3+2,\ 4-1\right\}=\{5,\ 3\}\] \[\overrightarrow{a}-\overrightarrow{b}=\left\{3-2,\ 4+1\right\}=\{1,\ 5\}\] \[3\overrightarrow{a}=\left\{3\cdot 3,3\cdot 4\right\}=\{9,12\}\]

Как ΠΎΠΏΡ€Π΅Π΄Π΅Π»ΡΡŽΡ‚ΡΡ ΠΊΠΎΠΎΡ€Π΄ΠΈΠ½Π°Ρ‚Ρ‹ Π²Π΅ΠΊΡ‚ΠΎΡ€Π° Π² пространствС. ΠšΠΎΠΎΡ€Π΄ΠΈΠ½Π°Ρ‚Ρ‹ Π²Π΅ΠΊΡ‚ΠΎΡ€Π°. ΠΠ°ΠΏΡ€Π°Π²Π»ΡΡŽΡ‰ΠΈΠ΅ косинусы. Бвойства скалярного произвСдСния Π²Π΅ΠΊΡ‚ΠΎΡ€ΠΎΠ²

НахоТдСниС ΠΊΠΎΠΎΡ€Π΄ΠΈΠ½Π°Ρ‚ Π²Π΅ΠΊΡ‚ΠΎΡ€Π° довольно часто встрСчаСмоС условиС ΠΌΠ½ΠΎΠ³ΠΈΡ… Π·Π°Π΄Π°Ρ‡ Π² ΠΌΠ°Ρ‚Π΅ΠΌΠ°Ρ‚ΠΈΠΊΠ΅. Π£ΠΌΠ΅Π½ΠΈΠ΅ Π½Π°Ρ…ΠΎΠ΄ΠΈΡ‚ΡŒ ΠΊΠΎΠΎΡ€Π΄ΠΈΠ½Π°Ρ‚Ρ‹ Π²Π΅ΠΊΡ‚ΠΎΡ€Π° ΠΏΠΎΠΌΠΎΠΆΠ΅Ρ‚ Π²Π°ΠΌ Π² Π΄Ρ€ΡƒΠ³ΠΈΡ…, Π±ΠΎΠ»Π΅Π΅ слоТных Π·Π°Π΄Π°Ρ‡Π°Ρ… со схоТСй Ρ‚Π΅ΠΌΠ°Ρ‚ΠΈΠΊΠΎΠΉ. Π’ Π΄Π°Π½Π½ΠΎΠΉ ΡΡ‚Π°Ρ‚ΡŒΠ΅ ΠΌΡ‹ рассмотрим Ρ„ΠΎΡ€ΠΌΡƒΠ»Ρƒ нахоТдСния ΠΊΠΎΠΎΡ€Π΄ΠΈΠ½Π°Ρ‚ Π²Π΅ΠΊΡ‚ΠΎΡ€Π° ΠΈ нСсколько Π·Π°Π΄Π°Ρ‡.

НахоТдСниС ΠΊΠΎΠΎΡ€Π΄ΠΈΠ½Π°Ρ‚ Π²Π΅ΠΊΡ‚ΠΎΡ€Π° Π² плоскости

Π§Ρ‚ΠΎ Ρ‚Π°ΠΊΠΎΠ΅ ΠΏΠ»ΠΎΡΠΊΠΎΡΡ‚ΡŒ? ΠŸΠ»ΠΎΡΠΊΠΎΡΡ‚ΡŒΡŽ считаСтся Π΄Π²ΡƒΡ…ΠΌΠ΅Ρ€Π½ΠΎΠ΅ пространство, пространство с двумя измСрСниями (ΠΈΠ·ΠΌΠ΅Ρ€Π΅Π½ΠΈΠ΅ x ΠΈ ΠΈΠ·ΠΌΠ΅Ρ€Π΅Π½ΠΈΠ΅ y). К ΠΏΡ€ΠΈΠΌΠ΅Ρ€Ρƒ, Π±ΡƒΠΌΠ°Π³Π° – ΠΏΠ»ΠΎΡΠΊΠΎΡΡ‚ΡŒ. ΠŸΠΎΠ²Π΅Ρ€Ρ…Π½ΠΎΡΡ‚ΡŒ стола – ΠΏΠ»ΠΎΡΠΊΠΎΡΡ‚ΡŒ. Какая-Π½ΠΈΠ±ΡƒΠ΄ΡŒ нСобъСмная Ρ„ΠΈΠ³ΡƒΡ€Π° (ΠΊΠ²Π°Π΄Ρ€Π°Ρ‚, Ρ‚Ρ€Π΅ΡƒΠ³ΠΎΠ»ΡŒΠ½ΠΈΠΊ, трапСция) Ρ‚ΠΎΠΆΠ΅ являСтся ΠΏΠ»ΠΎΡΠΊΠΎΡΡ‚ΡŒΡŽ. Π’Π°ΠΊΠΈΠΌ ΠΎΠ±Ρ€Π°Π·ΠΎΠΌ, Ссли Π² условии Π·Π°Π΄Π°Ρ‡ΠΈ Π½ΡƒΠΆΠ½ΠΎ Π½Π°ΠΉΡ‚ΠΈ ΠΊΠΎΠΎΡ€Π΄ΠΈΠ½Π°Ρ‚Ρ‹ Π²Π΅ΠΊΡ‚ΠΎΡ€Π°, ΠΊΠΎΡ‚ΠΎΡ€Ρ‹ΠΉ Π»Π΅ΠΆΠΈΡ‚ Π½Π° плоскости, сразу вспоминаСм ΠΏΡ€ΠΎ x ΠΈ y. Найти ΠΊΠΎΠΎΡ€Π΄ΠΈΠ½Π°Ρ‚Ρ‹ Ρ‚Π°ΠΊΠΎΠ³ΠΎ Π²Π΅ΠΊΡ‚ΠΎΡ€Π° ΠΌΠΎΠΆΠ½ΠΎ ΡΠ»Π΅Π΄ΡƒΡŽΡ‰ΠΈΠΌ ΠΎΠ±Ρ€Π°Π·ΠΎΠΌ: ΠšΠΎΠΎΡ€Π΄ΠΈΠ½Π°Ρ‚Ρ‹ AB Π²Π΅ΠΊΡ‚ΠΎΡ€Π° = (xB – xA; yB – xA). Из Ρ„ΠΎΡ€ΠΌΡƒΠ»Ρ‹ Π²ΠΈΠ΄Π½ΠΎ, Ρ‡Ρ‚ΠΎ ΠΎΡ‚ ΠΊΠΎΠΎΡ€Π΄ΠΈΠ½Π°Ρ‚ ΠΊΠΎΠ½Π΅Ρ‡Π½ΠΎΠΉ Ρ‚ΠΎΡ‡ΠΊΠΈ Π½ΡƒΠΆΠ½ΠΎ ΠΎΡ‚Π½ΡΡ‚ΡŒ ΠΊΠΎΠΎΡ€Π΄ΠΈΠ½Π°Ρ‚Ρ‹ Π½Π°Ρ‡Π°Π»ΡŒΠ½ΠΎΠΉ Ρ‚ΠΎΡ‡ΠΊΠΈ.

ΠŸΡ€ΠΈΠΌΠ΅Ρ€:

  • Π’Π΅ΠΊΡ‚ΠΎΡ€ CD ΠΈΠΌΠ΅Π΅Ρ‚ Π½Π°Ρ‡Π°Π»ΡŒΠ½Ρ‹Π΅ (5; 6) ΠΈ ΠΊΠΎΠ½Π΅Ρ‡Π½Ρ‹Π΅ (7; 8) ΠΊΠΎΠΎΡ€Π΄ΠΈΠ½Π°Ρ‚Ρ‹.
  • Найти ΠΊΠΎΠΎΡ€Π΄ΠΈΠ½Π°Ρ‚Ρ‹ самого Π²Π΅ΠΊΡ‚ΠΎΡ€Π°.
  • Π˜ΡΠΏΠΎΠ»ΡŒΠ·ΡƒΡ Π²Ρ‹ΡˆΠ΅ΡƒΠΏΠΎΠΌΡΠ½ΡƒΡ‚ΡƒΡŽ Ρ„ΠΎΡ€ΠΌΡƒΠ»Ρƒ, ΠΏΠΎΠ»ΡƒΡ‡ΠΈΠΌ ΡΠ»Π΅Π΄ΡƒΡŽΡ‰Π΅Π΅ Π²Ρ‹Ρ€Π°ΠΆΠ΅Π½ΠΈΠ΅: CD = (7-5; 8-6) = (2; 2).
  • Π’Π°ΠΊΠΈΠΌ ΠΎΠ±Ρ€Π°Π·ΠΎΠΌ, ΠΊΠΎΠΎΡ€Π΄ΠΈΠ½Π°Ρ‚Ρ‹ CD Π²Π΅ΠΊΡ‚ΠΎΡ€Π° = (2; 2).
  • БоотвСтствСнно, x ΠΊΠΎΠΎΡ€Π΄ΠΈΠ½Π°Ρ‚Π° Ρ€Π°Π²Π½Π° Π΄Π²ΡƒΠΌ, y ΠΊΠΎΠΎΡ€Π΄ΠΈΠ½Π°Ρ‚Π° – Ρ‚ΠΎΠΆΠ΅ Π΄Π²ΡƒΠΌ.

НахоТдСниС ΠΊΠΎΠΎΡ€Π΄ΠΈΠ½Π°Ρ‚ Π²Π΅ΠΊΡ‚ΠΎΡ€Π° Π² пространствС

Π§Ρ‚ΠΎ Ρ‚Π°ΠΊΠΎΠ΅ пространство? ΠŸΡ€ΠΎΡΡ‚Ρ€Π°Π½ΡΡ‚Π²ΠΎ это ΡƒΠΆΠ΅ Ρ‚Ρ€Π΅Ρ…ΠΌΠ΅Ρ€Π½ΠΎΠ΅ ΠΈΠ·ΠΌΠ΅Ρ€Π΅Π½ΠΈΠ΅, Π³Π΄Π΅ Π΄Π°Π½Ρ‹ 3 ΠΊΠΎΠΎΡ€Π΄ΠΈΠ½Π°Ρ‚Ρ‹: x, y, z. Π’ случаС, Ссли Π½ΡƒΠΆΠ½ΠΎ Π½Π°ΠΉΡ‚ΠΈ Π²Π΅ΠΊΡ‚ΠΎΡ€, ΠΊΠΎΡ‚ΠΎΡ€Ρ‹ΠΉ Π»Π΅ΠΆΠΈΡ‚ Π² пространствС, Ρ„ΠΎΡ€ΠΌΡƒΠ»Π° практичСски Π½Π΅ мСняСтся. ДобавляСтся Ρ‚ΠΎΠ»ΡŒΠΊΠΎ ΠΎΠ΄Π½Π° ΠΊΠΎΠΎΡ€Π΄ΠΈΠ½Π°Ρ‚Π°. Для нахоТдСния Π²Π΅ΠΊΡ‚ΠΎΡ€Π° Π½ΡƒΠΆΠ½ΠΎ ΠΎΡ‚ ΠΊΠΎΠΎΡ€Π΄ΠΈΠ½Π°Ρ‚ ΠΊΠΎΠ½Ρ†Π° ΠΎΡ‚Π½ΡΡ‚ΡŒ ΠΊΠΎΠΎΡ€Π΄ΠΈΠ½Π°Ρ‚Ρ‹ Π½Π°Ρ‡Π°Π»Π°. AB = (xB – xA; yB – yA; zB – zA)

ΠŸΡ€ΠΈΠΌΠ΅Ρ€:

  • Π’Π΅ΠΊΡ‚ΠΎΡ€ DF ΠΈΠΌΠ΅Π΅Ρ‚ Π½Π°Ρ‡Π°Π»ΡŒΠ½Ρ‹Π΅ (2; 3; 1) ΠΈ ΠΊΠΎΠ½Π΅Ρ‡Π½Ρ‹Π΅ (1; 5; 2).
  • ΠŸΡ€ΠΈΠΌΠ΅Π½ΡΡ Π²Ρ‹ΡˆΠ΅ΡƒΠΏΠΎΠΌΡΠ½ΡƒΡ‚ΡƒΡŽ Ρ„ΠΎΡ€ΠΌΡƒΠ»Ρƒ, ΠΏΠΎΠ»ΡƒΡ‡ΠΈΠΌ: ΠšΠΎΠΎΡ€Π΄ΠΈΠ½Π°Ρ‚Ρ‹ Π²Π΅ΠΊΡ‚ΠΎΡ€Π° DF = (1-2; 5-3; 2-1) = (-1; 2; 1).
  • ΠŸΠΎΠΌΠ½ΠΈΡ‚Π΅, Π·Π½Π°Ρ‡Π΅Π½ΠΈΠ΅ ΠΊΠΎΠΎΡ€Π΄ΠΈΠ½Π°Ρ‚ ΠΌΠΎΠΆΠ΅Ρ‚ Π±Ρ‹Ρ‚ΡŒ ΠΈ ΠΎΡ‚Ρ€ΠΈΡ†Π°Ρ‚Π΅Π»ΡŒΠ½Ρ‹ΠΌ, Π² этом Π½Π΅Ρ‚ Π½ΠΈΠΊΠ°ΠΊΠΎΠΉ ΠΏΡ€ΠΎΠ±Π»Π΅ΠΌΡ‹.


Как Π½Π°ΠΉΡ‚ΠΈ ΠΊΠΎΠΎΡ€Π΄ΠΈΠ½Π°Ρ‚Ρ‹ Π²Π΅ΠΊΡ‚ΠΎΡ€Π° ΠΎΠ½Π»Π°ΠΉΠ½?

Если ΠΏΠΎ ΠΊΠ°ΠΊΠΈΠΌ-Ρ‚ΠΎ ΠΏΡ€ΠΈΡ‡ΠΈΠ½Π°ΠΌ Π²Π°ΠΌ Π½Π΅ хочСтся Π½Π°Ρ…ΠΎΠ΄ΠΈΡ‚ΡŒ ΠΊΠΎΠΎΡ€Π΄ΠΈΠ½Π°Ρ‚Ρ‹ ΡΠ°ΠΌΠΎΡΡ‚ΠΎΡΡ‚Π΅Π»ΡŒΠ½ΠΎ, ΠΌΠΎΠΆΠ½ΠΎ Π²ΠΎΡΠΏΠΎΠ»ΡŒΠ·ΠΎΠ²Π°Ρ‚ΡŒΡΡ ΠΎΠ½Π»Π°ΠΉΠ½ ΠΊΠ°Π»ΡŒΠΊΡƒΠ»ΡΡ‚ΠΎΡ€ΠΎΠΌ . Для Π½Π°Ρ‡Π°Π»Π°, Π²Ρ‹Π±Π΅Ρ€ΠΈΡ‚Π΅ Ρ€Π°Π·ΠΌΠ΅Ρ€Π½ΠΎΡΡ‚ΡŒ Π²Π΅ΠΊΡ‚ΠΎΡ€Π°. Π Π°Π·ΠΌΠ΅Ρ€Π½ΠΎΡΡ‚ΡŒ Π²Π΅ΠΊΡ‚ΠΎΡ€Π° ΠΎΡ‚Π²Π΅Ρ‡Π°Π΅Ρ‚ Π·Π° Π΅Π³ΠΎ измСрСния. Π Π°Π·ΠΌΠ΅Ρ€Π½ΠΎΡΡ‚ΡŒ 3 ΠΎΠ·Π½Π°Ρ‡Π°Π΅Ρ‚, Ρ‡Ρ‚ΠΎ Π²Π΅ΠΊΡ‚ΠΎΡ€ находится Π² пространствС, Ρ€Π°Π·ΠΌΠ΅Ρ€Π½ΠΎΡΡ‚ΡŒ 2 – Ρ‡Ρ‚ΠΎ Π½Π° плоскости. Π”Π°Π»Π΅Π΅ Π²ΡΡ‚Π°Π²ΡŒΡ‚Π΅ ΠΊΠΎΠΎΡ€Π΄ΠΈΠ½Π°Ρ‚Ρ‹ Ρ‚ΠΎΡ‡Π΅ΠΊ Π² ΡΠΎΠΎΡ‚Π²Π΅Ρ‚ΡΡ‚Π²ΡƒΡŽΡ‰ΠΈΠ΅ поля ΠΈ ΠΏΡ€ΠΎΠ³Ρ€Π°ΠΌΠΌΠ° ΠΎΠΏΡ€Π΅Π΄Π΅Π»ΠΈΡ‚ Π²Π°ΠΌ ΠΊΠΎΠΎΡ€Π΄ΠΈΠ½Π°Ρ‚Ρ‹ самого Π²Π΅ΠΊΡ‚ΠΎΡ€Π°. ВсС ΠΎΡ‡Π΅Π½ΡŒ просто.


НаТав Π½Π° ΠΊΠ½ΠΎΠΏΠΊΡƒ, страница автоматичСски прокрутится Π²Π½ΠΈΠ· ΠΈ выдаст Π²Π°ΠΌ ΠΏΡ€Π°Π²ΠΈΠ»ΡŒΠ½Ρ‹ΠΉ ΠΎΡ‚Π²Π΅Ρ‚ вмСстС с этапами Ρ€Π΅ΡˆΠ΅Π½ΠΈΡ.


Π Π΅ΠΊΠΎΠΌΠ΅Π½Π΄ΠΎΠ²Π°Π½ΠΎ Ρ…ΠΎΡ€ΠΎΡˆΠΎ ΠΈΠ·ΡƒΡ‡ΠΈΡ‚ΡŒ Π΄Π°Π½Π½ΡƒΡŽ Ρ‚Π΅ΠΌΡƒ, ΠΏΠΎΡ‚ΠΎΠΌΡƒ Ρ‡Ρ‚ΠΎ понятиС Π²Π΅ΠΊΡ‚ΠΎΡ€Π° встрСчаСтся Π½Π΅ Ρ‚ΠΎΠ»ΡŒΠΊΠΎ Π² ΠΌΠ°Ρ‚Π΅ΠΌΠ°Ρ‚ΠΈΠΊΠ΅, Π½ΠΎ ΠΈ Π² Ρ„ΠΈΠ·ΠΈΠΊΠ΅. Π‘Ρ‚ΡƒΠ΄Π΅Π½Ρ‚Ρ‹ Ρ„Π°ΠΊΡƒΠ»ΡŒΡ‚Π΅Ρ‚Π° Π˜Π½Ρ„ΠΎΡ€ΠΌΠ°Ρ†ΠΈΠΎΠ½Π½Ρ‹Ρ… Π’Π΅Ρ…Π½ΠΎΠ»ΠΎΠ³ΠΈΠΉ Ρ‚ΠΎΠΆΠ΅ ΠΈΠ·ΡƒΡ‡Π°ΡŽΡ‚ Ρ‚Π΅ΠΌΡƒ Π²Π΅ΠΊΡ‚ΠΎΡ€ΠΎΠ², Π½ΠΎ Π½Π° Π±ΠΎΠ»Π΅Π΅ слоТном ΡƒΡ€ΠΎΠ²Π½Π΅.

НСобходимо ΠΎΠΏΡ€Π΅Π΄Π΅Π»ΠΈΡ‚ΡŒ Π²Π΅ΠΊΡ‚ΠΎΡ€ Π½Π° плоскости ΠΈΠ»ΠΈ Π² пространствС, Ρ‚ΠΎ Π΅ΡΡ‚ΡŒ Π΄Π°Ρ‚ΡŒ ΠΈΠ½Ρ„ΠΎΡ€ΠΌΠ°Ρ†ΠΈΡŽ ΠΎ Π΅Π³ΠΎ Π½Π°ΠΏΡ€Π°Π²Π»Π΅Π½ΠΈΠΈ ΠΈ Π΄Π»ΠΈΠ½Π΅.

ΠšΠΎΠΎΡ€Π΄ΠΈΠ½Π°Ρ‚Ρ‹ Π²Π΅ΠΊΡ‚ΠΎΡ€Π°

ΠŸΡƒΡΡ‚ΡŒ Π·Π°Π΄Π°Π½Π° ΠΏΡ€ΡΠΌΠΎΡƒΠ³ΠΎΠ»ΡŒΠ½Π°Ρ Π΄Π΅ΠΊΠ°Ρ€Ρ‚ΠΎΠ²Π° систСма ΠΊΠΎΠΎΡ€Π΄ΠΈΠ½Π°Ρ‚ (ΠŸΠ”Π‘Πš) $x O y$ ΠΈ ΠΏΡ€ΠΎΠΈΠ·Π²ΠΎΠ»ΡŒΠ½Ρ‹ΠΉ Π²Π΅ΠΊΡ‚ΠΎΡ€ $\overline{a}$, Π½Π°Ρ‡Π°Π»ΠΎ ΠΊΠΎΡ‚ΠΎΡ€ΠΎΠ³ΠΎ совпадаСт с Π½Π°Ρ‡Π°Π»ΠΎΠΌ систСмы ΠΊΠΎΠΎΡ€Π΄ΠΈΠ½Π°Ρ‚ (рис. 1).

ΠžΠΏΡ€Π΅Π΄Π΅Π»Π΅Π½ΠΈΠ΅

ΠšΠΎΠΎΡ€Π΄ΠΈΠ½Π°Ρ‚Π°ΠΌΠΈ Π²Π΅ΠΊΡ‚ΠΎΡ€Π° $\overline{a}$ Π½Π°Π·Ρ‹Π²Π°ΡŽΡ‚ΡΡ ΠΏΡ€ΠΎΠ΅ΠΊΡ†ΠΈΠΈ $a_{x}$ ΠΈ $a_{y}$ Π΄Π°Π½Π½ΠΎΠ³ΠΎ Π²Π΅ΠΊΡ‚ΠΎΡ€Π° Π½Π° оси $O x$ ΠΈ $O y$ соотвСтствСнно:

Π’Π΅Π»ΠΈΡ‡ΠΈΠ½Π° $a_{x}$ называСтся абсциссой Π²Π΅ΠΊΡ‚ΠΎΡ€Π° $\overline{a}$, Π° число $a_{y}$ — Π΅Π³ΠΎ ΠΎΡ€Π΄ΠΈΠ½Π°Ρ‚ΠΎΠΉ . Π’ΠΎ, Ρ‡Ρ‚ΠΎ Π²Π΅ΠΊΡ‚ΠΎΡ€ $\overline{a}$ ΠΈΠΌΠ΅Π΅Ρ‚ ΠΊΠΎΠΎΡ€Π΄ΠΈΠ½Π°Ρ‚Ρ‹ $a_{x}$ ΠΈ $a_{y}$, записываСтся ΡΠ»Π΅Π΄ΡƒΡŽΡ‰ΠΈΠΌ ΠΎΠ±Ρ€Π°Π·ΠΎΠΌ: $\overline{a}=\left(a_{x} ; a_{y}\right)$.

ΠŸΡ€ΠΈΠΌΠ΅Ρ€

Π—Π°ΠΏΠΈΡΡŒ $\overline{a}=(5 ;-2)$ ΠΎΠ·Π½Π°Ρ‡Π°Π΅Ρ‚, Ρ‡Ρ‚ΠΎ Π²Π΅ΠΊΡ‚ΠΎΡ€ $\overline{a}$ ΠΈΠΌΠ΅Π΅Ρ‚ ΡΠ»Π΅Π΄ΡƒΡŽΡ‰ΠΈΠ΅ ΠΊΠΎΠΎΡ€Π΄ΠΈΠ½Π°Ρ‚Ρ‹: абсцисса Ρ€Π°Π²Π½Π° 5, ΠΎΡ€Π΄ΠΈΠ½Π°Ρ‚Π° Ρ€Π°Π²Π½Π° -2.

Π‘ΡƒΠΌΠΌΠ° Π΄Π²ΡƒΡ… Π²Π΅ΠΊΡ‚ΠΎΡ€ΠΎΠ², Π·Π°Π΄Π°Π½Π½Ρ‹Ρ… ΠΊΠΎΠΎΡ€Π΄ΠΈΠ½Π°Ρ‚Π°ΠΌΠΈ

ΠŸΡƒΡΡ‚ΡŒ Π·Π°Π΄Π°Π½Ρ‹ $\overline{a}=\left(a_{x} ; a_{y}\right)$ ΠΈ $\overline{b}=\left(b_{x} ; b_{y}\right)$, Ρ‚ΠΎΠ³Π΄Π° Π²Π΅ΠΊΡ‚ΠΎΡ€ $\overline{c}=\overline{a}+\overline{b}$ ΠΈΠΌΠ΅Π΅Ρ‚ ΠΊΠΎΠΎΡ€Π΄ΠΈΠ½Π°Ρ‚Ρ‹ $\left(a_{x}+b_{x} ; a_{y}+b_{y}\right)$ (рис. 2).

ΠžΠΏΡ€Π΅Π΄Π΅Π»Π΅Π½ΠΈΠ΅

Π§Ρ‚ΠΎΠ±Ρ‹ Π½Π°ΠΉΡ‚ΠΈ сумму Π΄Π²ΡƒΡ… Π²Π΅ΠΊΡ‚ΠΎΡ€ΠΎΠ² , Π·Π°Π΄Π°Π½Π½Ρ‹Ρ… своими ΠΊΠΎΠΎΡ€Π΄ΠΈΠ½Π°Ρ‚Π°ΠΌΠΈ, Π½Π°Π΄ΠΎ ΡΠ»ΠΎΠΆΠΈΡ‚ΡŒ ΠΈΡ… ΡΠΎΠΎΡ‚Π²Π΅Ρ‚ΡΡ‚Π²ΡƒΡŽΡ‰ΠΈΠ΅ ΠΊΠΎΠΎΡ€Π΄ΠΈΠ½Π°Ρ‚Ρ‹.

ΠŸΡ€ΠΈΠΌΠ΅Ρ€

Π—Π°Π΄Π°Π½ΠΈΠ΅. Π—Π°Π΄Π°Π½Ρ‹ $\overline{a}=(-3 ; 5)$ ΠΈ $\overline{b}=(0 ;-1)$. Найти ΠΊΠΎΠΎΡ€Π΄ΠΈΠ½Π°Ρ‚Ρ‹ Π²Π΅ΠΊΡ‚ΠΎΡ€Π° $\overline{c}=\overline{a}+\overline{b}$

РСшСниС. $\overline{c}=\overline{a}+\overline{b}=(-3 ; 5)+(0 ;-1)=(-3+0 ; 5+(-1))=(-3 ; 4)$

Π£ΠΌΠ½ΠΎΠΆΠ΅Π½ΠΈΠ΅ Π²Π΅ΠΊΡ‚ΠΎΡ€Π° Π½Π° число

Если Π·Π°Π΄Π°Π½ $\overline{a}=\left(a_{x} ; a_{y}\right)$, Ρ‚ΠΎ Ρ‚ΠΎΠ³Π΄Π° Π²Π΅ΠΊΡ‚ΠΎΡ€ $m \overline{a}$ ΠΈΠΌΠ΅Π΅Ρ‚ ΠΊΠΎΠΎΡ€Π΄ΠΈΠ½Π°Ρ‚Ρ‹ $m \overline{a}=\left(m a_{x} ; m a_{y}\right)$, здСсь $m$ — Π½Π΅ΠΊΠΎΡ‚ΠΎΡ€ΠΎΠ΅ число (рис. 3).

ΠŸΡ€ΠΈΠΌΠ΅Ρ€

Π—Π°Π΄Π°Π½ΠΈΠ΅. Π’Π΅ΠΊΡ‚ΠΎΡ€ $\overline{a}=(3 ;-2)$. Найти ΠΊΠΎΠΎΡ€Π΄ΠΈΠ½Π°Ρ‚Ρ‹ Π²Π΅ΠΊΡ‚ΠΎΡ€Π° 2$\overline{a}$

РСшСниС. $2 \overline{a}=2 \cdot(3 ;-2)=(2 \cdot 3 ; 2 \cdot(-2))=(6 ;-4)$

Рассмотрим Π΄Π°Π»Π΅Π΅ случай, ΠΊΠΎΠ³Π΄Π° Π½Π°Ρ‡Π°Π»ΠΎ Π²Π΅ΠΊΡ‚ΠΎΡ€Π° Π½Π΅ совпадаСт с Π½Π°Ρ‡Π°Π»ΠΎΠΌ систСмы ΠΊΠΎΠΎΡ€Π΄ΠΈΠ½Π°Ρ‚. ΠŸΡ€Π΅Π΄ΠΏΠΎΠ»ΠΎΠΆΠΈΠΌ, Ρ‡Ρ‚ΠΎ Π² ΠŸΠ”Π‘Πš Π·Π°Π΄Π°Π½Ρ‹ Π΄Π²Π΅ Ρ‚ΠΎΡ‡ΠΊΠΈ $A\left(a_{x} ; a_{y}\right)$ ΠΈ $B\left(b_{x} ; b_{y}\right)$. Π’ΠΎΠ³Π΄Π° ΠΊΠΎΠΎΡ€Π΄ΠΈΠ½Π°Ρ‚Ρ‹ Π²Π΅ΠΊΡ‚ΠΎΡ€Π° $\overline{A B}=\left(x_{1} ; y_{1}\right)$ находятся ΠΏΠΎ Ρ„ΠΎΡ€ΠΌΡƒΠ»Π°ΠΌ (рис. 4):

$x_{1}=b_{x}-a_{x}, y_{1}=b_{y}-a_{y}$

ΠžΠΏΡ€Π΅Π΄Π΅Π»Π΅Π½ΠΈΠ΅

Π§Ρ‚ΠΎΠ±Ρ‹ Π½Π°ΠΉΡ‚ΠΈ ΠΊΠΎΠΎΡ€Π΄ΠΈΠ½Π°Ρ‚Ρ‹ Π²Π΅ΠΊΡ‚ΠΎΡ€Π° , Π·Π°Π΄Π°Π½Π½ΠΎΠ³ΠΎ ΠΊΠΎΠΎΡ€Π΄ΠΈΠ½Π°Ρ‚Π°ΠΌΠΈ Π½Π°Ρ‡Π°Π»Π° ΠΈ ΠΊΠΎΠ½Ρ†Π°, Π½Π°Π΄ΠΎ ΠΎΡ‚ ΠΊΠΎΠΎΡ€Π΄ΠΈΠ½Π°Ρ‚ ΠΊΠΎΠ½Ρ†Π° ΠΎΡ‚Π½ΡΡ‚ΡŒ ΡΠΎΠΎΡ‚Π²Π΅Ρ‚ΡΡ‚Π²ΡƒΡŽΡ‰ΠΈΠ΅ ΠΊΠΎΠΎΡ€Π΄ΠΈΠ½Π°Ρ‚Ρ‹ Π½Π°Ρ‡Π°Π»Π°.

ΠŸΡ€ΠΈΠΌΠ΅Ρ€

Π—Π°Π΄Π°Π½ΠΈΠ΅. Найти ΠΊΠΎΠΎΡ€Π΄ΠΈΠ½Π°Ρ‚Ρ‹ Π²Π΅ΠΊΡ‚ΠΎΡ€Π° $\overline{A B}$, Ссли $A(-4 ; 2), B(1 ;-3)$

РСшСниС. $\overline{A B}=(1-(-4) ;-3-2)=(5 ;-5)$

ΠΠ°ΠΏΡ€Π°Π²Π»ΡΡŽΡ‰ΠΈΠ΅ косинусы

ΠžΠΏΡ€Π΅Π΄Π΅Π»Π΅Π½ΠΈΠ΅

ΠΠ°ΠΏΡ€Π°Π²Π»ΡΡŽΡ‰ΠΈΠΌΠΈ косинусами Π²Π΅ΠΊΡ‚ΠΎΡ€Π° Π½Π°Π·Ρ‹Π²Π°ΡŽΡ‚ΡΡ косинусы ΡƒΠ³Π»ΠΎΠ², ΠΎΠ±Ρ€Π°Π·ΠΎΠ²Π°Π½Π½Ρ‹Ρ… Π²Π΅ΠΊΡ‚ΠΎΡ€ΠΎΠΌ с ΠΏΠΎΠ»ΠΎΠΆΠΈΡ‚Π΅Π»ΡŒΠ½Ρ‹ΠΌΠΈ направлСниями осСй ΠΊΠΎΠΎΡ€Π΄ΠΈΠ½Π°Ρ‚. {2}}}$

Π—Π΄Π΅ΡΡŒ $\alpha$, $\beta$ ΠΈ $\gamma$ — ΡƒΠ³Π»Ρ‹, ΠΊΠΎΡ‚ΠΎΡ€Ρ‹Π΅ составляСт Π²Π΅ΠΊΡ‚ΠΎΡ€ с ΠΏΠΎΠ»ΠΎΠΆΠΈΡ‚Π΅Π»ΡŒΠ½Ρ‹ΠΌΠΈ направлСниями осСй $O x$, $O y$ ΠΈ $O z$ соотвСтствСнно.

На оси абсцисс ΠΈ ΠΎΡ€Π΄ΠΈΠ½Π°Ρ‚ Π½Π°Π·Ρ‹Π²Π°ΡŽΡ‚ΡΡ ΠΊΠΎΠΎΡ€Π΄ΠΈΠ½Π°Ρ‚Π°ΠΌΠΈ Π²Π΅ΠΊΡ‚ΠΎΡ€Π° . ΠšΠΎΠΎΡ€Π΄ΠΈΠ½Π°Ρ‚Ρ‹ Π²Π΅ΠΊΡ‚ΠΎΡ€Π° общСпринято ΡƒΠΊΠ°Π·Ρ‹Π²Π°Ρ‚ΡŒ Π² Π²ΠΈΠ΄Π΅ (Ρ…, Ρƒ) , Π° сам Π²Π΅ΠΊΡ‚ΠΎΡ€ ΠΊΠ°ΠΊ: =(Ρ…, Ρƒ).

Π€ΠΎΡ€ΠΌΡƒΠ»Π° опрСдСлСния ΠΊΠΎΠΎΡ€Π΄ΠΈΠ½Π°Ρ‚ Π²Π΅ΠΊΡ‚ΠΎΡ€Π° для Π΄Π²ΡƒΡ…ΠΌΠ΅Ρ€Π½Ρ‹Ρ… Π·Π°Π΄Π°Ρ‡.

Π’ случаС Π΄Π²ΡƒΡ…ΠΌΠ΅Ρ€Π½ΠΎΠΉ Π·Π°Π΄Π°Ρ‡ΠΈ Π²Π΅ΠΊΡ‚ΠΎΡ€ с извСстными ΠΊΠΎΠΎΡ€Π΄ΠΈΠ½Π°Ρ‚Π°ΠΌΠΈ Ρ‚ΠΎΡ‡Π΅ΠΊ A(Ρ… 1 ;Ρƒ 1) ΠΈ B(x 2 ; y 2 ) ΠΌΠΎΠΆΠ½ΠΎ Π²Ρ‹Ρ‡ΠΈΡΠ»ΠΈΡ‚ΡŒ:

= (x 2 — x 1 ; y 2 — y 1).

Π€ΠΎΡ€ΠΌΡƒΠ»Π° опрСдСлСния ΠΊΠΎΠΎΡ€Π΄ΠΈΠ½Π°Ρ‚ Π²Π΅ΠΊΡ‚ΠΎΡ€Π° для пространствСнных Π·Π°Π΄Π°Ρ‡.

Π’ случаС пространствСнной Π·Π°Π΄Π°Ρ‡ΠΈ Π²Π΅ΠΊΡ‚ΠΎΡ€ с извСстными ΠΊΠΎΠΎΡ€Π΄ΠΈΠ½Π°Ρ‚Π°ΠΌΠΈ Ρ‚ΠΎΡ‡Π΅ΠΊ A(Ρ… 1 ;Ρƒ 1 ; z 1 ) ΠΈ B(x 2 ; y 2 ; z 2 ) ΠΌΠΎΠΆΠ½ΠΎ Π²Ρ‹Ρ‡ΠΈΡΠ»ΠΈΡ‚ΡŒ ΠΏΡ€ΠΈΠΌΠ΅Π½ΠΈΠ² Ρ„ΠΎΡ€ΠΌΡƒΠ»Ρƒ:

= (x 2 x 1 ; y 2 y 1 ; z 2 z 1 ).

ΠšΠΎΠΎΡ€Π΄ΠΈΠ½Π°Ρ‚Ρ‹ Π΄Π°ΡŽΡ‚ Π²ΡΠ΅ΠΎΠ±ΡŠΠ΅ΠΌΠ»ΡŽΡ‰ΡƒΡŽ характСристику Π²Π΅ΠΊΡ‚ΠΎΡ€Π°, ΠΏΠΎΡΠΊΠΎΠ»ΡŒΠΊΡƒ ΠΏΠΎ ΠΊΠΎΠΎΡ€Π΄ΠΈΠ½Π°Ρ‚Π°ΠΌ Π΅ΡΡ‚ΡŒ Π²ΠΎΠ·ΠΌΠΎΠΆΠ½ΠΎΡΡ‚ΡŒ ΠΏΠΎΡΡ‚Ρ€ΠΎΠΈΡ‚ΡŒ ΠΈ сам Π²Π΅ΠΊΡ‚ΠΎΡ€. Зная ΠΊΠΎΠΎΡ€Π΄ΠΈΠ½Π°Ρ‚Ρ‹, Π»Π΅Π³ΠΊΠΎ Π²Ρ‹Ρ‡ΠΈΡΠ»ΠΈΡ‚ΡŒ ΠΈ Π΄Π»ΠΈΠ½Ρƒ Π²Π΅ΠΊΡ‚ΠΎΡ€Π° . (Бвойство 3, ΠΏΡ€ΠΈΠ²Π΅Π΄Π΅Π½Π½ΠΎΠ΅ Π½ΠΈΠΆΠ΅).

Бвойства ΠΊΠΎΠΎΡ€Π΄ΠΈΠ½Π°Ρ‚ Π²Π΅ΠΊΡ‚ΠΎΡ€Π°.

1. Π›ΡŽΠ±Ρ‹Π΅ Ρ€Π°Π²Π½Ρ‹Π΅ Π²Π΅ΠΊΡ‚ΠΎΡ€Ρ‹ Π² Π΅Π΄ΠΈΠ½ΠΎΠΉ систСмС ΠΊΠΎΠΎΡ€Π΄ΠΈΠ½Π°Ρ‚ ΠΈΠΌΠ΅ΡŽΡ‚ Ρ€Π°Π²Π½Ρ‹Π΅ ΠΊΠΎΠΎΡ€Π΄ΠΈΠ½Π°Ρ‚Ρ‹ .

2. ΠšΠΎΠΎΡ€Π΄ΠΈΠ½Π°Ρ‚Ρ‹ ΠΊΠΎΠ»Π»ΠΈΠ½Π΅Π°Ρ€Π½Ρ‹Ρ… Π²Π΅ΠΊΡ‚ΠΎΡ€ΠΎΠ² ΠΏΡ€ΠΎΠΏΠΎΡ€Ρ†ΠΈΠΎΠ½Π°Π»ΡŒΠ½Ρ‹. ΠŸΡ€ΠΈ условии, Ρ‡Ρ‚ΠΎ Π½ΠΈ ΠΎΠ΄ΠΈΠ½ ΠΈΠ· Π²Π΅ΠΊΡ‚ΠΎΡ€ΠΎΠ² Π½Π΅ Ρ€Π°Π²Π΅Π½ Π½ΡƒΠ»ΡŽ.

3. ΠšΠ²Π°Π΄Ρ€Π°Ρ‚ Π΄Π»ΠΈΠ½Ρ‹ любого Π²Π΅ΠΊΡ‚ΠΎΡ€Π° Ρ€Π°Π²Π΅Π½ суммС ΠΊΠ²Π°Π΄Ρ€Π°Ρ‚ΠΎΠ² Π΅Π³ΠΎ ΠΊΠΎΠΎΡ€Π΄ΠΈΠ½Π°Ρ‚ .

4.ΠŸΡ€ΠΈ ΠΎΠΏΠ΅Ρ€Π°Ρ†ΠΈΠΈ умноТСния Π²Π΅ΠΊΡ‚ΠΎΡ€Π° Π½Π° Π΄Π΅ΠΉΡΡ‚Π²ΠΈΡ‚Π΅Π»ΡŒΠ½ΠΎΠ΅ число каТдая Π΅Π³ΠΎ ΠΊΠΎΠΎΡ€Π΄ΠΈΠ½Π°Ρ‚Π° умноТаСтся Π½Π° это число.

5. ΠŸΡ€ΠΈ ΠΎΠΏΠ΅Ρ€Π°Ρ†ΠΈΠΈ слоТСния Π²Π΅ΠΊΡ‚ΠΎΡ€ΠΎΠ² вычисляСм сумму ΡΠΎΠΎΡ‚Π²Π΅Ρ‚ΡΡ‚Π²ΡƒΡŽΡ‰ΠΈΠ΅ ΠΊΠΎΠΎΡ€Π΄ΠΈΠ½Π°Ρ‚Ρ‹ Π²Π΅ΠΊΡ‚ΠΎΡ€ΠΎΠ² .

6. БкалярноС ΠΏΡ€ΠΎΠΈΠ·Π²Π΅Π΄Π΅Π½ΠΈΠ΅ Π΄Π²ΡƒΡ… Π²Π΅ΠΊΡ‚ΠΎΡ€ΠΎΠ² равняСтся суммС ΠΏΡ€ΠΎΠΈΠ·Π²Π΅Π΄Π΅Π½ΠΈΠΉ ΠΈΡ… ΡΠΎΠΎΡ‚Π²Π΅Ρ‚ΡΡ‚Π²ΡƒΡŽΡ‰ΠΈΡ… ΠΊΠΎΠΎΡ€Π΄ΠΈΠ½Π°Ρ‚.

Для Π½Π°Ρ‡Π°Π»Π° Π΄Π°Π΄ΠΈΠΌ ΠΎΠΏΡ€Π΅Π΄Π΅Π»Π΅Π½ΠΈΠ΅ ΠΊΠΎΠΎΡ€Π΄ΠΈΠ½Π°Ρ‚ Π²Π΅ΠΊΡ‚ΠΎΡ€Π° Π² Π·Π°Π΄Π°Π½Π½ΠΎΠΉ систСмС ΠΊΠΎΠΎΡ€Π΄ΠΈΠ½Π°Ρ‚. Π§Ρ‚ΠΎΠ±Ρ‹ ввСсти Π΄Π°Π½Π½ΠΎΠ΅ понятиС, ΠΎΠΏΡ€Π΅Π΄Π΅Π»ΠΈΠΌ Ρ‡Ρ‚ΠΎ ΠΌΡ‹ Π½Π°Π·Ρ‹Π²Π°Π΅ΠΌ ΠΏΡ€ΡΠΌΠΎΡƒΠ³ΠΎΠ»ΡŒΠ½ΠΎΠΉ ΠΈΠ»ΠΈ Π΄Π΅ΠΊΠ°Ρ€Ρ‚ΠΎΠ²ΠΎΠΉ систСмой ΠΊΠΎΠΎΡ€Π΄ΠΈΠ½Π°Ρ‚.

ΠžΠΏΡ€Π΅Π΄Π΅Π»Π΅Π½ΠΈΠ΅ 1

ΠŸΡ€ΡΠΌΠΎΡƒΠ³ΠΎΠ»ΡŒΠ½Π°Ρ систСма ΠΊΠΎΠΎΡ€Π΄ΠΈΠ½Π°Ρ‚ прСдставляСт ΠΈΠ· сСбя ΠΏΡ€ΡΠΌΠΎΠ»ΠΈΠ½Π΅ΠΉΠ½ΡƒΡŽ систСму ΠΊΠΎΠΎΡ€Π΄ΠΈΠ½Π°Ρ‚ с Π²Π·Π°ΠΈΠΌΠ½ΠΎ пСрпСндикулярными осями Π½Π° плоскости ΠΈΠ»ΠΈ Π² пространствС.

Π‘ ΠΏΠΎΠΌΠΎΡ‰ΡŒΡŽ ввСдСния ΠΏΡ€ΡΠΌΠΎΡƒΠ³ΠΎΠ»ΡŒΠ½ΠΎΠΉ систСмы ΠΊΠΎΠΎΡ€Π΄ΠΈΠ½Π°Ρ‚ Π½Π° плоскости ΠΈΠ»ΠΈ Π² Ρ‚Ρ€Π΅Ρ…ΠΌΠ΅Ρ€Π½ΠΎΠΌ пространствС становится Π²ΠΎΠ·ΠΌΠΎΠΆΠ½Ρ‹ΠΌ описываниС гСомСтричСских Ρ„ΠΈΠ³ΡƒΡ€ вмСстС с ΠΈΡ… свойствами ΠΏΡ€ΠΈ ΠΏΠΎΠΌΠΎΡ‰ΠΈ ΡƒΡ€Π°Π²Π½Π΅Π½ΠΈΠΉ ΠΈ нСравСнств, Ρ‚ΠΎ Π΅ΡΡ‚ΡŒ ΠΈΡΠΏΠΎΠ»ΡŒΠ·ΠΎΠ²Π°Ρ‚ΡŒ алгСбраичСскиС ΠΌΠ΅Ρ‚ΠΎΠ΄Ρ‹ ΠΏΡ€ΠΈ Ρ€Π΅ΡˆΠ΅Π½ΠΈΠΈ гСомСтричСских Π·Π°Π΄Π°Ρ‡.

Π’Π΅ΠΌ самым, ΠΌΡ‹ ΠΌΠΎΠΆΠ΅ΠΌ ΠΏΡ€ΠΈΠ²ΡΠ·Π°Ρ‚ΡŒ ΠΊ Π·Π°Π΄Π°Π½Π½ΠΎΠΉ систСмС ΠΊΠΎΠΎΡ€Π΄ΠΈΠ½Π°Ρ‚ Π²Π΅ΠΊΡ‚ΠΎΡ€Ρ‹. Π­Ρ‚ΠΎ Π·Π½Π°Ρ‡ΠΈΡ‚Π΅Π»ΡŒΠ½ΠΎ Ρ€Π°ΡΡˆΠΈΡ€ΠΈΡ‚ наши возмоТности ΠΏΡ€ΠΈ Ρ€Π΅ΡˆΠ΅Π½ΠΈΠΈ ΠΎΠΏΡ€Π΅Π΄Π΅Π»Π΅Π½Π½Ρ‹Ρ… Π·Π°Π΄Π°Ρ‡

ΠŸΡ€ΡΠΌΠΎΡƒΠ³ΠΎΠ»ΡŒΠ½Π°Ρ систСма ΠΊΠΎΠΎΡ€Π΄ΠΈΠ½Π°Ρ‚ Π½Π° плоскости ΠΎΠ±Ρ‹Ρ‡Π½ΠΎ обозначаСтся O x y , Π³Π΄Π΅ O x ΠΈ O y – оси ΠΊΠΎΠΎΡ€Π΄Π½Π°Ρ‚. Ось O x Π½Π°Π·Ρ‹Π²Π°ΡŽΡ‚ осью абсцисс, Π° ось O y – осью ΠΎΡ€Π΄ΠΈΠ½Π°Ρ‚ (Π² пространствС появляСтся Π΅Ρ‰Ρ‘ ΠΎΠ΄Π½Π° ось O z , которая пСрпСндикулярна ΠΈ O x ΠΈ O y).

ΠŸΡ€ΠΈΠΌΠ΅Ρ€ 1

Π˜Ρ‚Π°ΠΊ, Π½Π°ΠΌ Π΄Π°Π½Π° ΠΏΡ€ΡΠΌΠΎΡƒΠ³ΠΎΠ»ΡŒΠ½Π°Ρ Π΄Π΅ΠΊΠ°Ρ€Ρ‚ΠΎΠ²Π° систСма ΠΊΠΎΠΎΡ€Π΄ΠΈΠ½Π°Ρ‚ O x y Π½Π° плоскости Ссли ΠΌΡ‹ ΠΎΡ‚Π»ΠΎΠΆΠΈΠΌ ΠΎΡ‚ Π½Π°Ρ‡Π°Π»Π° ΠΊΠΎΠΎΡ€Π΄ΠΈΠ½Π°Ρ‚ Π²Π΅ΠΊΡ‚ΠΎΡ€Ρ‹ i β†’ ΠΈ j β†’ , Π½Π°ΠΏΡ€Π°Π²Π»Π΅Π½ΠΈΠ΅ ΠΊΠΎΡ‚ΠΎΡ€Ρ‹Ρ… соотвСтствСнно совпадСт с ΠΏΠΎΠ»ΠΎΠΆΠΈΡ‚Π΅Π»ΡŒΠ½Ρ‹ΠΌΠΈ направлСниями осСй O x ΠΈ O y , ΠΈ ΠΈΡ… Π΄Π»ΠΈΠ½Π° Π±ΡƒΠ΄Π΅Ρ‚ Ρ€Π°Π²Π½Π° условной Π΅Π΄ΠΈΠ½ΠΈΡ†Π΅, ΠΌΡ‹ ΠΏΠΎΠ»ΡƒΡ‡ΠΈΠΌ ΠΊΠΎΠΎΡ€Π΄ΠΈΠ½Π°Ρ‚Π½Ρ‹Π΅ Π²Π΅ΠΊΡ‚ΠΎΡ€Ρ‹. Π’ΠΎ Π΅ΡΡ‚ΡŒ Π² Π΄Π°Π½Π½ΠΎΠΌ случаС i β†’ ΠΈ j β†’ ΡΠ²Π»ΡΡŽΡ‚ΡΡ ΠΊΠΎΠΎΡ€Π΄ΠΈΠ½Π°Ρ‚Π½Ρ‹ΠΌΠΈ Π²Π΅ΠΊΡ‚ΠΎΡ€Π°ΠΌΠΈ.

ΠšΠΎΠΎΡ€Π΄ΠΈΠ½Π°Ρ‚Π½Ρ‹Π΅ Π²Π΅ΠΊΡ‚ΠΎΡ€Ρ‹

ΠžΠΏΡ€Π΅Π΄Π΅Π»Π΅Π½ΠΈΠ΅ 2

Π’Π΅ΠΊΡ‚ΠΎΡ€Ρ‹ i β†’ ΠΈ j β†’ Π½Π°Π·Ρ‹Π²Π°ΡŽΡ‚ΡΡ ΠΊΠΎΠΎΡ€Π΄ΠΈΠ½Π°Ρ‚Π½Ρ‹ΠΌΠΈ Π²Π΅ΠΊΡ‚ΠΎΡ€Π°ΠΌΠΈ для Π·Π°Π΄Π°Π½Π½ΠΎΠΉ систСмы ΠΊΠΎΠΎΡ€Π΄ΠΈΠ½Π°Ρ‚.

ΠŸΡ€ΠΈΠΌΠ΅Ρ€ 2

ΠžΡ‚ΠΊΠ»Π°Π΄Ρ‹Π²Π°Π΅ΠΌ ΠΎΡ‚ Π½Π°Ρ‡Π°Π»Π° ΠΊΠΎΠΎΡ€Π΄ΠΈΠ½Π°Ρ‚ ΠΏΡ€ΠΎΠΈΠ·Π²ΠΎΠ»ΡŒΠ½Ρ‹ΠΉ Π²Π΅ΠΊΡ‚ΠΎΡ€ a β†’ . ΠžΠΏΠΈΡ€Π°ΡΡΡŒ Π½Π° гСомСтричСскоС ΠΎΠΏΡ€Π΅Π΄Π΅Π»Π΅Π½ΠΈΠ΅ ΠΎΠΏΠ΅Ρ€Π°Ρ†ΠΈΠΉ Π½Π°Π΄ Π²Π΅ΠΊΡ‚ΠΎΡ€Π°ΠΌΠΈ, Π²Π΅ΠΊΡ‚ΠΎΡ€ a β†’ ΠΌΠΎΠΆΠ΅Ρ‚ Π±Ρ‹Ρ‚ΡŒ прСдставлСн Π² Π²ΠΈΠ΄Π΅ a β†’ = a x Β· i β†’ + a y Β· j β†’ , Π³Π΄Π΅ коэффициСнты a x ΠΈ a y — СдинствСнныС Π² своСм Ρ€ΠΎΠ΄Π΅, ΠΈΡ… Π΅Π΄ΠΈΠ½ΡΡ‚Π²Π΅Π½Π½ΠΎΡΡ‚ΡŒ достаточно просто Π΄ΠΎΠΊΠ°Π·Π°Ρ‚ΡŒ ΠΌΠ΅Ρ‚ΠΎΠ΄ΠΎΠΌ ΠΎΡ‚ ΠΏΡ€ΠΎΡ‚ΠΈΠ²Π½ΠΎΠ³ΠΎ.

Π Π°Π·Π»ΠΎΠΆΠ΅Π½ΠΈΠ΅ Π²Π΅ΠΊΡ‚ΠΎΡ€Π°

ΠžΠΏΡ€Π΅Π΄Π΅Π»Π΅Π½ΠΈΠ΅ 3

Π Π°Π·Π»ΠΎΠΆΠ΅Π½ΠΈΠ΅ΠΌ Π²Π΅ΠΊΡ‚ΠΎΡ€Π° a β†’ ΠΏΠΎ ΠΊΠΎΠΎΡ€Π΄ΠΈΠ½Π°Ρ‚Π½Ρ‹ΠΌ Π²Π΅ΠΊΡ‚ΠΎΡ€Π°ΠΌ i β†’ ΠΈ j β†’ Π½Π° плоскости называСтся прСдставлСниС Π²ΠΈΠ΄Π° a β†’ = a x Β· i β†’ + a y Β· j β†’ .

ΠžΠΏΡ€Π΅Π΄Π΅Π»Π΅Π½ΠΈΠ΅ 4

ΠšΠΎΡΡ„Ρ„ΠΈΡ†ΠΈΠ΅Π½Ρ‚Ρ‹ a x ΠΈ a y Π½Π°Π·Ρ‹Π²Π°ΡŽΡ‚ΡΡ ΠΊΠΎΠΎΡ€Π΄ΠΈΠ½Π°Ρ‚Π°ΠΌΠΈ Π²Π΅ΠΊΡ‚ΠΎΡ€Π° Π² Π΄Π°Π½Π½ΠΎΠΉ систСмС ΠΊΠΎΠΎΡ€Π΄ΠΈΠ½Π°Ρ‚ Π½Π° плоскости.

ΠšΠΎΠΎΡ€Π΄ΠΈΠ½Π°Ρ‚Ρ‹ Π²Π΅ΠΊΡ‚ΠΎΡ€Π° Π² Π΄Π°Π½Π½ΠΎΠΉ систСмС ΠΊΠΎΠΎΡ€Π΄ΠΈΠ½Π°Ρ‚ принято Π·Π°ΠΏΠΈΡΡ‹Π²Π°Ρ‚ΡŒ Π² ΠΊΡ€ΡƒΠ³Π»Ρ‹Ρ… скобках, Ρ‡Π΅Ρ€Π΅Π· Π·Π°ΠΏΡΡ‚ΡƒΡŽ, ΠΏΡ€ΠΈ этом Π·Π°Π΄Π°Π½Π½Ρ‹Π΅ ΠΊΠΎΠΎΡ€Π΄ΠΈΠ½Π°Ρ‚Ρ‹ слСдуСт ΠΎΡ‚Π΄Π΅Π»ΡΡ‚ΡŒ ΠΎΡ‚ обозначСния Π²Π΅ΠΊΡ‚ΠΎΡ€Π° Π·Π½Π°ΠΊΠΎΠΌ равСнства. К ΠΏΡ€ΠΈΠΌΠ΅Ρ€Ρƒ, запись a β†’ = (2 ; — 3) ΠΎΠ·Π½Π°Ρ‡Π°Π΅Ρ‚, Ρ‡Ρ‚ΠΎ Π²Π΅ΠΊΡ‚ΠΎΡ€ a β†’ ΠΈΠΌΠ΅Π΅Ρ‚ ΠΊΠΎΠΎΡ€Π΄ΠΈΠ½Π°Ρ‚Ρ‹ (2 ; — 3) Π² Π΄Π°Π½Π½ΠΎΠΉ систСмС ΠΊΠΎΠΎΡ€Π΄ΠΈΠ½Π°Ρ‚ ΠΈ ΠΌΠΎΠΆΠ΅Ρ‚ Π±Ρ‹Ρ‚ΡŒ прСдставлСн Π² Π²ΠΈΠ΄Π΅ разлоТСния ΠΏΠΎ ΠΊΠΎΠΎΡ€Π΄ΠΈΠ½Π°Ρ‚Π½Ρ‹ΠΌ Π²Π΅ΠΊΡ‚ΠΎΡ€Π°ΠΌ i β†’ ΠΈ j β†’ ΠΊΠ°ΠΊ a β†’ = 2 Β· i β†’ — 3 Β· j β†’ .

Π—Π°ΠΌΠ΅Ρ‡Π°Π½ΠΈΠ΅

Π‘Π»Π΅Π΄ΡƒΠ΅Ρ‚ ΠΎΠ±Ρ€Π°Ρ‚ΠΈΡ‚ΡŒ Π²Π½ΠΈΠΌΠ°Π½ΠΈΠ΅, Ρ‡Ρ‚ΠΎ порядок записи ΠΊΠΎΠΎΡ€Π΄ΠΈΠ½Π°Ρ‚, ΠΈΠΌΠ΅Π΅Ρ‚ Π²Π°ΠΆΠ½ΠΎΠ΅ Π·Π½Π°Ρ‡Π΅Π½ΠΈΠ΅, Ссли Π²Ρ‹ Π·Π°ΠΏΠΈΡˆΠΈΡ‚Π΅ ΠΊΠΎΠΎΡ€Π΄ΠΈΠ½Π°Ρ‚Ρ‹ Π²Π΅ΠΊΡ‚ΠΎΡ€Π° Π² Π΄Ρ€ΡƒΠ³ΠΎΠΌ порядкС, Π²Ρ‹ ΠΏΠΎΠ»ΡƒΡ‡ΠΈΡ‚Π΅ ΡΠΎΠ²Π΅Ρ€ΡˆΠ΅Π½Π½ΠΎ Π΄Ρ€ΡƒΠ³ΠΎΠΉ Π²Π΅ΠΊΡ‚ΠΎΡ€.

ΠžΠΏΠΈΡ€Π°ΡΡΡŒ Π½Π° опрСдСлСния ΠΊΠΎΠΎΡ€Π΄ΠΈΠ½Π°Ρ‚ Π²Π΅ΠΊΡ‚ΠΎΡ€Π° ΠΈ ΠΈΡ… разлоТСния становится ΠΎΡ‡Π΅Π²ΠΈΠ΄Π½Ρ‹ΠΌ, Ρ‡Ρ‚ΠΎ Π΅Π΄ΠΈΠ½ΠΈΡ‡Π½Ρ‹Π΅ Π²Π΅ΠΊΡ‚ΠΎΡ€Ρ‹ i β†’ ΠΈ j β†’ ΠΈΠΌΠ΅ΡŽΡ‚ ΠΊΠΎΠΎΡ€Π΄ΠΈΠ½Π°Ρ‚Ρ‹ (1 ; 0) ΠΈ (0 ; 1) соотвСтствСнно, ΠΈ ΠΎΠ½ΠΈ ΠΌΠΎΠ³ΡƒΡ‚ Π±Ρ‹Ρ‚ΡŒ прСдставлСны Π² Π²ΠΈΠ΄Π΅ ΡΠ»Π΅Π΄ΡƒΡŽΡ‰ΠΈΡ… Ρ€Π°Π·Π»ΠΎΠΆΠ΅Π½ΠΈΠΉ i β†’ = 1 Β· i β†’ + 0 Β· j β†’ ; j β†’ = 0 Β· i β†’ + 1 Β· j β†’ .

Π’Π°ΠΊΠΆΠ΅ ΠΈΠΌΠ΅Π΅Ρ‚ мСсто Π±Ρ‹Ρ‚ΡŒ Π½ΡƒΠ»Π΅Π²ΠΎΠΉ Π²Π΅ΠΊΡ‚ΠΎΡ€ 0 β†’ с ΠΊΠΎΠΎΡ€Π΄ΠΈΠ½Π°Ρ‚Π°ΠΌΠΈ (0 ; 0) ΠΈ Ρ€Π°Π·Π»ΠΎΠΆΠ΅Π½ΠΈΠ΅ΠΌ 0 β†’ = 0 Β· i β†’ + 0 Β· j β†’ .

Π Π°Π²Π½Ρ‹Π΅ ΠΈ ΠΏΡ€ΠΎΡ‚ΠΈΠ²ΠΎΠΏΠΎΠ»ΠΎΠΆΠ½Ρ‹Π΅ Π²Π΅ΠΊΡ‚ΠΎΡ€Ρ‹

ΠžΠΏΡ€Π΅Π΄Π΅Π»Π΅Π½ΠΈΠ΅ 5

Π’Π΅ΠΊΡ‚ΠΎΡ€Ρ‹ a β†’ ΠΈ b β†’ Ρ€Π°Π²Π½Ρ‹ Ρ‚ΠΎΠ³Π΄Π°, ΠΊΠΎΠ³Π΄Π° ΠΈΡ… ΡΠΎΠΎΡ‚Π²Π΅Ρ‚ΡΡ‚Π²ΡƒΡŽΡ‰ΠΈΠ΅ ΠΊΠΎΠΎΡ€Π΄ΠΈΠ½Π°Ρ‚Ρ‹ Ρ€Π°Π²Π½Ρ‹.

ΠžΠΏΡ€Π΅Π΄Π΅Π»Π΅Π½ΠΈΠ΅ 6

ΠŸΡ€ΠΎΡ‚ΠΈΠ²ΠΎΠΏΠΎΠ»ΠΎΠΆΠ½Ρ‹ΠΌ Π²Π΅ΠΊΡ‚ΠΎΡ€ΠΎΠΌ называСтся Π²Π΅ΠΊΡ‚ΠΎΡ€ ΠΏΡ€ΠΎΡ‚ΠΈΠ²ΠΎΠΏΠΎΠ»ΠΎΠΆΠ½Ρ‹ΠΉ Π΄Π°Π½Π½ΠΎΠΌΡƒ.

ΠžΡ‚ΡΡŽΠ΄Π° слСдуСт, Ρ‡Ρ‚ΠΎ ΠΊΠΎΠΎΡ€Π΄ΠΈΠ½Π°Ρ‚Ρ‹ Ρ‚Π°ΠΊΠΎΠ³ΠΎ Π²Π΅ΠΊΡ‚ΠΎΡ€Π° Π±ΡƒΠ΄ΡƒΡ‚ ΠΏΡ€ΠΎΡ‚ΠΈΠ²ΠΎΠΏΠΎΠ»ΠΎΠΆΠ½Ρ‹ ΠΊΠΎΠΎΡ€Π΄ΠΈΠ½Π°Ρ‚Π°ΠΌ Π΄Π°Π½Π½ΠΎΠ³ΠΎ Π²Π΅ΠΊΡ‚ΠΎΡ€Π°, Ρ‚ΠΎ Π΅ΡΡ‚ΡŒ, — a β†’ = (- a x ; — a y) .

ВсС Π²Ρ‹ΡˆΠ΅ΠΈΠ·Π»ΠΎΠΆΠ΅Π½Π½ΠΎΠ΅ ΠΌΠΎΠΆΠ½ΠΎ Π°Π½Π°Π»ΠΎΠ³ΠΈΡ‡Π½ΠΎ ΠΎΠΏΡ€Π΅Π΄Π΅Π»ΠΈΡ‚ΡŒ ΠΈ для ΠΏΡ€ΡΠΌΠΎΡƒΠ³ΠΎΠ»ΡŒΠ½ΠΎΠΉ систСмы ΠΊΠΎΠΎΡ€Π΄ΠΈΠ½Π°Ρ‚, Π·Π°Π΄Π°Π½Π½ΠΎΠΉ Π² Ρ‚Ρ€Π΅Ρ…ΠΌΠ΅Ρ€Π½ΠΎΠΌ пространствС. Π’ Ρ‚Π°ΠΊΠΎΠΉ систСмС ΠΊΠΎΠΎΡ€Π΄ΠΈΠ½Π°Ρ‚ ΠΈΠΌΠ΅Π΅Ρ‚ мСсто Π±Ρ‹Ρ‚ΡŒ Ρ‚Ρ€ΠΎΠΉΠΊΠ° ΠΊΠΎΠΎΡ€Π΄ΠΈΠ½Π°Ρ‚Π½Ρ‹Ρ… Π²Π΅ΠΊΡ‚ΠΎΡ€ΠΎΠ² i β†’ , j β†’ , k β†’ , Π° ΠΏΡ€ΠΎΠΈΠ·Π²ΠΎΠ»ΡŒΠ½Ρ‹ΠΉ Π²Π΅ΠΊΡ‚ΠΎΡ€ a β†’ раскладываСтся Π½Π΅ ΠΏΠΎ Π΄Π²ΡƒΠΌ, Π° ΡƒΠΆΠ΅ ΠΏΠΎ Ρ‚Ρ€Π΅ΠΌ ΠΊΠΎΠΎΡ€Π΄ΠΈΠ½Π°Ρ‚Π°ΠΌ, ΠΏΡ€ΠΈΡ‡Π΅ΠΌ СдинствСнным ΠΎΠ±Ρ€Π°Π·ΠΎΠΌ ΠΈ ΠΈΠΌΠ΅Π΅Ρ‚ Π²ΠΈΠ΄ a β†’ = a x Β· i β†’ + a y Β· j β†’ + a z Β· k β†’ , Π° коэффициСнты этого разлоТСния (a x ; a y ; a z) Π½Π°Π·Ρ‹Π²Π°ΡŽΡ‚ΡΡ ΠΊΠΎΠΎΡ€Π΄ΠΈΠ½Π°Ρ‚Π°ΠΌΠΈ Π²Π΅ΠΊΡ‚ΠΎΡ€Π° Π² Π΄Π°Π½Π½ΠΎΠΉ (Ρ‚Ρ€Π΅Ρ…ΠΌΠ΅Ρ€Π½ΠΎΠΉ) систСмС ΠΊΠΎΠΎΡ€Π΄ΠΈΠ½Π°Ρ‚.

Π‘Π»Π΅Π΄ΠΎΠ²Π°Ρ‚Π΅Π»ΡŒΠ½ΠΎ, ΠΊΠΎΠΎΡ€Π΄ΠΈΠ½Π°Ρ‚Π½Ρ‹Π΅ Π²Π΅ΠΊΡ‚ΠΎΡ€Ρ‹ Π² Ρ‚Ρ€Π΅Ρ…ΠΌΠ΅Ρ€Π½ΠΎΠΌ пространствС ΠΏΡ€ΠΈΠ½ΠΈΠΌΠ°ΡŽΡ‚ Ρ‚Π°ΠΊΠΆΠ΅ Π·Π½Π°Ρ‡Π΅Π½ΠΈΠ΅ 1 ΠΈ ΠΈΠΌΠ΅ΡŽΡ‚ ΠΊΠΎΠΎΡ€Π΄ΠΈΠ½Π°Ρ‚Ρ‹ i β†’ = (1 ; 0 ; 0) , j β†’ = (0 ; 1 ; 0) , k β†’ = (0 ; 0 ; 1) , ΠΊΠΎΠΎΡ€Π΄ΠΈΠ½Π°Ρ‚Ρ‹ Π½ΡƒΠ»Π΅Π²ΠΎΠ³ΠΎ Π²Π΅ΠΊΡ‚ΠΎΡ€Π° Ρ‚Π°ΠΊΠΆΠ΅ Ρ€Π°Π²Π½Ρ‹ Π½ΡƒΠ»ΡŽ 0 β†’ = (0 ; 0 ; 0) , ΠΈ Π² Ρ‚Π°ΠΊΠΎΠΌ случаС Π΄Π²Π° Π²Π΅ΠΊΡ‚ΠΎΡ€Π° Π±ΡƒΠ΄ΡƒΡ‚ ΡΡ‡ΠΈΡ‚Π°Ρ‚ΡŒΡΡ Ρ€Π°Π²Π½Ρ‹ΠΌΠΈ, Ссли всС Ρ‚Ρ€ΠΈ ΡΠΎΠΎΡ‚Π²Π΅Ρ‚ΡΡ‚Π²ΡƒΡŽΡ‰ΠΈΠ΅ ΠΊΠΎΠΎΡ€Π΄ΠΈΠ½Π°Ρ‚Ρ‹ Π²Π΅ΠΊΡ‚ΠΎΡ€ΠΎΠ² ΠΌΠ΅ΠΆΠ΄Ρƒ собой Ρ€Π°Π²Π½Ρ‹ a β†’ = b β†’ ⇔ a x = b x , a y = b y , a z = b z , ΠΈ ΠΊΠΎΠΎΡ€Π΄ΠΈΠ½Π°Ρ‚Ρ‹ ΠΏΡ€ΠΎΡ‚ΠΈΠ²ΠΎΠΏΠΎΠ»ΠΎΠΆΠ½ΠΎΠ³ΠΎ Π²Π΅ΠΊΡ‚ΠΎΡ€Π° a β†’ ΠΏΡ€ΠΎΡ‚ΠΈΠ²ΠΎΠΏΠΎΠ»ΠΎΠΆΠ½Ρ‹ ΡΠΎΠΎΡ‚Π²Π΅Ρ‚ΡΡ‚Π²ΡƒΡŽΡ‰ΠΈΠΌ ΠΊΠΎΠΎΡ€Π΄ΠΈΠ½Π°Ρ‚Π°ΠΌ Π²Π΅ΠΊΡ‚ΠΎΡ€Π° a β†’ , Ρ‚ΠΎ Π΅ΡΡ‚ΡŒ, — a β†’ = (- a x ; — a y ; — a z) .

Π§Ρ‚ΠΎΠ±Ρ‹ ввСсти Π΄Π°Π½Π½ΠΎΠ΅ ΠΎΠΏΡ€Π΅Π΄Π΅Π»Π΅Π½ΠΈΠ΅, трСбуСтся ΠΏΠΎΠΊΠ°Π·Π°Ρ‚ΡŒ Π² Π΄Π°Π½Π½ΠΎΠΉ систСмС ΠΊΠΎΠΎΡ€Π΄ΠΈΠ½Π°Ρ‚ связь ΠΊΠΎΠΎΡ€Π΄ΠΈΠ½Π°Ρ‚ Ρ‚ΠΎΡ‡ΠΊΠΈ ΠΈ ΠΊΠΎΠΎΡ€Π΄ΠΈΠ½Π°Ρ‚ Π²Π΅ΠΊΡ‚ΠΎΡ€Π°.

ΠŸΡƒΡΡ‚ΡŒ Π½Π°ΠΌ Π΄Π°Π½Π° нСкоторая ΠΏΡ€ΡΠΌΠΎΡƒΠ³ΠΎΠ»ΡŒΠ½Π°Ρ Π΄Π΅ΠΊΠ°Ρ€Ρ‚ΠΎΠ²Π° систСма ΠΊΠΎΠΎΡ€Π΄ΠΈΠ½Π°Ρ‚ O x y ΠΈ Π½Π° Π½Π΅ΠΉ Π·Π°Π΄Π°Π½Π° ΠΏΡ€ΠΎΠΈΠ·Π²ΠΎΠ»ΡŒΠ½Π°Ρ Ρ‚ΠΎΡ‡ΠΊΠ° M с ΠΊΠΎΠΎΡ€Π΄ΠΈΠ½Π°Ρ‚Π°ΠΌΠΈ M (x M ; y M) .

ΠžΠΏΡ€Π΅Π΄Π΅Π»Π΅Π½ΠΈΠ΅ 7

Π’Π΅ΠΊΡ‚ΠΎΡ€ O M β†’ называСтся радиус-Π²Π΅ΠΊΡ‚ΠΎΡ€ΠΎΠΌ Ρ‚ΠΎΡ‡ΠΊΠΈ M .

ΠžΠΏΡ€Π΅Π΄Π΅Π»ΠΈΠΌ, ΠΊΠ°ΠΊΠΈΠ΅ ΠΊΠΎΠΎΡ€Π΄ΠΈΠ½Π°Ρ‚Ρ‹ Π² Π΄Π°Π½Π½ΠΎΠΉ систСмС ΠΊΠΎΠΎΡ€Π΄ΠΈΠ½Π°Ρ‚ ΠΈΠΌΠ΅Π΅Ρ‚ радиус-Π²Π΅ΠΊΡ‚ΠΎΡ€ Ρ‚ΠΎΡ‡ΠΊΠΈ

Π’Π΅ΠΊΡ‚ΠΎΡ€ O M β†’ ΠΈΠΌΠ΅Π΅Ρ‚ Π²ΠΈΠ΄ суммы O M β†’ = O M x β†’ + O M y β†’ = x M Β· i β†’ + y M Β· j β†’ , Π³Π΄Π΅ Ρ‚ΠΎΡ‡ΠΊΠΈ M x ΠΈ M y это ΠΏΡ€ΠΎΠ΅ΠΊΡ†ΠΈΠΈ Ρ‚ΠΎΡ‡ΠΊΠΈ М Π½Π° ΠΊΠΎΠΎΡ€Π΄ΠΈΠ½Π°Ρ‚Π½Ρ‹Π΅ прямыС Ox ΠΈ Oy соотвСтствСнно (Π΄Π°Π½Π½Ρ‹Π΅ рассуТдСния ΡΠ»Π΅Π΄ΡƒΡŽΡ‚ ΠΈΠ· опрСдСлСния проСкция Ρ‚ΠΎΡ‡ΠΊΠΈ Π½Π° ΠΏΡ€ΡΠΌΡƒΡŽ), Π° i β†’ ΠΈ j β†’ — ΠΊΠΎΠΎΡ€Π΄ΠΈΠ½Π°Ρ‚Π½Ρ‹Π΅ Π²Π΅ΠΊΡ‚ΠΎΡ€Ρ‹, ΡΠ»Π΅Π΄ΠΎΠ²Π°Ρ‚Π΅Π»ΡŒΠ½ΠΎ, Π²Π΅ΠΊΡ‚ΠΎΡ€ O M β†’ ΠΈΠΌΠ΅Π΅Ρ‚ ΠΊΠΎΠΎΡ€Π΄ΠΈΠ½Π°Ρ‚Ρ‹ (x M ; y M) Π² Π΄Π°Π½Π½ΠΎΠΉ систСмС ΠΊΠΎΠΎΡ€Π΄ΠΈΠ½Π°Ρ‚.

Π˜Π½Π°Ρ‡Π΅ говоря, ΠΊΠΎΠΎΡ€Π΄ΠΈΠ½Π°Ρ‚Ρ‹ радиус-Π²Π΅ΠΊΡ‚ΠΎΡ€Π° Ρ‚ΠΎΡ‡ΠΊΠΈ М Ρ€Π°Π²Π½Ρ‹ ΡΠΎΠΎΡ‚Π²Π΅Ρ‚ΡΡ‚Π²ΡƒΡŽΡ‰ΠΈΠΌ ΠΊΠΎΠΎΡ€Π΄ΠΈΠ½Π°Ρ‚Π°ΠΌ Ρ‚ΠΎΡ‡ΠΊΠΈ М Π² ΠΏΡ€ΡΠΌΠΎΡƒΠ³ΠΎΠ»ΡŒΠ½ΠΎΠΉ Π΄Π΅ΠΊΠ°Ρ€Ρ‚ΠΎΠ²ΠΎΠΉ систСмС ΠΊΠΎΠΎΡ€Π΄ΠΈΠ½Π°Ρ‚.

Аналогично Π² Ρ‚Ρ€Π΅Ρ…ΠΌΠ΅Ρ€Π½ΠΎΠΌ пространствС радиус-Π²Π΅ΠΊΡ‚ΠΎΡ€ Ρ‚ΠΎΡ‡ΠΊΠΈ M (x M ; y M ; z M) разлагаСтся ΠΏΠΎ ΠΊΠΎΠΎΡ€Π΄ΠΈΠ½Π°Ρ‚Π½Ρ‹ΠΌ Π²Π΅ΠΊΡ‚ΠΎΡ€Π°ΠΌ ΠΊΠ°ΠΊ O M β†’ = O M x β†’ + O M y β†’ + O M z β†’ = x M Β· i β†’ + y M Β· j β†’ + z M Β· k β†’ , ΡΠ»Π΅Π΄ΠΎΠ²Π°Ρ‚Π΅Π»ΡŒΠ½ΠΎ, O M β†’ = (x M ; y M ; z M) .

Если Π²Ρ‹ Π·Π°ΠΌΠ΅Ρ‚ΠΈΠ»ΠΈ ΠΎΡˆΠΈΠ±ΠΊΡƒ Π² тСкстС, поТалуйста, Π²Ρ‹Π΄Π΅Π»ΠΈΡ‚Π΅ Π΅Ρ‘ ΠΈ Π½Π°ΠΆΠΌΠΈΡ‚Π΅ Ctrl+Enter

Π”ΠΎ сих ΠΏΠΎΡ€ ΡΡ‡ΠΈΡ‚Π°Π»ΠΎΡΡŒ, Ρ‡Ρ‚ΠΎ Π²Π΅ΠΊΡ‚ΠΎΡ€Ρ‹ Ρ€Π°ΡΡΠΌΠ°Ρ‚Ρ€ΠΈΠ²Π°ΡŽΡ‚ΡΡ Π² пространствС. Начиная с этого ΠΌΠΎΠΌΠ΅Π½Ρ‚Π° Π±ΡƒΠ΄ΠΈΠΌ ΡΡ‡ΠΈΡ‚Π°Ρ‚ΡŒ, Ρ‡Ρ‚ΠΎ всС Π²Π΅ΠΊΡ‚ΠΎΡ€Ρ‹ Ρ€Π°ΡΡΠΌΠ°Ρ‚Ρ€ΠΈΠ²Π°ΡŽΡ‚ΡΡ Π½Π° плоскости. Π‘ΡƒΠ΄Π΅ΠΌ Ρ‚Π°ΠΊΠΆΠ΅ ΠΏΠΎΠ»Π°Π³Π°Ρ‚ΡŒ, Ρ‡Ρ‚ΠΎ Π½Π° плоскости Π·Π°Π΄Π°Π½Π° Π”Π΅ΠΊΠ°Ρ€Ρ‚ΠΎΠ²Π° систСма ΠΊΠΎΠΎΡ€Π΄ΠΈΠ½Π°Ρ‚ (Π΄Π°ΠΆΠ΅ Ссли ΠΎΠ± этом Π½Π΅ говорится), ΠΏΡ€Π΅Π΄ΡΡ‚Π°Π²Π»ΡΡŽΡ‰Π°Ρ Π΄Π²Π΅ Π²Π·Π°ΠΈΠΌΠ½ΠΎ пСрпСндикулярныС числовыС оси – Π³ΠΎΡ€ΠΈΠ·ΠΎΠ½Ρ‚Π°Π»ΡŒΠ½Π°Ρ ось ΠΈ Π²Π΅Ρ€Ρ‚ΠΈΠΊΠ°Π»ΡŒΠ½Π°Ρ ось. Π’ΠΎΠ³Π΄Π° ΠΊΠ°ΠΆΠ΄ΠΎΠΉ Ρ‚ΠΎΡ‡ΠΊΠ΅
Π½Π° плоскости ставится Π² соотвСтствиС ΠΏΠ°Ρ€Π° чисСл
, ΠΊΠΎΡ‚ΠΎΡ€Ρ‹Π΅ ΡΠ²Π»ΡΡŽΡ‚ΡΡ Π΅Π΅ ΠΊΠΎΠΎΡ€Π΄ΠΈΠ½Π°Ρ‚Π°ΠΌΠΈ. ΠžΠ±Ρ€Π°Ρ‚Π½ΠΎ, ΠΊΠ°ΠΆΠ΄ΠΎΠΉ ΠΏΠ°Ρ€Π΅ чисСл
соотвСтствуСт Ρ‚ΠΎΡ‡ΠΊΠ° плоскости такая, Ρ‡Ρ‚ΠΎ ΠΏΠ°Ρ€Π° чисСл
ΡΠ²Π»ΡΡŽΡ‚ΡΡ Π΅Π΅ ΠΊΠΎΠΎΡ€Π΄ΠΈΠ½Π°Ρ‚Π°ΠΌΠΈ.

Из элСмСнтарной Π³Π΅ΠΎΠΌΠ΅Ρ‚Ρ€ΠΈΠΈ извСстно, Ρ‡Ρ‚ΠΎ Ссли Π½Π° плоскости ΠΈΠΌΠ΅ΡŽΡ‚ΡΡ Π΄Π²Π΅ Ρ‚ΠΎΡ‡ΠΊΠΈ
ΠΈ
, Ρ‚ΠΎ расстояниС
ΠΌΠ΅ΠΆΠ΄Ρƒ этими Ρ‚ΠΎΡ‡ΠΊΠ°ΠΌΠΈ выраТаСтся Ρ‡Π΅Ρ€Π΅Π· ΠΈΡ… ΠΊΠΎΠΎΡ€Π΄ΠΈΠ½Π°Ρ‚Ρ‹ ΠΏΠΎ Ρ„ΠΎΡ€ΠΌΡƒΠ»Π΅

ΠŸΡƒΡΡ‚ΡŒ Π½Π° плоскости Π·Π°Π΄Π°Π½Π° Π”Π΅ΠΊΠ°Ρ€Ρ‚ΠΎΠ²Π° систСма ΠΊΠΎΠΎΡ€Π΄ΠΈΠ½Π°Ρ‚. ΠžΡ€Ρ‚ оси Π±ΡƒΠ΄Π΅ΠΌ ΠΎΠ±ΠΎΠ·Π½Π°Ρ‡Π°Ρ‚ΡŒ символом, Π° ΠΎΡ€Ρ‚ осисимволом. ΠŸΡ€ΠΎΠ΅ΠΊΡ†ΠΈΡŽ ΠΏΡ€ΠΎΠΈΠ·Π²ΠΎΠ»ΡŒΠ½ΠΎΠ³ΠΎΠ²Π΅ΠΊΡ‚ΠΎΡ€Π°Π½Π° ΠΎΡΡŒΠ±ΡƒΠ΄Π΅ΠΌ ΠΎΠ±ΠΎΠ·Π½Π°Ρ‡Π°Ρ‚ΡŒ символом
, Π° ΠΏΡ€ΠΎΠ΅ΠΊΡ†ΠΈΡŽ Π½Π° осьсимволом
.

ΠŸΡƒΡΡ‚ΡŒ — ΠΏΡ€ΠΎΠΈΠ·Π²ΠΎΠ»ΡŒΠ½Ρ‹ΠΉ Π²Π΅ΠΊΡ‚ΠΎΡ€ Π½Π° плоскости. Π˜ΠΌΠ΅Π΅Ρ‚ мСсто ΡΠ»Π΅Π΄ΡƒΡŽΡ‰Π°Ρ Ρ‚Π΅ΠΎΡ€Π΅ΠΌΠ°.

Π’Π΅ΠΎΡ€Π΅ΠΌΠ° 22.

Для любого Π²Π΅ΠΊΡ‚ΠΎΡ€Π° Π½Π° плоскости сущСствуСт ΠΏΠ°Ρ€Π° чисСл

.

ΠŸΡ€ΠΈ этом
,
.

Π”ΠΎΠΊΠ°Π·Π°Ρ‚Π΅Π»ΡŒΡΡ‚Π²ΠΎ.

ΠŸΡƒΡΡ‚ΡŒ Π΄Π°Π½ Π²Π΅ΠΊΡ‚ΠΎΡ€. ΠžΡ‚Π»ΠΎΠΆΠΈΠΌ Π²Π΅ΠΊΡ‚ΠΎΡ€ΠΎΡ‚ Π½Π°Ρ‡Π°Π»Π° ΠΊΠΎΠΎΡ€Π΄ΠΈΠ½Π°Ρ‚. ΠžΠ±ΠΎΠ·Π½Π°Ρ‡ΠΈΠΌ Ρ‡Π΅Ρ€Π΅Π·Π²Π΅ΠΊΡ‚ΠΎΡ€-ΠΏΡ€ΠΎΠ΅ΠΊΡ†ΠΈΡŽ Π²Π΅ΠΊΡ‚ΠΎΡ€Π°Π½Π° ось, Π° Ρ‡Π΅Ρ€Π΅Π·Π²Π΅ΠΊΡ‚ΠΎΡ€-ΠΏΡ€ΠΎΠ΅ΠΊΡ†ΠΈΡŽ Π²Π΅ΠΊΡ‚ΠΎΡ€Π°Π½Π° ось. Π’ΠΎΠ³Π΄Π°, ΠΊΠ°ΠΊ Π²ΠΈΠ΄Π½ΠΎ ΠΈΠ· рисунка 21, ΠΈΠΌΠ΅Π΅Ρ‚ мСсто равСнство

.

Богласно Ρ‚Π΅ΠΎΡ€Π΅ΠΌΠ΅ 9,

,

.

ΠžΠ±ΠΎΠ·Π½Π°Ρ‡ΠΈΠΌ
,
. Π’ΠΎΠ³Π΄Π° ΠΏΠΎΠ»ΡƒΡ‡Π°Π΅ΠΌ

.

Π˜Ρ‚Π°ΠΊ, Π΄ΠΎΠΊΠ°Π·Π°Π½ΠΎ, Ρ‡Ρ‚ΠΎ для любого Π²Π΅ΠΊΡ‚ΠΎΡ€Π° сущСствуСт ΠΏΠ°Ρ€Π° чисСл
Ρ‚Π°ΠΊΠΈΡ…, Ρ‡Ρ‚ΠΎ справСдливо равСнство

,

,

.

ΠŸΡ€ΠΈ Π΄Ρ€ΡƒΠ³ΠΎΠΌ располоТСнии Π²Π΅ΠΊΡ‚ΠΎΡ€Π° ΠΎΡ‚Π½ΠΎΡΠΈΡ‚Π΅Π»ΡŒΠ½ΠΎ осСй Π΄ΠΎΠΊΠ°Π·Π°Ρ‚Π΅Π»ΡŒΡΡ‚Π²ΠΎ Π°Π½Π°Π»ΠΎΠ³ΠΈΡ‡Π½ΠΎ.

ΠžΠΏΡ€Π΅Π΄Π΅Π»Π΅Π½ΠΈΠ΅.

ΠŸΠ°Ρ€Π° чисСл ΠΈΡ‚Π°ΠΊΠΈΡ…, Ρ‡Ρ‚ΠΎ
, Π½Π°Π·Ρ‹Π²Π°ΡŽΡ‚ΡΡ ΠΊΠΎΠΎΡ€Π΄ΠΈΠ½Π°Ρ‚Π°ΠΌΠΈ Π²Π΅ΠΊΡ‚ΠΎΡ€Π°. ЧислоназываСтся иксовой ΠΊΠΎΠΎΡ€Π΄ΠΈΠ½Π°Ρ‚ΠΎΠΉ, Π° числоигрСковой ΠΊΠΎΠΎΡ€Π΄ΠΈΠ½Π°Ρ‚ΠΎΠΉ.

ΠžΠΏΡ€Π΅Π΄Π΅Π»Π΅Π½ΠΈΠ΅.

ΠŸΠ°Ρ€Π° ΠΎΡ€Ρ‚ΠΎΠ² осСй ΠΊΠΎΠΎΡ€Π΄ΠΈΠ½Π°Ρ‚
называСтся ΠΎΡ€Ρ‚ΠΎΠ½ΠΎΡ€ΠΌΠΈΡ€ΠΎΠ²Π°Π½Π½Ρ‹ΠΌ базисом Π½Π° плоскости. ΠŸΡ€Π΅Π΄ΡΡ‚Π°Π²Π»Π΅Π½ΠΈΠ΅ любого Π²Π΅ΠΊΡ‚ΠΎΡ€Π°Π² Π²ΠΈΠ΄Π΅
называСтся Ρ€Π°Π·Π»ΠΎΠΆΠ΅Π½ΠΈΠ΅ΠΌ Π²Π΅ΠΊΡ‚ΠΎΡ€Π°ΠΏΠΎ базису
.

НСпосрСдствСнно ΠΈΠ· опрСдСлСния ΠΊΠΎΠΎΡ€Π΄ΠΈΠ½Π°Ρ‚ Π²Π΅ΠΊΡ‚ΠΎΡ€Π° слСдуСт, Ρ‡Ρ‚ΠΎ Ссли ΠΊΠΎΠΎΡ€Π΄ΠΈΠ½Π°Ρ‚Ρ‹ Π²Π΅ΠΊΡ‚ΠΎΡ€ΠΎΠ² Ρ€Π°Π²Π½Ρ‹, Ρ‚ΠΎ Ρ€Π°Π²Π½Ρ‹ ΠΈ сами Π²Π΅ΠΊΡ‚ΠΎΡ€Ρ‹. Π‘ΠΏΡ€Π°Π²Π΅Π΄Π»ΠΈΠ²ΠΎ Ρ‚Π°ΠΊΠΆΠ΅ ΠΈ ΠΎΠ±Ρ€Π°Ρ‚Π½ΠΎΠ΅ ΡƒΡ‚Π²Π΅Ρ€ΠΆΠ΄Π΅Π½ΠΈΠ΅.

Π’Π΅ΠΎΡ€Π΅ΠΌΠ°.

Π Π°Π²Π½Ρ‹Π΅ Π²Π΅ΠΊΡ‚ΠΎΡ€Ρ‹ ΠΈΠΌΠ΅ΡŽΡ‚ Ρ€Π°Π²Π½Ρ‹Π΅ ΠΊΠΎΠΎΡ€Π΄ΠΈΠ½Π°Ρ‚Ρ‹.

Π”ΠΎΠΊΠ°Π·Π°Ρ‚Π΅Π»ΡŒΡΡ‚Π²ΠΎ.

,

ΠΈ
. Π”ΠΎΠΊΠ°ΠΆΠ΅ΠΌ, Ρ‡Ρ‚ΠΎ
,
.

Из равСнства Π²Π΅ΠΊΡ‚ΠΎΡ€ΠΎΠ² слСдуСт, Ρ‡Ρ‚ΠΎ

.

Допустим, Ρ‡Ρ‚ΠΎ
, Π°
.

Π’ΠΎΠ³Π΄Π°
ΠΈ Π·Π½Π°Ρ‡ΠΈΡ‚
, Ρ‡Ρ‚ΠΎ Π½Π΅ Π²Π΅Ρ€Π½ΠΎ. Аналогично, Ссли
, Π½ΠΎ
, Ρ‚ΠΎ
. ΠžΡ‚ΡΡŽΠ΄Π°
, Ρ‡Ρ‚ΠΎ Π½Π΅ Π²Π΅Ρ€Π½ΠΎ. НаконСц, Ссли Π΄ΠΎΠΏΡƒΡΡ‚ΠΈΡ‚ΡŒ, Ρ‡Ρ‚ΠΎ
ΠΈ
, Ρ‚ΠΎ ΠΏΠΎΠ»ΡƒΡ‡Π°Π΅ΠΌ, Ρ‡Ρ‚ΠΎ

.

Π­Ρ‚ΠΎ ΠΎΠ·Π½Π°Ρ‡Π°Π΅Ρ‚, Ρ‡Ρ‚ΠΎ Π²Π΅ΠΊΡ‚ΠΎΡ€Ρ‹ ΠΈΠΊΠΎΠ»Π»ΠΈΠ½Π΅Π°Ρ€Π΅Ρ‹. Но это Π½Π΅ Π²Π΅Ρ€Π½ΠΎ, Ρ‚Π°ΠΊ ΠΊΠ°ΠΊ ΠΎΠ½ΠΈ пСрпСндикулярны. Π‘Π»Π΅Π΄ΠΎΠ²Π°Ρ‚Π΅Π»ΡŒΠ½ΠΎ, остаСтся, Ρ‡Ρ‚ΠΎ
,
, Ρ‡Ρ‚ΠΎ ΠΈ Ρ‚Ρ€Π΅Π±ΠΎΠ²Π°Π»ΠΎΡΡŒ Π΄ΠΎΠΊΠ°Π·Π°Ρ‚ΡŒ.

Π’Π°ΠΊΠΈΠΌ ΠΎΠ±Ρ€Π°Π·ΠΎΠΌ, ΠΊΠΎΠΎΡ€Π΄ΠΈΠ½Π°Ρ‚Ρ‹ Π²Π΅ΠΊΡ‚ΠΎΡ€Π° ΠΏΠΎΠ»Π½ΠΎΡΡ‚ΡŒΡŽ ΠΎΠΏΡ€Π΅Π΄Π΅Π»ΡΡŽΡ‚ сам Π²Π΅ΠΊΡ‚ΠΎΡ€. Зная ΠΊΠΎΠΎΡ€Π΄ΠΈΠ½Π°Ρ‚Ρ‹ ΠΈΠ²Π΅ΠΊΡ‚ΠΎΡ€Π°ΠΌΠΎΠΆΠ½ΠΎ ΠΏΠΎΡΡ‚Ρ€ΠΎΠΈΡ‚ΡŒ сам Π²Π΅ΠΊΡ‚ΠΎΡ€, построив Π²Π΅ΠΊΡ‚ΠΎΡ€Ρ‹
ΠΈ
ΠΈ слоТив ΠΈΡ…. ΠŸΠΎΡΡ‚ΠΎΠΌΡƒ часто сам Π²Π΅ΠΊΡ‚ΠΎΡ€ΠΎΠ±ΠΎΠ·Π½Π°Ρ‡Π°ΡŽΡ‚ Π² Π²ΠΈΠ΄Π΅ ΠΏΠ°Ρ€Ρ‹ Π΅Π³ΠΎ ΠΊΠΎΠΎΡ€Π΄ΠΈΠ½Π°Ρ‚ ΠΈ ΠΏΠΈΡˆΡƒΡ‚
. Вакая запись ΠΎΠ·Π½Π°Ρ‡Π°Π΅Ρ‚, Ρ‡Ρ‚ΠΎ
.

НСпосрСдствСнно ΠΈΠ· опрСдСлСния ΠΊΠΎΠΎΡ€Π΄ΠΈΠ½Π°Ρ‚ Π²Π΅ΠΊΡ‚ΠΎΡ€Π° слСдуСт ΡΠ»Π΅Π΄ΡƒΡŽΡ‰Π°Ρ Ρ‚Π΅ΠΎΡ€Π΅ΠΌΠ°.

Π’Π΅ΠΎΡ€Π΅ΠΌΠ°.

ΠŸΡ€ΠΈ слоТСнии Π²Π΅ΠΊΡ‚ΠΎΡ€ΠΎΠ² ΠΈΡ… ΠΊΠΎΠΎΡ€Π΄ΠΈΠ½Π°Ρ‚Ρ‹ ΡΠΊΠ»Π°Π΄Ρ‹Π²Π°ΡŽΡ‚ΡΡ Π° ΠΏΡ€ΠΈ ΡƒΠΌΠ½ΠΎΠΆΠ΅Π½ΠΈΠΈ Π²Π΅ΠΊΡ‚ΠΎΡ€Π° Π½Π° число Π΅Π³ΠΎ ΠΊΠΎΠΎΡ€Π΄ΠΈΠ½Π°Ρ‚Ρ‹ ΡƒΠΌΠ½ΠΎΠΆΠ°ΡŽΡ‚ΡΡ Π½Π° это число. Π—Π°ΠΏΠΈΡΡ‹Π²Π°ΡŽΡ‚ΡΡ эти утвСрТдСния Π² Π²ΠΈΠ΄Π΅

.

Π”ΠΎΠΊΠ°Π·Π°Ρ‚Π΅Π»ΡŒΡΡ‚Π²ΠΎ.

,

Π’Π΅ΠΎΡ€Π΅ΠΌΠ°.

ΠŸΡƒΡΡ‚ΡŒ
, ΠΏΡ€ΠΈΡ‡Π΅ΠΌ Π½Π°Ρ‡Π°Π»ΠΎ Π²Π΅ΠΊΡ‚ΠΎΡ€Π° Ρ‚ΠΎΡ‡ΠΊΠ°ΠΈΠΌΠ΅Π΅Ρ‚ ΠΊΠΎΠΎΡ€Π΄ΠΈΠ½Π°Ρ‚Ρ‹
, Π° ΠΊΠΎΠ½Π΅Ρ† Π²Π΅ΠΊΡ‚ΠΎΡ€Π° Π΅ΡΡ‚ΡŒ Ρ‚ΠΎΡ‡ΠΊΠ°
. Π’ΠΎΠ³Π΄Π° ΠΊΠΎΠΎΡ€Π΄ΠΈΠ½Π°Ρ‚Ρ‹ Π²Π΅ΠΊΡ‚ΠΎΡ€Π° связаны с ΠΊΠΎΠΎΡ€Π΄ΠΈΠ½Π°Ρ‚Π°ΠΌΠΈ Π΅Π³ΠΎ ΠΊΠΎΠ½Ρ†ΠΎΠ² ΡΠ»Π΅Π΄ΡƒΡŽΡ‰ΠΈΠΌΠΈ ΡΠΎΠΎΡ‚Π½ΠΎΡˆΠ΅Π½ΠΈΡΠΌΠΈ

,

.

Π”ΠΎΠΊΠ°Π·Π°Ρ‚Π΅Π»ΡŒΡΡ‚Π²ΠΎ.

ΠŸΡƒΡΡ‚ΡŒ
ΠΈ ΠΏΡƒΡΡ‚ΡŒ Π²Π΅ΠΊΡ‚ΠΎΡ€-проСкция Π²Π΅ΠΊΡ‚ΠΎΡ€Π°Π½Π° ΠΎΡΡŒΡΠΎΠ½Π°ΠΏΡ€Π°Π²Π»Π΅Π½ с осью(см. рис. 22). Π’ΠΎΠ³Π΄Π°

Ρ‚Π°ΠΊ ΠΊΠ°ΠΊ Π΄Π»ΠΈΠ½Π° ΠΎΡ‚Ρ€Π΅Π·ΠΊΠ° Π½Π° числовой осиравна ΠΊΠΎΠΎΡ€Π΄ΠΈΠ½Π°Ρ‚Π΅ ΠΏΡ€Π°Π²ΠΎΠ³ΠΎ ΠΊΠΎΠ½Ρ†Π° минус ΠΊΠΎΠΎΡ€Π΄ΠΈΠ½Π°Ρ‚Π° Π»Π΅Π²ΠΎΠ³ΠΎ ΠΊΠΎΠ½Ρ†Π°. Если Π²Π΅ΠΊΡ‚ΠΎΡ€

ΠΏΡ€ΠΎΡ‚ΠΈΠ²ΠΎΠ½Π°ΠΏΡ€Π°Π²Π»Π΅Π½ оси(ΠΊΠ°ΠΊ Π½Π° Рис. 23), Ρ‚ΠΎ

Рис. 23.

Если
, Ρ‚ΠΎ Π² этом случаС
ΠΈ Ρ‚ΠΎΠ³Π΄Π° ΠΏΠΎΠ»ΡƒΡ‡Π°Π΅ΠΌ

.

Π’Π°ΠΊΠΈΠΌ ΠΎΠ±Ρ€Π°Π·ΠΎΠΌ, ΠΏΡ€ΠΈ любом располоТСнии Π²Π΅ΠΊΡ‚ΠΎΡ€Π°
ΠΎΡ‚Π½ΠΎΡΠΈΡ‚Π΅Π»ΡŒΠ½ΠΎ осСй ΠΊΠΎΠΎΡ€Π΄ΠΈΠ½Π°Ρ‚ Π΅Π³ΠΎ ΠΊΠΎΠΎΡ€Π΄ΠΈΠ½Π°Ρ‚Π°Ρ€Π°Π²Π½Π°

.

Аналогично доказываСтся, Ρ‡Ρ‚ΠΎ

.

ΠŸΡ€ΠΈΠΌΠ΅Ρ€.

Π”Π°Π½Ρ‹ ΠΊΠΎΠΎΡ€Π΄ΠΈΠ½Π°Ρ‚Ρ‹ ΠΊΠΎΠ½Ρ†ΠΎΠ² Π²Π΅ΠΊΡ‚ΠΎΡ€Π°
:
. Найти ΠΊΠΎΠΎΡ€Π΄ΠΈΠ½Π°Ρ‚Ρ‹ Π²Π΅ΠΊΡ‚ΠΎΡ€Π°
.

РСшСниС.

Π’ ΡΠ»Π΅Π΄ΡƒΡŽΡ‰Π΅ΠΉ Ρ‚Π΅ΠΎΡ€Π΅ΠΌΠ΅ приводится Π²Ρ‹Ρ€Π°ΠΆΠ΅Π½ΠΈΠ΅ Π΄Π»ΠΈΠ½Ρ‹ Π²Π΅ΠΊΡ‚ΠΎΡ€Π° Ρ‡Π΅Ρ€Π΅Π· Π΅Π³ΠΎ ΠΊΠΎΠΎΡ€Π΄ΠΈΠ½Π°Ρ‚Ρ‹.

Π’Π΅ΠΎΡ€Π΅ΠΌΠ° 15.

ΠŸΡƒΡΡ‚ΡŒ
.Π’ΠΎΠ³Π΄Π°

.

Π”ΠΎΠΊΠ°Π·Π°Ρ‚Π΅Π»ΡŒΡΡ‚Π²ΠΎ.

ΠŸΡƒΡΡ‚ΡŒ ΠΈ- Π²Π΅ΠΊΡ‚ΠΎΡ€-ΠΏΡ€ΠΎΠ΅ΠΊΡ†ΠΈΠΈ Π²Π΅ΠΊΡ‚ΠΎΡ€Π°Π½Π° осии, соотвСтствСнно. Π’ΠΎΠ³Π΄Π°, ΠΊΠ°ΠΊ ΠΏΠΎΠΊΠ°Π·Π°Π½ΠΎ ΠΏΡ€ΠΈ Π΄ΠΎΠΊΠ°Π·Π°Ρ‚Π΅Π»ΡŒΡΡ‚Π²Π΅ Ρ‚Π΅ΠΎΡ€Π΅ΠΌΡ‹ 9, ΠΈΠΌΠ΅Π΅Ρ‚ мСсто равСнство

.

ΠŸΡ€ΠΈ этом, Π²Π΅ΠΊΡ‚ΠΎΡ€Ρ‹ ΠΈΠ²Π·Π°ΠΈΠΌΠ½ΠΎ пСрпСндикулярны. ΠŸΡ€ΠΈ слоТСнии этих Π²Π΅ΠΊΡ‚ΠΎΡ€ΠΎΠ² ΠΏΠΎ ΠΏΡ€Π°Π²ΠΈΠ»Ρƒ Ρ‚Ρ€Π΅ΡƒΠ³ΠΎΠ»ΡŒΠ½ΠΈΠΊΠ° ΠΏΠΎΠ»ΡƒΡ‡Π°Π΅ΠΌ ΠΏΡ€ΡΠΌΠΎΡƒΠ³ΠΎΠ»ΡŒΠ½Ρ‹ΠΉ Ρ‚Ρ€Π΅ΡƒΠ³ΠΎΠ»ΡŒΠ½ΠΈΠΊ (см. Рис. 24).

По Ρ‚Π΅ΠΎΡ€Π΅ΠΌΠ΅ ΠŸΠΈΡ„Π°Π³ΠΎΡ€Π° ΠΈΠΌΠ΅Π΅ΠΌ

.

,

.

Π‘Π»Π΅Π΄ΠΎΠ²Π°Ρ‚Π΅Π»ΡŒΠ½ΠΎ

,

.

.

.

ΠŸΡ€ΠΈΠΌΠ΅Ρ€.

.Найти.

Π’Π²Π΅Π΄Π΅ΠΌ понятиС Π½Π°ΠΏΡ€Π°Π²Π»ΡΡŽΡ‰ΠΈΡ… косинусов Π²Π΅ΠΊΡ‚ΠΎΡ€Π°.

ΠžΠΏΡ€Π΅Π΄Π΅Π»Π΅Π½ΠΈΠ΅.

ΠŸΡƒΡΡ‚ΡŒ Π²Π΅ΠΊΡ‚ΠΎΡ€
составляСт с ΠΎΡΡŒΡŽΡƒΠ³ΠΎΠ», Π° с ΠΎΡΡŒΡŽΡƒΠ³ΠΎΠ»(см. Рис. 25).

,

.

Π‘Π»Π΅Π΄ΠΎΠ²Π°Ρ‚Π΅Π»ΡŒΠ½ΠΎ,

Π’Π°ΠΊ ΠΊΠ°ΠΊ для любого Π²Π΅ΠΊΡ‚ΠΎΡ€Π° ΠΈΠΌΠ΅Π΅Ρ‚ мСсто равСнство

,

Π“Π΄Π΅ — ΠΎΡ€Ρ‚ Π²Π΅ΠΊΡ‚ΠΎΡ€Π°, Ρ‚ΠΎ Π΅ΡΡ‚ΡŒ Π²Π΅ΠΊΡ‚ΠΎΡ€ Π΅Π΄ΠΈΠ½ΠΈΡ‡Π½ΠΎΠΉ Π΄Π»ΠΈΠ½Ρ‹, сонаправлСнный с Π²Π΅ΠΊΡ‚ΠΎΡ€ΠΎΠΌ, Ρ‚ΠΎ

Π’Π΅ΠΊΡ‚ΠΎΡ€ опрСдСляСт Π½Π°ΠΏΡ€Π°Π²Π»Π΅Π½ΠΈΠ΅ Π²Π΅ΠΊΡ‚ΠΎΡ€Π°. Π•Π³ΠΎ ΠΊΠΎΠΎΡ€Π΄ΠΈΠ½Π°Ρ‚Ρ‹
ΠΈ
Π½Π°Π·Ρ‹Π²Π°ΡŽΡ‚ΡΡ Π½Π°ΠΏΡ€Π°Π²Π»ΡΡŽΡ‰ΠΈΠΌΠΈ косинусами Π²Π΅ΠΊΡ‚ΠΎΡ€Π°. ΠΠ°ΠΏΡ€Π°Π²Π»ΡΡŽΡ‰ΠΈΠ΅ косинусы Π²Π΅ΠΊΡ‚ΠΎΡ€Π° ΠΌΠΎΠΆΠ½ΠΎ Π²Ρ‹Ρ€Π°Π·ΠΈΡ‚ΡŒ Ρ‡Π΅Ρ€Π΅Π· Π΅Π³ΠΎ ΠΊΠΎΠΎΡ€Π΄ΠΈΠ½Π°Ρ‚Ρ‹ ΠΏΠΎ Ρ„ΠΎΡ€ΠΌΡƒΠ»Π°ΠΌ

,

.

Π˜ΠΌΠ΅Π΅Ρ‚ мСсто ΡΠΎΠΎΡ‚Π½ΠΎΡˆΠ΅Π½ΠΈΠ΅

.

Π”ΠΎ настоящСго ΠΌΠΎΠΌΠ΅Π½Ρ‚Π° Π² этом ΠΏΠ°Ρ€Π°Π³Ρ€Π°Ρ„Π΅ ΡΡ‡ΠΈΡ‚Π°Π»ΠΎΡΡŒ, Ρ‡Ρ‚ΠΎ всС Π²Π΅ΠΊΡ‚ΠΎΡ€Ρ‹ Ρ€Π°ΡΠΏΠΎΠ»Π°Π³Π°ΡŽΡ‚ΡΡ Π² ΠΎΠ΄Π½ΠΎΠΉ ΠΈ Ρ‚ΠΎΠΉ ΠΆΠ΅ плоскости. Π’Π΅ΠΏΠ΅Ρ€ΡŒ сдСлаСм ΠΎΠ±ΠΎΠ±Ρ‰Π΅Π½ΠΈΠ΅ для Π²Π΅ΠΊΡ‚ΠΎΡ€ΠΎΠ² Π² пространствС.

Π‘ΡƒΠ΄Π΅ΠΌ ΡΡ‡ΠΈΡ‚Π°Ρ‚ΡŒ, Ρ‡Ρ‚ΠΎ Π² пространствС Π·Π°Π΄Π°Π½Π° Π”Π΅ΠΊΠ°Ρ€Ρ‚ΠΎΠ²Π° систСма ΠΊΠΎΠΎΡ€Π΄ΠΈΠ½Π°Ρ‚ с осями ,ΠΈ.

ΠžΡ€Ρ‚Ρ‹ осСй ,ΠΈΠ±ΡƒΠ΄Π΅ΠΌ ΠΎΠ±ΠΎΠ·Π½Π°Ρ‡Π°Ρ‚ΡŒ символами,ΠΈ, соотвСтствСнно (Рис. 26).

МоТно ΠΏΠΎΠΊΠ°Π·Π°Ρ‚ΡŒ, Ρ‡Ρ‚ΠΎ всС понятия ΠΈ Ρ„ΠΎΡ€ΠΌΡƒΠ»Ρ‹, ΠΊΠΎΡ‚ΠΎΡ€Ρ‹Π΅ Π±Ρ‹Π»ΠΈ ΠΏΠΎΠ»ΡƒΡ‡Π΅Π½Ρ‹ для Π²Π΅ΠΊΡ‚ΠΎΡ€ΠΎΠ² Π½Π° плоскости, ΠΎΠ±ΠΎΠ±Ρ‰Π°ΡŽΡ‚ΡΡ для

Рис. 26.

Π²Π΅ΠΊΡ‚ΠΎΡ€ΠΎΠ² Π² пространствС. Π’Ρ€ΠΎΠΉΠΊΠ° Π²Π΅ΠΊΡ‚ΠΎΡ€ΠΎΠ²
называСтся ΠΎΡ€Ρ‚ΠΎΠ½ΠΎΡ€ΠΌΠΈΡ€ΠΎΠ²Π°Π½Π½Ρ‹ΠΌ базисом Π² пространствС.

ΠŸΡƒΡΡ‚ΡŒ ,ΠΈ- Π²Π΅ΠΊΡ‚ΠΎΡ€-ΠΏΡ€ΠΎΠ΅ΠΊΡ†ΠΈΠΈ Π²Π΅ΠΊΡ‚ΠΎΡ€Π°Π½Π° оси,ΠΈ, соотвСтствСнно. Π’ΠΎΠ³Π΄Π°

.

Π’ свою ΠΎΡ‡Π΅Ρ€Π΅Π΄ΡŒ

,

,

.

Если ΠΎΠ±ΠΎΠ·Π½Π°Ρ‡ΠΈΡ‚ΡŒ

,

,

,

Π’ΠΎ ΠΏΠΎΠ»ΡƒΡ‡Π°Π΅ΠΌ равСнство

.

ΠšΠΎΡΡ„Ρ„ΠΈΡ†ΠΈΠ΅Π½Ρ‚Ρ‹ ΠΏΠ΅Ρ€Π΅Π΄ базисными Π²Π΅ΠΊΡ‚ΠΎΡ€Π°ΠΌΠΈ ,ΠΈΠ½Π°Π·Ρ‹Π²Π°ΡŽΡ‚ΡΡ ΠΊΠΎΠΎΡ€Π΄ΠΈΠ½Π°Ρ‚Π°ΠΌΠΈ Π²Π΅ΠΊΡ‚ΠΎΡ€Π°. Π’Π°ΠΊΠΈΠΌ ΠΎΠ±Ρ€Π°Π·ΠΎΠΌ, для любого Π²Π΅ΠΊΡ‚ΠΎΡ€Π°Π² пространствС сущСствуСт Ρ‚Ρ€ΠΎΠΉΠΊΠ° чисСл,,, Π½Π°Π·Ρ‹Π²Π°Π΅ΠΌΡ‹Ρ… ΠΊΠΎΠΎΡ€Π΄ΠΈΠ½Π°Ρ‚Π°ΠΌΠΈ Π²Π΅ΠΊΡ‚ΠΎΡ€Π°Ρ‚Π°ΠΊΠΈΡ…, Ρ‡Ρ‚ΠΎ для этого Π²Π΅ΠΊΡ‚ΠΎΡ€Π° справСдливо прСдставлСниС

.

Π’Π΅ΠΊΡ‚ΠΎΡ€ Π² этом случаС Ρ‚Π°ΠΊΠΆΠ΅ ΠΎΠ±ΠΎΠ·Π½Π°Ρ‡Π°ΡŽΡ‚ Π² Π²ΠΈΠ΄Π΅
. ΠŸΡ€ΠΈ этом, ΠΊΠΎΠΎΡ€Π΄ΠΈΠ½Π°Ρ‚Ρ‹ Π²Π΅ΠΊΡ‚ΠΎΡ€Π° Ρ€Π°Π²Π½Ρ‹ проСкциям этого Π²Π΅ΠΊΡ‚ΠΎΡ€Π° Π½Π° ΠΊΠΎΠΎΡ€Π΄ΠΈΠ½Π°Ρ‚Π½Ρ‹Π΅ оси

,

,

,

Π³Π΄Π΅ — ΡƒΠ³ΠΎΠ» ΠΌΠ΅ΠΆΠ΄Ρƒ Π²Π΅ΠΊΡ‚ΠΎΡ€ΠΎΠΌΠΈ осью,- ΡƒΠ³ΠΎΠ» ΠΌΠ΅ΠΆΠ΄Ρƒ Π²Π΅ΠΊΡ‚ΠΎΡ€ΠΎΠΌΠΈ осью,- ΡƒΠ³ΠΎΠ» ΠΌΠ΅ΠΆΠ΄Ρƒ Π²Π΅ΠΊΡ‚ΠΎΡ€ΠΎΠΌΠΈ осью.

Π”Π»ΠΈΠ½Π° Π²Π΅ΠΊΡ‚ΠΎΡ€Π° выраТаСтся Ρ‡Π΅Ρ€Π΅Π· Π΅Π³ΠΎ ΠΊΠΎΠΎΡ€Π΄ΠΈΠ½Π°Ρ‚Ρ‹ ΠΏΠΎ Ρ„ΠΎΡ€ΠΌΡƒΠ»Π΅

.

Π‘ΠΏΡ€Π°Π²Π΅Π΄Π»ΠΈΠ²Ρ‹ утвСрТдСния ΠΎ Ρ‚ΠΎΠΌ, Ρ‡Ρ‚ΠΎ Ρ€Π°Π²Π½Ρ‹Π΅ Π²Π΅ΠΊΡ‚ΠΎΡ€Ρ‹ ΠΈΠΌΠ΅ΡŽΡ‚ Ρ€Π°Π²Π½Ρ‹Π΅ ΠΊΠΎΠΎΡ€Π΄ΠΈΠ½Π°Ρ‚Ρ‹, ΠΏΡ€ΠΈ слоТСнии Π²Π΅ΠΊΡ‚ΠΎΡ€ΠΎΠ² ΠΈΡ… ΠΊΠΎΠΎΡ€Π΄ΠΈΠ½Π°Ρ‚Ρ‹ ΡΠΊΠ»Π°Π΄Ρ‹Π²Π°ΡŽΡ‚ΡΡ, Π° ΠΏΡ€ΠΈ ΡƒΠΌΠ½ΠΎΠΆΠ΅Π½ΠΈΠΈ Π²Π΅ΠΊΡ‚ΠΎΡ€Π° Π½Π° число Π΅Π³ΠΎ ΠΊΠΎΠΎΡ€Π΄ΠΈΠ½Π°Ρ‚Ρ‹ ΡƒΠΌΠ½ΠΎΠΆΠ°ΡŽΡ‚ΡΡ Π½Π° это число. 2\), ΠΎΡ‚ΠΊΡƒΠ΄Π° ΠΏΠΎΠ»ΡƒΡ‡Π°Π΅ΠΌ Ρ‚Ρ€Π΅Π±ΡƒΠ΅ΠΌΠΎΠ΅ равСнство.

Β 

Π£Ρ‚Π²Π΅Ρ€ΠΆΠ΄Π΅Π½ΠΈΠ΅

Если Π² ΠΏΡ€ΡΠΌΠΎΡƒΠ³ΠΎΠ»ΡŒΠ½ΠΎΠΉ систСмС ΠΊΠΎΠΎΡ€Π΄ΠΈΠ½Π°Ρ‚ Ρ‚ΠΎΡ‡ΠΊΠ° \(M\) – сСрСдина ΠΎΡ‚Ρ€Π΅Π·ΠΊΠ° \(PQ\), Π³Π΄Π΅ \(P(x_1;y_1), \ Q(x_2;y_2)\), Ρ‚ΠΎ

\[M\left(\dfrac{x_1 + x_2}{2}; \dfrac{y_1 + y_2}{2}\right)\]

Π”ΠΎΠΊΠ°Π·Π°Ρ‚Π΅Π»ΡŒΡΡ‚Π²ΠΎ

ΠŸΡƒΡΡ‚ΡŒ \(M(a;b)\).

Β 

1) ΠŸΡƒΡΡ‚ΡŒ \(PQ\parallel Oy \Rightarrow x_1=x_2=a\). Π—Π½Π°Ρ‡ΠΈΡ‚, \(a=\dfrac{x_1+x_2}2=\dfrac{a+a}2\) – Π²Π΅Ρ€Π½ΠΎ.

Β 

Π’.ΠΊ. \(PM=MQ\), ΡΠ»Π΅Π΄ΠΎΠ²Π°Ρ‚Π΅Π»ΡŒΠ½ΠΎ, \(|y_2-b|=|y_1-b| \Rightarrow y_2-b=y_1-b\) ΠΈΠ»ΠΈ \(y_2-b=b-y_1\), Ρ‡Ρ‚ΠΎ Ρ€Π°Π²Π½ΠΎΡΠΈΠ»ΡŒΠ½ΠΎ \(y_2=y_1\) ΠΈΠ»ΠΈ \(b=\dfrac{y_1+y_2}2\). ΠŸΠ΅Ρ€Π²ΠΎΠ΅ равСнство Π½Π΅Π²ΠΎΠ·ΠΌΠΎΠΆΠ½ΠΎ (Ρ‚.ΠΊ. Ρ‚ΠΎΠ³Π΄Π° Ρ‚ΠΎΡ‡ΠΊΠΈ \(P\) ΠΈ \(Q\) ΡΠΎΠ²ΠΏΠ°Π΄Π°ΡŽΡ‚).

Β 

2) Π‘Π»ΡƒΡ‡Π°ΠΉ \(PQ\parallel Ox \Rightarrow y_1=y_2=b\) доказываСтся Π°Π½Π°Π»ΠΎΠ³ΠΈΡ‡Π½ΠΎ.

Β 

3) \(x_1\ne x_2, y_1\ne y_2\).


Β 

Π’ΠΎΠ³Π΄Π° \(Ma=b\) – срСдняя линия Ρ‚Ρ€Π°ΠΏΠ΅Ρ†ΠΈΠΈ \(x_1PQx_2\), ΡΠ»Π΅Π΄ΠΎΠ²Π°Ρ‚Π΅Π»ΡŒΠ½ΠΎ, Ρ€Π°Π²Π½Π° полусуммС оснований, Ρ‚ΠΎ Π΅ΡΡ‚ΡŒ \(b=\dfrac{y_1+y_2}2\).

Β 

Аналогично \(a=\dfrac{x_1+x_2}2\). Β 

\[{\Large{\text{Π’Π΅ΠΊΡ‚ΠΎΡ€Ρ‹ Π½Π° ΠΊΠΎΠΎΡ€Π΄ΠΈΠ½Π°Ρ‚Π½ΠΎΠΉ плоскости}}}\]

Π›Π΅ΠΌΠΌΠ°

Если Π²Π΅ΠΊΡ‚ΠΎΡ€Ρ‹ \(\overrightarrow a\) ΠΈ \(\overrightarrow b\) ΠΊΠΎΠ»Π»ΠΈΠ½Π΅Π°Ρ€Π½Ρ‹, Ρ‚ΠΎ сущСствуСт Ρ‚Π°ΠΊΠΎΠ΅ число \(\lambda\ne 0\), Ρ‡Ρ‚ΠΎ \(\overrightarrow a=\lambda\overrightarrow b\).

Β 

Π”ΠΎΠΊΠ°Π·Π°Ρ‚Π΅Π»ΡŒΡΡ‚Π²ΠΎ

1) Если \(\overrightarrow a\uparrow \uparrow \overrightarrow b\).

Β 

Рассмотрим Π²Π΅ΠΊΡ‚ΠΎΡ€ \(\dfrac1{|\overrightarrow a|}\overrightarrow a\). Π”Π°Π½Π½Ρ‹ΠΉ Π²Π΅ΠΊΡ‚ΠΎΡ€ сонавправлСн с \(\overrightarrow a\), Π° Π΅Π³ΠΎ Π΄Π»ΠΈΠ½Π° Ρ€Π°Π²Π½Π° \(1\). Π’ΠΎΠ³Π΄Π° Π²Π΅ΠΊΡ‚ΠΎΡ€ \(\dfrac{|\overrightarrow b|}{|\overrightarrow a|}\overrightarrow a\) Ρ‚Π°ΠΊΠΆΠ΅ сонаправлСн с \(\overrightarrow a\), Π½ΠΎ Π΅Π³ΠΎ Π΄Π»ΠΈΠ½Π° Ρ€Π°Π²Π½Π° \(|\overrightarrow b|\). Π’ΠΎ Π΅ΡΡ‚ΡŒ Ρ€Π°Π²Π΅Π½ Π²Π΅ΠΊΡ‚ΠΎΡ€Ρƒ \(\overrightarrow b\).

Β 

2) Если \(\overrightarrow a\uparrow \downarrow \overrightarrow b\).

Β 

Аналогично доказываСтся, Ρ‡Ρ‚ΠΎ \(\overrightarrow b=-\dfrac{|\overrightarrow b|}{|\overrightarrow a|}\overrightarrow a\).

Β 

ΠžΠΏΡ€Π΅Π΄Π΅Π»Π΅Π½ΠΈΠ΅

Если Π²Π΅ΠΊΡ‚ΠΎΡ€ \(\overrightarrow p\) прСдставлСн ΠΊΠ°ΠΊ линСйная комбинация Π΄Π²ΡƒΡ… Π²Π΅ΠΊΡ‚ΠΎΡ€ΠΎΠ²: \(\overrightarrow p=\alpha\overrightarrow a+\beta \overrightarrow b\), Ρ‚ΠΎ говорят, Ρ‡Ρ‚ΠΎ Π²Π΅ΠΊΡ‚ΠΎΡ€ \(\overrightarrow p\) Ρ€Π°Π·Π»ΠΎΠΆΠ΅Π½ ΠΏΠΎ Π²Π΅ΠΊΡ‚ΠΎΡ€Π°ΠΌ \(\overrightarrow a\) ΠΈ \(\overrightarrow b\).

Β 

\(\alpha, \beta\) – коэффициСнты разлоТСния. Β 

ΠŸΡƒΡΡ‚ΡŒ Π²Π΅ΠΊΡ‚ΠΎΡ€Ρ‹ \(\overrightarrow i\), \(\overrightarrow j\) – Π²Π΅ΠΊΡ‚ΠΎΡ€Ρ‹, Π΄Π»ΠΈΠ½Ρ‹ ΠΊΠΎΡ‚ΠΎΡ€Ρ‹Ρ… Ρ€Π°Π²Π½Ρ‹ \(1\), Π° Π½Π°ΠΏΡ€Π°Π²Π»Π΅Π½ΠΈΠ΅ совпадаСт с Π½Π°ΠΏΡ€Π°Π²Π»Π΅Π½ΠΈΠ΅ΠΌ осСй \(Ox\) ΠΈ \(Oy\) соотвСтствСнно. Π’Π°ΠΊΠΈΠ΅ Π²Π΅ΠΊΡ‚ΠΎΡ€Ρ‹ Π½Π°Π·Ρ‹Π²Π°ΡŽΡ‚ΡΡ Π΅Π΄ΠΈΠ½ΠΈΡ‡Π½Ρ‹ΠΌΠΈ Π²Π΅ΠΊΡ‚ΠΎΡ€Π°ΠΌΠΈ.

Β 

Π’ΠΎΠ³Π΄Π° Ссли \(\overrightarrow p=a\overrightarrow i+b\overrightarrow j\), Ρ‚ΠΎ \(\{a;b\}\) – ΠΊΠΎΠΎΡ€Π΄ΠΈΠ½Π°Ρ‚Ρ‹ Π²Π΅ΠΊΡ‚ΠΎΡ€Π° \(\overrightarrow p\).


Β 

Бвойства ΠΊΠΎΠΎΡ€Π΄ΠΈΠ½Π°Ρ‚ Π²Π΅ΠΊΡ‚ΠΎΡ€Π°

1. Π Π°Π²Π½Ρ‹Π΅ Π²Π΅ΠΊΡ‚ΠΎΡ€Ρ‹ ΠΈΠΌΠ΅ΡŽΡ‚ Ρ€Π°Π²Π½Ρ‹Π΅ ΠΊΠΎΠΎΡ€Π΄ΠΈΠ½Π°Ρ‚Ρ‹.

Β 

2. ΠšΠΎΠΎΡ€Π΄ΠΈΠ½Π°Ρ‚Ρ‹ суммы Π²Π΅ΠΊΡ‚ΠΎΡ€ΠΎΠ² Ρ€Π°Π²Π½Ρ‹ суммС ΠΊΠΎΠΎΡ€Π΄ΠΈΠ½Π°Ρ‚ ΠΊΠ°ΠΆΠ΄ΠΎΠ³ΠΎ Π²Π΅ΠΊΡ‚ΠΎΡ€Π°: Ссли \(\overrightarrow a\{x_1;y_1\}, \ \overrightarrow b\{x_2;y_2\}\), Ρ‚ΠΎ \(\overrightarrow a+\overrightarrow b=\{x_1+x_2;y_1+y_2\}\).

Β 

3. КаТдая ΠΊΠΎΠΎΡ€Π΄ΠΈΠ½Π°Ρ‚Π° произвСдСния Π²Π΅ΠΊΡ‚ΠΎΡ€Π° Π½Π° число Ρ€Π°Π²Π½Π° ΠΏΡ€ΠΎΠΈΠ·Π²Π΅Π΄Π΅Π½ΠΈΡŽ ΡΠΎΠΎΡ‚Π²Π΅Ρ‚ΡΡ‚Π²ΡƒΡŽΡ‰Π΅ΠΉ ΠΊΠΎΠΎΡ€Π΄ΠΈΠ½Π°Ρ‚Ρ‹ Π΄Π°Π½Π½ΠΎΠ³ΠΎ Π²Π΅ΠΊΡ‚ΠΎΡ€Π° Π½Π° это число: \(\overrightarrow a\{x;y\}, \ \lambda \) – число, Ρ‚ΠΎ \(\lambda\overrightarrow a\{\lambda x;\lambda y\}\). 2}\). Β 

\[{\Large{\text{БкалярноС ΠΏΡ€ΠΎΠΈΠ·Π²Π΅Π΄Π΅Π½ΠΈΠ΅ Π²Π΅ΠΊΡ‚ΠΎΡ€ΠΎΠ²}}}\]

ΠžΠΏΡ€Π΅Π΄Π΅Π»Π΅Π½ΠΈΠ΅

ΠŸΡƒΡΡ‚ΡŒ ΠΎΡ‚ ΠΎΠ΄Π½ΠΎΠΉ Ρ‚ΠΎΡ‡ΠΊΠΈ ΠΎΡ‚Π»ΠΎΠΆΠ΅Π½Ρ‹ Π΄Π²Π° Π²Π΅ΠΊΡ‚ΠΎΡ€Π° \(\overrightarrow {AB}\) ΠΈ \(\overrightarrow {AC}\). Π’ΠΎΠ³Π΄Π° ΡƒΠ³ΠΎΠ» ΠΌΠ΅ΠΆΠ΄Ρƒ этими Π²Π΅ΠΊΡ‚ΠΎΡ€Π°ΠΌΠΈ – это ΡƒΠ³ΠΎΠ» \(\angle BAC\), Π½Π΅ ΠΏΡ€Π΅Π²Ρ‹ΡˆΠ°ΡŽΡ‰ΠΈΠΉ Ρ€Π°Π·Π²Π΅Ρ€Π½ΡƒΡ‚ΠΎΠ³ΠΎ ΡƒΠ³Π»Π°.


Β 

БкалярноС ΠΏΡ€ΠΎΠΈΠ·Π²Π΅Π΄Π΅Π½ΠΈΠ΅ Π²Π΅ΠΊΡ‚ΠΎΡ€ΠΎΠ² \(\overrightarrow a\) ΠΈ \(\overrightarrow b\) – это число, Ρ€Π°Π²Π½ΠΎΠ΅ ΠΏΡ€ΠΎΠΈΠ·Π²Π΅Π΄Π΅Π½ΠΈΡŽ Π΄Π»ΠΈΠ½ этих Π²Π΅ΠΊΡ‚ΠΎΡ€ΠΎΠ² Π½Π° косинус ΡƒΠ³Π»Π° ΠΌΠ΅ΠΆΠ΄Ρƒ Π½ΠΈΠΌΠΈ.
ΠžΠ±ΠΎΠ·Π½Π°Ρ‡Π΅Π½ΠΈΠ΅: \(\overrightarrow a\cdot \overrightarrow b\) ΠΈΠ»ΠΈ \((\overrightarrow a, \overrightarrow b)\). \[(\overrightarrow a, \overrightarrow b)=|\overrightarrow a|\cdot |\overrightarrow b|\cdot \cos\widehat{(\overrightarrow a, \overrightarrow b)}\]

БлСдствия

1. Если Π½Π΅Π½ΡƒΠ»Π΅Π²Ρ‹Π΅ Π²Π΅ΠΊΡ‚ΠΎΡ€Ρ‹ Π²Π·Π°ΠΈΠΌΠ½ΠΎ пСрпСндикулярны, Ρ‚ΠΎ косинус ΡƒΠ³Π»Π° ΠΌΠ΅ΠΆΠ΄Ρƒ Π½ΠΈΠΌΠΈ Ρ€Π°Π²Π΅Π½ Π½ΡƒΠ»ΡŽ, ΡΠ»Π΅Π΄ΠΎΠ²Π°Ρ‚Π΅Π»ΡŒΠ½ΠΎ, ΠΈ ΠΈΡ… скалярноС ΠΏΡ€ΠΎΠΈΠ·Π²Π΅Π΄Π΅Π½ΠΈΠ΅ Ρ€Π°Π²Π½ΠΎ Π½ΡƒΠ»ΡŽ.

Β 

2. Если ΡƒΠ³ΠΎΠ» ΠΌΠ΅ΠΆΠ΄Ρƒ Π½Π΅Π½ΡƒΠ»Π΅Π²Ρ‹ΠΌΠΈ Π²Π΅ΠΊΡ‚ΠΎΡ€Π°ΠΌΠΈ острый, Ρ‚ΠΎ скалярноС ΠΏΡ€ΠΎΠΈΠ·Π²Π΅Π΄Π΅Π½ΠΈΠ΅ ΠΏΠΎΠ»ΠΎΠΆΠΈΡ‚Π΅Π»ΡŒΠ½ΠΎ. 2=0 \Leftrightarrow |\overrightarrow a|=0\).

Β 

2. ΠŸΠ΅Ρ€Π΅ΠΌΠ΅ΡΡ‚ΠΈΡ‚Π΅Π»ΡŒΠ½Ρ‹ΠΉ Π·Π°ΠΊΠΎΠ½: \(\overrightarrow a\cdot \overrightarrow b=\overrightarrow b\cdot \overrightarrow a\).

Β 

3. Π Π°ΡΠΏΡ€Π΅Π΄Π΅Π»ΠΈΡ‚Π΅Π»ΡŒΠ½Ρ‹ΠΉ Π·Π°ΠΊΠΎΠ½: \(\overrightarrow a \cdot (\overrightarrow b+\overrightarrow c)=\overrightarrow a\cdot \overrightarrow b+\overrightarrow a\cdot \overrightarrow c\).

Β 

4. Π‘ΠΎΡ‡Π΅Ρ‚Π°Ρ‚Π΅Π»ΡŒΠ½Ρ‹ΠΉ Π·Π°ΠΊΠΎΠ½: \((\lambda\overrightarrow a)\cdot \overrightarrow b=\lambda (\overrightarrow a\cdot \overrightarrow b)\).

Как Π½Π°ΠΉΡ‚ΠΈ ΠΊΠΎΠΎΡ€Π΄ΠΈΠ½Π°Ρ‚Ρ‹ Π²Π΅ΠΊΡ‚ΠΎΡ€Π° Π² Ρ‚Ρ€Π΅Ρ…ΠΌΠ΅Ρ€Π½ΠΎΠΌ пространствС

Π•Ρ‰Π΅ ΠΈΠ· школьного курса Π°Π»Π³Π΅Π±Ρ€Ρ‹ ΠΈ Π³Π΅ΠΎΠΌΠ΅Ρ‚Ρ€ΠΈΠΈ ΠΌΡ‹ Π·Π½Π°Π΅ΠΌ ΠΎ понятии Ρ‚Ρ€Π΅Ρ…ΠΌΠ΅Ρ€Π½ΠΎΠ³ΠΎ пространства. Если Ρ€Π°Π·ΠΎΠ±Ρ€Π°Ρ‚ΡŒΡΡ, сам Ρ‚Π΅Ρ€ΠΌΠΈΠ½ Β«Ρ‚Ρ€Π΅Ρ…ΠΌΠ΅Ρ€Π½ΠΎΠ΅ пространство» опрСдСляСтся ΠΊΠ°ΠΊ систСма ΠΊΠΎΠΎΡ€Π΄ΠΈΠ½Π°Ρ‚ с трСмя измСрСниями (это Π·Π½Π°ΡŽΡ‚ всС). По сути, ΠΎΠΏΠΈΡΠ°Ρ‚ΡŒ любой ΠΎΠ±ΡŠΠ΅ΠΌΠ½Ρ‹ΠΉ ΠΎΠ±ΡŠΠ΅ΠΊΡ‚ ΠΌΠΎΠΆΠ½ΠΎ ΠΏΡ€ΠΈ ΠΏΠΎΠΌΠΎΡ‰ΠΈ Π΄Π»ΠΈΠ½Ρ‹, ΡˆΠΈΡ€ΠΈΠ½Ρ‹ ΠΈ высоты Π² классичСском ΠΏΠΎΠ½ΠΈΠΌΠ°Π½ΠΈΠΈ. Однако Π΄Π°Π²Π°ΠΉΡ‚Π΅, ΠΊΠ°ΠΊ говорится, ΠΊΠΎΠΏΠ½Π΅ΠΌ нСсколько Π³Π»ΡƒΠ±ΠΆΠ΅.

Π§Ρ‚ΠΎ Ρ‚Π°ΠΊΠΎΠ΅ Ρ‚Ρ€Π΅Ρ…ΠΌΠ΅Ρ€Π½ΠΎΠ΅ пространство

Как ΡƒΠΆΠ΅ стало ясно, ΠΏΠΎΠ½ΠΈΠΌΠ°Π½ΠΈΠ΅ Ρ‚Ρ€Π΅Ρ…ΠΌΠ΅Ρ€Π½ΠΎΠ³ΠΎ пространства ΠΈ ΠΎΠ±ΡŠΠ΅ΠΊΡ‚ΠΎΠ², способных ΡΡƒΡ‰Π΅ΡΡ‚Π²ΠΎΠ²Π°Ρ‚ΡŒ Π²Π½ΡƒΡ‚Ρ€ΠΈ Π½Π΅Π³ΠΎ, опрСдСляСтся трСмя основными понятиями. ΠŸΡ€Π°Π²Π΄Π°, Π² случаС с Ρ‚ΠΎΡ‡ΠΊΠΎΠΉ это ΠΈΠΌΠ΅Π½Π½ΠΎ Ρ‚Ρ€ΠΈ значСния, Π° Π² случаС с прямыми, ΠΊΡ€ΠΈΠ²Ρ‹ΠΌΠΈ, Π»ΠΎΠΌΠ°Π½Ρ‹ΠΌΠΈ линиями ΠΈΠ»ΠΈ ΠΎΠ±ΡŠΠ΅ΠΌΠ½Ρ‹ΠΌΠΈ ΠΎΠ±ΡŠΠ΅ΠΊΡ‚Π°ΠΌΠΈ ΡΠΎΠΎΡ‚Π²Π΅Ρ‚ΡΡ‚Π²ΡƒΡŽΡ‰ΠΈΡ… ΠΊΠΎΠΎΡ€Π΄ΠΈΠ½Π°Ρ‚ ΠΌΠΎΠΆΠ΅Ρ‚ Π±Ρ‹Ρ‚ΡŒ большС.

Π’ Π΄Π°Π½Π½ΠΎΠΌ случаС всС зависит ΠΈΠΌΠ΅Π½Π½ΠΎ ΠΎΡ‚ Ρ‚ΠΈΠΏΠ° ΠΎΠ±ΡŠΠ΅ΠΊΡ‚Π° ΠΈ примСняСмой систСмы ΠΊΠΎΠΎΡ€Π΄ΠΈΠ½Π°Ρ‚. БСгодня Π½Π°ΠΈΠ±ΠΎΠ»Π΅Π΅ распространСнной (классичСской) считаСтся Π”Π΅ΠΊΠ°Ρ€Ρ‚ΠΎΠ²Π° систСма, ΠΊΠΎΡ‚ΠΎΡ€ΡƒΡŽ ΠΈΠ½ΠΎΠ³Π΄Π° Π΅Ρ‰Π΅ Π½Π°Π·Ρ‹Π²Π°ΡŽΡ‚ ΠΏΡ€ΡΠΌΠΎΡƒΠ³ΠΎΠ»ΡŒΠ½ΠΎΠΉ. Она ΠΈ Π½Π΅ΠΊΠΎΡ‚ΠΎΡ€Ρ‹Π΅ Π΄Ρ€ΡƒΠ³ΠΈΠ΅ разновидности Π±ΡƒΠ΄ΡƒΡ‚ рассмотрСны нСсколько ΠΏΠΎΠ·ΠΆΠ΅.

ΠšΡ€ΠΎΠΌΠ΅ всСго ΠΏΡ€ΠΎΡ‡Π΅Π³ΠΎ, здСсь Π½ΡƒΠΆΠ½ΠΎ Ρ€Π°Π·Π³Ρ€Π°Π½ΠΈΡ‡ΠΈΠ²Π°Ρ‚ΡŒ абстрактныС понятия (Ссли ΠΌΠΎΠΆΠ½ΠΎ Ρ‚Π°ΠΊ ΡΠΊΠ°Π·Π°Ρ‚ΡŒ, бСсформСнныС) Π²Ρ€ΠΎΠ΄Π΅ Ρ‚ΠΎΡ‡Π΅ΠΊ, прямых ΠΈΠ»ΠΈ плоскостСй ΠΈ Ρ„ΠΈΠ³ΡƒΡ€Ρ‹, ΠΎΠ±Π»Π°Π΄Π°ΡŽΡ‰ΠΈΠ΅ ΠΊΠΎΠ½Π΅Ρ‡Π½Ρ‹ΠΌΠΈ Ρ€Π°Π·ΠΌΠ΅Ρ€Π°ΠΌΠΈ ΠΈΠ»ΠΈ Π΄Π°ΠΆΠ΅ объСмом. Для ΠΊΠ°ΠΆΠ΄ΠΎΠ³ΠΎ ΠΈΠ· Ρ‚Π°ΠΊΠΈΡ… ΠΎΠΏΡ€Π΅Π΄Π΅Π»Π΅Π½ΠΈΠΉ ΡΡƒΡ‰Π΅ΡΡ‚Π²ΡƒΡŽΡ‚ ΠΈ свои уравнСния, ΠΎΠΏΠΈΡΡ‹Π²Π°ΡŽΡ‰ΠΈΠ΅ ΠΈΡ… Π²ΠΎΠ·ΠΌΠΎΠΆΠ½ΠΎΠ΅ ΠΏΠΎΠ»ΠΎΠΆΠ΅Π½ΠΈΠ΅ Π² Ρ‚Ρ€Π΅Ρ…ΠΌΠ΅Ρ€Π½ΠΎΠΌ пространствС. Но сСйчас Π½Π΅ ΠΎΠ± этом.

ΠŸΠΎΠ½ΡΡ‚ΠΈΠ΅ Ρ‚ΠΎΡ‡ΠΊΠΈ Π² Ρ‚Ρ€Π΅Ρ…ΠΌΠ΅Ρ€Π½ΠΎΠΌ пространствС

Для Π½Π°Ρ‡Π°Π»Π° опрСдСлимся, Ρ‡Ρ‚ΠΎ прСдставляСт собой Ρ‚ΠΎΡ‡ΠΊΠ° Π² Ρ‚Ρ€Π΅Ρ…ΠΌΠ΅Ρ€Π½ΠΎΠΌ пространствС. Π’ ΠΎΠ±Ρ‰Π΅ΠΌ-Ρ‚ΠΎ, Π΅Π΅ ΠΌΠΎΠΆΠ½ΠΎ Π½Π°Π·Π²Π°Ρ‚ΡŒ Π½Π΅ΠΊΠΎΠΉ основной Π΅Π΄ΠΈΠ½ΠΈΡ†Π΅ΠΉ, ΠΎΠΏΡ€Π΅Π΄Π΅Π»ΡΡŽΡ‰Π΅ΠΉ Π»ΡŽΠ±ΡƒΡŽ ΠΏΠ»ΠΎΡΠΊΡƒΡŽ ΠΈΠ»ΠΈ ΠΎΠ±ΡŠΠ΅ΠΌΠ½ΡƒΡŽ Ρ„ΠΈΠ³ΡƒΡ€Ρƒ, ΠΏΡ€ΡΠΌΡƒΡŽ, ΠΎΡ‚Ρ€Π΅Π·ΠΎΠΊ, Π²Π΅ΠΊΡ‚ΠΎΡ€, ΠΏΠ»ΠΎΡΠΊΠΎΡΡ‚ΡŒ ΠΈ Ρ‚. Π΄.

Π‘Π°ΠΌΠ° ΠΆΠ΅ Ρ‚ΠΎΡ‡ΠΊΠ° характСризуСтся трСмя основными ΠΊΠΎΠΎΡ€Π΄ΠΈΠ½Π°Ρ‚Π°ΠΌΠΈ. Для Π½ΠΈΡ… Π² ΠΏΡ€ΡΠΌΠΎΡƒΠ³ΠΎΠ»ΡŒΠ½ΠΎΠΉ систСмС ΠΏΡ€ΠΈΠΌΠ΅Π½ΡΡŽΡ‚ΡΡ ΡΠΏΠ΅Ρ†ΠΈΠ°Π»ΡŒΠ½Ρ‹Π΅ Π½Π°ΠΏΡ€Π°Π²Π»ΡΡŽΡ‰ΠΈΠ΅, Π½Π°Π·Ρ‹Π²Π°Π΅ΠΌΡ‹Π΅ осями X, Y ΠΈ Z, ΠΏΡ€ΠΈΡ‡Π΅ΠΌ ΠΏΠ΅Ρ€Π²Ρ‹Π΅ Π΄Π²Π΅ оси слуТат для выраТСния Π³ΠΎΡ€ΠΈΠ·ΠΎΠ½Ρ‚Π°Π»ΡŒΠ½ΠΎΠ³ΠΎ полоТСния ΠΎΠ±ΡŠΠ΅ΠΊΡ‚Π°, Π° Ρ‚Ρ€Π΅Ρ‚ΡŒΡ относится ΠΊ Π²Π΅Ρ€Ρ‚ΠΈΠΊΠ°Π»ΡŒΠ½ΠΎΠΌΡƒ заданию ΠΊΠΎΠΎΡ€Π΄ΠΈΠ½Π°Ρ‚. ЕстСствСнно, для удобства выраТСния полоТСния ΠΎΠ±ΡŠΠ΅ΠΊΡ‚Π° ΠΎΡ‚Π½ΠΎΡΠΈΡ‚Π΅Π»ΡŒΠ½ΠΎ Π½ΡƒΠ»Π΅Π²Ρ‹Ρ… ΠΊΠΎΠΎΡ€Π΄ΠΈΠ½Π°Ρ‚ Π² систСмС приняты ΠΏΠΎΠ»ΠΎΠΆΠΈΡ‚Π΅Π»ΡŒΠ½Ρ‹Π΅ ΠΈ ΠΎΡ‚Ρ€ΠΈΡ†Π°Ρ‚Π΅Π»ΡŒΠ½Ρ‹Π΅ значСния. Однако ΠΆΠ΅ сСгодня ΠΌΠΎΠΆΠ½ΠΎ Π½Π°ΠΉΡ‚ΠΈ ΠΈ Π΄Ρ€ΡƒΠ³ΠΈΠ΅ систСмы.

Разновидности систСм ΠΊΠΎΠΎΡ€Π΄ΠΈΠ½Π°Ρ‚

Как ΡƒΠΆΠ΅ Π³ΠΎΠ²ΠΎΡ€ΠΈΠ»ΠΎΡΡŒ, ΠΏΡ€ΡΠΌΠΎΡƒΠ³ΠΎΠ»ΡŒΠ½Π°Ρ систСма ΠΊΠΎΠΎΡ€Π΄ΠΈΠ½Π°Ρ‚, созданная Π”Π΅ΠΊΠ°Ρ€Ρ‚ΠΎΠΌ, сСгодня являСтся основной. Π’Π΅ΠΌ Π½Π΅ ΠΌΠ΅Π½Π΅Π΅ Π² Π½Π΅ΠΊΠΎΡ‚ΠΎΡ€Ρ‹Ρ… ΠΌΠ΅Ρ‚ΠΎΠ΄ΠΈΠΊΠ°Ρ… задания мСстополоТСния ΠΎΠ±ΡŠΠ΅ΠΊΡ‚Π° Π² Ρ‚Ρ€Π΅Ρ…ΠΌΠ΅Ρ€Π½ΠΎΠΌ пространствС ΠΏΡ€ΠΈΠΌΠ΅Π½ΡΡŽΡ‚ΡΡ ΠΈ Π½Π΅ΠΊΠΎΡ‚ΠΎΡ€Ρ‹Π΅ Π΄Ρ€ΡƒΠ³ΠΈΠ΅ разновидности.

НаиболСС извСстными ΡΡ‡ΠΈΡ‚Π°ΡŽΡ‚ΡΡ цилиндричСская ΠΈ сфСричСская систСмы. ΠžΡ‚Π»ΠΈΡ‡ΠΈΠ΅ ΠΎΡ‚ классичСской состоит Π² Ρ‚ΠΎΠΌ, Ρ‡Ρ‚ΠΎ ΠΏΡ€ΠΈ Π·Π°Π΄Π°Π½ΠΈΠΈ Ρ‚Π΅Ρ… ΠΆΠ΅ Ρ‚Ρ€Π΅Ρ… Π²Π΅Π»ΠΈΡ‡ΠΈΠ½, ΠΎΠΏΡ€Π΅Π΄Π΅Π»ΡΡŽΡ‰ΠΈΡ… мСстополоТСниС Ρ‚ΠΎΡ‡ΠΊΠΈ Π² Ρ‚Ρ€Π΅Ρ…ΠΌΠ΅Ρ€Π½ΠΎΠΌ пространствС, ΠΎΠ΄Π½ΠΎ ΠΈΠ· Π·Π½Π°Ρ‡Π΅Π½ΠΈΠΉ являСтся ΡƒΠ³Π»ΠΎΠ²Ρ‹ΠΌ. Π˜Π½Ρ‹ΠΌΠΈ словами, Π² Ρ‚Π°ΠΊΠΈΡ… систСмах ΠΈΡΠΏΠΎΠ»ΡŒΠ·ΡƒΠ΅Ρ‚ΡΡ ΠΎΠΊΡ€ΡƒΠΆΠ½ΠΎΡΡ‚ΡŒ, ΡΠΎΠΎΡ‚Π²Π΅Ρ‚ΡΡ‚Π²ΡƒΡŽΡ‰Π°Ρ ΡƒΠ³Π»Ρƒ Π² 360 градусов. ΠžΡ‚ΡΡŽΠ΄Π° ΠΈ спСцифичноС Π·Π°Π΄Π°Π½ΠΈΠ΅ ΠΊΠΎΠΎΡ€Π΄ΠΈΠ½Π°Ρ‚, Π²ΠΊΠ»ΡŽΡ‡Π°ΡŽΡ‰Π΅Π΅ Ρ‚Π°ΠΊΠΈΠ΅ элСмСнты, ΠΊΠ°ΠΊ радиус, ΡƒΠ³ΠΎΠ» ΠΈ ΠΎΠ±Ρ€Π°Π·ΡƒΡŽΡ‰Π°Ρ. ΠšΠΎΠΎΡ€Π΄ΠΈΠ½Π°Ρ‚Ρ‹ Π² Ρ‚Ρ€Π΅Ρ…ΠΌΠ΅Ρ€Π½ΠΎΠΌ пространствС (систСмС) Ρ‚Π°ΠΊΠΎΠ³ΠΎ Ρ‚ΠΈΠΏΠ° ΠΏΠΎΠ΄Ρ‡ΠΈΠ½ΡΡŽΡ‚ΡΡ нСсколько Π΄Ρ€ΡƒΠ³ΠΈΠΌ закономСрностям. Π˜Ρ… Π·Π°Π΄Π°Π½ΠΈΠ΅ Π² Π΄Π°Π½Π½ΠΎΠΌ случаС контролируСтся ΠΏΡ€Π°Π²ΠΈΠ»ΠΎΠΌ ΠΏΡ€Π°Π²ΠΎΠΉ Ρ€ΡƒΠΊΠΈ: Ссли ΡΠΎΠ²ΠΌΠ΅ΡΡ‚ΠΈΡ‚ΡŒ большой ΠΈ ΡƒΠΊΠ°Π·Π°Ρ‚Π΅Π»ΡŒΠ½Ρ‹ΠΉ ΠΏΠ°Π»Π΅Ρ† с осями X ΠΈ Y, соотвСтствСнно, ΠΎΡΡ‚Π°Π»ΡŒΠ½Ρ‹Π΅ ΠΏΠ°Π»ΡŒΡ†Ρ‹ Π² ΠΈΠ·ΠΎΠ³Π½ΡƒΡ‚ΠΎΠΌ ΠΏΠΎΠ»ΠΎΠΆΠ΅Π½ΠΈΠΈ ΡƒΠΊΠ°ΠΆΡƒΡ‚ Π½Π° Π½Π°ΠΏΡ€Π°Π²Π»Π΅Π½ΠΈΠ΅ оси Z.

ΠŸΠΎΠ½ΡΡ‚ΠΈΠ΅ прямой Π² Ρ‚Ρ€Π΅Ρ…ΠΌΠ΅Ρ€Π½ΠΎΠΌ пространствС

Π’Π΅ΠΏΠ΅Ρ€ΡŒ нСсколько слов ΠΎ Ρ‚ΠΎΠΌ, Ρ‡Ρ‚ΠΎ прСдставляСт собой прямая Π² Ρ‚Ρ€Π΅Ρ…ΠΌΠ΅Ρ€Π½ΠΎΠΌ пространствС. Π˜ΡΡ…ΠΎΠ΄Ρ ΠΈΠ· основного понятия прямой, это нСкая бСсконСчная линия, провСдСнная Ρ‡Π΅Ρ€Π΅Π· Ρ‚ΠΎΡ‡ΠΊΡƒ ΠΈΠ»ΠΈ Π΄Π²Π΅, Π½Π΅ считая мноТСства Ρ‚ΠΎΡ‡Π΅ΠΊ, располоТСнных Π² ΠΏΠΎΡΠ»Π΅Π΄ΠΎΠ²Π°Ρ‚Π΅Π»ΡŒΠ½ΠΎΡΡ‚ΠΈ, Π½Π΅ ΠΈΠ·ΠΌΠ΅Π½ΡΡŽΡ‰Π΅ΠΉ прямоС ΠΏΡ€ΠΎΡ…ΠΎΠΆΠ΄Π΅Π½ΠΈΠ΅ Π»ΠΈΠ½ΠΈΠΈ Ρ‡Π΅Ρ€Π΅Π· Π½ΠΈΡ….

Если ΠΏΠΎΡΠΌΠΎΡ‚Ρ€Π΅Ρ‚ΡŒ Π½Π° ΠΏΡ€ΡΠΌΡƒΡŽ, ΠΏΡ€ΠΎΠ²Π΅Π΄Π΅Π½Π½ΡƒΡŽ Ρ‡Π΅Ρ€Π΅Π· Π΄Π²Π΅ Ρ‚ΠΎΡ‡ΠΊΠΈ Π² Ρ‚Ρ€Π΅Ρ…ΠΌΠ΅Ρ€Π½ΠΎΠΌ пространствС, придСтся ΡƒΡ‡ΠΈΡ‚Ρ‹Π²Π°Ρ‚ΡŒ ΠΏΠΎ Ρ‚Ρ€ΠΈ ΠΊΠΎΠΎΡ€Π΄ΠΈΠ½Π°Ρ‚Ρ‹ ΠΎΠ±Π΅ΠΈΡ… Ρ‚ΠΎΡ‡Π΅ΠΊ. Π’ΠΎ ΠΆΠ΅ самоС относится ΠΊ ΠΎΡ‚Ρ€Π΅Π·ΠΊΠ°ΠΌ ΠΈ Π²Π΅ΠΊΡ‚ΠΎΡ€Π°ΠΌ. ПослСдниС ΠΎΠΏΡ€Π΅Π΄Π΅Π»ΡΡŽΡ‚ базис Ρ‚Ρ€Π΅Ρ…ΠΌΠ΅Ρ€Π½ΠΎΠ³ΠΎ пространства ΠΈ Π΅Π³ΠΎ Ρ€Π°Π·ΠΌΠ΅Ρ€Π½ΠΎΡΡ‚ΡŒ.

ΠžΠΏΡ€Π΅Π΄Π΅Π»Π΅Π½ΠΈΠ΅ Π²Π΅ΠΊΡ‚ΠΎΡ€ΠΎΠ² ΠΈ базиса Ρ‚Ρ€Π΅Ρ…ΠΌΠ΅Ρ€Π½ΠΎΠ³ΠΎ пространства

Как принято ΡΡ‡ΠΈΡ‚Π°Ρ‚ΡŒ, Π² Ρ‚Ρ€Π΅Ρ…ΠΌΠ΅Ρ€Π½ΠΎΠΉ систСмС ΠΊΠΎΠΎΡ€Π΄ΠΈΠ½Π°Ρ‚ ΠΌΠΎΠΆΠ΅Ρ‚ ΡΡƒΡ‰Π΅ΡΡ‚Π²ΠΎΠ²Π°Ρ‚ΡŒ Ρ‚Ρ€ΠΈ основных Π²Π΅ΠΊΡ‚ΠΎΡ€Π°, ΠΊΠΎΡ‚ΠΎΡ€Ρ‹Π΅ ΠΎΠΏΡ€Π΅Π΄Π΅Π»ΡΡŽΡ‚ базис. ΠŸΡ€ΠΈ этом базисов с ΡΠΎΠΎΡ‚Π²Π΅Ρ‚ΡΡ‚Π²ΡƒΡŽΡ‰ΠΈΠΌΠΈ нСзависимыми трСмя Π²Π΅ΠΊΡ‚ΠΎΡ€Π°ΠΌΠΈ ΠΌΠΎΠΆΠ΅Ρ‚ Π±Ρ‹Ρ‚ΡŒ бСсчислСнноС мноТСство.

Π—Π°ΠΌΠ΅Ρ‚ΡŒΡ‚Π΅, это ΠΌΠΎΠ³ΡƒΡ‚ Π±Ρ‹Ρ‚ΡŒ Ρ‚ΠΎΠ»ΡŒΠΊΠΎ Ρ‚Ρ€ΠΈ Π²Π΅ΠΊΡ‚ΠΎΡ€Π°, Π½ΠΎ Π²ΠΎΡ‚ Ρ‚Ρ€ΠΎΠ΅ΠΊ Π²Π΅ΠΊΡ‚ΠΎΡ€ΠΎΠ² ΠΌΠΎΠΆΠ½ΠΎ ΠΎΠΏΡ€Π΅Π΄Π΅Π»ΠΈΡ‚ΡŒ сколько ΡƒΠ³ΠΎΠ΄Π½ΠΎ. Π Π°Π·ΠΌΠ΅Ρ€Π½ΠΎΡΡ‚ΡŒ пространства опрСдСляСтся количСством Π»ΠΈΠ½Π΅ΠΉΠ½ΠΎ-нСзависимых Π²Π΅ΠΊΡ‚ΠΎΡ€ΠΎΠ² (Π² нашСм случаС – Ρ‚Ρ€ΠΈ). И пространство, Π² ΠΊΠΎΡ‚ΠΎΡ€ΠΎΠΌ имССтся ΠΊΠΎΠ½Π΅Ρ‡Π½ΠΎΠ΅ число Ρ‚Π°ΠΊΠΈΡ… Π²Π΅ΠΊΡ‚ΠΎΡ€ΠΎΠ², называСтся ΠΊΠΎΠ½Π΅Ρ‡Π½ΠΎΠΌΠ΅Ρ€Π½Ρ‹ΠΌ.

ЗависимыС ΠΈ нСзависимыС Π²Π΅ΠΊΡ‚ΠΎΡ€Ρ‹

Π§Ρ‚ΠΎ касаСтся опрСдСлСния зависимых ΠΈ нСзависимых Π²Π΅ΠΊΡ‚ΠΎΡ€ΠΎΠ², Π»ΠΈΠ½Π΅ΠΉΠ½ΠΎ-нСзависимыми принято ΡΡ‡ΠΈΡ‚Π°Ρ‚ΡŒ Π²Π΅ΠΊΡ‚ΠΎΡ€Ρ‹, ΡΠ²Π»ΡΡŽΡ‰ΠΈΠ΅ΡΡ проСкциями (Π½Π°ΠΏΡ€ΠΈΠΌΠ΅Ρ€, Π²Π΅ΠΊΡ‚ΠΎΡ€Ρ‹ оси X, спроСцированныС Π½Π° ось Y).

Как ΡƒΠΆΠ΅ понятно, любой Ρ‡Π΅Ρ‚Π²Π΅Ρ€Ρ‚Ρ‹ΠΉ Π²Π΅ΠΊΡ‚ΠΎΡ€ являСтся зависимым (тСория Π»ΠΈΠ½Π΅ΠΉΠ½Ρ‹Ρ… пространств). А Π²ΠΎΡ‚ Ρ‚Ρ€ΠΈ нСзависимых Π²Π΅ΠΊΡ‚ΠΎΡ€Π° Π² Ρ‚Ρ€Π΅Ρ…ΠΌΠ΅Ρ€Π½ΠΎΠΌ пространствС Π² ΠΎΠ±ΡΠ·Π°Ρ‚Π΅Π»ΡŒΠ½ΠΎΠΌ порядкС Π½Π΅ Π΄ΠΎΠ»ΠΆΠ½Ρ‹ Π»Π΅ΠΆΠ°Ρ‚ΡŒ Π² ΠΎΠ΄Π½ΠΎΠΉ плоскости. ΠšΡ€ΠΎΠΌΠ΅ Ρ‚ΠΎΠ³ΠΎ, Ссли ΠΎΠΏΡ€Π΅Π΄Π΅Π»ΡΡ‚ΡŒ нСзависимыС Π²Π΅ΠΊΡ‚ΠΎΡ€Ρ‹ Π² Ρ‚Ρ€Π΅Ρ…ΠΌΠ΅Ρ€Π½ΠΎΠΌ пространствС, ΠΎΠ½ΠΈ Π½Π΅ ΠΌΠΎΠ³ΡƒΡ‚ ΡΠ²Π»ΡΡ‚ΡŒΡΡ, Ρ‚Π°ΠΊ ΡΠΊΠ°Π·Π°Ρ‚ΡŒ, ΠΎΠ΄ΠΈΠ½ ΠΏΡ€ΠΎΠ΄ΠΎΠ»ΠΆΠ΅Π½ΠΈΠ΅ΠΌ Π΄Ρ€ΡƒΠ³ΠΎΠ³ΠΎ. Как ΡƒΠΆΠ΅ понятно, Π² рассматриваСмом Π½Π°ΠΌΠΈ случаС с трСмя измСрСниями, согласно ΠΎΠ±Ρ‰Π΅ΠΉ Ρ‚Π΅ΠΎΡ€ΠΈΠΈ, ΠΌΠΎΠΆΠ½ΠΎ ΠΏΠΎΡΡ‚Ρ€ΠΎΠΈΡ‚ΡŒ ΠΈΡΠΊΠ»ΡŽΡ‡ΠΈΡ‚Π΅Π»ΡŒΠ½ΠΎ Ρ‚ΠΎΠ»ΡŒΠΊΠΎ Ρ‚Ρ€ΠΎΠΉΠΊΠΈ Π»ΠΈΠ½Π΅ΠΉΠ½ΠΎ-нСзависимых Π²Π΅ΠΊΡ‚ΠΎΡ€ΠΎΠ² Π² ΠΎΠΏΡ€Π΅Π΄Π΅Π»Π΅Π½Π½ΠΎΠΉ систСмС ΠΊΠΎΠΎΡ€Π΄ΠΈΠ½Π°Ρ‚ (Π±Π΅Π· Ρ€Π°Π·Π½ΠΈΡ†Ρ‹, ΠΊΠ°ΠΊΠΎΠ³ΠΎ Ρ‚ΠΈΠΏΠ°).

ΠŸΠ»ΠΎΡΠΊΠΎΡΡ‚ΡŒ Π² Ρ‚Ρ€Π΅Ρ…ΠΌΠ΅Ρ€Π½ΠΎΠΌ пространствС

Если Ρ€Π°ΡΡΠΌΠ°Ρ‚Ρ€ΠΈΠ²Π°Ρ‚ΡŒ понятиС плоскости, Π½Π΅ вдаваясь Π² матСматичСскиС опрСдСлСния, для Π±ΠΎΠ»Π΅Π΅ простого понимания этого Ρ‚Π΅Ρ€ΠΌΠΈΠ½Π°, Ρ‚Π°ΠΊΠΎΠΉ ΠΎΠ±ΡŠΠ΅ΠΊΡ‚ ΠΌΠΎΠΆΠ½ΠΎ Ρ€Π°ΡΡΠΌΠ°Ρ‚Ρ€ΠΈΠ²Π°Ρ‚ΡŒ ΠΈΡΠΊΠ»ΡŽΡ‡ΠΈΡ‚Π΅Π»ΡŒΠ½ΠΎ ΠΊΠ°ΠΊ Π΄Π²ΡƒΠΌΠ΅Ρ€Π½Ρ‹ΠΉ. Π˜Π½Ρ‹ΠΌΠΈ словами, это бСсконСчная ΡΠΎΠ²ΠΎΠΊΡƒΠΏΠ½ΠΎΡΡ‚ΡŒ Ρ‚ΠΎΡ‡Π΅ΠΊ, Ρƒ ΠΊΠΎΡ‚ΠΎΡ€Ρ‹Ρ… ΠΎΠ΄Π½Π° ΠΈΠ· ΠΊΠΎΠΎΡ€Π΄ΠΈΠ½Π°Ρ‚ являСтся постоянной (константой).

К ΠΏΡ€ΠΈΠΌΠ΅Ρ€Ρƒ, ΠΏΠ»ΠΎΡΠΊΠΎΡΡ‚ΡŒΡŽ ΠΌΠΎΠΆΠ½ΠΎ Π½Π°Π·Π²Π°Ρ‚ΡŒ любоС количСство Ρ‚ΠΎΡ‡Π΅ΠΊ с Ρ€Π°Π·Π½Ρ‹ΠΌΠΈ ΠΊΠΎΠΎΡ€Π΄ΠΈΠ½Π°Ρ‚Π°ΠΌΠΈ ΠΏΠΎ осям X ΠΈ Y, Π½ΠΎ ΠΎΠ΄ΠΈΠ½Π°ΠΊΠΎΠ²Ρ‹ΠΌΠΈ ΠΊΠΎΠΎΡ€Π΄ΠΈΠ½Π°Ρ‚Π°ΠΌΠΈ ΠΏΠΎ оси Z. Π’ любом случаС ΠΎΠ΄Π½Π° ΠΈΠ· Ρ‚Ρ€Π΅Ρ…ΠΌΠ΅Ρ€Π½Ρ‹Ρ… ΠΊΠΎΠΎΡ€Π΄ΠΈΠ½Π°Ρ‚ остаСтся Π½Π΅ΠΈΠ·ΠΌΠ΅Π½Π½ΠΎΠΉ. Однако это, Ρ‚Π°ΠΊ ΡΠΊΠ°Π·Π°Ρ‚ΡŒ, ΠΎΠ±Ρ‰ΠΈΠΉ случай. Π’ Π½Π΅ΠΊΠΎΡ‚ΠΎΡ€Ρ‹Ρ… ситуациях Ρ‚Ρ€Π΅Ρ…ΠΌΠ΅Ρ€Π½ΠΎΠ΅ пространство ΠΌΠΎΠΆΠ΅Ρ‚ ΠΏΠ΅Ρ€Π΅ΡΠ΅ΠΊΠ°Ρ‚ΡŒΡΡ ΠΏΠ»ΠΎΡΠΊΠΎΡΡ‚ΡŒΡŽ ΠΏΠΎ всСм осям.

БущСствуСт Π»ΠΈ Π±ΠΎΠ»Π΅Π΅ Ρ‚Ρ€Π΅Ρ… ΠΈΠ·ΠΌΠ΅Ρ€Π΅Π½ΠΈΠΉ

Вопрос ΠΎ Ρ‚ΠΎΠΌ, сколько ΠΌΠΎΠΆΠ΅Ρ‚ ΡΡƒΡ‰Π΅ΡΡ‚Π²ΠΎΠ²Π°Ρ‚ΡŒ ΠΈΠ·ΠΌΠ΅Ρ€Π΅Π½ΠΈΠΉ, достаточно интСрСсСн. Как считаСтся, ΠΌΡ‹ ΠΆΠΈΠ²Π΅ΠΌ Π½Π΅ Π² Ρ‚Ρ€Π΅Ρ…ΠΌΠ΅Ρ€Π½ΠΎΠΌ с классичСской Ρ‚ΠΎΡ‡ΠΊΠΈ зрСния пространствС, Π° Π² Ρ‡Π΅Ρ‚Ρ‹Ρ€Π΅Ρ…ΠΌΠ΅Ρ€Π½ΠΎΠΌ. ΠšΡ€ΠΎΠΌΠ΅ извСстных всСм Π΄Π»ΠΈΠ½Ρ‹, ΡˆΠΈΡ€ΠΈΠ½Ρ‹ ΠΈ высоты, Ρ‚Π°ΠΊΠΎΠ΅ пространство Π²ΠΊΠ»ΡŽΡ‡Π°Π΅Ρ‚ Π² сСбя Π΅Ρ‰Π΅ ΠΈ врСмя сущСствования ΠΎΠ±ΡŠΠ΅ΠΊΡ‚Π°, ΠΏΡ€ΠΈΡ‡Π΅ΠΌ врСмя ΠΈ пространство ΠΌΠ΅ΠΆΠ΄Ρƒ собой взаимосвязаны достаточно сильно. Π­Ρ‚ΠΎ Π΄ΠΎΠΊΠ°Π·Π°Π» Π΅Ρ‰Π΅ Π­ΠΉΠ½ΡˆΡ‚Π΅ΠΉΠ½ Π² своСй Ρ‚Π΅ΠΎΡ€ΠΈΠΈ ΠΎΡ‚Π½ΠΎΡΠΈΡ‚Π΅Π»ΡŒΠ½ΠΎΡΡ‚ΠΈ, хотя это большС относится ΠΊ Ρ„ΠΈΠ·ΠΈΠΊΠ΅, Π½Π΅ΠΆΠ΅Π»ΠΈ ΠΊ Π°Π»Π³Π΅Π±Ρ€Π΅ ΠΈ Π³Π΅ΠΎΠΌΠ΅Ρ‚Ρ€ΠΈΠΈ.

Π˜Π½Ρ‚Π΅Ρ€Π΅ΡΠ΅Π½ ΠΈ Ρ‚ΠΎΡ‚ Ρ„Π°ΠΊΡ‚, Ρ‡Ρ‚ΠΎ сСгодня ΡƒΡ‡Π΅Π½Ρ‹Π΅ ΡƒΠΆΠ΅ Π΄ΠΎΠΊΠ°Π·Π°Π»ΠΈ сущСствованиС ΠΊΠ°ΠΊ ΠΌΠΈΠ½ΠΈΠΌΡƒΠΌ Π΄Π²Π΅Π½Π°Π΄Ρ†Π°Ρ‚ΠΈ ΠΈΠ·ΠΌΠ΅Ρ€Π΅Π½ΠΈΠΉ. ΠšΠΎΠ½Π΅Ρ‡Π½ΠΎ, ΠΏΠΎΠ½ΡΡ‚ΡŒ, Ρ‡Ρ‚ΠΎ ΠΎΠ½ΠΈ собой ΠΏΡ€Π΅Π΄ΡΡ‚Π°Π²Π»ΡΡŽΡ‚, смоТСт Π΄Π°Π»Π΅ΠΊΠΎ Π½Π΅ ΠΊΠ°ΠΆΠ΄Ρ‹ΠΉ, ΠΏΠΎΡΠΊΠΎΠ»ΡŒΠΊΡƒ это относится скорСС ΠΊ Π½Π΅ΠΊΠΎΠΉ абстрактной области, которая находится Π²Π½Π΅ чСловСчСского восприятия ΠΌΠΈΡ€Π°. Π’Π΅ΠΌ Π½Π΅ ΠΌΠ΅Π½Π΅Π΅ Ρ„Π°ΠΊΡ‚ остаСтся Ρ„Π°ΠΊΡ‚ΠΎΠΌ. И Π½Π΅ зря ΠΆΠ΅ ΠΌΠ½ΠΎΠ³ΠΈΠ΅ Π°Π½Ρ‚Ρ€ΠΎΠΏΠΎΠ»ΠΎΠ³ΠΈ ΠΈ историки ΡƒΡ‚Π²Π΅Ρ€ΠΆΠ΄Π°ΡŽΡ‚, Ρ‡Ρ‚ΠΎ наши ΠΏΡ€Π°Ρ‰ΡƒΡ€Ρ‹ ΠΌΠΎΠ³Π»ΠΈ ΠΈΠΌΠ΅Ρ‚ΡŒ Π½Π΅ΠΊΠΈΠ΅ спСцифичныС Ρ€Π°Π·Π²ΠΈΡ‚Ρ‹Π΅ ΠΎΡ€Π³Π°Π½Ρ‹ чувств Π²Ρ€ΠΎΠ΄Π΅ Ρ‚Ρ€Π΅Ρ‚ΡŒΠ΅Π³ΠΎ Π³Π»Π°Π·Π°, ΠΊΠΎΡ‚ΠΎΡ€Ρ‹Π΅ ΠΏΠΎΠΌΠΎΠ³Π°Π»ΠΈ Π²ΠΎΡΠΏΡ€ΠΈΠ½ΠΈΠΌΠ°Ρ‚ΡŒ ΠΌΠ½ΠΎΠ³ΠΎΠΌΠ΅Ρ€Π½ΡƒΡŽ Π΄Π΅ΠΉΡΡ‚Π²ΠΈΡ‚Π΅Π»ΡŒΠ½ΠΎΡΡ‚ΡŒ, Π° Π½Π΅ ΠΈΡΠΊΠ»ΡŽΡ‡ΠΈΡ‚Π΅Π»ΡŒΠ½ΠΎ Ρ‚Ρ€Π΅Ρ…ΠΌΠ΅Ρ€Π½ΠΎΠ΅ пространство.

ΠšΡΡ‚Π°Ρ‚ΠΈ ΡΠΊΠ°Π·Π°Ρ‚ΡŒ, сСгодня сущСствуСт достаточно ΠΌΠ½ΠΎΠ³ΠΎ ΠΌΠ½Π΅Π½ΠΈΠΉ ΠΏΠΎ ΠΏΠΎΠ²ΠΎΠ΄Ρƒ Ρ‚ΠΎΠ³ΠΎ, Ρ‡Ρ‚ΠΎ экстрасСнсорика Ρ‚ΠΎΠΆΠ΅ являСтся ΠΎΠ΄Π½ΠΈΠΌ ΠΈΠ· проявлСний восприятия ΠΌΠ½ΠΎΠ³ΠΎΠΌΠ΅Ρ€Π½ΠΎΠ³ΠΎ ΠΌΠΈΡ€Π°, ΠΈ Ρ‚ΠΎΠΌΡƒ ΠΌΠΎΠΆΠ½ΠΎ Π½Π°ΠΉΡ‚ΠΈ достаточно ΠΌΠ½ΠΎΠ³ΠΎ ΠΏΠΎΠ΄Ρ‚Π²Π΅Ρ€ΠΆΠ΄Π΅Π½ΠΈΠΉ.

Π—Π°ΠΌΠ΅Ρ‚ΡŒΡ‚Π΅, Ρ‡Ρ‚ΠΎ соврСмСнными Π±Π°Π·ΠΎΠ²Ρ‹ΠΌΠΈ уравнСниями ΠΈ Ρ‚Π΅ΠΎΡ€Π΅ΠΌΠ°ΠΌΠΈ ΠΎΠΏΠΈΡΠ°Ρ‚ΡŒ ΠΌΠ½ΠΎΠ³ΠΎΠΌΠ΅Ρ€Π½Ρ‹Π΅ пространства, ΠΎΡ‚Π»ΠΈΡ‡Π°ΡŽΡ‰ΠΈΠ΅ΡΡ ΠΎΡ‚ нашСго Ρ‡Π΅Ρ‚Ρ‹Ρ€Π΅Ρ…ΠΌΠ΅Ρ€Π½ΠΎΠ³ΠΎ ΠΌΠΈΡ€Π°, Ρ‚ΠΎΠΆΠ΅ Π½Π΅ всСгда прСдставляСтся Π²ΠΎΠ·ΠΌΠΎΠΆΠ½Ρ‹ΠΌ. Π”Π° ΠΈ Π½Π°ΡƒΠΊΠ° Π² этой области относится скорСС ΠΊ области Ρ‚Π΅ΠΎΡ€ΠΈΠΉ ΠΈ ΠΏΡ€Π΅Π΄ΠΏΠΎΠ»ΠΎΠΆΠ΅Π½ΠΈΠΉ, Π½Π΅ΠΆΠ΅Π»ΠΈ ΠΊ Ρ‚ΠΎΠΌΡƒ, Ρ‡Ρ‚ΠΎ ΠΌΠΎΠΆΠ½ΠΎ явно ΠΎΡ‰ΡƒΡ‚ΠΈΡ‚ΡŒ ΠΈΠ»ΠΈ, Ρ‚Π°ΠΊ ΡΠΊΠ°Π·Π°Ρ‚ΡŒ, ΠΏΠΎΡ‚Ρ€ΠΎΠ³Π°Ρ‚ΡŒ ΠΈΠ»ΠΈ ΡƒΠ²ΠΈΠ΄Π΅Ρ‚ΡŒ Π²ΠΎΠΎΡ‡ΠΈΡŽ. Π’Π΅ΠΌ Π½Π΅ ΠΌΠ΅Π½Π΅Π΅ косвСнныС Π΄ΠΎΠΊΠ°Π·Π°Ρ‚Π΅Π»ΡŒΡΡ‚Π²Π° сущСствования ΠΌΠ½ΠΎΠ³ΠΎΠΌΠ΅Ρ€Π½Ρ‹Ρ… ΠΌΠΈΡ€ΠΎΠ², Π² ΠΊΠΎΡ‚ΠΎΡ€Ρ‹Ρ… ΠΌΠΎΠΆΠ΅Ρ‚ ΡΡƒΡ‰Π΅ΡΡ‚Π²ΠΎΠ²Π°Ρ‚ΡŒ Ρ‡Π΅Ρ‚Ρ‹Ρ€Π΅ ΠΈ Π±ΠΎΠ»Π΅Π΅ ΠΈΠ·ΠΌΠ΅Ρ€Π΅Π½ΠΈΠΉ, сСгодня Π½ΠΈ Ρƒ ΠΊΠΎΠ³ΠΎ Π½Π΅ Π²Ρ‹Π·Ρ‹Π²Π°ΡŽΡ‚ сомнСний.

Π—Π°ΠΊΠ»ΡŽΡ‡Π΅Π½ΠΈΠ΅

Π’ Ρ†Π΅Π»ΠΎΠΌ ΠΆΠ΅, ΠΌΡ‹ ΠΎΡ‡Π΅Π½ΡŒ ΠΊΡ€Π°Ρ‚ΠΊΠΎ рассмотрСли основныС понятия, относящиСся ΠΊ Ρ‚Ρ€Π΅Ρ…ΠΌΠ΅Ρ€Π½ΠΎΠΌΡƒ пространству ΠΈ Π±Π°Π·ΠΎΠ²Ρ‹ΠΌ опрСдСлСниям. ЕстСствСнно, сущСствуСт мноТСство частных случаСв, связанных с Ρ€Π°Π·Π½Ρ‹ΠΌΠΈ систСмами ΠΊΠΎΠΎΡ€Π΄ΠΈΠ½Π°Ρ‚. К Ρ‚ΠΎΠΌΡƒ ΠΆΠ΅ ΠΌΡ‹ ΠΏΠΎΡΡ‚Π°Ρ€Π°Π»ΠΈΡΡŒ особо Π½Π΅ Π»Π΅Π·Ρ‚ΡŒ Π² матСматичСскиС Π΄Π΅Π±Ρ€ΠΈ для объяснСния основных Ρ‚Π΅Ρ€ΠΌΠΈΠ½ΠΎΠ² Ρ‚ΠΎΠ»ΡŒΠΊΠΎ для Ρ‚ΠΎΠ³ΠΎ, Ρ‡Ρ‚ΠΎΠ±Ρ‹ вопрос, связанный с Π½ΠΈΠΌΠΈ, Π±Ρ‹Π» понятСн Π»ΡŽΠ±ΠΎΠΌΡƒ ΡˆΠΊΠΎΠ»ΡŒΠ½ΠΈΠΊΡƒ (Ρ‚Π°ΠΊ ΡΠΊΠ°Π·Π°Ρ‚ΡŒ, объяснСниС Β«Π½Π° ΠΏΠ°Π»ΡŒΡ†Π°Ρ…Β»).

Π’Π΅ΠΌ Π½Π΅ ΠΌΠ΅Π½Π΅Π΅, думаСтся, Π΄Π°ΠΆΠ΅ ΠΈΠ· Ρ‚Π°ΠΊΠΈΡ… простых Ρ‚Ρ€Π°ΠΊΡ‚ΠΎΠ²ΠΎΠΊ ΠΌΠΎΠΆΠ½ΠΎ ΡΠ΄Π΅Π»Π°Ρ‚ΡŒ Π²Ρ‹Π²ΠΎΠ΄ ΠΎ матСматичСском аспСктС всСх ΡΠΎΡΡ‚Π°Π²Π»ΡΡŽΡ‰ΠΈΡ…, входящих Π² Π±Π°Π·ΠΎΠ²Ρ‹ΠΉ ΡˆΠΊΠΎΠ»ΡŒΠ½Ρ‹ΠΉ курс Π°Π»Π³Π΅Π±Ρ€Ρ‹ ΠΈ Π³Π΅ΠΎΠΌΠ΅Ρ‚Ρ€ΠΈΠΈ.

БущСствуСт Π΄Π²Π° способа Ρ€Π΅ΡˆΠ΅Π½ΠΈΡ Π·Π°Π΄Π°Ρ‡ ΠΏΠΎ стСрСомСтрии

ΠŸΠ΅Ρ€Π²Ρ‹ΠΉ β€” классичСский β€” Ρ‚Ρ€Π΅Π±ΡƒΠ΅Ρ‚ ΠΎΡ‚Π»ΠΈΡ‡Π½ΠΎΠ³ΠΎ знания аксиом ΠΈ Ρ‚Π΅ΠΎΡ€Π΅ΠΌ стСрСомСтрии, Π»ΠΎΠ³ΠΈΠΊΠΈ, умСния ΠΏΠΎΡΡ‚Ρ€ΠΎΠΈΡ‚ΡŒ Ρ‡Π΅Ρ€Ρ‚Π΅ΠΆ ΠΈ свСсти ΠΎΠ±ΡŠΠ΅ΠΌΠ½ΡƒΡŽ Π·Π°Π΄Π°Ρ‡Ρƒ ΠΊ планимСтричСской. Бпособ Ρ…ΠΎΡ€ΠΎΡˆ Ρ‚Π΅ΠΌ, Ρ‡Ρ‚ΠΎ Ρ€Π°Π·Π²ΠΈΠ²Π°Π΅Ρ‚ ΠΌΠΎΠ·Π³ΠΈ ΠΈ пространствСнноС Π²ΠΎΠΎΠ±Ρ€Π°ΠΆΠ΅Π½ΠΈΠ΅.

Π”Ρ€ΡƒΠ³ΠΎΠΉ ΠΌΠ΅Ρ‚ΠΎΠ΄ β€” ΠΏΡ€ΠΈΠΌΠ΅Π½Π΅Π½ΠΈΠ΅ Π²Π΅ΠΊΡ‚ΠΎΡ€ΠΎΠ² ΠΈ ΠΊΠΎΠΎΡ€Π΄ΠΈΠ½Π°Ρ‚. Π­Ρ‚ΠΎ простыС Ρ„ΠΎΡ€ΠΌΡƒΠ»Ρ‹, Π°Π»Π³ΠΎΡ€ΠΈΡ‚ΠΌΡ‹ ΠΈ ΠΏΡ€Π°Π²ΠΈΠ»Π°. Он ΠΎΡ‡Π΅Π½ΡŒ ΡƒΠ΄ΠΎΠ±Π΅Π½, особСнно ΠΊΠΎΠ³Π΄Π° Π²Ρ€Π΅ΠΌΠ΅Π½ΠΈ Π΄ΠΎ экзамСна ΠΌΠ°Π»ΠΎ, Π° Ρ€Π΅ΡˆΠΈΡ‚ΡŒ Π·Π°Π΄Π°Ρ‡Ρƒ хочСтся.

Если Π²Ρ‹ освоили Π²Π΅ΠΊΡ‚ΠΎΡ€Ρ‹ Π½Π° плоскости ΠΈ дСйствия с Π½ΠΈΠΌΠΈ β€” Ρ‚ΠΎ ΠΈ с Π²Π΅ΠΊΡ‚ΠΎΡ€Π°ΠΌΠΈ Π² пространствС Ρ€Π°Π·Π±Π΅Ρ€Π΅Ρ‚Π΅ΡΡŒ. МногиС понятия окаТутся Π·Π½Π°ΠΊΠΎΠΌΡ‹ΠΌΠΈ.

БистСма ΠΊΠΎΠΎΡ€Π΄ΠΈΠ½Π°Ρ‚ Π² пространствС

Π’Ρ‹Π±Π΅Ρ€Π΅ΠΌ Π½Π°Ρ‡Π°Π»ΠΎ ΠΊΠΎΠΎΡ€Π΄ΠΈΠ½Π°Ρ‚. ΠŸΡ€ΠΎΠ²Π΅Π΄Π΅ΠΌ Ρ‚Ρ€ΠΈ Π²Π·Π°ΠΈΠΌΠ½ΠΎ пСрпСндикулярныС оси X, Y ΠΈ Z. Π—Π°Π΄Π°Π΄ΠΈΠΌ ΡƒΠ΄ΠΎΠ±Π½Ρ‹ΠΉ ΠΌΠ°ΡΡˆΡ‚Π°Π±.

ΠŸΠΎΠ»ΡƒΡ‡ΠΈΠ»Π°ΡΡŒ систСма ΠΊΠΎΠΎΡ€Π΄ΠΈΠ½Π°Ρ‚ Π² Ρ‚Ρ€Π΅Ρ…ΠΌΠ΅Ρ€Π½ΠΎΠΌ пространствС. Π’Π΅ΠΏΠ΅Ρ€ΡŒ каТдая Π΅Π³ΠΎ Ρ‚ΠΎΡ‡ΠΊΠ° характСризуСтся трСмя числами β€” ΠΊΠΎΠΎΡ€Π΄ΠΈΠ½Π°Ρ‚Π°ΠΌΠΈ ΠΏΠΎ X, Y ΠΈ Z. НапримСр, запись M(βˆ’1; 3; 2) ΠΎΠ·Π½Π°Ρ‡Π°Π΅Ρ‚, Ρ‡Ρ‚ΠΎ ΠΊΠΎΠΎΡ€Π΄ΠΈΠ½Π°Ρ‚Π° Ρ‚ΠΎΡ‡ΠΊΠΈ M ΠΏΠΎ X (абсцисса) Ρ€Π°Π²Π½Π° βˆ’1, ΠΊΠΎΠΎΡ€Π΄ΠΈΠ½Π°Ρ‚Π° ΠΏΠΎ Y (ΠΎΡ€Π΄ΠΈΠ½Π°Ρ‚Π°) Ρ€Π°Π²Π½Π° 3, Π° ΠΊΠΎΠΎΡ€Π΄ΠΈΠ½Π°Ρ‚Π° ΠΏΠΎ Z (Π°ΠΏΠΏΠ»ΠΈΠΊΠ°Ρ‚Π°) Ρ€Π°Π²Π½Π° 2.

Π’Π΅ΠΊΡ‚ΠΎΡ€Ρ‹ Π² пространствС ΠΎΠΏΡ€Π΅Π΄Π΅Π»ΡΡŽΡ‚ΡΡ Ρ‚Π°ΠΊ ΠΆΠ΅, ΠΊΠ°ΠΊ ΠΈ Π½Π° плоскости. Π­Ρ‚ΠΎ Π½Π°ΠΏΡ€Π°Π²Π»Π΅Π½Π½Ρ‹Π΅ ΠΎΡ‚Ρ€Π΅Π·ΠΊΠΈ, ΠΈΠΌΠ΅ΡŽΡ‰ΠΈΠ΅ Π½Π°Ρ‡Π°Π»ΠΎ ΠΈ ΠΊΠΎΠ½Π΅Ρ†. Волько Π² пространствС Π²Π΅ΠΊΡ‚ΠΎΡ€ задаСтся трСмя ΠΊΠΎΠΎΡ€Π΄ΠΈΠ½Π°Ρ‚Π°ΠΌΠΈ x, y ΠΈ z:

Как Π½Π°ΠΉΡ‚ΠΈ ΠΊΠΎΠΎΡ€Π΄ΠΈΠ½Π°Ρ‚Ρ‹ Π²Π΅ΠΊΡ‚ΠΎΡ€Π°? Как ΠΈ Π½Π° плоскости β€” ΠΈΠ· ΠΊΠΎΠΎΡ€Π΄ΠΈΠ½Π°Ρ‚Ρ‹ ΠΊΠΎΠ½Ρ†Π° Π²Ρ‹Ρ‡ΠΈΡ‚Π°Π΅ΠΌ ΠΊΠΎΠΎΡ€Π΄ΠΈΠ½Π°Ρ‚Ρƒ Π½Π°Ρ‡Π°Π»Π°.


Π”Π»ΠΈΠ½Π° Π²Π΅ΠΊΡ‚ΠΎΡ€Π° Π² пространствС – это расстояниС ΠΌΠ΅ΠΆΠ΄Ρƒ Ρ‚ΠΎΡ‡ΠΊΠ°ΠΌΠΈ A ΠΈ B. Находится ΠΊΠ°ΠΊ ΠΊΠΎΡ€Π΅Π½ΡŒ ΠΊΠ²Π°Π΄Ρ€Π°Ρ‚Π½Ρ‹ΠΉ ΠΈΠ· суммы ΠΊΠ²Π°Π΄Ρ€Π°Ρ‚ΠΎΠ² ΠΊΠΎΠΎΡ€Π΄ΠΈΠ½Π°Ρ‚ Π²Π΅ΠΊΡ‚ΠΎΡ€Π°.

ΠŸΡƒΡΡ‚ΡŒ Ρ‚ΠΎΡ‡ΠΊΠ° M – сСрСдина ΠΎΡ‚Ρ€Π΅Π·ΠΊΠ° AB. Π•Π΅ ΠΊΠΎΠΎΡ€Π΄ΠΈΠ½Π°Ρ‚Ρ‹ находятся ΠΏΠΎ Ρ„ΠΎΡ€ΠΌΡƒΠ»Π΅:

Для слоТСния Π²Π΅ΠΊΡ‚ΠΎΡ€ΠΎΠ² примСняСм ΡƒΠΆΠ΅ Π·Π½Π°ΠΊΠΎΠΌΡ‹Π΅ ΠΏΡ€Π°Π²ΠΈΠ»ΠΎ Ρ‚Ρ€Π΅ΡƒΠ³ΠΎΠ»ΡŒΠ½ΠΈΠΊΠ° ΠΈ ΠΏΡ€Π°Π²ΠΈΠ»ΠΎ ΠΏΠ°Ρ€Π°Π»Π»Π΅Π»ΠΎΠ³Ρ€Π°ΠΌΠΌΠ°

Π‘ΡƒΠΌΠΌΠ° Π²Π΅ΠΊΡ‚ΠΎΡ€ΠΎΠ², ΠΈΡ… Ρ€Π°Π·Π½ΠΎΡΡ‚ΡŒ, ΠΏΡ€ΠΎΠΈΠ·Π²Π΅Π΄Π΅Π½ΠΈΠ΅ Π²Π΅ΠΊΡ‚ΠΎΡ€Π° Π½Π° число ΠΈ скалярноС ΠΏΡ€ΠΎΠΈΠ·Π²Π΅Π΄Π΅Π½ΠΈΠ΅ Π²Π΅ΠΊΡ‚ΠΎΡ€ΠΎΠ² ΠΎΠΏΡ€Π΅Π΄Π΅Π»ΡΡŽΡ‚ΡΡ Ρ‚Π°ΠΊ ΠΆΠ΅, ΠΊΠ°ΠΊ ΠΈ Π½Π° плоскости. Волько ΠΊΠΎΠΎΡ€Π΄ΠΈΠ½Π°Ρ‚ Π½Π΅ Π΄Π²Π΅, Π° Ρ‚Ρ€ΠΈ. Π’ΠΎΠ·ΡŒΠΌΠ΅ΠΌ Π²Π΅ΠΊΡ‚ΠΎΡ€Ρ‹ ΠΈ .

ΠŸΡ€ΠΎΠΈΠ·Π²Π΅Π΄Π΅Π½ΠΈΠ΅ Π²Π΅ΠΊΡ‚ΠΎΡ€Π° Π½Π° число:

БкалярноС ΠΏΡ€ΠΎΠΈΠ·Π²Π΅Π΄Π΅Π½ΠΈΠ΅ Π²Π΅ΠΊΡ‚ΠΎΡ€ΠΎΠ²:

ΠšΠΎΡΠΈΠ½ΡƒΡ ΡƒΠ³Π»Π° ΠΌΠ΅ΠΆΠ΄Ρƒ Π²Π΅ΠΊΡ‚ΠΎΡ€Π°ΠΌΠΈ:

ПослСдняя Ρ„ΠΎΡ€ΠΌΡƒΠ»Π° ΡƒΠ΄ΠΎΠ±Π½Π° для нахоТдСния ΡƒΠ³Π»Π° ΠΌΠ΅ΠΆΠ΄Ρƒ прямыми Π² пространствС. ОсобСнно Ссли эти прямыС – ΡΠΊΡ€Π΅Ρ‰ΠΈΠ²Π°ΡŽΡ‚ΡΡ. Напомним, Ρ‡Ρ‚ΠΎ Ρ‚Π°ΠΊ Π½Π°Π·Ρ‹Π²Π°ΡŽΡ‚ΡΡ прямыС, ΠΊΠΎΡ‚ΠΎΡ€Ρ‹Π΅ Π½Π΅ ΠΏΠ°Ρ€Π°Π»Π»Π΅Π»ΡŒΠ½Ρ‹ ΠΈ Π½Π΅ ΠΏΠ΅Ρ€Π΅ΡΠ΅ΠΊΠ°ΡŽΡ‚ΡΡ. Они Π»Π΅ΠΆΠ°Ρ‚ Π² ΠΏΠ°Ρ€Π°Π»Π»Π΅Π»ΡŒΠ½Ρ‹Ρ… плоскостях.

1. Π’ ΠΊΡƒΠ±Π΅ ABCDA1B1C1D1 Ρ‚ΠΎΡ‡ΠΊΠΈ E ΠΈ K β€” сСрСдины Ρ€Π΅Π±Π΅Ρ€ соотвСтствСнно A1B1 ΠΈ B1C1. НайдитС косинус ΡƒΠ³Π»Π° ΠΌΠ΅ΠΆΠ΄Ρƒ прямыми AE ΠΈ BK.

Если Π²Π°ΠΌ достался ΠΊΡƒΠ± β€” Π·Π½Π°Ρ‡ΠΈΡ‚, ΠΏΠΎΠ²Π΅Π·Π»ΠΎ. Он ΠΎΡ‚Π»ΠΈΡ‡Π½ΠΎ вписываСтся Π² ΠΏΡ€ΡΠΌΠΎΡƒΠ³ΠΎΠ»ΡŒΠ½ΡƒΡŽ систСму ΠΊΠΎΠΎΡ€Π΄ΠΈΠ½Π°Ρ‚. Π‘Ρ‚Ρ€ΠΎΠΈΠΌ Ρ‡Π΅Ρ€Ρ‚Π΅ΠΆ:

Π”Π»ΠΈΠ½Π° Ρ€Π΅Π±Ρ€Π° ΠΊΡƒΠ±Π° Π½Π΅ Π΄Π°Π½Π°. Какой Π±Ρ‹ ΠΎΠ½Π° Π½ΠΈ Π±Ρ‹Π»Π°, ΡƒΠ³ΠΎΠ» ΠΌΠ΅ΠΆΠ΄Ρƒ AE ΠΈ BK ΠΎΡ‚ Π½Π΅Π΅ Π½Π΅ зависит. ΠŸΠΎΡΡ‚ΠΎΠΌΡƒ возьмСм Π΅Π΄ΠΈΠ½ΠΈΡ‡Π½Ρ‹ΠΉ ΠΊΡƒΠ±, всС Ρ€Π΅Π±Ρ€Π° ΠΊΠΎΡ‚ΠΎΡ€ΠΎΠ³ΠΎ Ρ€Π°Π²Π½Ρ‹ 1.

ΠŸΡ€ΡΠΌΡ‹Π΅ AE ΠΈ BK β€” ΡΠΊΡ€Π΅Ρ‰ΠΈΠ²Π°ΡŽΡ‚ΡΡ. НайдСм ΡƒΠ³ΠΎΠ» ΠΌΠ΅ΠΆΠ΄Ρƒ Π²Π΅ΠΊΡ‚ΠΎΡ€Π°ΠΌΠΈ ΠΈ . Для этого Π½ΡƒΠΆΠ½Ρ‹ ΠΈΡ… ΠΊΠΎΠΎΡ€Π΄ΠΈΠ½Π°Ρ‚Ρ‹.

Π—Π°ΠΏΠΈΡˆΠ΅ΠΌ ΠΊΠΎΠΎΡ€Π΄ΠΈΠ½Π°Ρ‚Ρ‹ Π²Π΅ΠΊΡ‚ΠΎΡ€ΠΎΠ²:

ΠΈ Π½Π°ΠΉΠ΄Π΅ΠΌ косинус ΡƒΠ³Π»Π° ΠΌΠ΅ΠΆΠ΄Ρƒ Π²Π΅ΠΊΡ‚ΠΎΡ€Π°ΠΌΠΈ ΠΈ :

2. Π’ ΠΏΡ€Π°Π²ΠΈΠ»ΡŒΠ½ΠΎΠΉ Ρ‡Π΅Ρ‚Ρ‹Ρ€Π΅Ρ…ΡƒΠ³ΠΎΠ»ΡŒΠ½ΠΎΠΉ ΠΏΠΈΡ€Π°ΠΌΠΈΠ΄Π΅ SABCD, всС Ρ€Π΅Π±Ρ€Π° ΠΊΠΎΡ‚ΠΎΡ€ΠΎΠΉ Ρ€Π°Π²Π½Ρ‹ 1, Ρ‚ΠΎΡ‡ΠΊΠΈ E, K β€” сСрСдины Ρ€Π΅Π±Π΅Ρ€ SB ΠΈ SC соотвСтствСнно. НайдитС косинус ΡƒΠ³Π»Π° ΠΌΠ΅ΠΆΠ΄Ρƒ прямыми AE ΠΈ BK.

Π›ΡƒΡ‡ΡˆΠ΅ всСго Π²Ρ‹Π±Ρ€Π°Ρ‚ΡŒ Π½Π°Ρ‡Π°Π»ΠΎ ΠΊΠΎΠΎΡ€Π΄ΠΈΠ½Π°Ρ‚ Π² Ρ†Π΅Π½Ρ‚Ρ€Π΅ основания ΠΏΠΈΡ€Π°ΠΌΠΈΠ΄Ρ‹, Π° оси X ΠΈ Y ΡΠ΄Π΅Π»Π°Ρ‚ΡŒ ΠΏΠ°Ρ€Π°Π»Π»Π΅Π»ΡŒΠ½Ρ‹ΠΌΠΈ сторонам основания.

ΠšΠΎΠΎΡ€Π΄ΠΈΠ½Π°Ρ‚Ρ‹ Ρ‚ΠΎΡ‡Π΅ΠΊ A, B ΠΈ C Π½Π°ΠΉΡ‚ΠΈ Π»Π΅Π³ΠΊΠΎ:

Из ΠΏΡ€ΡΠΌΠΎΡƒΠ³ΠΎΠ»ΡŒΠ½ΠΎΠ³ΠΎ Ρ‚Ρ€Π΅ΡƒΠ³ΠΎΠ»ΡŒΠ½ΠΈΠΊΠ° AOS Π½Π°ΠΉΠ΄Π΅ΠΌ

ΠšΠΎΠΎΡ€Π΄ΠΈΠ½Π°Ρ‚Ρ‹ Π²Π΅Ρ€ΡˆΠΈΠ½Ρ‹ ΠΏΠΈΡ€Π°ΠΌΠΈΠ΄Ρ‹:

Π’ΠΎΡ‡ΠΊΠ° E β€” сСрСдина SB, Π° K β€” сСрСдина SC. Π’ΠΎΡΠΏΠΎΠ»ΡŒΠ·ΡƒΠ΅ΠΌΡΡ Ρ„ΠΎΡ€ΠΌΡƒΠ»ΠΎΠΉ для ΠΊΠΎΠΎΡ€Π΄ΠΈΠ½Π°Ρ‚ сСрСдины ΠΎΡ‚Ρ€Π΅Π·ΠΊΠ° ΠΈ Π½Π°ΠΉΠ΄Π΅ΠΌ ΠΊΠΎΠΎΡ€Π΄ΠΈΠ½Π°Ρ‚Ρ‹ Ρ‚ΠΎΡ‡Π΅ΠΊ E ΠΈ K.

НайдСм ΠΊΠΎΠΎΡ€Π΄ΠΈΠ½Π°Ρ‚Ρ‹ Π²Π΅ΠΊΡ‚ΠΎΡ€ΠΎΠ² ΠΈ

ΠΈ ΡƒΠ³ΠΎΠ» ΠΌΠ΅ΠΆΠ΄Ρƒ Π½ΠΈΠΌΠΈ:

ПокаТСм Ρ‚Π΅ΠΏΠ΅Ρ€ΡŒ, ΠΊΠ°ΠΊ Π²ΠΏΠΈΡΠ°Ρ‚ΡŒ систСму ΠΊΠΎΠΎΡ€Π΄ΠΈΠ½Π°Ρ‚ Π² Ρ‚Ρ€Π΅ΡƒΠ³ΠΎΠ»ΡŒΠ½ΡƒΡŽ ΠΏΡ€ΠΈΠ·ΠΌΡƒ:

3. Π’ ΠΏΡ€Π°Π²ΠΈΠ»ΡŒΠ½ΠΎΠΉ Ρ‚Ρ€Π΅ΡƒΠ³ΠΎΠ»ΡŒΠ½ΠΎΠΉ ΠΏΡ€ΠΈΠ·ΠΌΠ΅ ABCA1B1C1, всС Ρ€Π΅Π±Ρ€Π° ΠΊΠΎΡ‚ΠΎΡ€ΠΎΠΉ Ρ€Π°Π²Π½Ρ‹ 1, Ρ‚ΠΎΡ‡ΠΊΠ° D β€” сСрСдина Ρ€Π΅Π±Ρ€Π° A1B1. НайдитС косинус ΡƒΠ³Π»Π° ΠΌΠ΅ΠΆΠ΄Ρƒ прямыми AD ΠΈ BC1

ΠŸΡƒΡΡ‚ΡŒ Ρ‚ΠΎΡ‡ΠΊΠ° A β€” Π½Π°Ρ‡Π°Π»ΠΎ ΠΊΠΎΠΎΡ€Π΄ΠΈΠ½Π°Ρ‚. Π’ΠΎΠ·ΡŒΠΌΠ΅ΠΌ ось X ΠΏΠ°Ρ€Π°Π»Π»Π΅Π»ΡŒΠ½ΠΎ сторонС BC, Π° ось Y пСрпСндикулярно Π΅ΠΉ. Π”Ρ€ΡƒΠ³ΠΈΠΌΠΈ словами, Π½Π° оси Y Π±ΡƒΠ΄Π΅Ρ‚ Π»Π΅ΠΆΠ°Ρ‚ΡŒ ΠΎΡ‚Ρ€Π΅Π·ΠΎΠΊ AH, ΡΠ²Π»ΡΡŽΡ‰ΠΈΠΉΡΡ высотой Ρ‚Ρ€Π΅ΡƒΠ³ΠΎΠ»ΡŒΠ½ΠΈΠΊΠ° ABC. НарисуСм ΠΎΡ‚Π΄Π΅Π»ΡŒΠ½ΠΎ Π½ΠΈΠΆΠ½Π΅Π΅ основаниС ΠΏΡ€ΠΈΠ·ΠΌΡ‹.

Π—Π°ΠΏΠΈΡˆΠ΅ΠΌ ΠΊΠΎΠΎΡ€Π΄ΠΈΠ½Π°Ρ‚Ρ‹ Ρ‚ΠΎΡ‡Π΅ΠΊ:

Π’ΠΎΡ‡ΠΊΠ° D β€” сСрСдина A1B1. Π—Π½Π°Ρ‡ΠΈΡ‚, ΠΏΠΎΠ»ΡŒΠ·ΡƒΠ΅ΠΌΡΡ Ρ„ΠΎΡ€ΠΌΡƒΠ»Π°ΠΌΠΈ для ΠΊΠΎΠΎΡ€Π΄ΠΈΠ½Π°Ρ‚ сСрСдины
ΠΎΡ‚Ρ€Π΅Π·ΠΊΠ°.

НайдСм ΠΊΠΎΠΎΡ€Π΄ΠΈΠ½Π°Ρ‚Ρ‹ Π²Π΅ΠΊΡ‚ΠΎΡ€ΠΎΠ² ΠΈ , Π° Π·Π°Ρ‚Π΅ΠΌ ΡƒΠ³ΠΎΠ» ΠΌΠ΅ΠΆΠ΄Ρƒ Π½ΠΈΠΌΠΈ:

Π‘ΠΌΠΎΡ‚Ρ€ΠΈΡ‚Π΅, ΠΊΠ°ΠΊ Π»Π΅Π³ΠΊΠΎ с ΠΏΠΎΠΌΠΎΡ‰ΡŒΡŽ Π²Π΅ΠΊΡ‚ΠΎΡ€ΠΎΠ² ΠΈ ΠΊΠΎΠΎΡ€Π΄ΠΈΠ½Π°Ρ‚ Π½Π°ΠΉΡ‚ΠΈ ΡƒΠ³ΠΎΠ» ΠΌΠ΅ΠΆΠ΄Ρƒ прямыми. А Ссли трСбуСтся Π½Π°ΠΉΡ‚ΠΈ ΡƒΠ³ΠΎΠ» ΠΌΠ΅ΠΆΠ΄Ρƒ плоскостями ΠΈΠ»ΠΈ ΠΌΠ΅ΠΆΠ΄Ρƒ прямой ΠΈ ΠΏΠ»ΠΎΡΠΊΠΎΡΡ‚ΡŒΡŽ? Для Ρ€Π΅ΡˆΠ΅Π½ΠΈΡ ΠΏΠΎΠ΄ΠΎΠ±Π½Ρ‹Ρ… Π·Π°Π΄Π°Ρ‡ Π½Π°ΠΌ понадобится ΡƒΡ€Π°Π²Π½Π΅Π½ΠΈΠ΅ плоскости Π² пространствС.

ΠŸΠ»ΠΎΡΠΊΠΎΡΡ‚ΡŒ Π² пространствС задаСтся ΡƒΡ€Π°Π²Π½Π΅Π½ΠΈΠ΅ΠΌ:

Π—Π΄Π΅ΡΡŒ числа A, B ΠΈ C β€” ΠΊΠΎΠΎΡ€Π΄ΠΈΠ½Π°Ρ‚Ρ‹ Π²Π΅ΠΊΡ‚ΠΎΡ€Π°, пСрпСндикулярного этой плоскости. Π•Π³ΠΎ Π½Π°Π·Ρ‹Π²Π°ΡŽΡ‚ Π½ΠΎΡ€ΠΌΠ°Π»ΡŒΡŽ ΠΊ плоскости.

ВмСсто x, y ΠΈ z ΠΌΠΎΠΆΠ½ΠΎ ΠΏΠΎΠ΄ΡΡ‚Π°Π²ΠΈΡ‚ΡŒ Π² ΡƒΡ€Π°Π²Π½Π΅Π½ΠΈΠ΅ ΠΊΠΎΠΎΡ€Π΄ΠΈΠ½Π°Ρ‚Ρ‹ любой Ρ‚ΠΎΡ‡ΠΊΠΈ, ΠΏΡ€ΠΈΠ½Π°Π΄Π»Π΅ΠΆΠ°Ρ‰Π΅ΠΉ Π΄Π°Π½Π½ΠΎΠΉ плоскости. ΠŸΠΎΠ»ΡƒΡ‡ΠΈΡ‚ΡΡ Π²Π΅Ρ€Π½ΠΎΠ΅ равСнство.

ΠŸΠ»ΠΎΡΠΊΠΎΡΡ‚ΡŒ Π² пространствС ΠΌΠΎΠΆΠ½ΠΎ провСсти Ρ‡Π΅Ρ€Π΅Π· Π»ΡŽΠ±Ρ‹Π΅ Ρ‚Ρ€ΠΈ Ρ‚ΠΎΡ‡ΠΊΠΈ, Π½Π΅ Π»Π΅ΠΆΠ°Ρ‰ΠΈΠ΅ Π½Π° ΠΎΠ΄Π½ΠΎΠΉ прямой. ΠŸΠΎΡΡ‚ΠΎΠΌΡƒ для Ρ‚ΠΎΠ³ΠΎ, Ρ‡Ρ‚ΠΎΠ±Ρ‹ Π½Π°ΠΏΠΈΡΠ°Ρ‚ΡŒ ΡƒΡ€Π°Π²Π½Π΅Π½ΠΈΠ΅ плоскости, Π±Π΅Ρ€Π΅ΠΌ ΠΊΠΎΠΎΡ€Π΄ΠΈΠ½Π°Ρ‚Ρ‹ Ρ‚Ρ€Π΅Ρ… ΠΏΡ€ΠΈΠ½Π°Π΄Π»Π΅ΠΆΠ°Ρ‰ΠΈΡ… Π΅ΠΉ Ρ‚ΠΎΡ‡Π΅ΠΊ. ΠŸΠΎΠ΄ΡΡ‚Π°Π²Π»ΡΠ΅ΠΌ ΠΈΡ… ΠΏΠΎ ΠΎΡ‡Π΅Ρ€Π΅Π΄ΠΈ Π² ΡƒΡ€Π°Π²Π½Π΅Π½ΠΈΠ΅ плоскости. РСшаСм ΠΏΠΎΠ»ΡƒΡ‡Π΅Π½Π½ΡƒΡŽ систСму.

ПокаТСм, ΠΊΠ°ΠΊ это дСлаСтся.

НапишСм ΡƒΡ€Π°Π²Π½Π΅Π½ΠΈΠ΅ плоскости, проходящСй Ρ‡Π΅Ρ€Π΅Π· Ρ‚ΠΎΡ‡ΠΊΠΈ M (1; 0; 1), N (2; βˆ’2; 0) ΠΈ K (4; 1; 2).

Π£Ρ€Π°Π²Π½Π΅Π½ΠΈΠ΅ плоскости выглядит Ρ‚Π°ΠΊ:

ΠŸΠΎΠ΄ΡΡ‚Π°Π²ΠΈΠΌ Π² Π½Π΅Π³ΠΎ ΠΏΠΎ ΠΎΡ‡Π΅Ρ€Π΅Π΄ΠΈ ΠΊΠΎΠΎΡ€Π΄ΠΈΠ½Π°Ρ‚Ρ‹ Ρ‚ΠΎΡ‡Π΅ΠΊ M, N ΠΈ K.

Π’ΠΎ Π΅ΡΡ‚ΡŒ A + C + D = 0.

Аналогично для Ρ‚ΠΎΡ‡ΠΊΠΈ K:

ΠŸΠΎΠ»ΡƒΡ‡ΠΈΠ»ΠΈ систСму ΠΈΠ· Ρ‚Ρ€Π΅Ρ… ΡƒΡ€Π°Π²Π½Π΅Π½ΠΈΠΉ:

Π’ Π½Π΅ΠΉ Ρ‡Π΅Ρ‚Ρ‹Ρ€Π΅ нСизвСстных: A, B, C ΠΈ D. ΠŸΠΎΡΡ‚ΠΎΠΌΡƒ ΠΎΠ΄Π½Ρƒ ΠΈΠ· Π½ΠΈΡ… ΠΌΡ‹ Π²Ρ‹Π±Π΅Ρ€Π΅ΠΌ сами, Π° Π΄Ρ€ΡƒΠ³ΠΈΠ΅ Π²Ρ‹Ρ€Π°Π·ΠΈΠΌ Ρ‡Π΅Ρ€Π΅Π· Π½Π΅Π΅. ΠŸΡ€Π°Π²ΠΈΠ»ΠΎ простоС β€” вмСсто ΠΎΠ΄Π½ΠΎΠΉ ΠΈΠ· ΠΏΠ΅Ρ€Π΅ΠΌΠ΅Π½Π½Ρ‹Ρ… ΠΌΠΎΠΆΠ½ΠΎ Π²Π·ΡΡ‚ΡŒ любоС число, Π½Π΅ Ρ€Π°Π²Π½ΠΎΠ΅ Π½ΡƒΠ»ΡŽ.

ΠŸΡƒΡΡ‚ΡŒ, Π½Π°ΠΏΡ€ΠΈΠΌΠ΅Ρ€, D = βˆ’2. Π’ΠΎΠ³Π΄Π°:

Π’Ρ‹Ρ€Π°Π·ΠΈΠΌ C ΠΈ B Ρ‡Π΅Ρ€Π΅Π· A ΠΈ подставим Π² Ρ‚Ρ€Π΅Ρ‚ΡŒΠ΅ ΡƒΡ€Π°Π²Π½Π΅Π½ΠΈΠ΅:

РСшив систСму, ΠΏΠΎΠ»ΡƒΡ‡ΠΈΠΌ:

Π£Ρ€Π°Π²Π½Π΅Π½ΠΈΠ΅ плоскости MNK ΠΈΠΌΠ΅Π΅Ρ‚ Π²ΠΈΠ΄:

Π£ΠΌΠ½ΠΎΠΆΠΈΠΌ ΠΎΠ±Π΅ части уравнСния Π½Π° βˆ’3. Π’ΠΎΠ³Π΄Π° коэффициСнты станут Ρ†Π΅Π»Ρ‹ΠΌΠΈ:

Π’Π΅ΠΊΡ‚ΠΎΡ€ β€” это Π½ΠΎΡ€ΠΌΠ°Π»ΡŒ ΠΊ плоскости MNK.

Π£Ρ€Π°Π²Π½Π΅Π½ΠΈΠ΅ плоскости, проходящСй Ρ‡Π΅Ρ€Π΅Π· Π·Π°Π΄Π°Π½Π½ΡƒΡŽ Ρ‚ΠΎΡ‡ΠΊΡƒ ΠΈΠΌΠ΅Π΅Ρ‚ Π²ΠΈΠ΄:

Π£Π³ΠΎΠ» ΠΌΠ΅ΠΆΠ΄Ρƒ плоскостями Ρ€Π°Π²Π΅Π½ ΡƒΠ³Π»Ρƒ ΠΌΠ΅ΠΆΠ΄Ρƒ нормалями ΠΊ этим плоскостям:

НС ΠΏΡ€Π°Π²Π΄Π° Π»ΠΈ, знакомая Ρ„ΠΎΡ€ΠΌΡƒΠ»Π°? БкалярноС ΠΏΡ€ΠΎΠΈΠ·Π²Π΅Π΄Π΅Π½ΠΈΠ΅ Π½ΠΎΡ€ΠΌΠ°Π»Π΅ΠΉ ΠΏΠΎΠ΄Π΅Π»ΠΈΠ»ΠΈ Π½Π° ΠΏΡ€ΠΎΠΈΠ·Π²Π΅Π΄Π΅Π½ΠΈΠ΅ ΠΈΡ… Π΄Π»ΠΈΠ½.

Π—Π°ΠΌΠ΅Ρ‚ΠΈΠΌ, Ρ‡Ρ‚ΠΎ ΠΏΡ€ΠΈ пСрСсСчСнии Π΄Π²ΡƒΡ… плоскостСй Π²ΠΎΠΎΠ±Ρ‰Π΅-Ρ‚ΠΎ образуСтся Ρ‡Π΅Ρ‚Ρ‹Ρ€Π΅ ΡƒΠ³Π»Π°.

ΠœΡ‹ Π±Π΅Ρ€Π΅ΠΌ мСньший ΠΈΠ· Π½ΠΈΡ…. ΠŸΠΎΡΡ‚ΠΎΠΌΡƒ Π² Ρ„ΠΎΡ€ΠΌΡƒΠ»Π΅ стоит ΠΌΠΎΠ΄ΡƒΠ»ΡŒ скалярного произвСдСния β€” Ρ‡Ρ‚ΠΎΠ±Ρ‹ косинус ΡƒΠ³Π»Π° Π±Ρ‹Π» Π½Π΅ΠΎΡ‚Ρ€ΠΈΡ†Π°Ρ‚Π΅Π»Π΅Π½.

4. Π’ ΠΊΡƒΠ±Π΅ ABCDA1B1C1D1 Ρ‚ΠΎΡ‡ΠΊΠΈ E ΠΈ F β€” сСрСдины Ρ€Π΅Π±Π΅Ρ€ соотвСтствСнно A1B1 ΠΈ A1D1. НайдитС тангСнс ΡƒΠ³Π»Π° ΠΌΠ΅ΠΆΠ΄Ρƒ плоскостями AEF ΠΈ BDD1.

Π‘Ρ‚Ρ€ΠΎΠΈΠΌ Ρ‡Π΅Ρ€Ρ‚Π΅ΠΆ. Π’ΠΈΠ΄Π½ΠΎ, Ρ‡Ρ‚ΠΎ плоскости AEF ΠΈ BDD1 ΠΏΠ΅Ρ€Π΅ΡΠ΅ΠΊΠ°ΡŽΡ‚ΡΡ Π³Π΄Π΅-Ρ‚ΠΎ Π²Π½Π΅ ΠΊΡƒΠ±Π°. Π’ классичСском Ρ€Π΅ΡˆΠ΅Π½ΠΈΠΈ ΠΏΡ€ΠΈΡˆΠ»ΠΎΡΡŒ Π±Ρ‹ ΡΡ‚Ρ€ΠΎΠΈΡ‚ΡŒ линию ΠΈΡ… пСрСсСчСния. Но Π²Π΅ΠΊΡ‚ΠΎΡ€Π½ΠΎ-ΠΊΠΎΠΎΡ€Π΄ΠΈΠ½Π°Ρ‚Π½Ρ‹ΠΉ ΠΌΠ΅Ρ‚ΠΎΠ΄ Π·Π½Π°Ρ‡ΠΈΡ‚Π΅Π»ΡŒΠ½ΠΎ всё ΡƒΠΏΡ€ΠΎΡ‰Π°Π΅Ρ‚. НС Π±ΡƒΠ΄Π΅ΠΌ Π»ΠΎΠΌΠ°Ρ‚ΡŒ Π³ΠΎΠ»ΠΎΠ²Ρƒ Π½Π°Π΄ Ρ‚Π΅ΠΌ, ΠΏΠΎ ΠΊΠ°ΠΊΠΎΠΉ прямой ΠΏΠ΅Ρ€Π΅ΡΠ΅ΠΊΠ°ΡŽΡ‚ΡΡ плоскости. ΠŸΡ€ΠΎΡΡ‚ΠΎ ΠΎΡ‚ΠΌΠ΅Ρ‚ΠΈΠΌ ΠΊΠΎΠΎΡ€Π΄ΠΈΠ½Π°Ρ‚Ρ‹ Π½ΡƒΠΆΠ½Ρ‹Ρ… Π½Π°ΠΌ Ρ‚ΠΎΡ‡Π΅ΠΊ ΠΈ Π½Π°ΠΉΠ΄Π΅ΠΌ ΡƒΠ³ΠΎΠ» ΠΌΠ΅ΠΆΠ΄Ρƒ нормалями ΠΊ плоскостям AEF ΠΈ BDD1.

Π‘Π½Π°Ρ‡Π°Π»Π° β€” Π½ΠΎΡ€ΠΌΠ°Π»ΡŒ ΠΊ плоскости BDD1. ΠšΠΎΠ½Π΅Ρ‡Π½ΠΎ, ΠΌΡ‹ ΠΌΠΎΠΆΠ΅ΠΌ ΠΏΠΎΠ΄ΡΡ‚Π°Π²ΠΈΡ‚ΡŒ ΠΊΠΎΠΎΡ€Π΄ΠΈΠ½Π°Ρ‚Ρ‹ Ρ‚ΠΎΡ‡Π΅ΠΊ B, D ΠΈ D1 Π² ΡƒΡ€Π°Π²Π½Π΅Π½ΠΈΠ΅ плоскости ΠΈ Π½Π°ΠΉΡ‚ΠΈ коэффициСнты, ΠΊΠΎΡ‚ΠΎΡ€Ρ‹Π΅ ΠΈ Π±ΡƒΠ΄ΡƒΡ‚ ΠΊΠΎΠΎΡ€Π΄ΠΈΠ½Π°Ρ‚Π°ΠΌΠΈ Π²Π΅ΠΊΡ‚ΠΎΡ€Π° Π½ΠΎΡ€ΠΌΠ°Π»ΠΈ. А ΠΌΠΎΠΆΠ΅ΠΌ ΡΠ΄Π΅Π»Π°Ρ‚ΡŒ Ρ…ΠΈΡ‚Ρ€Π΅Π΅ β€” ΡƒΠ²ΠΈΠ΄Π΅Ρ‚ΡŒ Π½ΡƒΠΆΠ½ΡƒΡŽ Π½ΠΎΡ€ΠΌΠ°Π»ΡŒ прямо Π½Π° Ρ‡Π΅Ρ€Ρ‚Π΅ΠΆΠ΅. Π’Π΅Π΄ΡŒ ΠΏΠ»ΠΎΡΠΊΠΎΡΡ‚ΡŒ BDD1 β€” это диагональноС сСчСниС ΠΊΡƒΠ±Π°. Π’Π΅ΠΊΡ‚ΠΎΡ€ пСрпСндикулярСн этой плоскости.

Π˜Ρ‚Π°ΠΊ, ΠΏΠ΅Ρ€Π²Ρ‹ΠΉ Π²Π΅ΠΊΡ‚ΠΎΡ€ Π½ΠΎΡ€ΠΌΠ°Π»ΠΈ Ρƒ нас ΡƒΠΆΠ΅ Π΅ΡΡ‚ΡŒ:

НапишСм ΡƒΡ€Π°Π²Π½Π΅Π½ΠΈΠ΅ плоскости AEF.

Π‘Π΅Ρ€Π΅ΠΌ ΡƒΡ€Π°Π²Π½Π΅Π½ΠΈΠ΅ плоскости ΠΈ ΠΏΠΎ ΠΎΡ‡Π΅Ρ€Π΅Π΄ΠΈ подставляСм Π² Π½Π΅Π³ΠΎ, вмСсто x, y ΠΈ z, ΡΠΎΠΎΡ‚Π²Π΅Ρ‚ΡΡ‚Π²ΡƒΡŽΡ‰ΠΈΠ΅ ΠΊΠΎΠΎΡ€Π΄ΠΈΠ½Π°Ρ‚Ρ‹ Ρ‚ΠΎΡ‡Π΅ΠΊ A, E ΠΈ F.

ΠŸΡƒΡΡ‚ΡŒ Π‘ = -1. Π’ΠΎΠ³Π΄Π° A = B = 2.

Π£Ρ€Π°Π²Π½Π΅Π½ΠΈΠ΅ плоскости AEF:

ΠΠΎΡ€ΠΌΠ°Π»ΡŒ ΠΊ плоскости AEF:

НайдСм ΡƒΠ³ΠΎΠ» ΠΌΠ΅ΠΆΠ΄Ρƒ плоскостями:

5. ОснованиС прямой Ρ‡Π΅Ρ‚Ρ‹Ρ€Π΅Ρ…ΡƒΠ³ΠΎΠ»ΡŒΠ½ΠΎΠΉ ΠΏΡ€ΠΈΠ·ΠΌΡ‹ BCDA1B1C1D1 β€” ΠΏΡ€ΡΠΌΠΎΡƒΠ³ΠΎΠ»ΡŒΠ½ΠΈΠΊ ABCD, Π² ΠΊΠΎΡ‚ΠΎΡ€ΠΎΠΌ AB = 5, AD = √33. НайдитС тангСнс ΡƒΠ³Π»Π° ΠΌΠ΅ΠΆΠ΄Ρƒ ΠΏΠ»ΠΎΡΠΊΠΎΡΡ‚ΡŒΡŽ Π³Ρ€Π°Π½ΠΈ AA1D1D ΠΈ ΠΏΠ»ΠΎΡΠΊΠΎΡΡ‚ΡŒΡŽ, проходящСй Ρ‡Π΅Ρ€Π΅Π· сСрСдину Ρ€Π΅Π±Ρ€Π° CD пСрпСндикулярно прямой B1D, Ссли расстояниС ΠΌΠ΅ΠΆΠ΄Ρƒ прямыми A1C1 ΠΈ BD Ρ€Π°Π²Π½ΠΎ √3.

Π­Ρ‚Π° Π·Π°Π΄Π°Ρ‡Π° наглядно ΠΏΠΎΠΊΠ°Π·Ρ‹Π²Π°Π΅Ρ‚, насколько Π²Π΅ΠΊΡ‚ΠΎΡ€Π½Ρ‹ΠΉ ΠΌΠ΅Ρ‚ΠΎΠ΄ ΠΏΡ€ΠΎΡ‰Π΅ классичСского. ΠŸΠΎΠΏΡ€ΠΎΠ±ΡƒΠΉΡ‚Π΅, для разнообразия, ΠΏΠΎΡΡ‚Ρ€ΠΎΠΈΡ‚ΡŒ Π½Π΅ΠΎΠ±Ρ…ΠΎΠ΄ΠΈΠΌΡ‹Π΅ сСчСния ΠΈ провСсти всС Π΄ΠΎΠΊΠ°Π·Π°Ρ‚Π΅Π»ΡŒΡΡ‚Π²Π° β€” ΠΊΠ°ΠΊ это дСлаСтся Π² «классикС» πŸ™‚

Π‘Ρ‚Ρ€ΠΎΠΈΠΌ Ρ‡Π΅Ρ€Ρ‚Π΅ΠΆ. ΠŸΡ€ΡΠΌΡƒΡŽ Ρ‡Π΅Ρ‚Ρ‹Ρ€Π΅Ρ…ΡƒΠ³ΠΎΠ»ΡŒΠ½ΡƒΡŽ ΠΏΡ€ΠΈΠ·ΠΌΡƒ ΠΌΠΎΠΆΠ½ΠΎ ΠΏΠΎ-Π΄Ρ€ΡƒΠ³ΠΎΠΌΡƒ Π½Π°Π·Π²Π°Ρ‚ΡŒ «ΠΏΠ°Ρ€Π°Π»Π»Π΅Π»Π΅ΠΏΠΈΠΏΠ΅Π΄».

Π—Π°ΠΌΠ΅Ρ‡Π°Π΅ΠΌ, Ρ‡Ρ‚ΠΎ Π΄Π»ΠΈΠ½Π° ΠΈ ΡˆΠΈΡ€ΠΈΠ½Π° ΠΏΠ°Ρ€Π°Π»Π»Π΅Π»Π΅ΠΏΠΈΠΏΠ΅Π΄Π° Ρƒ нас Π΅ΡΡ‚ΡŒ, Π° Π²ΠΎΡ‚ высота β€” Π²Ρ€ΠΎΠ΄Π΅ Π½Π΅ Π΄Π°Π½Π°. Как ΠΆΠ΅ Π΅Π΅ Π½Π°ΠΉΡ‚ΠΈ?

«РасстояниС ΠΌΠ΅ΠΆΠ΄Ρƒ прямыми A1C1 ΠΈ BD Ρ€Π°Π²Π½ΠΎ √3Β». ΠŸΡ€ΡΠΌΡ‹Π΅ A1C1 ΠΈ BD ΡΠΊΡ€Π΅Ρ‰ΠΈΠ²Π°ΡŽΡ‚ΡΡ. Одна ΠΈΠ· Π½ΠΈΡ… β€” диагональ Π²Π΅Ρ€Ρ…Π½Π΅Π³ΠΎ основания, другая β€” диагональ Π½ΠΈΠΆΠ½Π΅Π³ΠΎ. Вспомним, Ρ‡Ρ‚ΠΎ расстояниС ΠΌΠ΅ΠΆΠ΄Ρƒ ΡΠΊΡ€Π΅Ρ‰ΠΈΠ²Π°ΡŽΡ‰ΠΈΠΌΠΈΡΡ прямыми Ρ€Π°Π²Π½ΠΎ Π΄Π»ΠΈΠ½Π΅ ΠΈΡ… ΠΎΠ±Ρ‰Π΅Π³ΠΎ пСрпСндикуляра. ΠžΠ±Ρ‰ΠΈΠΉ пСрпСндикуляр ΠΊ A1C1 ΠΈ BD β€” это, ΠΎΡ‡Π΅Π²ΠΈΠ΄Π½ΠΎ, OO1, Π³Π΄Π΅ O β€” Ρ‚ΠΎΡ‡ΠΊΠ° пСрСсСчСния Π΄ΠΈΠ°Π³ΠΎΠ½Π°Π»Π΅ΠΉ Π½ΠΈΠΆΠ½Π΅Π³ΠΎ основания, O1 β€” Ρ‚ΠΎΡ‡ΠΊΠ° пСрСсСчСния Π΄ΠΈΠ°Π³ΠΎΠ½Π°Π»Π΅ΠΉ Π²Π΅Ρ€Ρ…Π½Π΅Π³ΠΎ. А ΠΎΡ‚Ρ€Π΅Π·ΠΎΠΊ OO1 ΠΈ Ρ€Π°Π²Π΅Π½ высотС ΠΏΠ°Ρ€Π°Π»Π»Π΅Π»Π΅ΠΏΠΈΠΏΠ΅Π΄Π°.

ΠŸΠ»ΠΎΡΠΊΠΎΡΡ‚ΡŒ AA1 D1 D β€” это задняя Π³Ρ€Π°Π½ΡŒ ΠΏΡ€ΠΈΠ·ΠΌΡ‹ Π½Π° нашСм Ρ‡Π΅Ρ€Ρ‚Π΅ΠΆΠ΅. ΠΠΎΡ€ΠΌΠ°Π»ΡŒ ΠΊ Π½Π΅ΠΉ β€” это любой Π²Π΅ΠΊΡ‚ΠΎΡ€, пСрпСндикулярный Π·Π°Π΄Π½Π΅ΠΉ Π³Ρ€Π°Π½ΠΈ, Π½Π°ΠΏΡ€ΠΈΠΌΠ΅Ρ€, Π²Π΅ΠΊΡ‚ΠΎΡ€ ΠΈΠ»ΠΈ, Π΅Ρ‰Π΅ ΠΏΡ€ΠΎΡ‰Π΅, Π²Π΅ΠΊΡ‚ΠΎΡ€ .

ΠžΡΡ‚Π°Π»Π°ΡΡŒ Π΅Ρ‰Π΅ Β«ΠΏΠ»ΠΎΡΠΊΠΎΡΡ‚ΡŒ, проходящая Ρ‡Π΅Ρ€Π΅Π· сСрСдину Ρ€Π΅Π±Ρ€Π° CD пСрпСндикулярно прямой B1DΒ». Но ΠΏΠΎΠ·Π²ΠΎΠ»ΡŒΡ‚Π΅, Ссли ΠΏΠ»ΠΎΡΠΊΠΎΡΡ‚ΡŒ пСрпСндикулярна прямой B1D β€” Π·Π½Π°Ρ‡ΠΈΡ‚, B1D ΠΈ Π΅ΡΡ‚ΡŒ Π½ΠΎΡ€ΠΌΠ°Π»ΡŒ ΠΊ этой плоскости! ΠšΠΎΠΎΡ€Π΄ΠΈΠ½Π°Ρ‚Ρ‹ Ρ‚ΠΎΡ‡Π΅ΠΊ B1 ΠΈ D извСстны:

ΠšΠΎΠΎΡ€Π΄ΠΈΠ½Π°Ρ‚Ρ‹ Π²Π΅ΠΊΡ‚ΠΎΡ€Π° β€” Ρ‚ΠΎΠΆΠ΅:

Находим ΡƒΠ³ΠΎΠ» ΠΌΠ΅ΠΆΠ΄Ρƒ плоскостями, Ρ€Π°Π²Π½Ρ‹ΠΉ ΡƒΠ³Π»Ρƒ ΠΌΠ΅ΠΆΠ΄Ρƒ нормалями ΠΊ Π½ΠΈΠΌ:

Зная косинус ΡƒΠ³Π»Π°, Π½Π°Ρ…ΠΎΠ΄ΠΈΠΌ Π΅Π³ΠΎ тангСнс ΠΏΠΎ Ρ„ΠΎΡ€ΠΌΡƒΠ»Π΅

ΠŸΠΎΠ»ΡƒΡ‡ΠΈΠΌ:

ΠžΡ‚Π²Π΅Ρ‚:

Π£Π³ΠΎΠ» ΠΌΠ΅ΠΆΠ΄Ρƒ прямой m ΠΈ ΠΏΠ»ΠΎΡΠΊΠΎΡΡ‚ΡŒΡŽ Ξ± Ρ‚ΠΎΠΆΠ΅ вычисляСтся с ΠΏΠΎΠΌΠΎΡ‰ΡŒΡŽ скалярного произвСдСния Π²Π΅ΠΊΡ‚ΠΎΡ€ΠΎΠ².

ΠŸΡƒΡΡ‚ΡŒ β€” Π²Π΅ΠΊΡ‚ΠΎΡ€, Π»Π΅ΠΆΠ°Ρ‰ΠΈΠΉ Π½Π° прямой m (ΠΈΠ»ΠΈ ΠΏΠ°Ρ€Π°Π»Π»Π΅Π»ΡŒΠ½Ρ‹ΠΉ Π΅ΠΉ), β€” Π½ΠΎΡ€ΠΌΠ°Π»ΡŒ ΠΊ плоскости Ξ±.

Находим синус ΡƒΠ³Π»Π° ΠΌΠ΅ΠΆΠ΄Ρƒ прямой m ΠΈ ΠΏΠ»ΠΎΡΠΊΠΎΡΡ‚ΡŒΡŽ Ξ± ΠΏΠΎ Ρ„ΠΎΡ€ΠΌΡƒΠ»Π΅:

6. Π’ ΠΊΡƒΠ±Π΅ ABCDA1B1C1D1 Ρ‚ΠΎΡ‡ΠΊΠ° E β€” сСрСдина Ρ€Π΅Π±Ρ€Π° A1B1. НайдитС синус ΡƒΠ³Π»Π° ΠΌΠ΅ΠΆΠ΄Ρƒ прямой AE ΠΈ ΠΏΠ»ΠΎΡΠΊΠΎΡΡ‚ΡŒΡŽ BDD1.

Как всСгда, рисуСм Ρ‡Π΅Ρ€Ρ‚Π΅ΠΆ ΠΈ Π²Ρ‹Π±ΠΈΡ€Π°Π΅ΠΌ систСму ΠΊΠΎΠΎΡ€Π΄ΠΈΠ½Π°Ρ‚

Находим ΠΊΠΎΠΎΡ€Π΄ΠΈΠ½Π°Ρ‚Ρ‹ Π²Π΅ΠΊΡ‚ΠΎΡ€Π° .

НуТно Π»ΠΈ Π½Π°ΠΌ ΡƒΡ€Π°Π²Π½Π΅Π½ΠΈΠ΅ плоскости BDD1? Π’ ΠΎΠ±Ρ‰Π΅ΠΌ-Ρ‚ΠΎ, Π±Π΅Π· Π½Π΅Π³ΠΎ ΠΌΠΎΠΆΠ½ΠΎ ΠΎΠ±ΠΎΠΉΡ‚ΠΈΡΡŒ. Π’Π΅Π΄ΡŒ эта ΠΏΠ»ΠΎΡΠΊΠΎΡΡ‚ΡŒ являСтся Π΄ΠΈΠ°Π³ΠΎΠ½Π°Π»ΡŒΠ½Ρ‹ΠΌ сСчСниСм ΠΊΡƒΠ±Π°, Π° Π·Π½Π°Ρ‡ΠΈΡ‚, Π½ΠΎΡ€ΠΌΠ°Π»ΡŒΡŽ ΠΊ Π½Π΅ΠΉ Π±ΡƒΠ΄Π΅Ρ‚ любой Π²Π΅ΠΊΡ‚ΠΎΡ€, Π΅ΠΉ пСрпСндикулярный. НапримСр, Π²Π΅ΠΊΡ‚ΠΎΡ€ .

НайдСм ΡƒΠ³ΠΎΠ» ΠΌΠ΅ΠΆΠ΄Ρƒ прямой ΠΈ ΠΏΠ»ΠΎΡΠΊΠΎΡΡ‚ΡŒΡŽ:

ΠžΡ‚Π²Π΅Ρ‚:

РасстояниС ΠΎΡ‚ Ρ‚ΠΎΡ‡ΠΊΠΈ M с ΠΊΠΎΠΎΡ€Π΄ΠΈΠ½Π°Ρ‚Π°ΠΌΠΈ x0, y0 ΠΈ z0 Π΄ΠΎ плоскости Ξ±, Π·Π°Π΄Π°Π½Π½ΠΎΠΉ ΡƒΡ€Π°Π²Π½Π΅Π½ΠΈΠ΅ΠΌ Ax + By + Cz + D = 0, ΠΌΠΎΠΆΠ½ΠΎ Π½Π°ΠΉΡ‚ΠΈ ΠΏΠΎ Ρ„ΠΎΡ€ΠΌΡƒΠ»Π΅:

7. Π’ основании ΠΏΡ€ΡΠΌΠΎΡƒΠ³ΠΎΠ»ΡŒΠ½ΠΎΠ³ΠΎ ΠΏΠ°Ρ€Π°Π»Π»Π΅Π»Π΅ΠΏΠΈΠΏΠ΅Π΄Π° BCDA1B1C1D1 Π»Π΅ΠΆΠΈΡ‚ ΠΏΡ€ΡΠΌΠΎΡƒΠ³ΠΎΠ»ΡŒΠ½ΠΈΠΊ ABCD со сторонами AB = , AD = . Высота ΠΏΠ°Ρ€Π°Π»Π»Π΅Π»Π΅ΠΏΠΈΠΏΠ΅Π΄Π° AA1 = . НайдитС расстояниС ΠΎΡ‚ Ρ‚ΠΎΡ‡ΠΊΠΈ A Π΄ΠΎ плоскости A1DB.

ΠŸΠΎΡΡ‚Ρ€ΠΎΠΈΠΌ Ρ‡Π΅Ρ€Ρ‚Π΅ΠΆ ΠΈ Π²Ρ‹ΠΏΠΈΡˆΠ΅ΠΌ ΠΊΠΎΠΎΡ€Π΄ΠΈΠ½Π°Ρ‚Ρ‹ Ρ‚ΠΎΡ‡Π΅ΠΊ:

Π—Π°ΠΏΠΈΡˆΠ΅ΠΌ ΡƒΡ€Π°Π²Π½Π΅Π½ΠΈΠ΅ плоскости A1DB. Π’Ρ‹ ΠΏΠΎΠΌΠ½ΠΈΡ‚Π΅, ΠΊΠ°ΠΊ это дСлаСтся β€” ΠΏΠΎ ΠΎΡ‡Π΅Ρ€Π΅Π΄ΠΈ подставляСм ΠΊΠΎΠΎΡ€Π΄ΠΈΠ½Π°Ρ‚Ρ‹ Ρ‚ΠΎΡ‡Π΅ΠΊ A1, D ΠΈ B Π² ΡƒΡ€Π°Π²Π½Π΅Π½ΠΈΠ΅ Ax + Be + Cz + D

РСшим эту систСму. Π’Ρ‹Π±Π΅Ρ€Π΅ΠΌ

Π’ΠΎΠ³Π΄Π°

Π£Ρ€Π°Π²Π½Π΅Π½ΠΈΠ΅ плоскости A1DB ΠΈΠΌΠ΅Π΅Ρ‚ Π²ΠΈΠ΄:

Π”Π°Π»ΡŒΡˆΠ΅ всС просто. Находим расстояниС ΠΎΡ‚ Ρ‚ΠΎΡ‡ΠΊΠΈ A Π΄ΠΎ плоскости A1DB:

Π’ Π½Π΅ΠΊΠΎΡ‚ΠΎΡ€Ρ‹Ρ… Π·Π°Π΄Π°Ρ‡Π°Ρ… ΠΏΠΎ стСрСомСтрии трСбуСтся Π½Π°ΠΉΡ‚ΠΈ расстояниС ΠΎΡ‚ прямой Π΄ΠΎ ΠΏΠ°Ρ€Π°Π»Π»Π΅Π»ΡŒΠ½ΠΎΠΉ Π΅ΠΉ плоскости. Π’ этом случаС ΠΌΠΎΠΆΠ½ΠΎ Π²Ρ‹Π±Ρ€Π°Ρ‚ΡŒ Π»ΡŽΠ±ΡƒΡŽ Ρ‚ΠΎΡ‡ΠΊΡƒ, ΠΏΡ€ΠΈΠ½Π°Π΄Π»Π΅ΠΆΠ°Ρ‰ΡƒΡŽ Π΄Π°Π½Π½ΠΎΠΉ прямой.

Π—Π²ΠΎΠ½ΠΈΡ‚Π΅ Π½Π°ΠΌ: 8 (800) 775-06-82 (бСсплатный Π·Π²ΠΎΠ½ΠΎΠΊ ΠΏΠΎ России) +7 (495) 984-09-27 (бСсплатный Π·Π²ΠΎΠ½ΠΎΠΊ ΠΏΠΎ МосквС)

Или Π½Π°ΠΆΠΌΠΈΡ‚Π΅ Π½Π° ΠΊΠ½ΠΎΠΏΠΊΡƒ Β«Π£Π·Π½Π°Ρ‚ΡŒ большС», Ρ‡Ρ‚ΠΎΠ±Ρ‹ Π·Π°ΠΏΠΎΠ»Π½ΠΈΡ‚ΡŒ ΠΊΠΎΠ½Ρ‚Π°ΠΊΡ‚Π½ΡƒΡŽ Ρ„ΠΎΡ€ΠΌΡƒ. ΠœΡ‹ ΠΎΠ±ΡΠ·Π°Ρ‚Π΅Π»ΡŒΠ½ΠΎ Π’Π°ΠΌ ΠΏΠ΅Ρ€Π΅Π·Π²ΠΎΠ½ΠΈΠΌ.

ΠšΠ°Π»ΡŒΠΊΡƒΠ»ΡΡ‚ΠΎΡ€ слуТит для вычислСния Π΄Π»ΠΈΠ½Ρ‹ Π²Π΅ΠΊΡ‚ΠΎΡ€Π° Π² Ρ‚Ρ€Π΅Ρ…ΠΌΠ΅Ρ€Π½ΠΎΠΌ пространствС.

Π”Π»ΠΈΠ½ΠΎΠΉ ΠΈΠ»ΠΈ ΠΌΠΎΠ΄ΡƒΠ»Π΅ΠΌ Π²Π΅ΠΊΡ‚ΠΎΡ€Π° ΠΌΡ‹ Π½Π°Π·Ρ‹Π²Π°Π΅ΠΌ Π΄Π»ΠΈΠ½Ρƒ ΠΎΡ‚Ρ€Π΅Π·ΠΊΠ°, ΠΈΠ·ΠΎΠ±Ρ€Π°ΠΆΠ°ΡŽΡ‰Π΅Π³ΠΎ Π΄Π°Π½Π½Ρ‹ΠΉ Π²Π΅ΠΊΡ‚ΠΎΡ€.

Π€ΠΎΡ€ΠΌΡƒΠ»Π° для опрСдСлСния Π΄Π»ΠΈΠ½Ρ‹ Π²Π΅ΠΊΡ‚ΠΎΡ€Π° a Π² Ρ‚Ρ€Π΅Ρ…ΠΌΠ΅Ρ€Π½ΠΎΠΌ пространствС Ρ‡Π΅Ρ€Π΅Π· Π΅Π³ΠΎ ΠΊΠΎΠΎΡ€Π΄ΠΈΠ½Π°Ρ‚Ρ‹ прСдставлСна Π½ΠΈΠΆΠ΅.

Π§Ρ‚ΠΎΠ±Ρ‹ произвСсти вычислСниС Π΄Π»ΠΈΠ½Ρ‹ Π²Π΅ΠΊΡ‚ΠΎΡ€Π° Π² Ρ‚Ρ€Π΅Ρ…ΠΌΠ΅Ρ€Π½ΠΎΠΌ пространствС, Π²Π²Π΅Π΄ΠΈΡ‚Π΅ Π² ΡΠΎΠΎΡ‚Π²Π΅Ρ‚ΡΡ‚Π²ΡƒΡŽΡ‰ΠΈΡ… полях Π·Π½Π°Ρ‡Π΅Π½ΠΈΠ΅ ΠΊΠΎΠΎΡ€Π΄ΠΈΠ½Π°Ρ‚Ρ‹ x ΠΈ ΠΊΠΎΠΎΡ€Π΄ΠΈΠ½Π°Ρ‚Ρ‹ y ΠΈ ΠΊΠΎΠΎΡ€Π΄ΠΈΠ½Π°Ρ‚Ρ‹ z, послС Ρ‡Π΅Π³ΠΎ Π½Π°ΠΆΠΌΠΈΡ‚Π΅ ΠΊΠ½ΠΎΠΏΠΊΡƒ » Π’Π«Π§Π˜Π‘Π›Π˜Π’Π¬ «.

Π Π΅Π·ΡƒΠ»ΡŒΡ‚Π°Ρ‚ΠΎΠΌ Ρ€Π°Π±ΠΎΡ‚Ρ‹ ΠΏΡ€ΠΎΠ³Ρ€Π°ΠΌΠΌΡ‹ Π±ΡƒΠ΄Π΅Ρ‚ Π·Π½Π°Ρ‡Π΅Π½ΠΈΠ΅ Π΄Π»ΠΈΠ½Ρ‹ Π²Π΅ΠΊΡ‚ΠΎΡ€Π° Π² Ρ‚Ρ€Π΅Ρ…ΠΌΠ΅Ρ€Π½ΠΎΠΌ пространствС.

Π—Π°Ρ‡Π΅Ρ‚Π½Ρ‹ΠΉ ΡƒΡ€ΠΎΠΊ ΠΏΠΎ Ρ‚Π΅ΠΌΠ΅: «ΠœΠ΅Ρ‚ΠΎΠ΄ ΠΊΠΎΠΎΡ€Π΄ΠΈΠ½Π°Ρ‚»

Π Π°Π·Π΄Π΅Π»Ρ‹: ΠœΠ°Ρ‚Π΅ΠΌΠ°Ρ‚ΠΈΠΊΠ°


Π¦Π΅Π»ΠΈ ΡƒΡ€ΠΎΠΊΠ°:

  • подвСсти ΠΈΡ‚ΠΎΠ³ ΠΏΠΎ ΠΈΠ·ΡƒΡ‡Π΅Π½ΠΈΡŽ Ρ‚Π΅ΠΌΡ‹,
  • ΠΏΡ€ΠΎΠ²Π΅Ρ€ΠΈΡ‚ΡŒ усвоСнныС знания,
  • ΠΏΠΎΠ΄Π³ΠΎΡ‚ΠΎΠ²ΠΊΠ° ΠΊ ΠΊΠΎΠ½Ρ‚Ρ€ΠΎΠ»ΡŒΠ½ΠΎΠΉ Ρ€Π°Π±ΠΎΡ‚Π΅.

Класс разбиваСтся Π½Π° 5 Π³Ρ€ΡƒΠΏΠΏ ΠΏΠΎ 5 Ρ‡Π΅Π»ΠΎΠ²Π΅ΠΊ. Π‘Ρ‚Π°Ρ€ΡˆΠΈΠΌΠΈ Π² Π³Ρ€ΡƒΠΏΠΏΠ΅ Π½Π°Π·Π½Π°Ρ‡Π°ΡŽΡ‚ΡΡ ΡƒΡΠΏΠ΅Π²Π°ΡŽΡ‰ΠΈΠ΅ ΡƒΡ‡Π΅Π½ΠΈΠΊΠΈ (ΠΏΠΎΠΌΠΎΡ‰Π½ΠΈΠΊΠΈ учитСля).

1 этап. ΠžΡ€Π³Π°Π½ΠΈΠ·Π°Ρ†ΠΈΠΎΠ½Π½Ρ‹ΠΉ ΠΌΠΎΠΌΠ΅Π½Ρ‚. ΠŸΠΎΡΡ‚Π°Π½ΠΎΠ²ΠΊΠ° Ρ†Π΅Π»Π΅ΠΉ

2 этап. ΠžΡ‚Π²Π΅Ρ‚Ρ‹ Π½Π° вопросы с использованиС ΡΠΈΠ³Π½Π°Π»ΡŒΠ½Ρ‹Ρ… ΠΊΠ°Ρ€Ρ‚ (15 ΠΌΠΈΠ½ΡƒΡ‚)

КаТдой Π³Ρ€ΡƒΠΏΠΏΠ΅ выдаСтся ΠΏΡƒΡ‚Π΅Π²ΠΎΠΉ лист, Π² ΠΊΠΎΡ‚ΠΎΡ€ΠΎΠΌ ΠΏΠΎΠΌΠΎΡ‰Π½ΠΈΠΊΠΈ Π±ΡƒΠ΄ΡƒΡ‚ ΠΎΡ‚ΠΌΠ΅Ρ‡Π°Ρ‚ΡŒ Π·Π½Π°Π½ΠΈΠ΅/Π½Π΅Π·Π½Π°Π½ΠΈΠ΅ ΠΎΡ‚Π²Π΅Ρ‚Π° Π½Π° вопрос. ΠšΠ°ΠΆΠ΄ΠΎΠΌΡƒ учащСмуся Π²Ρ‹Π΄Π°ΡŽΡ‚ΡΡ Π΄Π²Π΅ ΠΊΠ°Ρ€Ρ‚ΠΎΡ‡ΠΊΠΈ: зСлСная ΠΈ красная. Как Ρ‚ΠΎΠ»ΡŒΠΊΠΎ вопрос сформулирован, ΡƒΡ‡Π΅Π½ΠΈΠΊΠΈ ΠΏΠΎΠ΄Π½ΠΈΠΌΠ°ΡŽΡ‚ ΡΠΈΠ³Π½Π°Π»ΡŒΠ½Ρ‹Π΅ ΠΊΠ°Ρ€Ρ‚Ρ‹ (знаю ΠΎΡ‚Π²Π΅Ρ‚ – зСлСная ΠΊΠ°Ρ€Ρ‚ΠΎΡ‡ΠΊΠ°, Π½Π΅ знаю ΠΎΡ‚Π²Π΅Ρ‚ – красная ΠΊΠ°Ρ€Ρ‚ΠΎΡ‡ΠΊΠ°). Π—Π°Ρ‚Π΅ΠΌ ΠΎΠ΄ΠΈΠ½ ΡƒΡ‡Π΅Π½ΠΈΠΊ ΠΏΠΎ ΠΏΡ€ΠΎΡΡŒΠ±Π΅ учитСля вслух Π΄Π°Π΅Ρ‚ ΠΎΡ‚Π²Π΅Ρ‚ Π½Π° поставлСнный вопрос.

Вопрос 1. Π‘Ρ„ΠΎΡ€ΠΌΡƒΠ»ΠΈΡ€ΡƒΠΉΡ‚Π΅ Π»Π΅ΠΌΠΌΡƒ ΠΎ Π΄Π²ΡƒΡ… ΠΊΠΎΠ»Π»ΠΈΠ½Π΅Π°Ρ€Π½Ρ‹Ρ… Π²Π΅ΠΊΡ‚ΠΎΡ€Π°Ρ….

(Если Π²Π΅ΠΊΡ‚ΠΎΡ€Ρ‹ ΠΈ ΠΊΠΎΠ»Π»ΠΈΠ½Π΅Π°Ρ€Π½Ρ‹ ΠΈ , Ρ‚ΠΎ сущСствуСт Ρ‚Π°ΠΊΠΎΠ΅ число k, Ρ‡Ρ‚ΠΎ ).

Вопрос 2. Π‘Ρ„ΠΎΡ€ΠΌΡƒΠ»ΠΈΡ€ΡƒΠΉΡ‚Π΅ Ρ‚Π΅ΠΎΡ€Π΅ΠΌΡƒ ΠΎ Ρ€Π°Π·Π»ΠΎΠΆΠ΅Π½ΠΈΠΈ Π²Π΅ΠΊΡ‚ΠΎΡ€Π° ΠΏΠΎ Π΄Π²ΡƒΠΌ Π½Π΅ΠΊΠΎΠ»Π»ΠΈΠ½Π΅Π°Ρ€Π½Ρ‹ΠΌ Π²Π΅ΠΊΡ‚ΠΎΡ€Π°ΠΌ.

(Π›ΡŽΠ±ΠΎΠΉ Π²Π΅ΠΊΡ‚ΠΎΡ€ ΠΌΠΎΠΆΠ½ΠΎ Ρ€Π°Π·Π»ΠΎΠΆΠΈΡ‚ΡŒ ΠΏΠΎ Π΄Π²ΡƒΠΌ Π΄Π°Π½Π½Ρ‹ΠΌ Π½Π΅ΠΊΠΎΠ»Π»ΠΈΠ½Π΅Π°Ρ€Π½Ρ‹ΠΌ Π²Π΅ΠΊΡ‚ΠΎΡ€Π°ΠΌ, ΠΏΡ€ΠΈΡ‡Π΅ΠΌ коэффициСнты разлоТСния ΠΎΠΏΡ€Π΅Π΄Π΅Π»ΡΡŽΡ‚ΡΡ СдинствСнным ΠΎΠ±Ρ€Π°Π·ΠΎΠΌ).

Вопрос 3. Π§Ρ‚ΠΎ Ρ‚Π°ΠΊΠΎΠ΅ ΠΊΠΎΠΎΡ€Π΄ΠΈΠ½Π°Ρ‚Ρ‹ Π²Π΅ΠΊΡ‚ΠΎΡ€Π°?

(Π­Ρ‚ΠΎ коэффициСнты разлоТСния Π²Π΅ΠΊΡ‚ΠΎΡ€Π° ΠΏΠΎ ΠΊΠΎΠΎΡ€Π΄ΠΈΠ½Π°Ρ‚Π½Ρ‹ΠΌ Π²Π΅ΠΊΡ‚ΠΎΡ€Π°ΠΌ).

Вопрос 4. Π‘Ρ„ΠΎΡ€ΠΌΡƒΠ»ΠΈΡ€ΡƒΠΉΡ‚Π΅ ΠΏΡ€Π°Π²ΠΈΠ»ΠΎ нахоТдСния ΠΊΠΎΠΎΡ€Π΄ΠΈΠ½Π°Ρ‚ суммы Π²Π΅ΠΊΡ‚ΠΎΡ€ΠΎΠ².

(КаТдая ΠΊΠΎΠΎΡ€Π΄ΠΈΠ½Π°Ρ‚Π° суммы Π΄Π²ΡƒΡ… ΠΈΠ»ΠΈ Π±ΠΎΠ»Π΅Π΅ Π²Π΅ΠΊΡ‚ΠΎΡ€ΠΎΠ² Ρ€Π°Π²Π½Π° суммС ΡΠΎΠΎΡ‚Π²Π΅Ρ‚ΡΡ‚Π²ΡƒΡŽΡ‰ΠΈΡ… ΠΊΠΎΠΎΡ€Π΄ΠΈΠ½Π°Ρ‚ этих Π²Π΅ΠΊΡ‚ΠΎΡ€ΠΎΠ²).

Вопрос 5. Π‘Ρ„ΠΎΡ€ΠΌΡƒΠ»ΠΈΡ€ΡƒΠΉΡ‚Π΅ ΠΏΡ€Π°Π²ΠΈΠ»ΠΎ нахоТдСния ΠΊΠΎΠΎΡ€Π΄ΠΈΠ½Π°Ρ‚ разности Π²Π΅ΠΊΡ‚ΠΎΡ€ΠΎΠ².

(КаТдая ΠΊΠΎΠΎΡ€Π΄ΠΈΠ½Π°Ρ‚Π° разности Π΄Π²ΡƒΡ… ΠΈΠ»ΠΈ Π±ΠΎΠ»Π΅Π΅ Π²Π΅ΠΊΡ‚ΠΎΡ€ΠΎΠ² Ρ€Π°Π²Π½Π° разности ΡΠΎΠΎΡ‚Π²Π΅Ρ‚ΡΡ‚Π²ΡƒΡŽΡ‰ΠΈΡ… ΠΊΠΎΠΎΡ€Π΄ΠΈΠ½Π°Ρ‚ этих Π²Π΅ΠΊΡ‚ΠΎΡ€ΠΎΠ²).

Вопрос 6. Π‘Ρ„ΠΎΡ€ΠΌΡƒΠ»ΠΈΡ€ΡƒΠΉΡ‚Π΅ ΠΏΡ€Π°Π²ΠΈΠ»ΠΎ нахоТдСния ΠΊΠΎΠΎΡ€Π΄ΠΈΠ½Π°Ρ‚ произвСдСния Π²Π΅ΠΊΡ‚ΠΎΡ€Π° Π½Π° число.

(КаТдая ΠΊΠΎΠΎΡ€Π΄ΠΈΠ½Π°Ρ‚Π° произвСдСния Π²Π΅ΠΊΡ‚ΠΎΡ€Π° Π½Π° число Ρ€Π°Π²Π½Π° ΠΏΡ€ΠΎΠΈΠ·Π²Π΅Π΄Π΅Π½ΠΈΡŽ ΡΠΎΠΎΡ‚Π²Π΅Ρ‚ΡΡ‚Π²ΡƒΡŽΡ‰ΠΈΡ… ΠΊΠΎΠΎΡ€Π΄ΠΈΠ½Π°Ρ‚ Π²Π΅ΠΊΡ‚ΠΎΡ€Π° Π½Π° это число).

Вопрос 7. ΠΠ°ΠΏΠΈΡˆΠΈΡ‚Π΅ Π½Π° доскС Ρ„ΠΎΡ€ΠΌΡƒΠ»Ρƒ для вычислСния ΠΊΠΎΠΎΡ€Π΄ΠΈΠ½Π°Ρ‚ Π²Π΅ΠΊΡ‚ΠΎΡ€Π° ΠΏΠΎ ΠΊΠΎΠΎΡ€Π΄ΠΈΠ½Π°Ρ‚Π°ΠΌ Π΅Π³ΠΎ Π½Π°Ρ‡Π°Π»Π° ΠΈ ΠΊΠΎΠ½Ρ†Π°.

()

Вопрос 8. ΠΠ°ΠΏΠΈΡˆΠΈΡ‚Π΅ Π½Π° доскС Ρ„ΠΎΡ€ΠΌΡƒΠ»Ρƒ для вычислСния ΠΊΠΎΠΎΡ€Π΄ΠΈΠ½Π°Ρ‚ сСрСдины ΠΎΡ‚Ρ€Π΅Π·ΠΊΠ° ΠΏΠΎ ΠΊΠΎΠΎΡ€Π΄ΠΈΠ½Π°Ρ‚Π°ΠΌ Π΅Π³ΠΎ ΠΊΠΎΠ½Ρ†ΠΎΠ².

()

Вопрос 9. ΠΠ°ΠΏΠΈΡˆΠΈΡ‚Π΅ Π½Π° доскС Ρ„ΠΎΡ€ΠΌΡƒΠ»Ρƒ для вычислСния Π΄Π»ΠΈΠ½Ρ‹ Π²Π΅ΠΊΡ‚ΠΎΡ€Π° ΠΏΠΎ Π΅Π³ΠΎ ΠΊΠΎΠΎΡ€Π΄ΠΈΠ½Π°Ρ‚Π°ΠΌ.

()

Вопрос 10. ΠΠ°ΠΏΠΈΡˆΠΈΡ‚Π΅ Π½Π° доскС Ρ„ΠΎΡ€ΠΌΡƒΠ»Ρƒ для вычислСния расстояния ΠΌΠ΅ΠΆΠ΄Ρƒ двумя Ρ‚ΠΎΡ‡ΠΊΠ°ΠΌΠΈ.

()

Вопрос 11. ΠΠ°ΠΏΠΈΡˆΠΈΡ‚Π΅ Π½Π° доскС ΡƒΡ€Π°Π²Π½Π΅Π½ΠΈΠ΅ окруТности Π΄Π°Π½Π½ΠΎΠ³ΠΎ радиуса с Ρ†Π΅Π½Ρ‚Ρ€ΠΎΠΌ Π² Π΄ΠΎΠ½Π½ΠΎΠΉ Ρ‚ΠΎΡ‡ΠΊΠ΅ ΠΈ ΡƒΡ€Π°Π²Π½Π΅Π½ΠΈΠ΅ окруТности с Ρ†Π΅Π½Ρ‚Ρ€ΠΎΠΌ Π² Π½Π°Ρ‡Π°Π»Π΅ ΠΊΠΎΠΎΡ€Π΄ΠΈΠ½Π°Ρ‚.

Вопрос 12. ΠΠ°ΠΏΠΈΡˆΠΈΡ‚Π΅ ΡƒΡ€Π°Π²Π½Π΅Π½ΠΈΠ΅ Π΄Π°Π½Π½ΠΎΠΉ прямой Π² ΠΏΡ€ΡΠΌΠΎΡƒΠ³ΠΎΠ»ΡŒΠ½ΠΎΠΉ систСмС ΠΊΠΎΠΎΡ€Π΄ΠΈΠ½Π°Ρ‚.

(ax+by+c=0).

3 этап. Π’Ρ‹ΠΏΠΎΠ»Π½Π΅Π½ΠΈΠ΅ ΠΈΠ½Π΄ΠΈΠ²ΠΈΠ΄ΡƒΠ°Π»ΡŒΠ½ΠΎΠ³ΠΎ задания (10 ΠΌΠΈΠ½ΡƒΡ‚)

ΠšΠ°ΠΆΠ΄ΠΎΠΌΡƒ ΡƒΡ‡Π΅Π½ΠΈΠΊΡƒ Π²Ρ‹Π΄Π°ΡŽΡ‚ΡΡ задания, ΠΊΠΎΡ‚ΠΎΡ€Ρ‹Π΅ ΠΎΠ½ΠΈ Π΄ΠΎΠ»ΠΆΠ½Ρ‹ Π²Ρ‹ΠΏΠΎΠ»Π½ΠΈΡ‚ΡŒ ΡΠ°ΠΌΠΎΡΡ‚ΠΎΡΡ‚Π΅Π»ΡŒΠ½ΠΎ.

ΠŸΡ€ΠΈΠΌΠ΅Ρ€Π½Ρ‹Π΅ Π²Π°Ρ€ΠΈΠ°Π½Ρ‚Ρ‹ ΠΊΠ°Ρ€Ρ‚ΠΎΡ‡Π΅ΠΊ.

ΠšΠ°Ρ€Ρ‚ΠΎΡ‡ΠΊΠ° 1.

  1. Π›Π΅ΠΆΠΈΡ‚ Π»ΠΈ Ρ‚ΠΎΡ‡ΠΊΠ° A(2;-1) Π½Π° окруТности, Π·Π°Π΄Π°Π½Π½ΠΎΠΉ ΡƒΡ€Π°Π²Π½Π΅Π½ΠΈΠ΅ΠΌ ?
  2. НайдитС Π΄Π»ΠΈΠ½Ρƒ Π²Π΅ΠΊΡ‚ΠΎΡ€Π° .
  3. НайдитС ΠΊΠΎΠΎΡ€Π΄ΠΈΠ½Π°Ρ‚Ρ‹ сСрСдины ΠΎΡ‚Ρ€Π΅Π·ΠΊΠ° PQ, Ссли P(5;-3), Q(3;-7).

ΠšΠ°Ρ€Ρ‚ΠΎΡ‡ΠΊΠ° 2.

  1. ΠΠ°ΠΏΠΈΡˆΠΈΡ‚Π΅ ΡƒΡ€Π°Π²Π½Π΅Π½ΠΈΠ΅ окруТности, Ссли Π΅Π΅ Ρ†Π΅Π½Ρ‚Ρ€ – Ρ‚ΠΎΡ‡ΠΊΠ° (4; 5), Π° радиус Ρ€Π°Π²Π΅Π½ 3.
  2. НайдитС ΠΊΠΎΠΎΡ€Π΄ΠΈΠ½Π°Ρ‚Ρ‹ Π²Π΅ΠΊΡ‚ΠΎΡ€Π° , Ссли A(2;-5), B(-3;4).
  3. ΠΠ°ΠΏΠΈΡˆΠΈΡ‚Π΅ ΡƒΡ€Π°Π²Π½Π΅Π½ΠΈΠ΅ прямой, проходящСй Ρ‡Π΅Ρ€Π΅Π· Ρ‚ΠΎΡ‡ΠΊΡƒ M(3;-2) ΠΈ ΠΏΠ°Ρ€Π°Π»Π»Π΅Π»ΡŒΠ½ΠΎΠΉ оси ΠΎΡ€Π΄ΠΈΠ½Π°Ρ‚.

ΠšΠ°Ρ€Ρ‚ΠΎΡ‡ΠΊΠ° 3.

  1. ΠΠ°ΠΏΠΈΡˆΠΈΡ‚Π΅ ΡƒΡ€Π°Π²Π½Π΅Π½ΠΈΠ΅ окруТности с Ρ†Π΅Π½Ρ‚Ρ€ΠΎΠΌ Π² Π½Π°Ρ‡Π°Π»Π΅ ΠΊΠΎΠΎΡ€Π΄ΠΈΠ½Π°Ρ‚, Ссли ΠΎΠ½Π° ΠΏΡ€ΠΎΡ…ΠΎΠ΄ΠΈΡ‚ Ρ‡Π΅Ρ€Π΅Π· Ρ‚ΠΎΡ‡ΠΊΡƒ C(2;-1).
  2. НайдитС расстояниС ΠΌΠ΅ΠΆΠ΄Ρƒ Ρ‚ΠΎΡ‡ΠΊΠ°ΠΌΠΈ A(-1; 3) ΠΈ B(2; -1).
  3. НайдитС ΠΊΠΎΠΎΡ€Π΄ΠΈΠ½Π°Ρ‚Ρ‹ Π²Π΅ΠΊΡ‚ΠΎΡ€Π° , Ρ€Π°Π²Π½ΠΎΠ³ΠΎ суммС Π²Π΅ΠΊΡ‚ΠΎΡ€ΠΎΠ² ΠΈ , Ссли .

ΠšΠ°Ρ€Ρ‚ΠΎΡ‡ΠΊΠ° 4.

  1. Π›Π΅ΠΆΠΈΡ‚ Π»ΠΈ Ρ‚ΠΎΡ‡ΠΊΠ° A(2;-1) Π½Π° прямой, Π·Π°Π΄Π°Π½Π½ΠΎΠΉ ΡƒΡ€Π°Π²Π½Π΅Π½ΠΈΠ΅ΠΌ 2x-3y-7=0.
  2. ΠΠ°ΠΏΠΈΡˆΠΈΡ‚Π΅ ΡƒΡ€Π°Π²Π½Π΅Π½ΠΈΠ΅ окруТности, Ссли Π΅Π΅ Ρ†Π΅Π½Ρ‚Ρ€ – Ρ‚ΠΎΡ‡ΠΊΠ° (4;5), Π° радиус Ρ€Π°Π²Π΅Π½ 2.
  3. НайдитС ΠΊΠΎΠΎΡ€Π΄ΠΈΠ½Π°Ρ‚Ρ‹ Π²Π΅ΠΊΡ‚ΠΎΡ€Π° , Ρ€Π°Π²Π½ΠΎΠ³ΠΎ разности Π²Π΅ΠΊΡ‚ΠΎΡ€ΠΎΠ²Β  ΠΈ , Ссли .

ΠšΠ°Ρ€Ρ‚ΠΎΡ‡ΠΊΠ° 5.

  1. ΠΠ°ΠΏΠΈΡˆΠΈΡ‚Π΅ ΡƒΡ€Π°Π²Π½Π΅Π½ΠΈΠ΅ окруТности с Ρ†Π΅Π½Ρ‚Ρ€ΠΎΠΌ Π² Ρ‚ΠΎΡ‡ΠΊΠ΅ P(-2;-1), Ссли ΠΎΠ½Π° ΠΏΡ€ΠΎΡ…ΠΎΠ΄ΠΈΡ‚ Ρ‡Π΅Ρ€Π΅Π· Ρ‚ΠΎΡ‡ΠΊΡƒ Q(1;3).
  2. НайдитС ΠΊΠΎΠΎΡ€Π΄ΠΈΠ½Π°Ρ‚Ρ‹ Π²Π΅ΠΊΡ‚ΠΎΡ€Π° , Ссли C(-1; 6), D(3; -2).
  3. НайдитС ΠΊΠΎΠΎΡ€Π΄ΠΈΠ½Π°Ρ‚Ρ‹ Π²Π΅ΠΊΡ‚ΠΎΡ€Π° , Ссли , Π° .

4 этап. Π“Ρ€ΡƒΠΏΠΏΠΎΠ²ΠΎΠ΅ Ρ€Π΅ΡˆΠ΅Π½ΠΈΠ΅ Π·Π°Π΄Π°Ρ‡ΠΈ (15 ΠΌΠΈΠ½ΡƒΡ‚)

КаТдой Π³Ρ€ΡƒΠΏΠΏΠ΅ даСтся Π·Π°Π΄Π°Ρ‡Π°.

Π”Π°Π½Ρ‹ ΠΊΠΎΠΎΡ€Π΄ΠΈΠ½Π°Ρ‚Ρ‹ Ρ‚Ρ€Π΅Ρ… Π²Π΅Ρ€ΡˆΠΈΠ½ ΠΏΠ°Ρ€Π°Π»Π»Π΅Π»ΠΎΠ³Ρ€Π°ΠΌΠΌΠ° KLMN: K(-4; 2), L(0; 5), M(12; 0). НайдитС ΠΊΠΎΠΎΡ€Π΄ΠΈΠ½Π°Ρ‚Ρ‹ Ρ‡Π΅Ρ‚Π²Π΅Ρ€Ρ‚ΠΎΠΉ Π²Π΅Ρ€ΡˆΠΈΠ½Ρ‹ ΠΈ ΠΏΠ΅Ρ€ΠΈΠΌΠ΅Ρ‚Ρ€ Π΄Π°Π½Π½ΠΎΠ³ΠΎ ΠΏΠ°Ρ€Π°Π»Π»Π΅Π»ΠΎΠ³Ρ€Π°ΠΌΠΌΠ°.

ΠŸΡ€Π΅Π΄ΡΡ‚Π°Π²ΠΈΡ‚Π΅Π»ΡŒ Π³Ρ€ΡƒΠΏΠΏΡ‹, ΠΏΠ΅Ρ€Π²ΠΎΠΉ ΠΏΡ€Π°Π²ΠΈΠ»ΡŒΠ½ΠΎ Ρ€Π΅ΡˆΠΈΠ²ΡˆΠ΅ΠΉ Π·Π°Π΄Π°Ρ‡Ρƒ, ΠΎΠ±ΡŠΡΡΠ½ΡΠ΅Ρ‚ Ρ€Π΅ΡˆΠ΅Π½ΠΈΠ΅ Ρƒ доски.

5 этап. Π”ΠΎΠΌΠ°ΡˆΠ½Π΅Π΅ Π·Π°Π΄Π°Π½ΠΈΠ΅

По ΡƒΡ‡Π΅Π±Π½ΠΈΠΊΡƒ Π›. Π‘. Атанасяна, β„–β„– 996, 1001, 1005.

2$$ ΠΈ Π·Π°Π΄Π°ΠΉΡ‚Π΅ коэффициСнты $a,b,c$ Π² Π²ΠΈΠ΄Π΅ ΠΊΠΎΠΎΡ€Π΄ΠΈΠ½Π°Ρ‚Π½ΠΎΠ³ΠΎ Π²Π΅ΠΊΡ‚ΠΎΡ€Π° Π²ΠΈΠ΄Π° $\langle a,b,c \rangle$ — ΠΊΠΎΠΎΡ€Π΄ΠΈΠ½Π°Ρ‚Ρ‹ ΠΎΡ‚Π½ΠΎΡΠΈΡ‚Π΅Π»ΡŒΠ½ΠΎ Π½ΠΎΠ²ΠΎΠ³ΠΎ базиса.

Помимо Π»ΠΈΠ½Π΅ΠΉΠ½ΠΎ-алгСбраичСских ΠΌΠ΅Ρ‚ΠΎΠ΄ΠΎΠ² смСны базиса, ΠΌΠΎΠΆΠ½ΠΎ ΠΏΠ΅Ρ€Π΅ΠΏΠΈΡΠ°Ρ‚ΡŒ ΠΌΠ½ΠΎΠ³ΠΎΡ‡Π»Π΅Π½ Π² Ρ‚Ρ€Π΅Π±ΡƒΠ΅ΠΌΠΎΠΌ Π²ΠΈΠ΄Π΅ судСбным ΠΏΡ€ΠΈΠΌΠ΅Π½Π΅Π½ΠΈΠ΅ΠΌ дополнСния ΠΊΠ²Π°Π΄Ρ€Π°Ρ‚Π° ΠΈ разлоТСния Π½Π° ΠΌΠ½ΠΎΠΆΠΈΡ‚Π΅Π»ΠΈ, Π»ΠΈΠ±ΠΎ ΠΎΡ‡Π΅Π½ΡŒ простым ΠΏΡ€ΠΈΠΌΠ΅Π½Π΅Π½ΠΈΠ΅ΠΌ Ρ‚Π΅ΠΎΡ€Π΅ΠΌΡ‹ Π’Π΅ΠΉΠ»ΠΎΡ€Π° для нахоТдСния коэффициСнтов ряда Π’Π΅ΠΉΠ»ΠΎΡ€Π° с Ρ†Π΅Π½Ρ‚Ρ€ΠΎΠΌ Π² $t = 2$ (это, вСроятно, самый эффСктивный ΠΌΠ΅Ρ‚ΠΎΠ΄). Π’ качСствС Π°Π»ΡŒΡ‚Π΅Ρ€Π½Π°Ρ‚ΠΈΠ²Ρ‹ Π²Ρ‹ ΠΌΠΎΠΆΠ΅Ρ‚Π΅ Ρ€Π°Π·Π»ΠΎΠΆΠΈΡ‚ΡŒ ΠΏΠΎΠ»ΠΈΠ½ΠΎΠΌ Π²Ρ‹ΡˆΠ΅ ΠΈ Ρ€Π΅ΡˆΠΈΡ‚ΡŒ систСму ΡƒΡ€Π°Π²Π½Π΅Π½ΠΈΠΉ для Π½Π΅ΠΎΠΏΡ€Π΅Π΄Π΅Π»Π΅Π½Π½Ρ‹Ρ… коэффициСнтов. 92 — 4t + 4$ :

$${\bf T}= \left[\begin{array}{rrr} 1&-2&4\\ 0&1&-4\\ 0&0&1 \end{массив}\right]$$ Π’Ρ‹ Π²ΠΈΠ΄ΠΈΡ‚Π΅, ΠΊΠ°ΠΊΠΈΠ΅ числа Π΄ΠΎΠ»ΠΆΠ½Ρ‹ Π±Ρ‹Ρ‚ΡŒ ΠΏΠΎΠΌΠ΅Ρ‰Π΅Π½Ρ‹ Π³Π΄Π΅? А Π·Π°Ρ‚Π΅ΠΌ Ρ€Π΅ΡˆΠ°Π΅ΠΌ для $\bf v$ систСму ΡƒΡ€Π°Π²Π½Π΅Π½ΠΈΠΉ: $${\bf T}{\bf v} = \left[\begin{array}{r}2\\3\\-2\end{array }\right]$$

Π­Ρ‚ΠΎ Π±ΡƒΠ΄Π΅Ρ‚ Π»Π΅Π³ΠΊΠΎ, ΠΊΠΎΠ³Π΄Π° Π²Ρ‹ Π½Π°ΡƒΡ‡ΠΈΡ‚Π΅ΡΡŒ Ρ€Π΅ΡˆΠ°Ρ‚ΡŒ систСмы ΠΌΠ°Ρ‚Ρ€ΠΈΡ‡Π½Ρ‹Ρ… ΡƒΡ€Π°Π²Π½Π΅Π½ΠΈΠΉ.

Π§Ρ‚ΠΎΠ±Ρ‹ Ρ€Π°ΡΡˆΠΈΡ€ΠΈΡ‚ΡŒ Ρ€Π΅ΡˆΠ΅Π½ΠΈΠ΅ JnxF (Ρ‡Ρ‚ΠΎΠ±Ρ‹ ΠΈΠ·Π±Π΅ΠΆΠ°Ρ‚ΡŒ ΠΏΡƒΡ‚Π°Π½ΠΈΡ†Ρ‹), ΠΌΠ°Ρ‚Ρ€ΠΈΡ†Π°, ΠΈΡΠΏΠΎΠ»ΡŒΠ·ΡƒΠ΅ΠΌΠ°Ρ для умноТСния Π² Π΅Π³ΠΎ ΠΎΡ‚Π²Π΅Ρ‚Π΅, Π½Π° самом Π΄Π΅Π»Π΅ являСтся 92\}$, Ρ‚Π°ΠΊ Ρ‡Ρ‚ΠΎ Π½Π° самом Π΄Π΅Π»Π΅ это Β«ΠΏΠ΅Ρ€Π΅ΠΌΠ΅Ρ‰Π΅Π½ΠΈΠ΅Β» систСмы ΠΊΠΎΠΎΡ€Π΄ΠΈΠ½Π°Ρ‚ Π½Π°Π·Π°Π΄: $t\to t+2$, Ρ‡Ρ‚ΠΎ отмСняСт ΠΏΡ€Π΅Π΄Ρ‹Π΄ΡƒΡ‰Π΅Π΅ ΠΈΠ·ΠΌΠ΅Π½Π΅Π½ΠΈΠ΅ $t \to t-2$.

$\endgroup$

4

Π’Π²ΠΎΠΉ ΠΎΡ‚Π²Π΅Ρ‚

Π—Π°Ρ€Π΅Π³ΠΈΡΡ‚Ρ€ΠΈΡ€ΡƒΠΉΡ‚Π΅ΡΡŒ ΠΈΠ»ΠΈ Π²ΠΎΠΉΠ΄ΠΈΡ‚Π΅ Π² систСму

Π—Π°Ρ€Π΅Π³ΠΈΡΡ‚Ρ€ΠΈΡ€ΡƒΠΉΡ‚Π΅ΡΡŒ с ΠΏΠΎΠΌΠΎΡ‰ΡŒΡŽ Google

Π—Π°Ρ€Π΅Π³ΠΈΡΡ‚Ρ€ΠΈΡ€ΠΎΠ²Π°Ρ‚ΡŒΡΡ Ρ‡Π΅Ρ€Π΅Π· Facebook

Π—Π°Ρ€Π΅Π³ΠΈΡΡ‚Ρ€ΠΈΡ€ΡƒΠΉΡ‚Π΅ΡΡŒ, ΠΈΡΠΏΠΎΠ»ΡŒΠ·ΡƒΡ адрСс элСктронной ΠΏΠΎΡ‡Ρ‚Ρ‹ ΠΈ ΠΏΠ°Ρ€ΠΎΠ»ΡŒ

ΠžΠΏΡƒΠ±Π»ΠΈΠΊΠΎΠ²Π°Ρ‚ΡŒ ΠΊΠ°ΠΊ Π³ΠΎΡΡ‚ΡŒ

ЭлСктронная ΠΏΠΎΡ‡Ρ‚Π°

ВрСбуСтся, Π½ΠΎ Π½ΠΈΠΊΠΎΠ³Π΄Π° Π½Π΅ отобраТаСтся

ΠžΠΏΡƒΠ±Π»ΠΈΠΊΠΎΠ²Π°Ρ‚ΡŒ ΠΊΠ°ΠΊ Π³ΠΎΡΡ‚ΡŒ

ЭлСктронная ΠΏΠΎΡ‡Ρ‚Π°

ВрСбуСтся, Π½ΠΎ Π½Π΅ отобраТаСтся

НаТимая Β«ΠžΠΏΡƒΠ±Π»ΠΈΠΊΠΎΠ²Π°Ρ‚ΡŒ свой ΠΎΡ‚Π²Π΅Ρ‚Β», Π²Ρ‹ ΡΠΎΠ³Π»Π°ΡˆΠ°Π΅Ρ‚Π΅ΡΡŒ с нашими условиями обслуТивания, ΠΏΠΎΠ»ΠΈΡ‚ΠΈΠΊΠΎΠΉ ΠΊΠΎΠ½Ρ„ΠΈΠ΄Π΅Π½Ρ†ΠΈΠ°Π»ΡŒΠ½ΠΎΡΡ‚ΠΈ ΠΈ ΠΏΠΎΠ»ΠΈΡ‚ΠΈΠΊΠΎΠΉ использования Ρ„Π°ΠΉΠ»ΠΎΠ² cookie

линСйная Π°Π»Π³Π΅Π±Ρ€Π° — ΠžΠΏΡ€Π΅Π΄Π΅Π»Π΅Π½ΠΈΠ΅ ΠΊΠΎΠΎΡ€Π΄ΠΈΠ½Π°Ρ‚ Π²Π΅ΠΊΡ‚ΠΎΡ€Π° ΠΎΡ‚Π½ΠΎΡΠΈΡ‚Π΅Π»ΡŒΠ½ΠΎ базиса

Π·Π°Π΄Π°Π½Π½Ρ‹ΠΉ вопрос

ИзмСнСно 7 Π»Π΅Ρ‚, 2 мСсяца Π½Π°Π·Π°Π΄

ΠŸΡ€ΠΎΡΠΌΠΎΡ‚Ρ€Π΅Π½ΠΎ 67 Ρ€Π°Π·

$\begingroup$

Π—Π°Π΄Π°Ρ‡Π°: ΠŸΡƒΡΡ‚ΡŒ $V = \mathbb{R}[X]_{\leq 4}$ β€” Π²Π΅ΠΊΡ‚ΠΎΡ€Π½ΠΎΠ΅ пространство всСх ΠΌΠ½ΠΎΠ³ΠΎΡ‡Π»Π΅Π½ΠΎΠ² стСпСни Π½Π΅ Π²Ρ‹ΡˆΠ΅ $n$, ΠΈ ΠΏΡƒΡΡ‚ΡŒ $\alpha = \left\{1 , 1+x, (1+x)^2, (1+x)^3, (1+x)^4\right\}$ β€” базис для $V$. 4$ ΠΎΡ‚Π½ΠΎΡΠΈΡ‚Π΅Π»ΡŒΠ½ΠΎ этого базиса. 94) \end{align*} Π³Π΄Π΅ я Ρ€Π°ΡΡˆΠΈΡ€ΠΈΠ» базисныС Π²Π΅ΠΊΡ‚ΠΎΡ€Ρ‹. ΠŸΡ€ΠΈ ΠΏΡ€ΠΎΠ²Π΅Ρ€ΠΊΠ΅ ΡΠΎΠΎΡ‚Π²Π΅Ρ‚ΡΡ‚Π²ΡƒΡŽΡ‰ΠΈΡ… коэффициСнтов получаСтся ΡΠ»Π΅Π΄ΡƒΡŽΡ‰Π°Ρ систСма ΡƒΡ€Π°Π²Π½Π΅Π½ΠΈΠΉ: \begin{align*} \begin{cases} \lambda_1 + \lambda_2 + \lambda_3 + \lambda_4 + \lambda_5 &= 0 \\ \lambda_2 + 2 \ lambda_3 + 3\lambda_4 + 4 \lambda_5 &= 1 \\ \lambda_3 + 3 \lambda_4 + 6 \lambda_5 &= 0 \\ \lambda_4 + 4 \lambda_5 &= 1 \\ \lambda_5 &= 1 \end{cases} \end{align*} Π― помСстил это Π² Ρ€Π°ΡΡˆΠΈΡ€Π΅Π½Π½ΡƒΡŽ ΠΌΠ°Ρ‚Ρ€ΠΈΡ†Ρƒ ΠΈ ΡƒΠΌΠ΅Π½ΡŒΡˆΠΈΠ» строку: \begin{Π²Ρ‹Ρ€Π°Π²Π½ΠΈΠ²Π°Π½ΠΈΠ΅*} ΠΈ \left(\begin{массив}{ccccc|c} 1 ΠΈ 1 ΠΈ 1 ΠΈ 1 ΠΈ 1 ΠΈ 0 \\ 0 ΠΈ 1 ΠΈ 2 ΠΈ 3 ΠΈ 4 ΠΈ 1 \\ 0 ΠΈ 0 ΠΈ 1 ΠΈ 3 ΠΈ 6 ΠΈ 0 \\ 0 ΠΈ 0 ΠΈ 0 ΠΈ 1 ΠΈ 4 ΠΈ 1 \\ 0 ΠΈ 0 ΠΈ 0 ΠΈ 0 ΠΈ 1 ΠΈ 1 \ΠΊΠΎΠ½Π΅Ρ†{массив}\справа) \begin{matrix} \xrightarrow{R_4 \rightarrow R_4 — 4 R_5} \\ \xrightarrow{R_3 \rightarrow R_3 — 6 R_5} \\ \xrightarrow{R_2 \rightarrow R_2 — 4R_5} \\ \xrightarrow{R_1 \rightarrow R_1 — R_5} \end{matrix} \left(\begin{array}{ccccc|c} 1 ΠΈ 1 ΠΈ 1 ΠΈ 1 ΠΈ 0 ΠΈ -1 \\ 0 ΠΈ 1 ΠΈ 2 ΠΈ 3 ΠΈ 0 ΠΈ -3 \\ 0 ΠΈ 0 ΠΈ 1 ΠΈ 3 ΠΈ 0 ΠΈ -6 \\ 0 & 0 & 0 & 1 & 0 & -3 \\ 0 ΠΈ 0 ΠΈ 0 ΠΈ 0 ΠΈ 1 ΠΈ 1 \end{массив}\right) \\ \begin{matrix} \xrightarrow{R_3 \rightarrow R_3 — 3R_4} \\ \xrightarrow{R_2 \rightarrow R_2 — 3 R_4} \\ \xrightarrow{R_1 \rightarrow R_1 — R_4} \end{ΠΌΠ°Ρ‚Ρ€ΠΈΡ†Π°} & \left(\begin{массив}{ccccc|c} 1 ΠΈ 1 ΠΈ 1 ΠΈ 0 ΠΈ 0 ΠΈ -4 \\ 0 ΠΈ 1 ΠΈ 2 ΠΈ 0 ΠΈ 0 ΠΈ 6 \\ 0 ΠΈ 0 ΠΈ 1 ΠΈ 0 ΠΈ 0 ΠΈ 3 \\ 0 ΠΈ 0 ΠΈ 0 ΠΈ 1 ΠΈ 0 ΠΈ -3 \\ 0 ΠΈ 0 ΠΈ 0 ΠΈ 0 ΠΈ 1 ΠΈ 1 \ΠΊΠΎΠ½Π΅Ρ†{массив}\справа) \end{align*} Π—Π°Ρ‚Π΅ΠΌ с ΠΎΠ±Ρ€Π°Ρ‚Π½ΠΎΠΉ Π·Π°ΠΌΠ΅Π½ΠΎΠΉ я ΠΎΠ±Π½Π°Ρ€ΡƒΠΆΠΈΠ», Ρ‡Ρ‚ΠΎ \begin{align*} \begin{cases} \lambda_5 &= 1 \\ \lambda_4 &= -3 \\ \lambda_3 &= 3 \\ \lambda_2 &= 0 \\ \lambda_1 &= -7 \end{cases} \end{align*} Но ΠΊΠΎΠ³Π΄Π° я ΠΏΠΎΠ΄ΡΡ‚Π°Π²Π»ΡΡŽ это Π² ΡƒΡ€Π°Π²Π½Π΅Π½ΠΈΠ΅ Π²Ρ‹ΡˆΠ΅, это Π½Π΅ Π΄Π°Π΅Ρ‚ ΠΌΠ½Π΅ ΠΏΡ€Π°Π²ΠΈΠ»ΡŒΠ½Ρ‹ΠΉ Π²Π΅ΠΊΡ‚ΠΎΡ€. ΠœΠΎΠΆΠ΅Ρ‚ ΠΊΡ‚ΠΎ-Π½ΠΈΠ±ΡƒΠ΄ΡŒ ΡΠΊΠ°Π·Π°Ρ‚ΡŒ ΠΌΠ½Π΅, Π³Π΄Π΅ я ошибся, поТалуйста?

  • линСйная Π°Π»Π³Π΅Π±Ρ€Π°
  • Π²Π΅ΠΊΡ‚ΠΎΡ€Π½Ρ‹Π΅ пространства
  • систСмы ΡƒΡ€Π°Π²Π½Π΅Π½ΠΈΠΉ

$\endgroup$

2

$\begingroup$

Π’ сокращСнии послСднСй строки, ΠΊΠΎΠ³Π΄Π° Π²Ρ‹ выполняСтС $R_1\leftarrow R_1-R_4$, послСднСС число Π² $R_1$ Π΄ΠΎΠ»ΠΆΠ½ΠΎ Π±Ρ‹Ρ‚ΡŒ $-1-(-3)=-1+3=2$, Π½ΠΎ Π²Ρ‹ написали $-4 $. Π­Ρ‚ΠΎ Π΄Π΅Π»Π°Π΅Ρ‚ ΡΠ°ΠΌΡƒΡŽ послСднюю ΠΎΠ±Ρ€Π°Ρ‚Π½ΡƒΡŽ Π·Π°ΠΌΠ΅Π½Ρƒ $\lambda_1$ Π½Π΅ΠΏΡ€Π°Π²ΠΈΠ»ΡŒΠ½ΠΎΠΉ.

Π― Π½Π΅ понимаю, Ρ‡Ρ‚ΠΎ Π²Ρ‹ Π΄Π΅Π»Π°Π΅Ρ‚Π΅. Π’Π°ΡˆΠ° исходная ΠΌΠ°Ρ‚Ρ€ΠΈΡ†Π° Π±Ρ‹Π»Π° Π²Π΅Ρ€Ρ…Π½Π΅Ρ‚Ρ€Π΅ΡƒΠ³ΠΎΠ»ΡŒΠ½ΠΎΠΉ, поэтому ΠΎΡ‚Ρ‚ΡƒΠ΄Π° ΠΌΠΎΠΆΠ½ΠΎ Π±Ρ‹Π»ΠΎ Π²Ρ‹ΠΏΠΎΠ»Π½ΠΈΡ‚ΡŒ ΠΎΠ±Ρ€Π°Ρ‚Π½ΡƒΡŽ подстановку, Ρ‚Π°ΠΊ Ρ‡Ρ‚ΠΎ Π²ΠΎΠΎΠ±Ρ‰Π΅ Π½Π΅ Ρ‚Ρ€Π΅Π±ΠΎΠ²Π°Π»ΠΎΡΡŒ сокращСния строк. И Π²Ρ‹ ΠΎΡΡ‚Π°Π½ΠΎΠ²ΠΈΠ»ΠΈΡΡŒ Π΄ΠΎ Ρ‚ΠΎΠ³ΠΎ, ΠΊΠ°ΠΊ дошли Π΄ΠΎ Ρ„ΠΎΡ€ΠΌΡ‹ эшСлона с ΡƒΠΌΠ΅Π½ΡŒΡˆΠ΅Π½Π½Ρ‹ΠΌ числом строк, Π³Π΄Π΅ Π½Π΅ ΠΏΠΎΡ‚Ρ€Π΅Π±ΠΎΠ²Π°Π»Π°ΡΡŒ Π±Ρ‹ обратная Π·Π°ΠΌΠ΅Π½Π°. Какой Ρƒ тСбя Π±Ρ‹Π» смысл Π΄Π΅Π»Π°Ρ‚ΡŒ нСсколько сокращСний рядов, Π½Π΅ пСрСходя ΠΏΠΎΠ»Π½ΠΎΡΡ‚ΡŒΡŽ ΠΊ ΡƒΠΌΠ΅Π½ΡŒΡˆΠ΅Π½Π½ΠΎΠΉ ΡΡˆΠ΅Π»ΠΎΠ½ΠΈΡ€ΠΎΠ²Π°Π½Π½ΠΎΠΉ Ρ„ΠΎΡ€ΠΌΠ΅ рядов?

$\endgroup$

$\begingroup$

ΠšΠΎΠΎΡ€Π΄ΠΈΠ½Π°Ρ‚Ρ‹: {$-7,12,-3,-3,1$} см. ваши ΠΎΠΏΠ΅Ρ€Π°Ρ†ΠΈΠΈ

$\endgroup$

2

Π’Π²ΠΎΠΉ ΠΎΡ‚Π²Π΅Ρ‚

Π—Π°Ρ€Π΅Π³ΠΈΡΡ‚Ρ€ΠΈΡ€ΡƒΠΉΡ‚Π΅ΡΡŒ ΠΈΠ»ΠΈ Π²ΠΎΠΉΠ΄ΠΈΡ‚Π΅

Π—Π°Ρ€Π΅Π³ΠΈΡΡ‚Ρ€ΠΈΡ€ΠΎΠ²Π°Ρ‚ΡŒΡΡ Ρ‡Π΅Ρ€Π΅Π· Google

Π—Π°Ρ€Π΅Π³ΠΈΡΡ‚Ρ€ΠΈΡ€ΠΎΠ²Π°Ρ‚ΡŒΡΡ Ρ‡Π΅Ρ€Π΅Π· Facebook

Π—Π°Ρ€Π΅Π³ΠΈΡΡ‚Ρ€ΠΈΡ€ΡƒΠΉΡ‚Π΅ΡΡŒ, ΠΈΡΠΏΠΎΠ»ΡŒΠ·ΡƒΡ ΡΠ»Π΅ΠΊΡ‚Ρ€ΠΎΠ½Π½ΡƒΡŽ ΠΏΠΎΡ‡Ρ‚Ρƒ ΠΈ ΠΏΠ°Ρ€ΠΎΠ»ΡŒ

ΠžΠΏΡƒΠ±Π»ΠΈΠΊΠΎΠ²Π°Ρ‚ΡŒ ΠΊΠ°ΠΊ Π³ΠΎΡΡ‚ΡŒ

ЭлСктронная ΠΏΠΎΡ‡Ρ‚Π°

ВрСбуСтся, Π½ΠΎ Π½ΠΈΠΊΠΎΠ³Π΄Π° Π½Π΅ отобраТаСтся

ΠžΠΏΡƒΠ±Π»ΠΈΠΊΠΎΠ²Π°Ρ‚ΡŒ ΠΊΠ°ΠΊ Π³ΠΎΡΡ‚ΡŒ

ЭлСктронная ΠΏΠΎΡ‡Ρ‚Π°

ВрСбуСтся, Π½ΠΎ Π½Π΅ отобраТаСтся

НаТимая Β«ΠžΠΏΡƒΠ±Π»ΠΈΠΊΠΎΠ²Π°Ρ‚ΡŒ свой ΠΎΡ‚Π²Π΅Ρ‚Β», Π²Ρ‹ ΡΠΎΠ³Π»Π°ΡˆΠ°Π΅Ρ‚Π΅ΡΡŒ с нашими условиями обслуТивания, ΠΏΠΎΠ»ΠΈΡ‚ΠΈΠΊΠΎΠΉ ΠΊΠΎΠ½Ρ„ΠΈΠ΄Π΅Π½Ρ†ΠΈΠ°Π»ΡŒΠ½ΠΎΡΡ‚ΠΈ ΠΈ ΠΏΠΎΠ»ΠΈΡ‚ΠΈΠΊΠΎΠΉ использования Ρ„Π°ΠΉΠ»ΠΎΠ² cookie

ΠšΠΎΠΎΡ€Π΄ΠΈΠ½Π°Ρ‚Ρ‹ Ρ‚ΠΎΡ‡ΠΊΠΈ, ΠΊΠΎΠΌΠΏΠΎΠ½Π΅Π½Ρ‚Ρ‹ Π²Π΅ΠΊΡ‚ΠΎΡ€Π° ΠΈ сСрСдины ΠΎΡ‚Ρ€Π΅Π·ΠΊΠ°

ΠšΠΎΠΎΡ€Π΄ΠΈΠ½Π°Ρ‚Ρ‹ Ρ‚ΠΎΡ‡ΠΊΠΈ Π½Π° плоскости

Π”Π°Π²Π°ΠΉΡ‚Π΅ посмотрим, ΠΊΠ°ΠΊ Π²Π΅ΠΊΡ‚ΠΎΡ€Ρ‹ ΠΈΡΠΏΠΎΠ»ΡŒΠ·ΡƒΡŽΡ‚ΡΡ для присвоСния ΠΊΠΎΠΎΡ€Π΄ΠΈΠ½Π°Ρ‚ Ρ‚ΠΎΡ‡ΠΊΠ°ΠΌ Π½Π° плоскости.

Рассмотрим Ρ„ΠΈΠΊΡΠΈΡ€ΠΎΠ²Π°Π½Π½ΡƒΡŽ Ρ‚ΠΎΡ‡ΠΊΡƒ Π½Π° плоскости $$O$$ (ΠΈΠ·Π²Π΅ΡΡ‚Π½ΡƒΡŽ ΠΊΠ°ΠΊ Π½Π°Ρ‡Π°Π»ΠΎ ΠΊΠΎΠΎΡ€Π΄ΠΈΠ½Π°Ρ‚) ΠΈ базис $$B=\{\overrightarrow{u}, \overrightarrow{v}\}$$ $$V_2$ $ (пространствСнный Π²Π΅ΠΊΡ‚ΠΎΡ€ размСрности $$2$$).

Напомним, Ρ‡Ρ‚ΠΎ Π² основС $$V_2$$ Π»Π΅ΠΆΠ°Ρ‚ Π΄Π²Π° Π»ΠΈΠ½Π΅ΠΉΠ½ΠΎ нСзависимых Π²Π΅ΠΊΡ‚ΠΎΡ€Π°. ΠœΠ½ΠΎΠΆΠ΅ΡΡ‚Π²ΠΎ, ΠΎΠ±Ρ€Π°Π·ΠΎΠ²Π°Π½Π½ΠΎΠ΅ $$O$$ ΠΈ $$B=\{\overrightarrow{u}, \overrightarrow{v}\}$$, ΠΎΠ±Ρ€Π°Π·ΡƒΠ΅Ρ‚ систСму отсчСта Π½Π° плоскости, ΠΏΠΎΡΠΊΠΎΠ»ΡŒΠΊΡƒ позволяСт ΠΎΠΏΡ€Π΅Π΄Π΅Π»ΠΈΡ‚ΡŒ ΠΏΠΎΠ»ΠΎΠΆΠ΅Π½ΠΈΠ΅ Π»ΡŽΠ±Ρ‹Ρ… Π΄Ρ€ΡƒΠ³ΠΈΡ… Ρ‚ΠΎΡ‡ΠΊΠΈ Π½Π° плоскости.

Π­Ρ‚ΠΎ связано с Ρ‚Π΅ΠΌ, Ρ‡Ρ‚ΠΎ Π»ΡŽΠ±Ρ‹Π΅ Π΄Ρ€ΡƒΠ³ΠΈΠ΅ Ρ‚ΠΎΡ‡ΠΊΠΈ $$P$$ Π½Π° плоскости вмСстС с Ρ‚ΠΎΡ‡ΠΊΠΎΠΉ $$O$$ ΠΎΠΏΡ€Π΅Π΄Π΅Π»ΡΡŽΡ‚ Π²Π΅ΠΊΡ‚ΠΎΡ€ $$\overrightarrow{OP}$$. ΠŸΡƒΡΡ‚ΡŒ $$(p_1,p_2)$$ β€” ΠΊΠΎΠΌΠΏΠΎΠ½Π΅Π½Ρ‚Ρ‹ Π²Π΅ΠΊΡ‚ΠΎΡ€Π° Π² базисС $$B$$. Π’ΠΎΠ³Π΄Π° $$(p_1,p_2)$$ β€” это ΠΊΠΎΠΎΡ€Π΄ΠΈΠ½Π°Ρ‚Ρ‹ Ρ‚ΠΎΡ‡ΠΊΠΈ $$P$$ Π² систСмС отсчСта $$R=\{O;\overrightarrow{u}, \overrightarrow{v}\}$$, ΠΈ ΠΌΡ‹ пишСм $$P=(p_1,p_2)$$.

ΠŸΡ€ΠΎΡ†Π΅Π΄ΡƒΡ€Π° нахоТдСния ΠΊΠΎΠΎΡ€Π΄ΠΈΠ½Π°Ρ‚ Ρ‚ΠΎΡ‡ΠΊΠΈ $$P$$ Π² Π·Π°Π΄Π°Π½Π½ΠΎΠΉ систСмС отсчСта ΡΠ»Π΅Π΄ΡƒΡŽΡ‰Π°Ρ:

  1. Из Ρ‚ΠΎΡ‡Π΅ΠΊ $$O$$ ΠΈ $$P$$ опрСдСляСм Π²Π΅ΠΊΡ‚ΠΎΡ€ $$\overrightarrow{OP}$$

  2. Π’Ρ‹Ρ€Π°Π·ΠΈΠΌ Π²Π΅ΠΊΡ‚ΠΎΡ€ $$\overrightarrow{OP}$$ Π² Π²ΠΈΠ΄Π΅ Π»ΠΈΠ½Π΅ΠΉΠ½ΠΎΠΉ ΠΊΠΎΠΌΠ±ΠΈΠ½Π°Ρ†ΠΈΠΈ Π²Π΅ΠΊΡ‚ΠΎΡ€ΠΎΠ² базиса $$B=\{\overrightarrow{u}, \overrightarrow{v}\}$$, Ρ‚. Π΅. , $$\overrightarrow{OP}=p_1 \cdot \overrightarrow{u}+p_2 \cdot \overrightarrow{v}$$

  3. $$P=(p_1,p_2)$$

Π’Ρ‹Ρ€Π°Π·ΠΈΡ‚Π΅ Ρ‚ΠΎΡ‡ΠΊΡƒ $$P$$ Ρ‡Π΅Ρ€Ρ‚Π΅ΠΆΠ° Π² систСмС отсчСта $$R =\{O;\overrightarrow {u}, \overrightarrow{v}\}$$.

  • РисуСм Π²Π΅ΠΊΡ‚ΠΎΡ€ $$\overrightarrow{OP}$$:

  • Π’Ρ‹Ρ€Π°Π·ΠΈΠΌ Π²Π΅ΠΊΡ‚ΠΎΡ€ $$\overrightarrow{OP}$$ Π² Π²ΠΈΠ΄Π΅ Π»ΠΈΠ½Π΅ΠΉΠ½ΠΎΠΉ ΠΊΠΎΠΌΠ±ΠΈΠ½Π°Ρ†ΠΈΠΈ Π²Π΅ΠΊΡ‚ΠΎΡ€ΠΎΠ² базиса $$B=\{\overrightarrow{u}, \overrightarrow{v}\}$$:

  • ΠŸΠΎΠ»ΡƒΡ‡Π°Π΅ΠΌ $$\overrightarrow{OP}=\overrightarrow{u}+2\overrightarrow{v}$$ ΠΈ, ΡΠ»Π΅Π΄ΠΎΠ²Π°Ρ‚Π΅Π»ΡŒΠ½ΠΎ, ΠΊΠΎΠΎΡ€Π΄ΠΈΠ½Π°Ρ‚Ρ‹ Ρ‚ΠΎΡ‡ΠΊΠΈ $$P$$ Ρ€Π°Π²Π½Ρ‹ $$P = (1 , 2 )$$

Π’ дальнСйшСм Π² качСствС систСмы отсчСта $$R$$ Π±ΡƒΠ΄Π΅ΠΌ Ρ€Π°ΡΡΠΌΠ°Ρ‚Ρ€ΠΈΠ²Π°Ρ‚ΡŒ систСму, ΠΎΠ±Ρ€Π°Π·ΠΎΠ²Π°Π½Π½ΡƒΡŽ Π½Π°Ρ‡Π°Π»ΠΎΠΌ ΠΊΠΎΠΎΡ€Π΄ΠΈΠ½Π°Ρ‚ $$O = (0, 0)$$ ΠΈ каноничСским базисом $$V_2$$ $$B =\{\overrightarrow{i},\overrightarrow{j}\}$$.

ΠšΠΎΠΌΠΏΠΎΠ½Π΅Π½Ρ‚Ρ‹ Π²Π΅ΠΊΡ‚ΠΎΡ€Π°, опрСдСляСмого двумя Ρ‚ΠΎΡ‡ΠΊΠ°ΠΌΠΈ

Π’Π΅ΠΏΠ΅Ρ€ΡŒ посмотрим, ΠΊΠ°ΠΊ ΠΎΠΏΡ€Π΅Π΄Π΅Π»ΠΈΡ‚ΡŒ ΠΊΠΎΠΌΠΏΠΎΠ½Π΅Π½Ρ‚Ρ‹ Π²Π΅ΠΊΡ‚ΠΎΡ€Π°, Ссли извСстны ΠΊΠΎΠΎΡ€Π΄ΠΈΠ½Π°Ρ‚Ρ‹ Π΅Π³ΠΎ ΠΊΠΎΠ½Ρ†ΠΎΠ²:

ΠŸΡƒΡΡ‚ΡŒ $$P =(p_1,p_2)$$ ΠΈ $$ Q = (q_1,q_2)$$ β€” Π΄Π²Π΅ Ρ‚ΠΎΡ‡ΠΊΠΈ плоскости, Π° $$\overrightarrow{PQ}$$ β€” Π²Π΅ΠΊΡ‚ΠΎΡ€, ΠΈΠ΄ΡƒΡ‰ΠΈΠΉ ΠΈΠ· $$P$$ Π² $$Q$$. Π’ΠΎΠ³Π΄Π° ΠΊΠΎΠΌΠΏΠΎΠ½Π΅Π½Ρ‚Ρ‹ Π²Π΅ΠΊΡ‚ΠΎΡ€Π° $$\overrightarrow{PQ}$$ Ρ€Π°Π²Π½Ρ‹ $$\overrightarrow{PQ}=(q_1-p_1,q_2-p_2)$$.

Π”Π°Π½ΠΎ $$P = (2, 6)$$ ΠΈ $$Q = (-3, 9)$$. ΠšΠΎΠΌΠΏΠΎΠ½Π΅Π½Ρ‚Ρ‹ Π²Π΅ΠΊΡ‚ΠΎΡ€Π° $$\overrightarrow{PQ}$$ Ρ‚Π°ΠΊΠΎΠ²Ρ‹: $$\overrightarrow{PQ}= (-3 — 2, 9- 6) = (-5, 3)$$

ΠŸΡ€ΠΈΠΌΠ΅Π½Π΅Π½ΠΈΠ΅ Π²Π΅ΠΊΡ‚ΠΎΡ€Π° ΠΊ Ρ‚ΠΎΡ‡ΠΊΠ΅

Для Π΄Π°Π½Π½ΠΎΠΉ Ρ‚ΠΎΡ‡ΠΊΠΈ $$P$$ ΠΈ Π²Π΅ΠΊΡ‚ΠΎΡ€Π° $$\overrightarrow{v}$$ Ρ€Π΅Π·ΡƒΠ»ΡŒΡ‚Π°Ρ‚ примСнСния Π²Π΅ΠΊΡ‚ΠΎΡ€Π° ΠΊ Ρ‚ΠΎΡ‡ΠΊΠ° β€” это новая Ρ‚ΠΎΡ‡ΠΊΠ° $$Q$$, располоТСнная Π² Π½Π°ΠΏΡ€Π°Π²Π»Π΅Π½ΠΈΠΈ $$\overrightarrow{v}$$ ΠΈ Π½Π° расстоянии $$|\overrightarrow{v}|$$. (ΠΌΠΎΠ΄ΡƒΠ»ΡŒ Π²Π΅ΠΊΡ‚ΠΎΡ€Π° $$\overrightarrow{v}$$)

ΠšΠΎΠΎΡ€Π΄ΠΈΠ½Π°Ρ‚Ρ‹ этой Π½ΠΎΠ²ΠΎΠΉ Ρ‚ΠΎΡ‡ΠΊΠΈ $$Q$$ Π²Ρ‹Ρ‡ΠΈΡΠ»ΡΡŽΡ‚ΡΡ ΠΈΠ· ΠΊΠΎΠΎΡ€Π΄ΠΈΠ½Π°Ρ‚ $$P =(p_1,p_2)$$ ΠΈ $$\overrightarrow{ v}=(v_1,v_2)$$ Ρ‚Π°ΠΊΠΈΠΌ ΠΎΠ±Ρ€Π°Π·ΠΎΠΌ $$$Q = P +\overrightarrow{v}=(p_1+v_1,p_2+v_2)$$$

ΠŸΠ Π˜ΠœΠ•Π§ΠΠΠ˜Π•. ΠžΡ‡Π΅Π½ΡŒ Π²Π°ΠΆΠ½ΠΎ ΠΏΠΎΠΌΠ½ΠΈΡ‚ΡŒ, Ρ‡Ρ‚ΠΎ эта опСрация слоТСния ΠΈΠΌΠ΅Π΅Ρ‚ смысл Ρ‚ΠΎΠ»ΡŒΠΊΠΎ ΠΌΠ΅ΠΆΠ΄Ρƒ Ρ‚ΠΎΡ‡ΠΊΠΎΠΉ ΠΈ Π²Π΅ΠΊΡ‚ΠΎΡ€ΠΎΠΌ. ΠœΡ‹ Π½ΠΈΠΊΠΎΠ³Π΄Π° Π½Π΅ Π΄ΠΎΠ»ΠΆΠ½Ρ‹ ΡΠΊΠ»Π°Π΄Ρ‹Π²Π°Ρ‚ΡŒ Π΄Π²Π΅ Ρ‚ΠΎΡ‡ΠΊΠΈ, Π° Ρ€Π΅Π·ΡƒΠ»ΡŒΡ‚Π°Ρ‚ΠΎΠΌ слоТСния Π΄Π²ΡƒΡ… Π²Π΅ΠΊΡ‚ΠΎΡ€ΠΎΠ² Π±ΡƒΠ΄Π΅Ρ‚ Π΄Ρ€ΡƒΠ³ΠΎΠΉ Π²Π΅ΠΊΡ‚ΠΎΡ€, Π° Π½Π΅ Ρ‚ΠΎΡ‡ΠΊΠ°!

РассмотрСв ΡΠ»Π΅Π΄ΡƒΡŽΡ‰ΠΈΠΉ рисунок, ΠΎΠΏΡ€Π΅Π΄Π΅Π»ΠΈΡ‚Π΅ ΠΊΠΎΠΎΡ€Π΄ΠΈΠ½Π°Ρ‚Ρ‹ Ρ‚ΠΎΡ‡ΠΊΠΈ $$P$$ рисунка, ΠΏΠΎΠ»ΡƒΡ‡Π΅Π½Π½ΠΎΠ³ΠΎ ΠΏΡ€ΠΈΠ»ΠΎΠΆΠ΅Π½ΠΈΠ΅ΠΌ Π²Π΅ΠΊΡ‚ΠΎΡ€Π° $$\overrightarrow{v}$$ ΠΊ Ρ‚ΠΎΡ‡ΠΊΠ΅ $$A$$.

НачнСм с вычислСния ΠΊΠΎΠΌΠΏΠΎΠ½Π΅Π½Ρ‚ Π²Π΅ΠΊΡ‚ΠΎΡ€Π° $$\overrightarrow{v}$$:$$$\overrightarrow{v} = (2 — (-1), 4-2) = (3, 2) $$$ ΠŸΠΎΡΠΊΠΎΠ»ΡŒΠΊΡƒ $$P$$ являСтся Ρ€Π΅Π·ΡƒΠ»ΡŒΡ‚Π°Ρ‚ΠΎΠΌ примСнСния Π²Π΅ΠΊΡ‚ΠΎΡ€Π° $$\overrightarrow{v}$$ ΠΊ Ρ‚ΠΎΡ‡ΠΊΠ΅ $$A$$, ΠΈΠΌΠ΅Π΅ΠΌ $$$P=A+\overrightarrow{v}=(0,4) +(3,2)=(3,6)$$$

Π‘Π΅Ρ€Π΅Π΄ΠΈΠ½Π° ΠΎΡ‚Ρ€Π΅Π·ΠΊΠ°

Рассмотрим Ρ‚Π΅ΠΏΠ΅Ρ€ΡŒ ΠΎΡ‚Ρ€Π΅Π·ΠΎΠΊ с ΠΊΠΎΠ½Ρ†Π°ΠΌΠΈ $$A = (a_1,a_2)$$ ΠΈ $$B = (b_1,b_2)$$. ΠŸΡƒΡΡ‚ΡŒ $$M =(m_1,m_2)$$ β€” сСрСдина ΡƒΠΊΠ°Π·Π°Π½Π½ΠΎΠ³ΠΎ Π²Ρ‹ΡˆΠ΅ ΠΎΡ‚Ρ€Π΅Π·ΠΊΠ°. ΠžΡ‡Π΅Π²ΠΈΠ΄Π½ΠΎ, Ρ‡Ρ‚ΠΎ указанная Π²Ρ‹ΡˆΠ΅ Ρ‚ΠΎΡ‡ΠΊΠ° удовлСтворяСт Ρ‚ΠΎΠΌΡƒ, Ρ‡Ρ‚ΠΎ $$\overrightarrow{AB}=2\cdot \overrightarrow{AM}$$, ΠΈΠ»ΠΈ Ρ‡Ρ‚ΠΎ $$(b_1-a_1,b_2-a_2)=2\cdot (m_1-a_1,m_2- a_2)$$

РаздСляя ΠΏΠΎΠΊΠΎΠΌΠΏΠΎΠ½Π΅Π½Ρ‚Π½ΠΎ ΠΏΠΎΠ»ΡƒΡ‡Π°Π΅ΠΌ: $$$\begin{массив}{rcl} b_1-a_1 & = & 2 \cdot (m_1-a_1) \\ b_2-a_2 &=& 2\cdot (m_2-a_2) \end{массив}$$$ ΠΈ изолируя ΠΈΠΌΠ΅Π΅ΠΌ: $$$\begin{array}{rcl} m_1 & = & \displaystyle \frac{a_1+b_1}{2}\\ m_2 &=& \displaystyle \frac{a_2+b_2}{2} \end{array} $$$ Π’Π°ΠΊ Ρ‡Ρ‚ΠΎ ΠΌΡ‹ ΠΌΠΎΠΆΠ΅ΠΌ Π²Ρ‹Ρ‡ΠΈΡΠ»ΠΈΡ‚ΡŒ ΠΊΠΎΠΎΡ€Π΄ΠΈΠ½Π°Ρ‚Ρ‹ сСрСдины ΠΎΡ‚Ρ€Π΅Π·ΠΊΠ° ΠΏΠΎ ΠΊΠΎΠΎΡ€Π΄ΠΈΠ½Π°Ρ‚Π°ΠΌ Π΅Π³ΠΎ ΠΊΠΎΠ½Ρ†ΠΎΠ².

По Ρ‚ΠΎΡ‡ΠΊΠ°ΠΌ $$A = (-3, 7)$$ ΠΈ $$B = (1, 2)$$ Π½Π°ΠΉΠ΄ΠΈΡ‚Π΅ сСрСдину ΠΎΡ‚Ρ€Π΅Π·ΠΊΠ°, ΠΊΠΎΡ‚ΠΎΡ€Ρ‹ΠΉ ΠΎΠ½ΠΈ ΠΎΠΏΡ€Π΅Π΄Π΅Π»ΡΡŽΡ‚.

ΠŸΡ€ΠΈΠΌΠ΅Π½ΡΡ ΠΏΡ€Π΅Π΄Ρ‹Π΄ΡƒΡ‰ΠΈΠ΅ Ρ„ΠΎΡ€ΠΌΡƒΠ»Ρ‹, ΠΈΠΌΠ΅Π΅ΠΌ: $$$\begin{array}{rcl} m_1 & = & \displaystyle \frac{a_1+b_1}{2}= \frac{-3+2}{2}=-1\\ m_2 &=& \displaystyle \frac{a_2+b_2}{2}=\frac{7+2}{2}=\frac{9}{2} \end{array}$$$ Π‘Π»Π΅Π΄ΠΎΠ²Π°Ρ‚Π΅Π»ΡŒΠ½ΠΎ, сСрСдина ΠΎΡ‚Ρ€Π΅Π·ΠΊΠ° $$AB$$ Ρ€Π°Π²Π½Π° $$M = (-1, \displaystyle \frac{9}{2})$$

2.2 БистСмы ΠΊΠΎΠΎΡ€Π΄ΠΈΠ½Π°Ρ‚ ΠΈ ΠΊΠΎΠΌΠΏΠΎΠ½Π΅Π½Ρ‚Ρ‹ Π²Π΅ΠΊΡ‚ΠΎΡ€Π° — University Physics Volume 1

Π¦Π΅Π»ΠΈ обучСния

К ΠΊΠΎΠ½Ρ†Ρƒ этого Ρ€Π°Π·Π΄Π΅Π»Π° Π²Ρ‹ смоТСтС:

  • ΠžΠΏΠΈΡΡ‹Π²Π°Ρ‚ΡŒ Π²Π΅ΠΊΡ‚ΠΎΡ€Π° Π² Π΄Π²ΡƒΡ… ΠΈ Ρ‚Ρ€Π΅Ρ… измСрСниях с Ρ‚ΠΎΡ‡ΠΊΠΈ зрСния ΠΈΡ… ΠΊΠΎΠΌΠΏΠΎΠ½Π΅Π½Ρ‚ΠΎΠ², ΠΈΡΠΏΠΎΠ»ΡŒΠ·ΡƒΡ Π΅Π΄ΠΈΠ½ΠΈΡ‡Π½Ρ‹Π΅ Π²Π΅ΠΊΡ‚ΠΎΡ€Ρ‹ вдоль осСй.
  • Π Π°Π·Π»ΠΈΡ‡Π°Ρ‚ΡŒ Π²Π΅ΠΊΡ‚ΠΎΡ€Π½Ρ‹Π΅ ΠΊΠΎΠΌΠΏΠΎΠ½Π΅Π½Ρ‚Ρ‹ Π²Π΅ΠΊΡ‚ΠΎΡ€Π° ΠΈ скалярныС ΠΊΠΎΠΌΠΏΠΎΠ½Π΅Π½Ρ‚Ρ‹ Π²Π΅ΠΊΡ‚ΠΎΡ€Π°.
  • ΠžΠ±ΡŠΡΡΠ½ΠΈΡ‚Π΅, ΠΊΠ°ΠΊ Π²Π΅Π»ΠΈΡ‡ΠΈΠ½Π° Π²Π΅ΠΊΡ‚ΠΎΡ€Π° опрСдСляСтся Ρ‡Π΅Ρ€Π΅Π· ΠΊΠΎΠΌΠΏΠΎΠ½Π΅Π½Ρ‚Ρ‹ Π²Π΅ΠΊΡ‚ΠΎΡ€Π°.
  • ΠžΠΏΡ€Π΅Π΄Π΅Π»ΠΈΡ‚Π΅ ΡƒΠ³ΠΎΠ» направлСния Π²Π΅ΠΊΡ‚ΠΎΡ€Π° Π½Π° плоскости.
  • ΠžΠ±ΡŠΡΡΠ½ΠΈΡ‚Π΅ связь ΠΌΠ΅ΠΆΠ΄Ρƒ полярными ΠΊΠΎΠΎΡ€Π΄ΠΈΠ½Π°Ρ‚Π°ΠΌΠΈ ΠΈ Π΄Π΅ΠΊΠ°Ρ€Ρ‚ΠΎΠ²Ρ‹ΠΌΠΈ ΠΊΠΎΠΎΡ€Π΄ΠΈΠ½Π°Ρ‚Π°ΠΌΠΈ Π½Π° плоскости.

Π’Π΅ΠΊΡ‚ΠΎΡ€Ρ‹ ΠΎΠ±Ρ‹Ρ‡Π½ΠΎ ΠΎΠΏΠΈΡΡ‹Π²Π°ΡŽΡ‚ΡΡ ΠΈΡ… ΠΊΠΎΠΌΠΏΠΎΠ½Π΅Π½Ρ‚Π°ΠΌΠΈ Π² систСмС ΠΊΠΎΠΎΡ€Π΄ΠΈΠ½Π°Ρ‚. Π”Π°ΠΆΠ΅ Π² повсСднСвной ΠΆΠΈΠ·Π½ΠΈ ΠΌΡ‹ СстСствСнным ΠΎΠ±Ρ€Π°Π·ΠΎΠΌ ΠΏΠΎΠ»ΡŒΠ·ΡƒΠ΅ΠΌΡΡ понятиСм ΠΎΡ€Ρ‚ΠΎΠ³ΠΎΠ½Π°Π»ΡŒΠ½Ρ‹Ρ… ΠΏΡ€ΠΎΠ΅ΠΊΡ†ΠΈΠΉ Π² ΠΏΡ€ΡΠΌΠΎΡƒΠ³ΠΎΠ»ΡŒΠ½ΠΎΠΉ систСмС ΠΊΠΎΠΎΡ€Π΄ΠΈΠ½Π°Ρ‚. НапримСр, Ссли Π²Ρ‹ спроситС Ρƒ ΠΊΠΎΠ³ΠΎ-Π½ΠΈΠ±ΡƒΠ΄ΡŒ Π½Π°ΠΏΡ€Π°Π²Π»Π΅Π½ΠΈΠ΅ ΠΊ ΠΎΠΏΡ€Π΅Π΄Π΅Π»Π΅Π½Π½ΠΎΠΌΡƒ мСсту, Π²Π°ΠΌ, скорСС всСго, скаТут ΠΏΡ€ΠΎΠΉΡ‚ΠΈ 40 ΠΊΠΌ Π½Π° восток ΠΈ 30 ΠΊΠΌ Π½Π° сСвСр, Ρ‡Π΅ΠΌ 50 ΠΊΠΌ Π² Π½Π°ΠΏΡ€Π°Π²Π»Π΅Π½ΠΈΠΈ 37Β°37Β° ΠΊ сСвСру ΠΎΡ‚ востока.

Π’ ΠΏΡ€ΡΠΌΠΎΡƒΠ³ΠΎΠ»ΡŒΠ½ΠΎΠΉ (Π΄Π΅ΠΊΠ°Ρ€Ρ‚ΠΎΠ²ΠΎΠΉ) xy -систСмС ΠΊΠΎΠΎΡ€Π΄ΠΈΠ½Π°Ρ‚ Π½Π° плоскости Ρ‚ΠΎΡ‡ΠΊΠ° Π½Π° плоскости описываСтся ΠΏΠ°Ρ€ΠΎΠΉ ΠΊΠΎΠΎΡ€Π΄ΠΈΠ½Π°Ρ‚ ( x , y ). Аналогичным ΠΎΠ±Ρ€Π°Π·ΠΎΠΌ Π²Π΅ΠΊΡ‚ΠΎΡ€ Aβ†’Aβ†’ Π½Π° плоскости описываСтся ΠΏΠ°Ρ€ΠΎΠΉ Π΅Π³ΠΎ Π²Π΅ΠΊΡ‚ΠΎΡ€Π½Ρ‹Ρ… ΠΊΠΎΠΎΡ€Π΄ΠΈΠ½Π°Ρ‚. ΠšΠΎΠΎΡ€Π΄ΠΈΠ½Π°Ρ‚Π° x Π²Π΅ΠΊΡ‚ΠΎΡ€Π° Aβ†’Aβ†’ называСтся Π΅Π³ΠΎ x -ΠΊΠΎΠΌΠΏΠΎΠ½Π΅Π½Ρ‚ΠΎΠΉ, Π° y -ΠΊΠΎΠΎΡ€Π΄ΠΈΠ½Π°Ρ‚Π° Π²Π΅ΠΊΡ‚ΠΎΡ€Π° Aβ†’Aβ†’ называСтся Π΅Π³ΠΎ y -ΠΊΠΎΠΌΠΏΠΎΠ½Π΅Π½Ρ‚ΠΎΠΉ. ΠšΠΎΠΌΠΏΠΎΠ½Π΅Π½Ρ‚Π° Π²Π΅ΠΊΡ‚ΠΎΡ€Π° x прСдставляСт собой Π²Π΅ΠΊΡ‚ΠΎΡ€, ΠΎΠ±ΠΎΠ·Π½Π°Ρ‡Π°Π΅ΠΌΡ‹ΠΉ Aβ†’xAβ†’x. Π’Π΅ΠΊΡ‚ΠΎΡ€ y -ΠΊΠΎΠΌΠΏΠΎΠ½Π΅Π½Ρ‚Π° прСдставляСт собой Π²Π΅ΠΊΡ‚ΠΎΡ€, ΠΎΠ±ΠΎΠ·Π½Π°Ρ‡Π°Π΅ΠΌΡ‹ΠΉ Aβ†’yAβ†’y. Π’ Π΄Π΅ΠΊΠ°Ρ€Ρ‚ΠΎΠ²ΠΎΠΉ систСмС ΠΊΠΎΠΌΠΏΠΎΠ½Π΅Π½Ρ‚Ρ‹ Π²Π΅ΠΊΡ‚ΠΎΡ€Π° x ΠΈ y ΡΠ²Π»ΡΡŽΡ‚ΡΡ ΠΎΡ€Ρ‚ΠΎΠ³ΠΎΠ½Π°Π»ΡŒΠ½Ρ‹ΠΌΠΈ проСкциями этого Π²Π΅ΠΊΡ‚ΠΎΡ€Π° Π½Π° оси x ΠΈ y соотвСтствСнно. Π’Π°ΠΊΠΈΠΌ ΠΎΠ±Ρ€Π°Π·ΠΎΠΌ, слСдуя ΠΏΡ€Π°Π²ΠΈΠ»Ρƒ ΠΏΠ°Ρ€Π°Π»Π»Π΅Π»ΠΎΠ³Ρ€Π°ΠΌΠΌΠ° для слоТСния Π²Π΅ΠΊΡ‚ΠΎΡ€ΠΎΠ², ΠΊΠ°ΠΆΠ΄Ρ‹ΠΉ Π²Π΅ΠΊΡ‚ΠΎΡ€ Π½Π° Π΄Π΅ΠΊΠ°Ρ€Ρ‚ΠΎΠ²ΠΎΠΉ плоскости ΠΌΠΎΠΆΠ΅Ρ‚ Π±Ρ‹Ρ‚ΡŒ Π²Ρ‹Ρ€Π°ΠΆΠ΅Π½ ΠΊΠ°ΠΊ вСкторная сумма Π΅Π³ΠΎ Π²Π΅ΠΊΡ‚ΠΎΡ€Π½Ρ‹Ρ… ΠΊΠΎΠΌΠΏΠΎΠ½Π΅Π½Ρ‚ΠΎΠ²:

A→=A→x+A→y.A→=A→x+A→ у.

2.10

Как ΠΏΠΎΠΊΠ°Π·Π°Π½ΠΎ Π½Π° рис. 2.16, Π²Π΅ΠΊΡ‚ΠΎΡ€ Aβ†’Aβ†’ являСтся диагональю ΠΏΡ€ΡΠΌΠΎΡƒΠ³ΠΎΠ»ΡŒΠ½ΠΈΠΊΠ°, Π³Π΄Π΅ x -ΠΊΠΎΠΌΠΏΠΎΠ½Π΅Π½Ρ‚Π° Aβ†’xAβ†’x являСтся стороной, ΠΏΠ°Ρ€Π°Π»Π»Π΅Π»ΡŒΠ½ΠΎΠΉ оси x , Π° y -ΠΊΠΎΠΌΠΏΠΎΠ½Π΅Π½Ρ‚Π° Aβ†’yAβ†’y являСтся стороной, ΠΏΠ°Ρ€Π°Π»Π»Π΅Π»ΡŒΠ½ΠΎΠΉ оси y . ΠšΠΎΠΌΠΏΠΎΠ½Π΅Π½Ρ‚Π° Π²Π΅ΠΊΡ‚ΠΎΡ€Π° Aβ†’xAβ†’x ΠΎΡ€Ρ‚ΠΎΠ³ΠΎΠ½Π°Π»ΡŒΠ½Π° ΠΊΠΎΠΌΠΏΠΎΠ½Π΅Π½Ρ‚Π΅ Π²Π΅ΠΊΡ‚ΠΎΡ€Π° Aβ†’yAβ†’y.

Рисунок 2.16 Π’Π΅ΠΊΡ‚ΠΎΡ€ Aβ†’Aβ†’ Π½Π° плоскости Π² Π΄Π΅ΠΊΠ°Ρ€Ρ‚ΠΎΠ²ΠΎΠΉ систСмС ΠΊΠΎΠΎΡ€Π΄ΠΈΠ½Π°Ρ‚ прСдставляСт собой Π²Π΅ΠΊΡ‚ΠΎΡ€Π½ΡƒΡŽ сумму Π΅Π³ΠΎ Π²Π΅ΠΊΡ‚ΠΎΡ€Π½Ρ‹Ρ… x — ΠΈ y -ΠΊΠΎΠΌΠΏΠΎΠ½Π΅Π½Ρ‚. ΠšΠΎΠΌΠΏΠΎΠ½Π΅Π½Ρ‚Π° Π²Π΅ΠΊΡ‚ΠΎΡ€Π° Aβ†’xAβ†’x Ρ€Π°Π·ΠΌΠ΅Ρ€ΠΎΠΌ x являСтся ΠΎΡ€Ρ‚ΠΎΠ³ΠΎΠ½Π°Π»ΡŒΠ½ΠΎΠΉ ΠΏΡ€ΠΎΠ΅ΠΊΡ†ΠΈΠ΅ΠΉ Π²Π΅ΠΊΡ‚ΠΎΡ€Π° Aβ†’Aβ†’ Π½Π° 9.

2.12

Если ΠΌΡ‹ Π·Π½Π°Π΅ΠΌ ΠΊΠΎΠΎΡ€Π΄ΠΈΠ½Π°Ρ‚Ρ‹ b(xb,yb)b(xb,yb) Π½Π°Ρ‡Π°Π»ΡŒΠ½ΠΎΠΉ Ρ‚ΠΎΡ‡ΠΊΠΈ Π²Π΅ΠΊΡ‚ΠΎΡ€Π° (Π³Π΄Π΅ b ΠΎΠ·Π½Π°Ρ‡Π°Π΅Ρ‚ Β«Π½Π°Ρ‡Π°Π»ΠΎΒ») ΠΈ ΠΊΠΎΠΎΡ€Π΄ΠΈΠ½Π°Ρ‚Ρ‹ e(xe,ye)e(xe,ye ) ΠΊΠΎΠ½Π΅Ρ‡Π½ΠΎΠΉ Ρ‚ΠΎΡ‡ΠΊΠΈ Π²Π΅ΠΊΡ‚ΠΎΡ€Π° (Π³Π΄Π΅ e ΠΎΠ·Π½Π°Ρ‡Π°Π΅Ρ‚ Β«ΠΊΠΎΠ½Π΅Ρ†Β»), ΠΌΡ‹ ΠΌΠΎΠΆΠ΅ΠΌ ΠΏΠΎΠ»ΡƒΡ‡ΠΈΡ‚ΡŒ скалярныС ΠΊΠΎΠΌΠΏΠΎΠ½Π΅Π½Ρ‚Ρ‹ Π²Π΅ΠΊΡ‚ΠΎΡ€Π°, просто вычитая ΠΊΠΎΠΎΡ€Π΄ΠΈΠ½Π°Ρ‚Ρ‹ исходной Ρ‚ΠΎΡ‡ΠΊΠΈ ΠΈΠ· ΠΊΠΎΠΎΡ€Π΄ΠΈΠ½Π°Ρ‚ ΠΊΠΎΠ½Π΅Ρ‡Π½ΠΎΠΉ Ρ‚ΠΎΡ‡ΠΊΠΈ:

{Ax=xeβˆ’xbAy =ye-yb.{Ax=xe-xbAy=ye-yb.

2.13

ΠŸΡ€ΠΈΠΌΠ΅Ρ€ 2.3 9Π½Π°

y — ось ΡƒΠΊΠ°Π·Ρ‹Π²Π°Π΅Ρ‚ Π²Π΅Ρ€Ρ‚ΠΈΠΊΠ°Π»ΡŒΠ½ΠΎ Π²Π²Π΅Ρ€Ρ…. Начало Π²Π΅ΠΊΡ‚ΠΎΡ€Π° смСщСния располоТСно Π² Ρ‚ΠΎΡ‡ΠΊΠ΅ b(6.0, 1.6), Π° ΠΊΠΎΠ½Π΅Ρ† Π²Π΅ΠΊΡ‚ΠΎΡ€Π° смСщСния располоТСн Π² Ρ‚ΠΎΡ‡ΠΊΠ΅ e(2. 0, 4.5). ΠŸΠΎΠ΄ΡΡ‚Π°Π²ΡŒΡ‚Π΅ ΠΊΠΎΠΎΡ€Π΄ΠΈΠ½Π°Ρ‚Ρ‹ этих Ρ‚ΠΎΡ‡Π΅ΠΊ Π² ΡƒΡ€Π°Π²Π½Π΅Π½ΠΈΠ΅ 2.13, Ρ‡Ρ‚ΠΎΠ±Ρ‹ Π½Π°ΠΉΡ‚ΠΈ скалярныС ΠΊΠΎΠΌΠΏΠΎΠ½Π΅Π½Ρ‚Ρ‹ DxDx ΠΈ DyDy Π²Π΅ΠΊΡ‚ΠΎΡ€Π° смСщСния Dβ†’Dβ†’. НаконСц, ΠΏΠΎΠ΄ΡΡ‚Π°Π²ΡŒΡ‚Π΅ ΠΊΠΎΠΎΡ€Π΄ΠΈΠ½Π°Ρ‚Ρ‹ Π² ΡƒΡ€Π°Π²Π½Π΅Π½ΠΈΠ΅ 2.12, Ρ‡Ρ‚ΠΎΠ±Ρ‹ Π·Π°ΠΏΠΈΡΠ°Ρ‚ΡŒ Π²Π΅ΠΊΡ‚ΠΎΡ€ смСщСния Π² Ρ„ΠΎΡ€ΠΌΠ΅ ΠΊΠΎΠΌΠΏΠΎΠ½Π΅Π½Ρ‚Π° Π²Π΅ΠΊΡ‚ΠΎΡ€Π°.

Раствор

ΠœΡ‹ опрСдСляСм xb=6,0xb=6,0, xe=2,0xe=2,0, yb=1,6yb=1,6 ΠΈ ye=4,5ye=4,5, Π³Π΄Π΅ физичСской Π΅Π΄ΠΈΠ½ΠΈΡ†Π΅ΠΉ являСтся 1 см. Бкаляр 9)см.

2.14

Π­Ρ‚ΠΎ Ρ€Π΅ΡˆΠ΅Π½ΠΈΠ΅ ΠΏΠΎΠΊΠ°Π·Π°Π½ΠΎ Π½Π° рис. 2.17.

Рисунок 2.17 Π“Ρ€Π°Ρ„ΠΈΠΊ Π²Π΅ΠΊΡ‚ΠΎΡ€Π° смСщСния. Π’Π΅ΠΊΡ‚ΠΎΡ€ ΡƒΠΊΠ°Π·Ρ‹Π²Π°Π΅Ρ‚ ΠΎΡ‚ Π½Π°Ρ‡Π°Π»ΡŒΠ½ΠΎΠΉ Ρ‚ΠΎΡ‡ΠΊΠΈ b Π΄ΠΎ ΠΊΠΎΠ½Π΅Ρ‡Π½ΠΎΠΉ Ρ‚ΠΎΡ‡ΠΊΠΈ e .

Π—Π½Π°Ρ‡Π΅Π½ΠΈΠ΅

ΠžΠ±Ρ€Π°Ρ‚ΠΈΡ‚Π΅ Π²Π½ΠΈΠΌΠ°Π½ΠΈΠ΅, Ρ‡Ρ‚ΠΎ физичСская Π΅Π΄ΠΈΠ½ΠΈΡ†Π° β€” здСсь 1 см β€” ΠΌΠΎΠΆΠ΅Ρ‚ Π±Ρ‹Ρ‚ΡŒ ΠΏΠΎΠΌΠ΅Ρ‰Π΅Π½Π° Π»ΠΈΠ±ΠΎ с ΠΊΠ°ΠΆΠ΄Ρ‹ΠΌ ΠΊΠΎΠΌΠΏΠΎΠ½Π΅Π½Ρ‚ΠΎΠΌ нСпосрСдствСнно ΠΏΠ΅Ρ€Π΅Π΄ Π΅Π΄ΠΈΠ½ΠΈΡ‡Π½Ρ‹ΠΌ Π²Π΅ΠΊΡ‚ΠΎΡ€ΠΎΠΌ, Π»ΠΈΠ±ΠΎ глобально для ΠΎΠ±ΠΎΠΈΡ… ΠΊΠΎΠΌΠΏΠΎΠ½Π΅Π½Ρ‚ΠΎΠ², ΠΊΠ°ΠΊ Π² ΡƒΡ€Π°Π²Π½Π΅Π½ΠΈΠΈ 2.14. Часто послСдний способ ΡƒΠ΄ΠΎΠ±Π½Π΅Π΅, ΠΏΠΎΡ‚ΠΎΠΌΡƒ Ρ‡Ρ‚ΠΎ ΠΎΠ½ ΠΏΡ€ΠΎΡ‰Π΅. 9, ΠΊΠΎΡ‚ΠΎΡ€Ρ‹ΠΉ ΠΏΠ°Ρ€Π°Π»Π»Π΅Π»Π΅Π½ Π½Π°ΠΏΡ€Π°Π²Π»Π΅Π½ΠΈΡŽ оси + y -. Π‘Π»Π΅Π΄ΠΎΠ²Π°Ρ‚Π΅Π»ΡŒΠ½ΠΎ, y -ΠΊΠΎΠΌΠΏΠΎΠ½Π΅Π½Ρ‚Π½Ρ‹ΠΉ Π²Π΅ΠΊΡ‚ΠΎΡ€ Dβ†’yDβ†’y ΡƒΠΊΠ°Π·Ρ‹Π²Π°Π΅Ρ‚ Π²Π²Π΅Ρ€Ρ…, ΠΊΠ°ΠΊ ΠΏΠΎΠΊΠ°Π·Π°Π½ΠΎ Π½Π° рис. 2.17. Бкаляр y -ΠΊΠΎΠΌΠΏΠΎΠ½Π΅Π½Ρ‚Π° Π²Π΅ΠΊΡ‚ΠΎΡ€Π° Dβ†’Dβ†’ Ρ€Π°Π²Π΅Π½ Dy=+2,9Dy=+2,9. Π’Π΅ΠΊΡ‚ΠΎΡ€ смСщСния Dβ†’Dβ†’ являСтся Ρ€Π΅Π·ΡƒΠ»ΡŒΡ‚Π°Ρ‚ΠΎΠΌ Π΄Π²ΡƒΡ… Π΅Π³ΠΎ ΠΊΠΎΠΌΠΏΠΎΠ½Π΅Π½Ρ‚ Π²Π΅ΠΊΡ‚ΠΎΡ€Π° .

Π€ΠΎΡ€ΠΌΠ° Π²Π΅ΠΊΡ‚ΠΎΡ€Π½ΠΎΠΉ ΡΠΎΡΡ‚Π°Π²Π»ΡΡŽΡ‰Π΅ΠΉ Π²Π΅ΠΊΡ‚ΠΎΡ€Π° смСщСния Π£Ρ€Π°Π²Π½Π΅Π½ΠΈΠ΅ 2.14 Π³ΠΎΠ²ΠΎΡ€ΠΈΡ‚ Π½Π°ΠΌ, Ρ‡Ρ‚ΠΎ ΡƒΠΊΠ°Π·Π°Ρ‚Π΅Π»ΡŒ ΠΌΡ‹ΡˆΠΈ пСрСмСстился Π½Π° ΠΌΠΎΠ½ΠΈΡ‚ΠΎΡ€Π΅ Π½Π° 4,0 см Π²Π»Π΅Π²ΠΎ ΠΈ Π½Π° 2,9 см Π²Π²Π΅Ρ€Ρ… ΠΎΡ‚ своСго Π½Π°Ρ‡Π°Π»ΡŒΠ½ΠΎΠ³ΠΎ полоТСния.

ΠŸΡ€ΠΎΠ²Π΅Ρ€ΡŒΡ‚Π΅ своС ΠΏΠΎΠ½ΠΈΠΌΠ°Π½ΠΈΠ΅ 2,4

Биняя ΠΌΡƒΡ…Π° призСмляСтся Π½Π° лист ΠΌΠΈΠ»Π»ΠΈΠΌΠ΅Ρ‚Ρ€ΠΎΠ²ΠΎΠΉ Π±ΡƒΠΌΠ°Π³ΠΈ Π² Ρ‚ΠΎΡ‡ΠΊΠ΅, располоТСнной Π½Π° 10,0 см ΠΏΡ€Π°Π²Π΅Π΅ Π΅Π³ΠΎ Π»Π΅Π²ΠΎΠ³ΠΎ края ΠΈ Π½Π° 8,0 см Π²Ρ‹ΡˆΠ΅ Π΅Π³ΠΎ Π½ΠΈΠΆΠ½Π΅Π³ΠΎ края, ΠΈ ΠΌΠ΅Π΄Π»Π΅Π½Π½ΠΎ двиТСтся ΠΊ Ρ‚ΠΎΡ‡ΠΊΠ΅, располоТСнной Π½Π° расстоянии 5,0 см ΠΎΡ‚ Π»Π΅Π²ΠΎΠ³ΠΎ края ΠΈ 5,0 см ΠΎΡ‚ Π½ΠΈΠΆΠ½Π΅Π³ΠΎ края Π½ΠΈΠΆΠ½ΠΈΠΉ ΠΊΡ€Π°ΠΉ. Π’Ρ‹Π±Π΅Ρ€ΠΈΡ‚Π΅ ΠΏΡ€ΡΠΌΠΎΡƒΠ³ΠΎΠ»ΡŒΠ½ΡƒΡŽ систСму ΠΊΠΎΠΎΡ€Π΄ΠΈΠ½Π°Ρ‚ с Π½Π°Ρ‡Π°Π»ΠΎΠΌ Π² Π»Π΅Π²ΠΎΠΌ Π½ΠΈΠΆΠ½Π΅ΠΌ ΡƒΠ³Π»Ρƒ листа ΠΈ Π½Π°ΠΉΠ΄ΠΈΡ‚Π΅ Π²Π΅ΠΊΡ‚ΠΎΡ€ смСщСния ΠΌΡƒΡ…ΠΈ. ΠŸΡ€ΠΎΠΈΠ»Π»ΡŽΡΡ‚Ρ€ΠΈΡ€ΡƒΠΉΡ‚Π΅ своС Ρ€Π΅ΡˆΠ΅Π½ΠΈΠ΅ Π³Ρ€Π°Ρ„ΠΈΠΊΠΎΠΌ.

Зная скалярныС ΠΊΠΎΠΌΠΏΠΎΠ½Π΅Π½Ρ‚Ρ‹ AxAx ΠΈ AyAy Π²Π΅ΠΊΡ‚ΠΎΡ€Π° Aβ†’Aβ†’, ΠΌΡ‹ ΠΌΠΎΠΆΠ΅ΠΌ Π½Π°ΠΉΡ‚ΠΈ Π΅Π³ΠΎ Π²Π΅Π»ΠΈΡ‡ΠΈΠ½Ρƒ A ΠΈ Π΅Π³ΠΎ Π΄ΠΈΡ€Π΅ΠΊΡ†ΠΈΠΎΠ½Π½Ρ‹ΠΉ ΡƒΠ³ΠΎΠ» ΞΈAΞΈA. Π£Π³ΠΎΠ» направлСния β€” ΠΈΠ»ΠΈ, для краткости, Π½Π°ΠΏΡ€Π°Π²Π»Π΅Π½ΠΈΠ΅ β€” это ΡƒΠ³ΠΎΠ», ΠΊΠΎΡ‚ΠΎΡ€Ρ‹ΠΉ Π²Π΅ΠΊΡ‚ΠΎΡ€ ΠΎΠ±Ρ€Π°Π·ΡƒΠ΅Ρ‚ с ΠΏΠΎΠ»ΠΎΠΆΠΈΡ‚Π΅Π»ΡŒΠ½Ρ‹ΠΌ Π½Π°ΠΏΡ€Π°Π²Π»Π΅Π½ΠΈΠ΅ΠΌ Π½Π° оси x . Π£Π³ΠΎΠ» ΞΈAΞΈA измСряСтся Π² Π½Π°ΠΏΡ€Π°Π²Π»Π΅Π½ΠΈΠΈ ΠΏΡ€ΠΎΡ‚ΠΈΠ² часовой стрСлки ΠΎΡ‚ оси +x ΠΊ Π²Π΅ΠΊΡ‚ΠΎΡ€Ρƒ (рисунок 2.18). ΠŸΠΎΡΠΊΠΎΠ»ΡŒΠΊΡƒ Π΄Π»ΠΈΠ½Ρ‹ A , AxAx ΠΈ AyAy ΠΎΠ±Ρ€Π°Π·ΡƒΡŽΡ‚ ΠΏΡ€ΡΠΌΠΎΡƒΠ³ΠΎΠ»ΡŒΠ½Ρ‹ΠΉ Ρ‚Ρ€Π΅ΡƒΠ³ΠΎΠ»ΡŒΠ½ΠΈΠΊ, ΠΎΠ½ΠΈ связаны Ρ‚Π΅ΠΎΡ€Π΅ΠΌΠΎΠΉ ΠŸΠΈΡ„Π°Π³ΠΎΡ€Π°:

A2=Ax2+Ay2⇔A=Ax2+Ay2.A2=Ax2+Ay2⇔A=Ax2+Ay2.

2,15

Π­Ρ‚ΠΎ ΡƒΡ€Π°Π²Π½Π΅Π½ΠΈΠ΅ Ρ€Π°Π±ΠΎΡ‚Π°Π΅Ρ‚, Π΄Π°ΠΆΠ΅ Ссли скалярныС ΠΊΠΎΠΌΠΏΠΎΠ½Π΅Π½Ρ‚Ρ‹ Π²Π΅ΠΊΡ‚ΠΎΡ€Π° ΠΎΡ‚Ρ€ΠΈΡ†Π°Ρ‚Π΅Π»ΡŒΠ½Ρ‹. ΠΠ°ΠΏΡ€Π°Π²Π»ΡΡŽΡ‰ΠΈΠΉ ΡƒΠ³ΠΎΠ» ΞΈAΞΈA Π²Π΅ΠΊΡ‚ΠΎΡ€Π° опрСдСляСтся Ρ‡Π΅Ρ€Π΅Π· тангСнс Ρ„ΡƒΠ½ΠΊΡ†ΠΈΠΈ ΡƒΠ³Π»Π° ΞΈAΞΈA Π² Ρ‚Ρ€Π΅ΡƒΠ³ΠΎΠ»ΡŒΠ½ΠΈΠΊΠ΅, ΠΏΠΎΠΊΠ°Π·Π°Π½Π½ΠΎΠΌ Π½Π° рисункС 2.18: 2.18 Когда Π²Π΅ΠΊΡ‚ΠΎΡ€ Π»Π΅ΠΆΠΈΡ‚ Π»ΠΈΠ±ΠΎ Π² ΠΏΠ΅Ρ€Π²ΠΎΠΌ ΠΊΠ²Π°Π΄Ρ€Π°Π½Ρ‚Π΅, Π»ΠΈΠ±ΠΎ Π² Ρ‡Π΅Ρ‚Π²Π΅Ρ€Ρ‚ΠΎΠΌ ΠΊΠ²Π°Π΄Ρ€Π°Π½Ρ‚Π΅, Π³Π΄Π΅ ΡΠΎΡΡ‚Π°Π²Π»ΡΡŽΡ‰Π°Ρ AxAxis ΠΏΠΎΠ»ΠΎΠΆΠΈΡ‚Π΅Π»ΡŒΠ½Π° (рис. 2.19), ΡƒΠ³ΠΎΠ» направлСния ΞΈAΞΈA Π² ΡƒΡ€Π°Π²Π½Π΅Π½ΠΈΠΈ 2.16) ΠΈΠ΄Π΅Π½Ρ‚ΠΈΡ‡Π΅Π½ ΡƒΠ³Π»Ρƒ ΞΈΞΈ.

Когда Π²Π΅ΠΊΡ‚ΠΎΡ€ Π»Π΅ΠΆΠΈΡ‚ Π»ΠΈΠ±ΠΎ Π² ΠΏΠ΅Ρ€Π²ΠΎΠΌ ΠΊΠ²Π°Π΄Ρ€Π°Π½Ρ‚Π΅, Π»ΠΈΠ±ΠΎ Π² Ρ‡Π΅Ρ‚Π²Π΅Ρ€Ρ‚ΠΎΠΌ ΠΊΠ²Π°Π΄Ρ€Π°Π½Ρ‚Π΅, Π³Π΄Π΅ ΡΠΎΡΡ‚Π°Π²Π»ΡΡŽΡ‰Π°Ρ AxAx ΠΏΠΎΠ»ΠΎΠΆΠΈΡ‚Π΅Π»ΡŒΠ½Π° (рис. 2.19), ΡƒΠ³ΠΎΠ» ΞΈΞΈ Π² ΡƒΡ€Π°Π²Π½Π΅Π½ΠΈΠΈ 2.16 ΠΈΠ΄Π΅Π½Ρ‚ΠΈΡ‡Π΅Π½ ΡƒΠ³Π»Ρƒ направлСния ΞΈAΞΈA. Для Π²Π΅ΠΊΡ‚ΠΎΡ€ΠΎΠ² Π² Ρ‡Π΅Ρ‚Π²Π΅Ρ€Ρ‚ΠΎΠΌ ΠΊΠ²Π°Π΄Ρ€Π°Π½Ρ‚Π΅ ΡƒΠ³ΠΎΠ» ΞΈΞΈ ΠΎΡ‚Ρ€ΠΈΡ†Π°Ρ‚Π΅Π»Π΅Π½, Ρ‡Ρ‚ΠΎ ΠΎΠ·Π½Π°Ρ‡Π°Π΅Ρ‚, Ρ‡Ρ‚ΠΎ для этих Π²Π΅ΠΊΡ‚ΠΎΡ€ΠΎΠ² ΡƒΠ³ΠΎΠ» направлСния ΞΈAΞΈA измСряСтся Π½Π° ΠΏΠΎ часовой стрСлкС ΠΎΡ‚ ΠΏΠΎΠ»ΠΎΠΆΠΈΡ‚Π΅Π»ΡŒΠ½ΠΎΠΉ оси x . Π’ΠΎΡ‡Π½ΠΎ Ρ‚Π°ΠΊ ΠΆΠ΅ для Π²Π΅ΠΊΡ‚ΠΎΡ€ΠΎΠ² Π²ΠΎ Π²Ρ‚ΠΎΡ€ΠΎΠΌ ΠΊΠ²Π°Π΄Ρ€Π°Π½Ρ‚Π΅ ΡƒΠ³ΠΎΠ» ΞΈΞΈ ΠΎΡ‚Ρ€ΠΈΡ†Π°Ρ‚Π΅Π»Π΅Π½. Когда Π²Π΅ΠΊΡ‚ΠΎΡ€ Π»Π΅ΠΆΠΈΡ‚ Π»ΠΈΠ±ΠΎ Π²ΠΎ Π²Ρ‚ΠΎΡ€ΠΎΠΌ, Π»ΠΈΠ±ΠΎ Π² Ρ‚Ρ€Π΅Ρ‚ΡŒΠ΅ΠΌ ΠΊΠ²Π°Π΄Ρ€Π°Π½Ρ‚Π΅, Π³Π΄Π΅ ΠΊΠΎΠΌΠΏΠΎΠ½Π΅Π½Ρ‚Π° AxAx ΠΎΡ‚Ρ€ΠΈΡ†Π°Ρ‚Π΅Π»ΡŒΠ½Π°, ΡƒΠ³ΠΎΠ» направлСния Ρ€Π°Π²Π΅Π½ ΞΈA=ΞΈ+180Β°, ΞΈA=ΞΈ+180Β° (рис. 2.19).).

Рисунок 2.19 БкалярныС ΠΊΠΎΠΌΠΏΠΎΠ½Π΅Π½Ρ‚Ρ‹ Π²Π΅ΠΊΡ‚ΠΎΡ€Π° ΠΌΠΎΠ³ΡƒΡ‚ Π±Ρ‹Ρ‚ΡŒ ΠΏΠΎΠ»ΠΎΠΆΠΈΡ‚Π΅Π»ΡŒΠ½Ρ‹ΠΌΠΈ ΠΈΠ»ΠΈ ΠΎΡ‚Ρ€ΠΈΡ†Π°Ρ‚Π΅Π»ΡŒΠ½Ρ‹ΠΌΠΈ. Π’Π΅ΠΊΡ‚ΠΎΡ€Ρ‹ Π² ΠΏΠ΅Ρ€Π²ΠΎΠΌ ΠΊΠ²Π°Π΄Ρ€Π°Π½Ρ‚Π΅ (I) ΠΈΠΌΠ΅ΡŽΡ‚ ΠΎΠ±Π΅ скалярныС ΠΊΠΎΠΌΠΏΠΎΠ½Π΅Π½Ρ‚Ρ‹ ΠΏΠΎΠ»ΠΎΠΆΠΈΡ‚Π΅Π»ΡŒΠ½Ρ‹Π΅, Π° Π²Π΅ΠΊΡ‚ΠΎΡ€Ρ‹ Π² Ρ‚Ρ€Π΅Ρ‚ΡŒΠ΅ΠΌ ΠΊΠ²Π°Π΄Ρ€Π°Π½Ρ‚Π΅ ΠΈΠΌΠ΅ΡŽΡ‚ ΠΎΠ±Π΅ скалярныС ΠΊΠΎΠΌΠΏΠΎΠ½Π΅Π½Ρ‚Ρ‹ ΠΎΡ‚Ρ€ΠΈΡ†Π°Ρ‚Π΅Π»ΡŒΠ½Ρ‹Π΅. Для Π²Π΅ΠΊΡ‚ΠΎΡ€ΠΎΠ² Π² ΠΊΠ²Π°Π΄Ρ€Π°Π½Ρ‚Π°Ρ… II ΠΈ III ΡƒΠ³ΠΎΠ» направлСния Π²Π΅ΠΊΡ‚ΠΎΡ€Π° Ρ€Π°Π²Π΅Π½ ΞΈA=ΞΈ+180Β°, ΞΈA=ΞΈ+180Β°.

ΠŸΡ€ΠΈΠΌΠ΅Ρ€ 2,4

Π’Π΅Π»ΠΈΡ‡ΠΈΠ½Π° ΠΈ Π½Π°ΠΏΡ€Π°Π²Π»Π΅Π½ΠΈΠ΅ Π²Π΅ΠΊΡ‚ΠΎΡ€Π° смСщСния

Π’Ρ‹ ΠΏΠ΅Ρ€Π΅ΠΌΠ΅Ρ‰Π°Π΅Ρ‚Π΅ ΡƒΠΊΠ°Π·Π°Ρ‚Π΅Π»ΡŒ ΠΌΡ‹ΡˆΠΈ Π½Π° ΠΌΠΎΠ½ΠΈΡ‚ΠΎΡ€Π΅ ΠΈΠ· исходного полоТСния Π² Ρ‚ΠΎΡ‡ΠΊΠ΅ (6,0 см, 1,6 см) ΠΊ Π·Π½Π°Ρ‡ΠΊΡƒ, располоТСнному Π² Ρ‚ΠΎΡ‡ΠΊΠ΅ (2,0 см, 4,5 см). ΠšΠ°ΠΊΠΎΠ²Ρ‹ Π²Π΅Π»ΠΈΡ‡ΠΈΠ½Π° ΠΈ Π½Π°ΠΏΡ€Π°Π²Π»Π΅Π½ΠΈΠ΅ Π²Π΅ΠΊΡ‚ΠΎΡ€Π° смСщСния указатСля?

БтратСгия

Π’ ΠΏΡ€ΠΈΠΌΠ΅Ρ€Π΅ 2.3 ΠΌΡ‹ нашли Π²Π΅ΠΊΡ‚ΠΎΡ€ смСщСния Dβ†’Dβ†’ указатСля ΠΌΡ‹ΡˆΠΈ (см. ΡƒΡ€Π°Π²Π½Π΅Π½ΠΈΠ΅ 2.14). ΠœΡ‹ ΠΈΠ΄Π΅Π½Ρ‚ΠΈΡ„ΠΈΡ†ΠΈΡ€ΡƒΠ΅ΠΌ Π΅Π³ΠΎ скалярныС ΠΊΠΎΠΌΠΏΠΎΠ½Π΅Π½Ρ‚Ρ‹ Dx=-4,0 смDx=-4,0 см ΠΈ Dy=+2,9 смDy=+2,9 см ΠΈ подставляСм Π² уравнСния 2.15 ΠΈ 2.16, Ρ‡Ρ‚ΠΎΠ±Ρ‹ Π½Π°ΠΉΡ‚ΠΈ Π·Π²Π΅Π·Π΄Π½ΡƒΡŽ Π²Π΅Π»ΠΈΡ‡ΠΈΠ½Ρƒ D ΠΈ Π½Π°ΠΏΡ€Π°Π²Π»Π΅Π½ΠΈΠ΅ ΞΈDΞΈD соотвСтствСнно.

Раствор

ВСличина вСктора D→D→ равна

D=Dx2+Dy2=(βˆ’4,0 см)2+(2,9 см)2=(4,0)2+(2,9)2 см=4,9 см. D=Dx2+Dy2=(βˆ’4,0 см)2+(2,9 см) )2=(4,0)2+(2,9)см, Π½Π°ΠΉΡ‚ΠΈ Π΅Π³ΠΎ Π²Π΅Π»ΠΈΡ‡ΠΈΠ½Ρƒ ΠΈ Π½Π°ΠΏΡ€Π°Π²Π»Π΅Π½ΠΈΠ΅.

Π’ΠΎ ΠΌΠ½ΠΎΠ³ΠΈΡ… прилоТСниях Π²Π΅Π»ΠΈΡ‡ΠΈΠ½Ρ‹ ΠΈ направлСния Π²Π΅ΠΊΡ‚ΠΎΡ€Π½Ρ‹Ρ… Π²Π΅Π»ΠΈΡ‡ΠΈΠ½ извСстны, ΠΈ Π½Π°ΠΌ Π½ΡƒΠΆΠ½ΠΎ Π½Π°ΠΉΡ‚ΠΈ Ρ€Π°Π²Π½ΠΎΠ΄Π΅ΠΉΡΡ‚Π²ΡƒΡŽΡ‰ΡƒΡŽ ΠΌΠ½ΠΎΠ³ΠΈΡ… Π²Π΅ΠΊΡ‚ΠΎΡ€ΠΎΠ². НапримСр, ΠΏΡ€Π΅Π΄ΡΡ‚Π°Π²ΡŒΡ‚Π΅, Ρ‡Ρ‚ΠΎ 400 Π°Π²Ρ‚ΠΎΠΌΠΎΠ±ΠΈΠ»Π΅ΠΉ двиТутся ΠΏΠΎ мосту Π—ΠΎΠ»ΠΎΡ‚Ρ‹Π΅ Π’ΠΎΡ€ΠΎΡ‚Π° Π² Π‘Π°Π½-Ѐранциско ΠΏΡ€ΠΈ сильном Π²Π΅Ρ‚Ρ€Π΅. КаТдая машина Π΄Π°Π΅Ρ‚ мосту Ρ€Π°Π·Π½Ρ‹Π΅ Ρ‚ΠΎΠ»Ρ‡ΠΊΠΈ Π² Ρ€Π°Π·Π½Ρ‹Ρ… направлСниях, ΠΈ ΠΌΡ‹ Ρ…ΠΎΡ‚Π΅Π»ΠΈ Π±Ρ‹ Π·Π½Π°Ρ‚ΡŒ, насколько большим ΠΌΠΎΠΆΠ΅Ρ‚ Π±Ρ‹Ρ‚ΡŒ Ρ€Π΅Π·ΡƒΠ»ΡŒΡ‚ΠΈΡ€ΡƒΡŽΡ‰ΠΈΠΉ Ρ‚ΠΎΠ»Ρ‡ΠΎΠΊ. Π£ нас ΡƒΠΆΠ΅ Π΅ΡΡ‚ΡŒ Π½Π΅ΠΊΠΎΡ‚ΠΎΡ€Ρ‹ΠΉ ΠΎΠΏΡ‹Ρ‚ гСомСтричСского построСния Π²Π΅ΠΊΡ‚ΠΎΡ€Π½Ρ‹Ρ… сумм, поэтому ΠΌΡ‹ Π·Π½Π°Π΅ΠΌ, Ρ‡Ρ‚ΠΎ Π·Π°Π΄Π°Ρ‡Π° нахоТдСния Ρ€Π°Π²Π½ΠΎΠ΄Π΅ΠΉΡΡ‚Π²ΡƒΡŽΡ‰Π΅ΠΉ ΠΏΡƒΡ‚Π΅ΠΌ рисования Π²Π΅ΠΊΡ‚ΠΎΡ€ΠΎΠ² ΠΈ измСрСния ΠΈΡ… Π΄Π»ΠΈΠ½ ΠΈ ΡƒΠ³Π»ΠΎΠ² ΠΌΠΎΠΆΠ΅Ρ‚ довольно быстро ΡΡ‚Π°Ρ‚ΡŒ Π½Π΅Ρ€Π°Π·Ρ€Π΅ΡˆΠΈΠΌΠΎΠΉ, Ρ‡Ρ‚ΠΎ ΠΏΡ€ΠΈΠ²Π΅Π΄Π΅Ρ‚ ΠΊ ΠΎΠ³Ρ€ΠΎΠΌΠ½Ρ‹ΠΌ ошибкам. Подобного бСспокойства Π½Π΅ Π²ΠΎΠ·Π½ΠΈΠΊΠ°Π΅Ρ‚, ΠΊΠΎΠ³Π΄Π° ΠΌΡ‹ ΠΈΡΠΏΠΎΠ»ΡŒΠ·ΡƒΠ΅ΠΌ аналитичСскиС ΠΌΠ΅Ρ‚ΠΎΠ΄Ρ‹. Π‘Π°ΠΌΡ‹ΠΉ ΠΏΠ΅Ρ€Π²Ρ‹ΠΉ шаг Π² аналитичСском ΠΏΠΎΠ΄Ρ…ΠΎΠ΄Π΅ состоит Π² Ρ‚ΠΎΠΌ, Ρ‡Ρ‚ΠΎΠ±Ρ‹ Π½Π°ΠΉΡ‚ΠΈ ΠΊΠΎΠΌΠΏΠΎΠ½Π΅Π½Ρ‚Ρ‹ Π²Π΅ΠΊΡ‚ΠΎΡ€Π°, ΠΊΠΎΠ³Π΄Π° извСстны Π½Π°ΠΏΡ€Π°Π²Π»Π΅Π½ΠΈΠ΅ ΠΈ Π²Π΅Π»ΠΈΡ‡ΠΈΠ½Π° Π²Π΅ΠΊΡ‚ΠΎΡ€Π°.

ВСрнСмся ΠΊ ΠΏΡ€ΡΠΌΠΎΡƒΠ³ΠΎΠ»ΡŒΠ½ΠΎΠΌΡƒ Ρ‚Ρ€Π΅ΡƒΠ³ΠΎΠ»ΡŒΠ½ΠΈΠΊΡƒ Π½Π° рис. 2.18. ΠžΡ‚Π½ΠΎΡˆΠ΅Π½ΠΈΠ΅ ΠΏΡ€ΠΎΡ‚ΠΈΠ²ΠΎΠ»Π΅ΠΆΠ°Ρ‰Π΅Π³ΠΎ ΠΊΠ°Ρ‚Π΅Ρ‚Π° AxAx ΠΊ Π³ΠΈΠΏΠΎΡ‚Π΅Π½ΡƒΠ·Π΅ A являСтся Ρ„ΡƒΠ½ΠΊΡ†ΠΈΠ΅ΠΉ косинуса Π½Π°ΠΏΡ€Π°Π²Π»ΡΡŽΡ‰Π΅Π³ΠΎ ΡƒΠ³Π»Π° ΞΈAΞΈA, Ax/A=cosΞΈAAx/A=cosΞΈA, Π° ΠΎΡ‚Π½ΠΎΡˆΠ΅Π½ΠΈΠ΅ ΠΏΡ€ΠΎΡ‚ΠΈΠ²ΠΎΠ»Π΅ΠΆΠ°Ρ‰Π΅Π³ΠΎ ΠΊΠ°Ρ‚Π΅Ρ‚Π° AyAy ΠΊ Π³ΠΈΠΏΠΎΡ‚Π΅Π½ΡƒΠ·Π΅ A являСтся Ρ„ΡƒΠ½ΠΊΡ†ΠΈΠ΅ΠΉ синуса ΞΈAΞΈA, Ay/A=sinΞΈAAy/A=sinΞΈA. Когда извСстна Π²Π΅Π»ΠΈΡ‡ΠΈΠ½Π° A ΠΈ Π½Π°ΠΏΡ€Π°Π²Π»Π΅Π½ΠΈΠ΅ ΞΈAΞΈA, ΠΌΡ‹ ΠΌΠΎΠΆΠ΅ΠΌ Ρ€Π΅ΡˆΠΈΡ‚ΡŒ эти ΡΠΎΠΎΡ‚Π½ΠΎΡˆΠ΅Π½ΠΈΡ для скалярных ΡΠΎΡΡ‚Π°Π²Π»ΡΡŽΡ‰ΠΈΡ…:

{Ax=AcosΞΈAAy=AsinΞΈA.{Ax=AcosΞΈAAy=AsinΞΈA.

2.17

ΠŸΡ€ΠΈ расчСтС ΠΊΠΎΠΌΠΏΠΎΠ½Π΅Π½Ρ‚ΠΎΠ² Π²Π΅ΠΊΡ‚ΠΎΡ€Π° ΠΏΠΎ ΡƒΡ€Π°Π²Π½Π΅Π½ΠΈΡŽ 2.17 Π½Π΅ΠΎΠ±Ρ…ΠΎΠ΄ΠΈΠΌΠΎ ΡΠΎΠ±Π»ΡŽΠ΄Π°Ρ‚ΡŒ ΠΎΡΡ‚ΠΎΡ€ΠΎΠΆΠ½ΠΎΡΡ‚ΡŒ ΠΏΡ€ΠΈ Π²Ρ‹Π±ΠΎΡ€Π΅ ΡƒΠ³Π»Π°. Π£Π³ΠΎΠ» направлСния ΞΈAΞΈA Π²Π΅ΠΊΡ‚ΠΎΡ€Π° β€” это ΡƒΠ³ΠΎΠ», ΠΈΠ·ΠΌΠ΅Ρ€Π΅Π½Π½Ρ‹ΠΉ Π½Π° 90Β 289 ΠΏΡ€ΠΎΡ‚ΠΈΠ² часовой стрСлки Π½Π° 90Β 150 ΠΎΡ‚ ΠΏΠΎΠ»ΠΎΠΆΠΈΡ‚Π΅Π»ΡŒΠ½ΠΎΠ³ΠΎ направлСния ΠΏΠΎ оси 90Β 289 x 90Β 150 Π΄ΠΎ Π²Π΅ΠΊΡ‚ΠΎΡ€Π°. Π˜Π·ΠΌΠ΅Ρ€Π΅Π½ΠΈΠ΅ ΠΏΠΎ часовой стрСлкС Π΄Π°Π΅Ρ‚ ΠΎΡ‚Ρ€ΠΈΡ†Π°Ρ‚Π΅Π»ΡŒΠ½Ρ‹ΠΉ ΡƒΠ³ΠΎΠ».

ΠŸΡ€ΠΈΠΌΠ΅Ρ€ 2,5

ΠšΠΎΠΌΠΏΠΎΠ½Π΅Π½Ρ‚Ρ‹ Π²Π΅ΠΊΡ‚ΠΎΡ€ΠΎΠ² смСщСния

Π“Ρ€ΡƒΠΏΠΏΠ° спасСния ΠΏΡ€ΠΎΠΏΠ°Π²ΡˆΠ΅Π³ΠΎ Ρ€Π΅Π±Π΅Π½ΠΊΠ° слСдуСт Π·Π° поисковой собакой ΠΏΠΎ ΠΊΠ»ΠΈΡ‡ΠΊΠ΅ Π‘ΠΎΠ»Π΄Π°Ρ‚. ДСсантник ΠΌΠ½ΠΎΠ³ΠΎ Π±Ρ€ΠΎΠ΄ΠΈΡ‚ ΠΈ Π΄Π΅Π»Π°Π΅Ρ‚ ΠΌΠ½ΠΎΠ³ΠΎ ΠΏΡ€ΠΎΠ±Π½Ρ‹Ρ… ΠΎΠ±Π½ΡŽΡ…ΠΈΠ²Π°Π½ΠΈΠΉ ΠΏΠΎ Ρ€Π°Π·Π½Ρ‹ΠΌ Ρ‚Ρ€ΠΎΠΏΠ°ΠΌ. Π‘ΠΎΠ»Π΄Π°Ρ‚ Π² ΠΊΠΎΠ½Ρ†Π΅ ΠΊΠΎΠ½Ρ†ΠΎΠ² Π½Π°Ρ…ΠΎΠ΄ΠΈΡ‚ Ρ€Π΅Π±Π΅Π½ΠΊΠ°, ΠΈ Ρƒ истории счастливый ΠΊΠΎΠ½Π΅Ρ†, Π½ΠΎ Π΅Π³ΠΎ пСрСмСщСния Π½Π° Ρ€Π°Π·Π½Ρ‹Ρ… Π½ΠΎΠ³Π°Ρ… каТутся Π΄Π΅ΠΉΡΡ‚Π²ΠΈΡ‚Π΅Π»ΡŒΠ½ΠΎ Π·Π°ΠΏΡƒΡ‚Π°Π½Π½Ρ‹ΠΌΠΈ. На ΠΎΠ΄Π½ΠΎΠΉ ΠΈΠ· Π½ΠΎΠ³ ΠΎΠ½ ΠΏΡ€ΠΎΡ…ΠΎΠ΄ΠΈΡ‚ 200,0 ΠΌ Π½Π° юго-восток, Π·Π°Ρ‚Π΅ΠΌ Π±Π΅ΠΆΠΈΡ‚ Π½Π° сСвСр ΠΎΠΊΠΎΠ»ΠΎ 300,0 ΠΌ. На Ρ‚Ρ€Π΅Ρ‚ΡŒΠ΅ΠΌ этапС Π²Π½ΠΈΠΌΠ°Ρ‚Π΅Π»ΡŒΠ½ΠΎ исслСдуСт Π·Π°ΠΏΠ°Ρ…ΠΈ Π½Π° протяТСнии 50,0 ΠΌ Π² Π½Π°ΠΏΡ€Π°Π²Π»Π΅Π½ΠΈΠΈ 30Β°30Β° ΠΊ Π·Π°ΠΏΠ°Π΄Ρƒ ΠΎΡ‚ сСвСра. На Ρ‡Π΅Ρ‚Π²Π΅Ρ€Ρ‚ΠΎΠΌ этапС Trooper ΠΈΠ΄Π΅Ρ‚ прямо Π½Π° юг Π½Π° 80,0 ΠΌ, Π½Π°Ρ…ΠΎΠ΄ΠΈΡ‚ свСТий Π·Π°ΠΏΠ°Ρ… ΠΈ ΠΏΠΎΠ²ΠΎΡ€Π°Ρ‡ΠΈΠ²Π°Π΅Ρ‚ Π½Π° 23Β°23Β° ΠΊ Π·Π°ΠΏΠ°Π΄Ρƒ ΠΎΡ‚ юга Π½Π° 150,0 ΠΌ. НайдитС скалярныС ΠΊΠΎΠΌΠΏΠΎΠ½Π΅Π½Ρ‚Ρ‹ Π²Π΅ΠΊΡ‚ΠΎΡ€ΠΎΠ² смСщСния ДСсантника ΠΈ Π΅Π³ΠΎ Π²Π΅ΠΊΡ‚ΠΎΡ€ΠΎΠ² смСщСния Π² Ρ„ΠΎΡ€ΠΌΠ΅ Π²Π΅ΠΊΡ‚ΠΎΡ€Π½Ρ‹Ρ… ΠΊΠΎΠΌΠΏΠΎΠ½Π΅Π½Ρ‚ для ΠΊΠ°ΠΆΠ΄ΠΎΠΉ Π½ΠΎΠ³ΠΈ. 9оси ΠΈ ΡƒΠΊΠ°Π·Ρ‹Π²Π°Π΅Ρ‚ Π½Π° сСвСр. ДСсантник Π΄Π΅Π»Π°Π΅Ρ‚ ΠΏΡΡ‚ΡŒ Π½ΠΎΠ³, поэтому Π΅ΡΡ‚ΡŒ ΠΏΡΡ‚ΡŒ Π²Π΅ΠΊΡ‚ΠΎΡ€ΠΎΠ² смСщСния. НачнСм с опрСдСлСния ΠΈΡ… Π²Π΅Π»ΠΈΡ‡ΠΈΠ½ ΠΈ ΡƒΠ³Π»ΠΎΠ² направлСния, Π·Π°Ρ‚Π΅ΠΌ Π²ΠΎΡΠΏΠΎΠ»ΡŒΠ·ΡƒΠ΅ΠΌΡΡ ΡƒΡ€Π°Π²Π½Π΅Π½ΠΈΠ΅ΠΌ 2.17, Ρ‡Ρ‚ΠΎΠ±Ρ‹ Π½Π°ΠΉΡ‚ΠΈ скалярныС ΠΊΠΎΠΌΠΏΠΎΠ½Π΅Π½Ρ‚Ρ‹ смСщСний, ΠΈ ΡƒΡ€Π°Π²Π½Π΅Π½ΠΈΠ΅ΠΌ 2.12, Ρ‡Ρ‚ΠΎΠ±Ρ‹ Π½Π°ΠΉΡ‚ΠΈ Π²Π΅ΠΊΡ‚ΠΎΡ€Π° смСщСния.

Раствор

На ΠΏΠ΅Ρ€Π²ΠΎΠΌ участкС Π²Π΅Π»ΠΈΡ‡ΠΈΠ½Π° смСщСния L1=200,0ΠΌL1=200,0ΠΌ ΠΈ Π½Π°ΠΏΡ€Π°Π²Π»Π΅Π½ΠΈΠ΅ юго-восточноС. Π’ качСствС Π΄ΠΈΡ€Π΅ΠΊΡ†ΠΈΠΎΠ½Π½ΠΎΠ³ΠΎ ΡƒΠ³Π»Π° ΞΈ1ΞΈ1 ΠΌΡ‹ ΠΌΠΎΠΆΠ΅ΠΌ ΠΏΡ€ΠΈΠ½ΡΡ‚ΡŒ Π»ΠΈΠ±ΠΎ 45Β°45Β°, ΠΈΠ·ΠΌΠ΅Ρ€Π΅Π½Π½Ρ‹Π΅ ΠΏΠΎ часовой стрСлкС с восточного направлСния, Π»ΠΈΠ±ΠΎ 45Β°+270Β°45Β°+270Β°, ΠΈΠ·ΠΌΠ΅Ρ€Π΅Π½Π½Ρ‹Π΅ ΠΏΡ€ΠΎΡ‚ΠΈΠ² часовой стрСлки с востока. ΠŸΡ€ΠΈ ΠΏΠ΅Ρ€Π²ΠΎΠΌ Π²Ρ‹Π±ΠΎΡ€Π΅ ΞΈ1=βˆ’45°θ1=βˆ’45Β°. ΠŸΡ€ΠΈ Π²Ρ‚ΠΎΡ€ΠΎΠΌ Π²Ρ‹Π±ΠΎΡ€Π΅ ΞΈ1=+315°θ1=+315Β°. ΠœΡ‹ ΠΌΠΎΠΆΠ΅ΠΌ ΠΈΡΠΏΠΎΠ»ΡŒΠ·ΠΎΠ²Π°Ρ‚ΡŒ любой ΠΈΠ· этих Π΄Π²ΡƒΡ… ΡƒΠ³Π»ΠΎΠ². ΠšΠΎΠΌΠΏΠΎΠ½Π΅Π½Ρ‚Ρ‹ 9вдоль оси x ΠΈ оси y соотвСтствСнно. Π”Π΅ΠΊΠ°Ρ€Ρ‚ΠΎΠ²Ρƒ систСму ΠΊΠΎΠΎΡ€Π΄ΠΈΠ½Π°Ρ‚ ΠΎΡ‡Π΅Π½ΡŒ ΡƒΠ΄ΠΎΠ±Π½ΠΎ ΠΈΡΠΏΠΎΠ»ΡŒΠ·ΠΎΠ²Π°Ρ‚ΡŒ ΠΏΡ€ΠΈ описании ΠΏΠ΅Ρ€Π΅ΠΌΠ΅Ρ‰Π΅Π½ΠΈΠΉ ΠΈ скоростСй Ρ‚Π΅Π» ΠΈ Π΄Π΅ΠΉΡΡ‚Π²ΡƒΡŽΡ‰ΠΈΡ… Π½Π° Π½ΠΈΡ… сил. Однако это становится Π³Ρ€ΠΎΠΌΠΎΠ·Π΄ΠΊΠΈΠΌ, ΠΊΠΎΠ³Π΄Π° Π½Π°ΠΌ Π½ΡƒΠΆΠ½ΠΎ ΠΎΠΏΠΈΡΠ°Ρ‚ΡŒ Π²Ρ€Π°Ρ‰Π΅Π½ΠΈΠ΅ ΠΎΠ±ΡŠΠ΅ΠΊΡ‚ΠΎΠ². ΠŸΡ€ΠΈ описании вращСния ΠΌΡ‹ ΠΎΠ±Ρ‹Ρ‡Π½ΠΎ Ρ€Π°Π±ΠΎΡ‚Π°Π΅ΠΌ Π² полярной систСмС ΠΊΠΎΠΎΡ€Π΄ΠΈΠ½Π°Ρ‚.

Π’ полярной систСмС ΠΊΠΎΠΎΡ€Π΄ΠΈΠ½Π°Ρ‚ ΠΏΠΎΠ»ΠΎΠΆΠ΅Π½ΠΈΠ΅ Ρ‚ΠΎΡ‡ΠΊΠΈ P Π½Π° плоскости задаСтся двумя полярными ΠΊΠΎΠΎΡ€Π΄ΠΈΠ½Π°Ρ‚Π°ΠΌΠΈ (рис. 2.20). ΠŸΠ΅Ρ€Π²Π°Ρ полярная ΠΊΠΎΠΎΡ€Π΄ΠΈΠ½Π°Ρ‚Π° — это Ρ€Π°Π΄ΠΈΠ°Π»ΡŒΠ½Π°Ρ ΠΊΠΎΠΎΡ€Π΄ΠΈΠ½Π°Ρ‚Π° 9Π½Π°ΠΏΡ€Π°Π²Π»Π΅Π½ΠΈΠ΅ ΠΏΠΎΠΊΠ°Π·Ρ‹Π²Π°Π΅Ρ‚, ΠΊΠ°ΠΊ ΡƒΠ³ΠΎΠ» φφ измСняСтся Π² Π½Π°ΠΏΡ€Π°Π²Π»Π΅Π½ΠΈΠΈ ΠΏΡ€ΠΎΡ‚ΠΈΠ² часовой стрСлки. Π’Π°ΠΊΠΈΠΌ ΠΎΠ±Ρ€Π°Π·ΠΎΠΌ, Ρ‚ΠΎΡ‡ΠΊΠ° P , ΠΈΠΌΠ΅ΡŽΡ‰Π°Ρ ΠΊΠΎΠΎΡ€Π΄ΠΈΠ½Π°Ρ‚Ρ‹ ( x , y ) Π² ΠΏΡ€ΡΠΌΠΎΡƒΠ³ΠΎΠ»ΡŒΠ½ΠΎΠΉ систСмС, ΠΌΠΎΠΆΠ΅Ρ‚ Π±Ρ‹Ρ‚ΡŒ эквивалСнтно описана Π² полярной систСмС ΠΊΠΎΠΎΡ€Π΄ΠΈΠ½Π°Ρ‚ двумя полярными ΠΊΠΎΠΎΡ€Π΄ΠΈΠ½Π°Ρ‚Π°ΠΌΠΈ (r,Ο†)(r,Ο†). Π£Ρ€Π°Π²Π½Π΅Π½ΠΈΠ΅ 2.17 справСдливо для любого Π²Π΅ΠΊΡ‚ΠΎΡ€Π°, поэтому ΠΌΡ‹ ΠΌΠΎΠΆΠ΅ΠΌ ΠΈΡΠΏΠΎΠ»ΡŒΠ·ΠΎΠ²Π°Ρ‚ΡŒ Π΅Π³ΠΎ для выраТСния ΠΊΠΎΠΎΡ€Π΄ΠΈΠ½Π°Ρ‚ x ΠΈ y Π²Π΅ΠΊΡ‚ΠΎΡ€Π° rβ†’rβ†’. Π’Π°ΠΊΠΈΠΌ ΠΎΠ±Ρ€Π°Π·ΠΎΠΌ, ΠΌΡ‹ ΠΏΠΎΠ»ΡƒΡ‡Π°Π΅ΠΌ связь ΠΌΠ΅ΠΆΠ΄Ρƒ полярными ΠΊΠΎΠΎΡ€Π΄ΠΈΠ½Π°Ρ‚Π°ΠΌΠΈ ΠΈ ΠΏΡ€ΡΠΌΠΎΡƒΠ³ΠΎΠ»ΡŒΠ½Ρ‹ΠΌΠΈ ΠΊΠΎΠΎΡ€Π΄ΠΈΠ½Π°Ρ‚Π°ΠΌΠΈ Ρ‚ΠΎΡ‡ΠΊΠΈ 9опрСдСляСт ΠΏΠΎΠ»ΠΎΠΆΠΈΡ‚Π΅Π»ΡŒΠ½ΠΎΠ΅ Π½Π°ΠΏΡ€Π°Π²Π»Π΅Π½ΠΈΠ΅ вращСния ΡƒΠ³Π»ΠΎΠΌ φφ.

ΠŸΡ€ΠΈΠΌΠ΅Ρ€ 2,6

ΠŸΠΎΠ»ΡΡ€Π½Ρ‹Π΅ ΠΊΠΎΠΎΡ€Π΄ΠΈΠ½Π°Ρ‚Ρ‹

ΠšΠ»Π°Π΄ΠΎΠΈΡΠΊΠ°Ρ‚Π΅Π»ΡŒ Π½Π°Ρ…ΠΎΠ΄ΠΈΡ‚ ΠΎΠ΄Π½Ρƒ ΡΠ΅Ρ€Π΅Π±Ρ€ΡΠ½ΡƒΡŽ ΠΌΠΎΠ½Π΅Ρ‚Ρƒ Π½Π° расстоянии 20,0 ΠΌ ΠΎΡ‚ сухого ΠΊΠΎΠ»ΠΎΠ΄Ρ†Π° Π² Π½Π°ΠΏΡ€Π°Π²Π»Π΅Π½ΠΈΠΈ 20Β°20Β° сСвСрной востока ΠΈ ΠΎΠ΄Π½Ρƒ Π·ΠΎΠ»ΠΎΡ‚ΡƒΡŽ ΠΌΠΎΠ½Π΅Ρ‚Ρƒ Π½Π° расстоянии 10,0 ΠΌ ΠΎΡ‚ ΠΊΠΎΠ»ΠΎΠ΄Ρ†Π° Π² Π½Π°ΠΏΡ€Π°Π²Π»Π΅Π½ΠΈΠΈ 20Β°20Β° сСвСрной ΡˆΠΈΡ€ΠΎΡ‚Ρ‹. Π·Π°ΠΏΠ°Π΄. ΠšΠ°ΠΊΠΎΠ²Ρ‹ полярныС ΠΈ ΠΏΡ€ΡΠΌΠΎΡƒΠ³ΠΎΠ»ΡŒΠ½Ρ‹Π΅ ΠΊΠΎΠΎΡ€Π΄ΠΈΠ½Π°Ρ‚Ρ‹ этих Π½Π°Ρ…ΠΎΠ΄ΠΎΠΊ ΠΎΡ‚Π½ΠΎΡΠΈΡ‚Π΅Π»ΡŒΠ½ΠΎ скваТины?

БтратСгия

ΠšΠΎΠ»ΠΎΠ΄Π΅Ρ† ΠΎΡ‚ΠΌΠ΅Ρ‡Π°Π΅Ρ‚ Π½Π°Ρ‡Π°Π»ΠΎ систСмы ΠΊΠΎΠΎΡ€Π΄ΠΈΠ½Π°Ρ‚, Π° восток — + 9.0289 x — Π½Π°ΠΏΡ€Π°Π²Π»Π΅Π½ΠΈΠ΅. ΠœΡ‹ опрСдСляСм Ρ€Π°Π΄ΠΈΠ°Π»ΡŒΠ½Ρ‹Π΅ расстояния ΠΎΡ‚ мСстополоТСний Π΄ΠΎ Π½Π°Ρ‡Π°Π»Π° ΠΊΠΎΠΎΡ€Π΄ΠΈΠ½Π°Ρ‚, ΠΊΠΎΡ‚ΠΎΡ€Ρ‹Π΅ ΡΠΎΡΡ‚Π°Π²Π»ΡΡŽΡ‚ rS=20,0mrS=20,0ΠΌ (для сСрСбряной ΠΌΠΎΠ½Π΅Ρ‚Ρ‹) ΠΈ rG=10,0mrG=10,0ΠΌ (для Π·ΠΎΠ»ΠΎΡ‚ΠΎΠΉ ΠΌΠΎΠ½Π΅Ρ‚Ρ‹). Π§Ρ‚ΠΎΠ±Ρ‹ Π½Π°ΠΉΡ‚ΠΈ ΡƒΠ³Π»ΠΎΠ²Ρ‹Π΅ ΠΊΠΎΠΎΡ€Π΄ΠΈΠ½Π°Ρ‚Ρ‹, ΠΌΡ‹ ΠΊΠΎΠ½Π²Π΅Ρ€Ρ‚ΠΈΡ€ΡƒΠ΅ΠΌ 20Β°20Β° Π² Ρ€Π°Π΄ΠΈΠ°Π½Ρ‹: 20Β°=Ο€20/180=Ο€/920Β°=Ο€20/180=Ο€/9. ΠœΡ‹ ΠΈΡΠΏΠΎΠ»ΡŒΠ·ΡƒΠ΅ΠΌ ΡƒΡ€Π°Π²Π½Π΅Π½ΠΈΠ΅ 2.18, Ρ‡Ρ‚ΠΎΠ±Ρ‹ Π½Π°ΠΉΡ‚ΠΈ ΠΊΠΎΠΎΡ€Π΄ΠΈΠ½Π°Ρ‚Ρ‹ x ΠΈ y ΠΌΠΎΠ½Π΅Ρ‚.

Раствор

Угловая ΠΊΠΎΠΎΡ€Π΄ΠΈΠ½Π°Ρ‚Π° сСрСбряной ΠΌΠΎΠ½Π΅Ρ‚Ρ‹ Ο†S=Ο€/9Ο†S=Ο€/9, Ρ‚ΠΎΠ³Π΄Π° ΠΊΠ°ΠΊ угловая ΠΊΠΎΠΎΡ€Π΄ΠΈΠ½Π°Ρ‚Π° Π·ΠΎΠ»ΠΎΡ‚ΠΎΠΉ ΠΌΠΎΠ½Π΅Ρ‚Ρ‹ Ο†G=Ο€βˆ’Ο€/9.=8Ο€/9Ο†G=Ο€βˆ’Ο€/9=8Ο€/9. Π‘Π»Π΅Π΄ΠΎΠ²Π°Ρ‚Π΅Π»ΡŒΠ½ΠΎ, полярныС ΠΊΠΎΠΎΡ€Π΄ΠΈΠ½Π°Ρ‚Ρ‹ сСрСбряной ΠΌΠΎΠ½Π΅Ρ‚Ρ‹ Ρ€Π°Π²Π½Ρ‹ (rS,Ο†S)=(20,0m,Ο€/9)(rS,Ο†S)=(20,0m,Ο€/9), Π° полярныС ΠΊΠΎΠΎΡ€Π΄ΠΈΠ½Π°Ρ‚Ρ‹ Π·ΠΎΠ»ΠΎΡ‚ΠΎΠΉ ΠΌΠΎΠ½Π΅Ρ‚Ρ‹ Ρ€Π°Π²Π½Ρ‹ (rG,Ο†G) =(10,0ΠΌ,8Ο€/9)(rG,Ο†G)=(10,0ΠΌ,8Ο€/9). ΠœΡ‹ подставляСм эти ΠΊΠΎΠΎΡ€Π΄ΠΈΠ½Π°Ρ‚Ρ‹ Π² ΡƒΡ€Π°Π²Π½Π΅Π½ΠΈΠ΅ 2.18, Ρ‡Ρ‚ΠΎΠ±Ρ‹ ΠΏΠΎΠ»ΡƒΡ‡ΠΈΡ‚ΡŒ ΠΏΡ€ΡΠΌΠΎΡƒΠ³ΠΎΠ»ΡŒΠ½Ρ‹Π΅ ΠΊΠΎΠΎΡ€Π΄ΠΈΠ½Π°Ρ‚Ρ‹. Для Π·ΠΎΠ»ΠΎΡ‚ΠΎΠΉ ΠΌΠΎΠ½Π΅Ρ‚Ρ‹ ΠΊΠΎΠΎΡ€Π΄ΠΈΠ½Π°Ρ‚Ρ‹

{xG=rGcosΟ†G=(10,0m)cos8Ο€/9=βˆ’9,4myG=rGsinΟ†G=(10,0m)sin8Ο€/9=3,4mβ‡’(xG,yG)=(βˆ’9,4m,3,4m).{xG= rGcosΟ†G=(10,0m)cos8Ο€/9=-9,4myG=rGsinΟ†G=(10,0m)sin8Ο€/9=3,4mβ‡’(xG,yG)=(-9,4m,3,4m).

Для сСрСбряной ΠΌΠΎΠ½Π΅Ρ‚Ρ‹ ΠΊΠΎΠΎΡ€Π΄ΠΈΠ½Π°Ρ‚Ρ‹

{xS=rScosφS=(20,0m)cosπ/9=18,9myS=rSsinφS=(20,0m)sinπ/9=6,8m⇒(xS,yS)=(18,9m,6,8m). {xS=rScosφS= (20,0m)cosπ/9=18,9myS=rSsinφS=(20,0m)sinπ/9=6,8m⇒(xS,yS)=(18,9m,6,8m).

Π’Π΅ΠΊΡ‚ΠΎΡ€Ρ‹ Π² Ρ‚Ρ€Π΅Ρ… измСрСниях

Π§Ρ‚ΠΎΠ±Ρ‹ Π·Π°Π΄Π°Ρ‚ΡŒ ΠΏΠΎΠ»ΠΎΠΆΠ΅Π½ΠΈΠ΅ Ρ‚ΠΎΡ‡ΠΊΠΈ Π² пространствС, Π½Π°ΠΌ Π½ΡƒΠΆΠ½Ρ‹ Ρ‚Ρ€ΠΈ ΠΊΠΎΠΎΡ€Π΄ΠΈΠ½Π°Ρ‚Ρ‹ ( x , y , z ), Π³Π΄Π΅ ΠΊΠΎΠΎΡ€Π΄ΠΈΠ½Π°Ρ‚Ρ‹ x ΠΈ y Π·Π°Π΄Π°ΡŽΡ‚ располоТСниС Π½Π° плоскости, Π° ΠΊΠΎΠΎΡ€Π΄ΠΈΠ½Π°Ρ‚Π° z Π΄Π°Π΅Ρ‚ Π²Π΅Ρ€Ρ‚ΠΈΠΊΠ°Π»ΡŒΠ½ΠΎΠ΅ ΠΏΠΎΠ»ΠΎΠΆΠ΅Π½ΠΈΠ΅ Π½Π°Π΄ ΠΈΠ»ΠΈ ΠΏΠΎΠ΄ ΠΏΠ»ΠΎΡΠΊΠΎΡΡ‚ΡŒΡŽ. Π’Ρ€Π΅Ρ…ΠΌΠ΅Ρ€Π½ΠΎΠ΅ пространство ΠΈΠΌΠ΅Π΅Ρ‚ Ρ‚Ρ€ΠΈ ΠΎΡ€Ρ‚ΠΎΠ³ΠΎΠ½Π°Π»ΡŒΠ½Ρ‹Ρ… направлСния, поэтому Π½Π°ΠΌ Π½ΡƒΠΆΠ½ΠΎ Π½Π΅ Π΄Π²Π°, Π° 9.

2,19

Если извСстны ΠΊΠΎΠΎΡ€Π΄ΠΈΠ½Π°Ρ‚Ρ‹ Π΅Π³ΠΎ Π½Π°Ρ‡Π°Π»Π° b(xb,yb,zb)b(xb,yb,zb) ΠΈ ΠΊΠΎΠ½Ρ†Π° e(xe,ye,ze)e(xe,ye,ze), Π΅Π³ΠΎ скалярныС ΠΊΠΎΠΌΠΏΠΎΠ½Π΅Π½Ρ‚Ρ‹ Ρ€Π°Π²Π½Ρ‹ получаСтся ΠΏΡƒΡ‚Π΅ΠΌ взятия ΠΈΡ… разностСй: AxAx ΠΈ AyAy Π·Π°Π΄Π°ΡŽΡ‚ΡΡ ΡƒΡ€Π°Π²Π½Π΅Π½ΠΈΠ΅ΠΌ 2.13, Π° z -ΠΊΠΎΠΌΠΏΠΎΠ½Π΅Π½Ρ‚Π° задаСтся

Az=ze-zb.Az=ze-zb.

2,20

Π’Π΅Π»ΠΈΡ‡ΠΈΠ½Π° A получаСтся ΠΏΡƒΡ‚Π΅ΠΌ обобщСния уравнСния 2. 15 Π½Π° Ρ‚Ρ€ΠΈ измСрСния:

A=Ax2+Ay2+Az2.A=Ax2+Ay2+Az2.

2,21

Π­Ρ‚ΠΎ Π²Ρ‹Ρ€Π°ΠΆΠ΅Π½ΠΈΠ΅ для Π²Π΅Π»ΠΈΡ‡ΠΈΠ½Ρ‹ Π²Π΅ΠΊΡ‚ΠΎΡ€Π° получаСтся ΠΈΠ· Π΄Π²ΠΎΠΉΠ½ΠΎΠ³ΠΎ примСнСния Ρ‚Π΅ΠΎΡ€Π΅ΠΌΡ‹ ΠŸΠΈΡ„Π°Π³ΠΎΡ€Π°. Как Π²ΠΈΠ΄Π½ΠΎ Π½Π° рис. 2.22, диагональ Π² плоскости xy ΠΈΠΌΠ΅Π΅Ρ‚ Π΄Π»ΠΈΠ½Ρƒ Ax2+Ay2Ax2+Ay2, Π° Π΅Π΅ ΠΊΠ²Π°Π΄Ρ€Π°Ρ‚ Π² суммС с ΠΊΠ²Π°Π΄Ρ€Π°Ρ‚ΠΎΠΌ Az2Az2 Π΄Π°Π΅Ρ‚ A2A2. ΠžΠ±Ρ€Π°Ρ‚ΠΈΡ‚Π΅ Π²Π½ΠΈΠΌΠ°Π½ΠΈΠ΅, Ρ‡Ρ‚ΠΎ ΠΊΠΎΠ³Π΄Π° z -ΠΊΠΎΠΌΠΏΠΎΠ½Π΅Π½Ρ‚Π° Ρ€Π°Π²Π½Π° Π½ΡƒΠ»ΡŽ, Π²Π΅ΠΊΡ‚ΠΎΡ€ ΠΏΠΎΠ»Π½ΠΎΡΡ‚ΡŒΡŽ Π»Π΅ΠΆΠΈΡ‚ Π² xy -плоскости ΠΈ Π΅Π³ΠΎ описаниС сводится ΠΊ Π΄Π²ΡƒΠΌ измСрСниям.

Рисунок 2,22 Π’Π΅ΠΊΡ‚ΠΎΡ€ Π² Ρ‚Ρ€Π΅Ρ…ΠΌΠ΅Ρ€Π½ΠΎΠΌ пространствС прСдставляСт собой Π²Π΅ΠΊΡ‚ΠΎΡ€Π½ΡƒΡŽ сумму Ρ‚Ρ€Π΅Ρ… Π΅Π³ΠΎ Π²Π΅ΠΊΡ‚ΠΎΡ€Π½Ρ‹Ρ… ΠΊΠΎΠΌΠΏΠΎΠ½Π΅Π½Ρ‚.

ΠŸΡ€ΠΈΠΌΠ΅Ρ€ 2,7

Π’Π·Π»Π΅Ρ‚ Π΄Ρ€ΠΎΠ½Π°

ΠŸΡ€ΠΈ Π²Π·Π»Π΅Ρ‚Π΅ IAI Heron (рис. 2.23) Π΅Π³ΠΎ ΠΏΠΎΠ»ΠΎΠΆΠ΅Π½ΠΈΠ΅ ΠΎΡ‚Π½ΠΎΡΠΈΡ‚Π΅Π»ΡŒΠ½ΠΎ диспСтчСрской Π²Ρ‹ΡˆΠΊΠΈ составляСт 100 ΠΌ Π½Π°Π΄ Π·Π΅ΠΌΠ»Π΅ΠΉ, 300 ΠΌ восточнСС ΠΈ 200 ΠΌ сСвСрнСС. Π§Π΅Ρ€Π΅Π· ΠΎΠ΄Π½Ρƒ ΠΌΠΈΠ½ΡƒΡ‚Ρƒ Π΅Π³ΠΎ ΠΏΠΎΠ»ΠΎΠΆΠ΅Π½ΠΈΠ΅ 250 ΠΌ Π½Π°Π΄ Π·Π΅ΠΌΠ»Π΅ΠΉ, 1200 ΠΌ Π½Π° восток ΠΈ 2100 ΠΌ Π½Π° сСвСр. Каков Π²Π΅ΠΊΡ‚ΠΎΡ€ смСщСния Π΄Ρ€ΠΎΠ½Π° ΠΎΡ‚Π½ΠΎΡΠΈΡ‚Π΅Π»ΡŒΠ½ΠΎ диспСтчСрской? Какова Π²Π΅Π»ΠΈΡ‡ΠΈΠ½Π° Π΅Π³ΠΎ Π²Π΅ΠΊΡ‚ΠΎΡ€Π° смСщСния?

Рисунок 2,23 Π”Ρ€ΠΎΠ½ IAI Heron Π² ΠΏΠΎΠ»Π΅Ρ‚Π΅. (ΠΊΡ€Π΅Π΄ΠΈΡ‚: сСрТант РСйнальдо Π Π°ΠΌΠΎΠ½, Π’Π’Π‘ БША) 9, ΠΊΠΎΡ‚ΠΎΡ€Ρ‹ΠΉ ΡƒΠΊΠ°Π·Ρ‹Π²Π°Π΅Ρ‚ Π²Π²Π΅Ρ€Ρ… ΠΎΡ‚ Π·Π΅ΠΌΠ»ΠΈ. ΠŸΠ΅Ρ€Π²Π°Ρ позиция Π΄Ρ€ΠΎΠ½Π° β€” это Π½Π°Ρ‡Π°Π»ΠΎ (ΠΈΠ»ΠΈ, Ρ‡Ρ‚ΠΎ Ρ‚ΠΎ ΠΆΠ΅ самоС, Π½Π°Ρ‡Π°Π»ΠΎ) Π²Π΅ΠΊΡ‚ΠΎΡ€Π° смСщСния, Π° Π΅Π³ΠΎ вторая позиция β€” ΠΊΠΎΠ½Π΅Ρ† Π²Π΅ΠΊΡ‚ΠΎΡ€Π° смСщСния.

РСшСниС

ΠœΡ‹ ΠΈΠ΄Π΅Π½Ρ‚ΠΈΡ„ΠΈΡ†ΠΈΡ€ΡƒΠ΅ΠΌ b (300,0 ΠΌ, 200,0 ΠΌ, 100,0 ΠΌ) ΠΈ e (1200 ΠΌ, 2100 ΠΌ, 250 ΠΌ) ΠΈ ΠΈΡΠΏΠΎΠ»ΡŒΠ·ΡƒΠ΅ΠΌ уравнСния 2.13 ΠΈ 2.20, Ρ‡Ρ‚ΠΎΠ±Ρ‹ Π½Π°ΠΉΡ‚ΠΈ скалярныС ΠΊΠΎΠΌΠΏΠΎΠ½Π΅Π½Ρ‚Ρ‹ Π΄Ρ€ΠΎΠ½Π°. Π²Π΅ΠΊΡ‚ΠΎΡ€ смСщСния:

{Dx=xe-xb=1200,0 ΠΌ-300,0 ΠΌ=900,0 ΠΌ,Dy=ye-yb=2100,0 ΠΌ-200,0 ΠΌ=19) ΠΌ/с, ΠΊΠ°ΠΊΠΎΠ²Π° Π²Π΅Π»ΠΈΡ‡ΠΈΠ½Π° Π²Π΅ΠΊΡ‚ΠΎΡ€Π° скорости Π΄Ρ€ΠΎΠ½Π°?

Π’Π΅ΠΊΡ‚ΠΎΡ€ полоТСния — ΠΎΠΏΡ€Π΅Π΄Π΅Π»Π΅Π½ΠΈΠ΅, Ρ„ΠΎΡ€ΠΌΡƒΠ»Π°, ΠΏΡ€ΠΈΠΌΠ΅Ρ€Ρ‹, часто Π·Π°Π΄Π°Π²Π°Π΅ΠΌΡ‹Π΅ вопросы

Π’Π΅ΠΊΡ‚ΠΎΡ€ полоТСния ΠΈΡΠΏΠΎΠ»ΡŒΠ·ΡƒΠ΅Ρ‚ΡΡ, Ρ‡Ρ‚ΠΎΠ±Ρ‹ ΠΏΠΎΠΌΠΎΡ‡ΡŒ Π½Π°ΠΌ Π½Π°ΠΉΡ‚ΠΈ ΠΏΠΎΠ»ΠΎΠΆΠ΅Π½ΠΈΠ΅ ΠΎΠ΄Π½ΠΎΠ³ΠΎ ΠΎΠ±ΡŠΠ΅ΠΊΡ‚Π° ΠΎΡ‚Π½ΠΎΡΠΈΡ‚Π΅Π»ΡŒΠ½ΠΎ Π΄Ρ€ΡƒΠ³ΠΎΠ³ΠΎ ΠΎΠ±ΡŠΠ΅ΠΊΡ‚Π°. Π’Π΅ΠΊΡ‚ΠΎΡ€Ρ‹ полоТСния ΠΎΠ±Ρ‹Ρ‡Π½ΠΎ Π½Π°Ρ‡ΠΈΠ½Π°ΡŽΡ‚ΡΡ Π² Π½Π°Ρ‡Π°Π»Π΅ ΠΊΠΎΠΎΡ€Π΄ΠΈΠ½Π°Ρ‚, Π° Π·Π°Ρ‚Π΅ΠΌ Π·Π°ΠΊΠ°Π½Ρ‡ΠΈΠ²Π°ΡŽΡ‚ΡΡ Π² любой Π΄Ρ€ΡƒΠ³ΠΎΠΉ ΠΏΡ€ΠΎΠΈΠ·Π²ΠΎΠ»ΡŒΠ½ΠΎΠΉ Ρ‚ΠΎΡ‡ΠΊΠ΅. Π’Π°ΠΊΠΈΠΌ ΠΎΠ±Ρ€Π°Π·ΠΎΠΌ, эти Π²Π΅ΠΊΡ‚ΠΎΡ€Ρ‹ ΠΈΡΠΏΠΎΠ»ΡŒΠ·ΡƒΡŽΡ‚ΡΡ для опрСдСлСния полоТСния ΠΊΠΎΠ½ΠΊΡ€Π΅Ρ‚Π½ΠΎΠΉ Ρ‚ΠΎΡ‡ΠΊΠΈ ΠΎΡ‚Π½ΠΎΡΠΈΡ‚Π΅Π»ΡŒΠ½ΠΎ Π΅Π΅ Π½Π°Ρ‡Π°Π»Π°.

Π’ этой ΡΡ‚Π°Ρ‚ΡŒΠ΅ Π΄Π°Π²Π°ΠΉΡ‚Π΅ ΡƒΠ·Π½Π°Π΅ΠΌ ΠΎ Π²Π΅ΠΊΡ‚ΠΎΡ€Π°Ρ… полоТСния, ΠΈΡ… ΠΎΠΏΡ€Π΅Π΄Π΅Π»Π΅Π½ΠΈΠΈ, Ρ„ΠΎΡ€ΠΌΡƒΠ»Π°Ρ… с Ρ€Π΅ΡˆΠ΅Π½Π½Ρ‹ΠΌΠΈ ΠΏΡ€ΠΈΠΌΠ΅Ρ€Π°ΠΌΠΈ.

1. Π§Ρ‚ΠΎ Ρ‚Π°ΠΊΠΎΠ΅ Π²Π΅ΠΊΡ‚ΠΎΡ€ полоТСния?
2. Как Π½Π°ΠΉΡ‚ΠΈ Π²Π΅ΠΊΡ‚ΠΎΡ€ полоТСния?
3. Π€ΠΎΡ€ΠΌΡƒΠ»Π° Π²Π΅ΠΊΡ‚ΠΎΡ€Π° полоТСния
4. Часто Π·Π°Π΄Π°Π²Π°Π΅ΠΌΡ‹Π΅ вопросы ΠΎ Π²Π΅ΠΊΡ‚ΠΎΡ€Π΅ полоТСния

Π§Ρ‚ΠΎ Ρ‚Π°ΠΊΠΎΠ΅ Π²Π΅ΠΊΡ‚ΠΎΡ€ полоТСния?

Π’Π΅ΠΊΡ‚ΠΎΡ€ полоТСния прСдставляСт собой ΠΏΡ€ΡΠΌΡƒΡŽ линию, ΠΎΠ΄ΠΈΠ½ ΠΊΠΎΠ½Π΅Ρ† ΠΊΠΎΡ‚ΠΎΡ€ΠΎΠΉ ΠΏΡ€ΠΈΠΊΡ€Π΅ΠΏΠ»Π΅Π½ ΠΊ Ρ‚Π΅Π»Ρƒ, Π° Π΄Ρ€ΡƒΠ³ΠΎΠΉ ΠΊΠΎΠ½Π΅Ρ† ΠΏΡ€ΠΈΠΊΡ€Π΅ΠΏΠ»Π΅Π½ ΠΊ двиТущСйся Ρ‚ΠΎΡ‡ΠΊΠ΅, ΠΈ ΠΈΡΠΏΠΎΠ»ΡŒΠ·ΡƒΠ΅Ρ‚ΡΡ для описания полоТСния Ρ‚ΠΎΡ‡ΠΊΠΈ ΠΎΡ‚Π½ΠΎΡΠΈΡ‚Π΅Π»ΡŒΠ½ΠΎ Ρ‚Π΅Π»Π°. По ΠΌΠ΅Ρ€Π΅ пСрСмСщСния Ρ‚ΠΎΡ‡ΠΊΠΈ Π²Π΅ΠΊΡ‚ΠΎΡ€ полоТСния Π±ΡƒΠ΄Π΅Ρ‚ ΠΈΠ·ΠΌΠ΅Π½ΡΡ‚ΡŒΡΡ ΠΏΠΎ Π΄Π»ΠΈΠ½Π΅ ΠΈΠ»ΠΈ ΠΏΠΎ Π½Π°ΠΏΡ€Π°Π²Π»Π΅Π½ΠΈΡŽ, ΠΈΠ»ΠΈ ΠΏΠΎ Π΄Π»ΠΈΠ½Π΅ ΠΈ ΠΏΠΎ Π½Π°ΠΏΡ€Π°Π²Π»Π΅Π½ΠΈΡŽ ΠΎΠ΄Π½ΠΎΠ²Ρ€Π΅ΠΌΠ΅Π½Π½ΠΎ.

ΠžΠΏΡ€Π΅Π΄Π΅Π»Π΅Π½ΠΈΠ΅ Π²Π΅ΠΊΡ‚ΠΎΡ€Π° полоТСния

Π’Π΅ΠΊΡ‚ΠΎΡ€ полоТСния опрСдСляСтся ΠΊΠ°ΠΊ Π²Π΅ΠΊΡ‚ΠΎΡ€, ΠΊΠΎΡ‚ΠΎΡ€Ρ‹ΠΉ ΡƒΠΊΠ°Π·Ρ‹Π²Π°Π΅Ρ‚ Π»ΠΈΠ±ΠΎ ΠΏΠΎΠ»ΠΎΠΆΠ΅Π½ΠΈΠ΅, Π»ΠΈΠ±ΠΎ ΠΏΠΎΠ»ΠΎΠΆΠ΅Π½ΠΈΠ΅ любой Π·Π°Π΄Π°Π½Π½ΠΎΠΉ Ρ‚ΠΎΡ‡ΠΊΠΈ ΠΎΡ‚Π½ΠΎΡΠΈΡ‚Π΅Π»ΡŒΠ½ΠΎ любой ΠΏΡ€ΠΎΠΈΠ·Π²ΠΎΠ»ΡŒΠ½ΠΎΠΉ ΠΊΠΎΠ½Ρ‚Ρ€ΠΎΠ»ΡŒΠ½ΠΎΠΉ Ρ‚ΠΎΡ‡ΠΊΠΈ, Ρ‚Π°ΠΊΠΎΠΉ ΠΊΠ°ΠΊ Π½Π°Ρ‡Π°Π»ΠΎ ΠΊΠΎΠΎΡ€Π΄ΠΈΠ½Π°Ρ‚. НаправлСниС Π²Π΅ΠΊΡ‚ΠΎΡ€Π° полоТСния всСгда ΡƒΠΊΠ°Π·Ρ‹Π²Π°Π΅Ρ‚ ΠΎΡ‚ Π½Π°Ρ‡Π°Π»Π° этого Π²Π΅ΠΊΡ‚ΠΎΡ€Π° ΠΊ Π·Π°Π΄Π°Π½Π½ΠΎΠΉ Ρ‚ΠΎΡ‡ΠΊΠ΅.

  • Π’ Π΄Π΅ΠΊΠ°Ρ€Ρ‚ΠΎΠ²ΠΎΠΉ систСмС ΠΊΠΎΠΎΡ€Π΄ΠΈΠ½Π°Ρ‚, Ссли O являСтся Π½Π°Ρ‡Π°Π»ΠΎΠΌ ΠΊΠΎΠΎΡ€Π΄ΠΈΠ½Π°Ρ‚, Π° P(x 1 , y 1 ) являСтся Π΄Ρ€ΡƒΠ³ΠΎΠΉ Ρ‚ΠΎΡ‡ΠΊΠΎΠΉ, Ρ‚ΠΎ Π²Π΅ΠΊΡ‚ΠΎΡ€ полоТСния, Π½Π°ΠΏΡ€Π°Π²Π»Π΅Π½Π½Ρ‹ΠΉ ΠΈΠ· Ρ‚ΠΎΡ‡ΠΊΠΈ O Π² Ρ‚ΠΎΡ‡ΠΊΡƒ P, ΠΌΠΎΠΆΠ΅Ρ‚ Π±Ρ‹Ρ‚ΡŒ прСдставлСн ΠΊΠ°ΠΊ ОП.
  • Π’ Ρ‚Ρ€Π΅Ρ…ΠΌΠ΅Ρ€Π½ΠΎΠΌ пространствС, Ссли Π½Π°Ρ‡Π°Π»ΠΎ ΠΊΠΎΠΎΡ€Π΄ΠΈΠ½Π°Ρ‚ O = (0,0,0) ΠΈ P = (x 1 , y 1 , z 1 ), Ρ‚ΠΎ Π²Π΅ΠΊΡ‚ΠΎΡ€ полоТСния v Ρ‚ΠΎΡ‡ΠΊΠΈ P ΠΌΠΎΠΆΠ΅Ρ‚ Π±Ρ‹Ρ‚ΡŒ прСдставлСно ΠΊΠ°ΠΊ: v = x 1 i + y 1 j + z 1 k

Рассмотрим Π΄Π²Π° Π²Π΅ΠΊΡ‚ΠΎΡ€Π° P ΠΈ Q с Π²Π΅ΠΊΡ‚ΠΎΡ€Π°ΠΌΠΈ полоТСния p = (2,4) ΠΈ q = (3, 5) соотвСтствСнно. ΠšΠΎΠΎΡ€Π΄ΠΈΠ½Π°Ρ‚Ρ‹ Π²Π΅ΠΊΡ‚ΠΎΡ€ΠΎΠ² P ΠΈ Q ΠΌΠΎΠΆΠ½ΠΎ Π·Π°ΠΏΠΈΡΠ°Ρ‚ΡŒ Π² Π²ΠΈΠ΄Π΅:Β P = (2,4), Q = (3, 5). Π”Π°Π²Π°ΠΉΡ‚Π΅ рассмотрим Π½Π°Ρ‡Π°Π»ΠΎ ΠΊΠΎΠΎΡ€Π΄ΠΈΠ½Π°Ρ‚ O, ΠΊΠ°ΠΊ ΠΏΠΎΠΊΠ°Π·Π°Π½ΠΎ Π½Π° ΠΈΠ·ΠΎΠ±Ρ€Π°ΠΆΠ΅Π½ΠΈΠΈ Π½ΠΈΠΆΠ΅. ΠœΡ‹ Π±ΡƒΠ΄Π΅ΠΌ Ρ€Π°ΡΡΠΌΠ°Ρ‚Ρ€ΠΈΠ²Π°Ρ‚ΡŒ частицу, которая двиТСтся ΠΈΠ· Ρ‚ΠΎΡ‡ΠΊΠΈ P Π² Ρ‚ΠΎΡ‡ΠΊΡƒ Q. Π’Π΅ΠΊΡ‚ΠΎΡ€ полоТСния частицы ΠΌΠΎΠΆΠ½ΠΎ ΠΎΠΏΡ€Π΅Π΄Π΅Π»ΠΈΡ‚ΡŒ ΠΊΠ°ΠΊ Π²Π΅ΠΊΡ‚ΠΎΡ€, ΠΊΠΎΡ‚ΠΎΡ€Ρ‹ΠΉ начинаСтся ΠΎΡ‚ Π½Π°Ρ‡Π°Π»Π° ΠΊΠΎΠΎΡ€Π΄ΠΈΠ½Π°Ρ‚ Π΄ΠΎ Ρ‚ΠΎΡ‡ΠΊΠΈ, Π² ΠΊΠΎΡ‚ΠΎΡ€ΠΎΠΉ находится частица.

На ΠΏΡ€ΠΈΠ²Π΅Π΄Π΅Π½Π½ΠΎΠΉ Π²Ρ‹ΡˆΠ΅ Π΄ΠΈΠ°Π³Ρ€Π°ΠΌΠΌΠ΅ Π²Π΅ΠΊΡ‚ΠΎΡ€ полоТСния частицы, ΠΊΠΎΠ³Π΄Π° ΠΎΠ½Π° находится Π² Ρ‚ΠΎΡ‡ΠΊΠ΅ P, являСтся Π²Π΅ΠΊΡ‚ΠΎΡ€ΠΎΠΌ OP, Π° ΠΊΠΎΠ³Π΄Π° ΠΎΠ½Π° находится Π² Ρ‚ΠΎΡ‡ΠΊΠ΅ Q, это Π²Π΅ΠΊΡ‚ΠΎΡ€ OQ.

Как Π½Π°ΠΉΡ‚ΠΈ Π²Π΅ΠΊΡ‚ΠΎΡ€ полоТСния?

ΠžΡ‡Π΅Π½ΡŒ Π²Π°ΠΆΠ½ΠΎ сначала ΠΎΠΏΡ€Π΅Π΄Π΅Π»ΠΈΡ‚ΡŒ ΠΊΠΎΠΎΡ€Π΄ΠΈΠ½Π°Ρ‚Ρ‹ Ρ‚ΠΎΡ‡ΠΊΠΈ, ΠΏΡ€Π΅ΠΆΠ΄Π΅ Ρ‡Π΅ΠΌ Π½Π°ΠΉΡ‚ΠΈ Π²Π΅ΠΊΡ‚ΠΎΡ€ полоТСния этой Ρ‚ΠΎΡ‡ΠΊΠΈ. Рассмотрим Π΄Π²Π΅ Ρ‚ΠΎΡ‡ΠΊΠΈ, A ΠΈ B, Π³Π΄Π΅ A = (x 1 , y 1 ) ΠΈ B = (x 2 , y 2 ).

  • Π”Π°Π»Π΅Π΅ Π½Π°ΠΉΠ΄Π΅ΠΌ Π²Π΅ΠΊΡ‚ΠΎΡ€ полоТСния ΠΈΠ· Ρ‚ΠΎΡ‡ΠΊΠΈ А Π² Ρ‚ΠΎΡ‡ΠΊΡƒ Π’, Π²Π΅ΠΊΡ‚ΠΎΡ€ АВ.
  • Π§Ρ‚ΠΎΠ±Ρ‹ ΠΎΠΏΡ€Π΅Π΄Π΅Π»ΠΈΡ‚ΡŒ этот Π²Π΅ΠΊΡ‚ΠΎΡ€ полоТСния, Π½Π°ΠΌ Π½ΡƒΠΆΠ½ΠΎ Π²Ρ‹Ρ‡Π΅ΡΡ‚ΡŒ ΡΠΎΠΎΡ‚Π²Π΅Ρ‚ΡΡ‚Π²ΡƒΡŽΡ‰ΠΈΠ΅ ΠΊΠΎΠΌΠΏΠΎΠ½Π΅Π½Ρ‚Ρ‹ A ΠΈΠ· B: AB = (x 2 Β — x 1 , y 2 Β — y 1 ) = (x 2 Β — x 1 ) я + (Ρƒ 2 Β — Ρƒ 1 ) Ρƒ

Π€ΠΎΡ€ΠΌΡƒΠ»Π° Π²Π΅ΠΊΡ‚ΠΎΡ€Π° полоТСния

Если ΠΌΡ‹ Π·Π½Π°Π΅ΠΌ ΠΏΠΎΠ»ΠΎΠΆΠ΅Π½ΠΈΠ΅ любой Ρ‚ΠΎΡ‡ΠΊΠΈ Π² плоскости xy, Ρ‚ΠΎ ΠΌΡ‹ ΠΌΠΎΠΆΠ΅ΠΌ ΠΈΡΠΏΠΎΠ»ΡŒΠ·ΠΎΠ²Π°Ρ‚ΡŒ Ρ„ΠΎΡ€ΠΌΡƒΠ»Ρƒ для опрСдСлСния Π²Π΅ΠΊΡ‚ΠΎΡ€Π° полоТСния ΠΌΠ΅ΠΆΠ΄Ρƒ этими двумя Ρ‚ΠΎΡ‡ΠΊΠ°ΠΌΠΈ. НапримСр, рассмотрим Ρ‚ΠΎΡ‡ΠΊΡƒ A, которая ΠΈΠΌΠ΅Π΅Ρ‚ ΠΊΠΎΠΎΡ€Π΄ΠΈΠ½Π°Ρ‚Ρ‹ (x k , y k ) Π² плоскости xy ΠΈ Π΅Ρ‰Π΅ ΠΎΠ΄Π½Π° Ρ‚ΠΎΡ‡ΠΊΠ° B, ΠΈΠΌΠ΅ΡŽΡ‰Π°Ρ ΠΊΠΎΠΎΡ€Π΄ΠΈΠ½Π°Ρ‚Ρ‹ (x k+1 , y k+1 ).

  • Π€ΠΎΡ€ΠΌΡƒΠ»Π° для опрСдСлСния Π²Π΅ΠΊΡ‚ΠΎΡ€Π° полоТСния ΠΎΡ‚ A Π΄ΠΎ B: AB = (x k+1 Β — x k , y k+1 Β — y k ).
  • Π’Π΅ΠΊΡ‚ΠΎΡ€ полоТСния AB относится ΠΊ Π²Π΅ΠΊΡ‚ΠΎΡ€Ρƒ, ΠΊΠΎΡ‚ΠΎΡ€Ρ‹ΠΉ начинаСтся Π² Ρ‚ΠΎΡ‡ΠΊΠ΅ A ΠΈ заканчиваСтся Π² Ρ‚ΠΎΡ‡ΠΊΠ΅ B.
  • Аналогично, Ссли ΠΌΡ‹ Ρ…ΠΎΡ‚ΠΈΠΌ Π½Π°ΠΉΡ‚ΠΈ Π²Π΅ΠΊΡ‚ΠΎΡ€ полоТСния ΠΈΠ· Ρ‚ΠΎΡ‡ΠΊΠΈ B Π² Ρ‚ΠΎΡ‡ΠΊΡƒ A, Ρ‚ΠΎ ΠΌΡ‹ ΠΌΠΎΠΆΠ΅ΠΌ ΠΈΡΠΏΠΎΠ»ΡŒΠ·ΠΎΠ²Π°Ρ‚ΡŒ: BA = (x ΠΊ Β — Ρ… ΠΊ+1 , Ρƒ ΠΊ Β — Ρƒ ΠΊ+1 )

ΠŸΠΎΡ…ΠΎΠΆΠΈΠ΅ ΡΡ‚Π°Ρ‚ΡŒΠΈ ΠΎ Π²Π΅ΠΊΡ‚ΠΎΡ€Π΅ полоТСния

ΠžΠ·Π½Π°ΠΊΠΎΠΌΡŒΡ‚Π΅ΡΡŒ со ΡΠ»Π΅Π΄ΡƒΡŽΡ‰ΠΈΠΌΠΈ страницами, связанными с Π²Π΅ΠΊΡ‚ΠΎΡ€ΠΎΠΌ полоТСния

  • Π”ΠΎΠ±Π°Π²Π»Π΅Π½ΠΈΠ΅ ΠΊΠ°Π»ΡŒΠΊΡƒΠ»ΡΡ‚ΠΎΡ€Π° Π²Π΅ΠΊΡ‚ΠΎΡ€ΠΎΠ²
  • ΠšΠ°Π»ΡŒΠΊΡƒΠ»ΡΡ‚ΠΎΡ€ ΡƒΠ³Π»Π° ΠΌΠ΅ΠΆΠ΄Ρƒ двумя Π²Π΅ΠΊΡ‚ΠΎΡ€Π°ΠΌΠΈ
  • ΠžΠ±Ρ€Π°Π±ΠΎΡ‚ΠΊΠ° Π²Π΅ΠΊΡ‚ΠΎΡ€ΠΎΠ², ΡƒΠΊΠ°Π·Π°Π½Π½Ρ‹Ρ… Π² Ρ„ΠΎΡ€ΠΌΠ΅ i-j
  • НСравСнство Ρ‚Ρ€Π΅ΡƒΠ³ΠΎΠ»ΡŒΠ½ΠΈΠΊΠΎΠ² Π² Π²Π΅ΠΊΡ‚ΠΎΡ€Π°Ρ…
  • Π’Ρ‹Ρ‡ΠΈΡ‚Π°Π½ΠΈΠ΅ Π΄Π²ΡƒΡ… Π²Π΅ΠΊΡ‚ΠΎΡ€ΠΎΠ²

Π’Π°ΠΆΠ½Ρ‹Π΅ примСчания ΠΏΠΎ Π²Π΅ΠΊΡ‚ΠΎΡ€Ρƒ полоТСния

НиТС ΠΏΡ€ΠΈΠ²Π΅Π΄Π΅Π½ список Π½Π΅ΡΠΊΠΎΠ»ΡŒΠΊΠΈΡ… Ρ‚ΠΎΡ‡Π΅ΠΊ, ΠΊΠΎΡ‚ΠΎΡ€Ρ‹Π΅ слСдуСт ΠΏΠΎΠΌΠ½ΠΈΡ‚ΡŒ ΠΏΡ€ΠΈ ΠΈΠ·ΡƒΡ‡Π΅Π½ΠΈΠΈ Π²Π΅ΠΊΡ‚ΠΎΡ€Π° полоТСния ΠΎΡ‚Π½ΠΎΡΠΈΡ‚Π΅Π»ΡŒΠ½ΠΎ любой ΠΏΡ€ΠΎΠΈΠ·Π²ΠΎΠ»ΡŒΠ½ΠΎΠΉ Ρ‚ΠΎΡ‡ΠΊΠΈ отсчСта, Ρ‚Π°ΠΊΠΎΠΉ ΠΊΠ°ΠΊ Π½Π°Ρ‡Π°Π»ΠΎ ΠΊΠΎΠΎΡ€Π΄ΠΈΠ½Π°Ρ‚.

  • НаправлСниС Π²Π΅ΠΊΡ‚ΠΎΡ€Π° полоТСния всСгда ΡƒΠΊΠ°Π·Ρ‹Π²Π°Π΅Ρ‚ ΠΎΡ‚ Π½Π°Ρ‡Π°Π»Π° этого Π²Π΅ΠΊΡ‚ΠΎΡ€Π° ΠΊ Π·Π°Π΄Π°Π½Π½ΠΎΠΉ Ρ‚ΠΎΡ‡ΠΊΠ΅.
  • Β 

    ΠŸΡ€ΠΈΠΌΠ΅Ρ€Ρ‹ Π²Π΅ΠΊΡ‚ΠΎΡ€Π° полоТСния

    1. ΠŸΡ€ΠΈΠΌΠ΅Ρ€ 1: Для Π΄Π²ΡƒΡ… Ρ‚ΠΎΡ‡Π΅ΠΊ P = (-4, 6) ΠΈ Q = (5, 11) ΠΎΠΏΡ€Π΅Π΄Π΅Π»ΠΈΡ‚Π΅ Π²Π΅ΠΊΡ‚ΠΎΡ€ полоТСния PQ.
      РСшСниС: Если Π² систСмС ΠΊΠΎΠΎΡ€Π΄ΠΈΠ½Π°Ρ‚ xy Π·Π°Π΄Π°Π½Ρ‹ Π΄Π²Π΅ Ρ‚ΠΎΡ‡ΠΊΠΈ, Ρ‚ΠΎ ΠΌΡ‹ ΠΌΠΎΠΆΠ΅ΠΌ ΠΈΡΠΏΠΎΠ»ΡŒΠ·ΠΎΠ²Π°Ρ‚ΡŒ ΡΠ»Π΅Π΄ΡƒΡŽΡ‰ΡƒΡŽ Ρ„ΠΎΡ€ΠΌΡƒΠ»Ρƒ для нахоТдСния Π²Π΅ΠΊΡ‚ΠΎΡ€Π° полоТСния PQ: — y 1 )

      Π“Π΄Π΅ (x 1 , y 1 ) прСдставляСт ΠΊΠΎΠΎΡ€Π΄ΠΈΠ½Π°Ρ‚Ρ‹ Ρ‚ΠΎΡ‡ΠΊΠΈ P ΠΈ (x 2 , y 2 ) прСдставляСт ΠΊΠΎΠΎΡ€Π΄ΠΈΠ½Π°Ρ‚Ρ‹ Ρ‚ΠΎΡ‡ΠΊΠΈ Q. Π’Π°ΠΊΠΈΠΌ ΠΎΠ±Ρ€Π°Π·ΠΎΠΌ, просто подставив значСния Ρ‚ΠΎΡ‡Π΅ΠΊ P ΠΈ Q Π² ΠΏΡ€ΠΈΠ²Π΅Π΄Π΅Π½Π½ΠΎΠ΅ Π²Ρ‹ΡˆΠ΅ ΡƒΡ€Π°Π²Π½Π΅Π½ΠΈΠ΅, ΠΌΡ‹ ΠΌΠΎΠΆΠ΅ΠΌ Π½Π°ΠΉΡ‚ΠΈ Π²Π΅ΠΊΡ‚ΠΎΡ€ полоТСния PQ:

      PQ = (5-(-4), 11-6)

      PQ = ((5+ 4 ), 11-6)

      PQ = (9, 5) = 9 i + 5 j

      Π’Π°ΠΊΠΈΠΌ ΠΎΠ±Ρ€Π°Π·ΠΎΠΌ, Π²Π΅ΠΊΡ‚ΠΎΡ€ полоТСния PQ эквивалСнтСн Π²Π΅ΠΊΡ‚ΠΎΡ€Ρƒ, ΠΊΠΎΡ‚ΠΎΡ€Ρ‹ΠΉ начинаСтся Π² Π½Π°Ρ‡Π°Π»Π΅ ΠΊΠΎΠΎΡ€Π΄ΠΈΠ½Π°Ρ‚. Π­Ρ‚ΠΎΡ‚ Π²Π΅ΠΊΡ‚ΠΎΡ€ Π½Π°ΠΏΡ€Π°Π²Π»Π΅Π½ Π² Ρ‚ΠΎΡ‡ΠΊΡƒ Π½Π° 9 Π΅Π΄ΠΈΠ½ΠΈΡ† Π²ΠΏΡ€Π°Π²ΠΎ ΠΏΠΎ оси x ΠΈ Π½Π° 5 Π΅Π΄ΠΈΠ½ΠΈΡ† Π²Π²Π΅Ρ€Ρ… ΠΏΠΎ оси y.

    2. ΠŸΡ€ΠΈΠΌΠ΅Ρ€ 2: Для Π΄Π²ΡƒΡ… Ρ‚ΠΎΡ‡Π΅ΠΊ P = (-4, 6) ΠΈ Q = (5, 11) ΠΎΠΏΡ€Π΅Π΄Π΅Π»ΠΈΡ‚Π΅ Π²Π΅ΠΊΡ‚ΠΎΡ€ полоТСния QP.

      РСшСниС: Если Π² систСмС ΠΊΠΎΠΎΡ€Π΄ΠΈΠ½Π°Ρ‚ xy Π·Π°Π΄Π°Π½Ρ‹ Π΄Π²Π΅ Ρ‚ΠΎΡ‡ΠΊΠΈ, Ρ‚ΠΎ ΠΌΡ‹ ΠΌΠΎΠΆΠ΅ΠΌ ΠΈΡΠΏΠΎΠ»ΡŒΠ·ΠΎΠ²Π°Ρ‚ΡŒ ΡΠ»Π΅Π΄ΡƒΡŽΡ‰ΡƒΡŽ Ρ„ΠΎΡ€ΠΌΡƒΠ»Ρƒ для нахоТдСния Π²Π΅ΠΊΡ‚ΠΎΡ€Π° полоТСния QP: Β — y 2 )

      Π“Π΄Π΅ (x 1 , y 1 ) прСдставляСт ΠΊΠΎΠΎΡ€Π΄ΠΈΠ½Π°Ρ‚Ρ‹ Ρ‚ΠΎΡ‡ΠΊΠΈ P ΠΈ (x 2 , y 2 ) прСдставляСт ΠΊΠΎΠΎΡ€Π΄ΠΈΠ½Π°Ρ‚Ρ‹ Ρ‚ΠΎΡ‡ΠΊΠΈ Q. ΠžΠ±Ρ€Π°Ρ‚ΠΈΡ‚Π΅ Π²Π½ΠΈΠΌΠ°Π½ΠΈΠ΅, Ρ‡Ρ‚ΠΎ Π²Π΅ΠΊΡ‚ΠΎΡ€ полоТСния QP прСдставляСт собой Π²Π΅ΠΊΡ‚ΠΎΡ€, Π½Π°ΠΏΡ€Π°Π²Π»Π΅Π½Π½Ρ‹ΠΉ ΠΎΡ‚ Ρ‚ΠΎΡ‡ΠΊΠΈ Q ΠΊ Ρ‚ΠΎΡ‡ΠΊΠ΅ P. Он отличаСтся ΠΎΡ‚ Π²Π΅ΠΊΡ‚ΠΎΡ€Π° полоТСния PQ, ΠΊΠΎΡ‚ΠΎΡ€Ρ‹ΠΉ Π½Π°ΠΏΡ€Π°Π²Π»Π΅Π½ ΠΎΡ‚ P ΠΊ Q. Π’Π°ΠΊΠΈΠΌ ΠΎΠ±Ρ€Π°Π·ΠΎΠΌ, просто подставив значСния Ρ‚ΠΎΡ‡Π΅ΠΊ P ΠΈ Q Π² ΠΏΡ€ΠΈΠ²Π΅Π΄Π΅Π½Π½ΠΎΠ΅ Π²Ρ‹ΡˆΠ΅ ΡƒΡ€Π°Π²Π½Π΅Π½ΠΈΠ΅ , ΠΌΡ‹ ΠΌΠΎΠΆΠ΅ΠΌ Π½Π°ΠΉΡ‚ΠΈ Π²Π΅ΠΊΡ‚ΠΎΡ€ полоТСния QP:

      QP = (-4-5, 6-11)

      QP = (-9, -5) = -9 i — 5 j

      Π’Π°ΠΊΠΈΠΌ ΠΎΠ±Ρ€Π°Π·ΠΎΠΌ, Π²Π΅ΠΊΡ‚ΠΎΡ€ полоТСния QP Ρ€Π°Π²Π΅Π½ эквивалСнтСн Π²Π΅ΠΊΡ‚ΠΎΡ€Ρƒ, ΠΊΠΎΡ‚ΠΎΡ€Ρ‹ΠΉ начинаСтся Π² Π½Π°Ρ‡Π°Π»Π΅ ΠΊΠΎΠΎΡ€Π΄ΠΈΠ½Π°Ρ‚. Π­Ρ‚ΠΎΡ‚ Π²Π΅ΠΊΡ‚ΠΎΡ€ Π½Π°ΠΏΡ€Π°Π²Π»Π΅Π½ Π² Ρ‚ΠΎΡ‡ΠΊΡƒ 9Π΅Π΄ΠΈΠ½ΠΈΡ† Π²Π»Π΅Π²ΠΎ ΠΏΠΎ оси x ΠΈ Π½Π° 5 Π΅Π΄ΠΈΠ½ΠΈΡ† Π²Π½ΠΈΠ· ΠΏΠΎ оси y.

    ΠΏΠ΅Ρ€Π΅ΠΉΡ‚ΠΈ ΠΊ слайдупСрСйти ΠΊ слайду

    ΠžΡ‚Π»ΠΈΡ‡Π½ΠΎΠ΅ ΠΎΠ±ΡƒΡ‡Π΅Π½ΠΈΠ΅ Π² ΡΡ‚Π°Ρ€ΡˆΠ΅ΠΉ школС с использованиСм простых подсказок

    УвлСкаясь Π·ΡƒΠ±Ρ€Π΅ΠΆΠΊΠΎΠΉ, Π²Ρ‹, скорСС всСго, Π·Π°Π±ΡƒΠ΄Π΅Ρ‚Π΅ понятия. Π‘ Cuemath Π²Ρ‹ Π±ΡƒΠ΄Π΅Ρ‚Π΅ ΡƒΡ‡ΠΈΡ‚ΡŒΡΡ Π²ΠΈΠ·ΡƒΠ°Π»ΡŒΠ½ΠΎ ΠΈ Π±ΡƒΠ΄Π΅Ρ‚Π΅ ΡƒΠ΄ΠΈΠ²Π»Π΅Π½Ρ‹ Ρ€Π΅Π·ΡƒΠ»ΡŒΡ‚Π°Ρ‚Π°ΠΌΠΈ.

    Π—Π°ΠΏΠΈΡΠ°Ρ‚ΡŒΡΡ Π½Π° бСсплатный ΠΏΡ€ΠΎΠ±Π½Ρ‹ΠΉ ΡƒΡ€ΠΎΠΊ

    ΠŸΡ€Π°ΠΊΡ‚ΠΈΡ‡Π΅ΡΠΊΠΈΠ΅ вопросы ΠΏΠΎ ΠΏΠΎΠ·ΠΈΡ†ΠΈΠΎΠ½Π½ΠΎΠΌΡƒ Π²Π΅ΠΊΡ‚ΠΎΡ€Ρƒ

    Β 

    ΠΏΠ΅Ρ€Π΅ΠΉΡ‚ΠΈ ΠΊ слайдупСрСйти ΠΊ слайду

    Часто Π·Π°Π΄Π°Π²Π°Π΅ΠΌΡ‹Π΅ вопросы ΠΎ Π²Π΅ΠΊΡ‚ΠΎΡ€Π΅ полоТСния

    Π§Ρ‚ΠΎ Ρ‚Π°ΠΊΠΎΠ΅

    Π²Π΅ΠΊΡ‚ΠΎΡ€ полоТСния Π² ΠΌΠ°Ρ‚Π΅ΠΌΠ°Ρ‚ΠΈΠΊΠ΅?

    Π’Π΅ΠΊΡ‚ΠΎΡ€ полоТСния прСдставляСт собой ΠΏΡ€ΡΠΌΡƒΡŽ линию, ΠΎΠ΄ΠΈΠ½ ΠΊΠΎΠ½Π΅Ρ† ΠΊΠΎΡ‚ΠΎΡ€ΠΎΠΉ ΠΏΡ€ΠΈΠΊΡ€Π΅ΠΏΠ»Π΅Π½ ΠΊ Ρ‚Π΅Π»Ρƒ, Π° Π΄Ρ€ΡƒΠ³ΠΎΠΉ ΠΊΠΎΠ½Π΅Ρ† ΠΏΡ€ΠΈΠΊΡ€Π΅ΠΏΠ»Π΅Π½ ΠΊ двиТущСйся Ρ‚ΠΎΡ‡ΠΊΠ΅, ΠΈ ΠΈΡΠΏΠΎΠ»ΡŒΠ·ΡƒΠ΅Ρ‚ΡΡ для описания полоТСния Ρ‚ΠΎΡ‡ΠΊΠΈ ΠΎΡ‚Π½ΠΎΡΠΈΡ‚Π΅Π»ΡŒΠ½ΠΎ Ρ‚Π΅Π»Π°. НаправлСниС Π²Π΅ΠΊΡ‚ΠΎΡ€Π° полоТСния всСгда ΡƒΠΊΠ°Π·Ρ‹Π²Π°Π΅Ρ‚ ΠΎΡ‚ Π½Π°Ρ‡Π°Π»Π° этого Π²Π΅ΠΊΡ‚ΠΎΡ€Π° ΠΊ Π·Π°Π΄Π°Π½Π½ΠΎΠΉ Ρ‚ΠΎΡ‡ΠΊΠ΅.

    Как Π½Π°ΠΉΡ‚ΠΈ Π²Π΅ΠΊΡ‚ΠΎΡ€ полоТСния?

    ΠžΡ‡Π΅Π½ΡŒ Π²Π°ΠΆΠ½ΠΎ сначала ΠΎΠΏΡ€Π΅Π΄Π΅Π»ΠΈΡ‚ΡŒ ΠΊΠΎΠΎΡ€Π΄ΠΈΠ½Π°Ρ‚Ρ‹ Ρ‚ΠΎΡ‡ΠΊΠΈ, ΠΏΡ€Π΅ΠΆΠ΄Π΅ Ρ‡Π΅ΠΌ Π½Π°ΠΉΡ‚ΠΈ Π²Π΅ΠΊΡ‚ΠΎΡ€ полоТСния этой Ρ‚ΠΎΡ‡ΠΊΠΈ. Рассмотрим Π΄Π²Π΅ Ρ‚ΠΎΡ‡ΠΊΠΈ, A ΠΈ B, Π³Π΄Π΅ A = (x 1 , y 1 ) ΠΈ B = (x 2 , y 2 ).

    • Π”Π°Π»Π΅Π΅ Π½Π°ΠΉΠ΄Π΅ΠΌ Π²Π΅ΠΊΡ‚ΠΎΡ€ полоТСния ΠΈΠ· Ρ‚ΠΎΡ‡ΠΊΠΈ А Π² Ρ‚ΠΎΡ‡ΠΊΡƒ Π’, Π²Π΅ΠΊΡ‚ΠΎΡ€ АВ.
    • Π§Ρ‚ΠΎΠ±Ρ‹ ΠΎΠΏΡ€Π΅Π΄Π΅Π»ΠΈΡ‚ΡŒ этот Π²Π΅ΠΊΡ‚ΠΎΡ€ полоТСния, Π½Π°ΠΌ Π½ΡƒΠΆΠ½ΠΎ Π²Ρ‹Ρ‡Π΅ΡΡ‚ΡŒ ΡΠΎΠΎΡ‚Π²Π΅Ρ‚ΡΡ‚Π²ΡƒΡŽΡ‰ΠΈΠ΅ ΠΊΠΎΠΌΠΏΠΎΠ½Π΅Π½Ρ‚Ρ‹ A ΠΈΠ· B: AB = (x 2 Β — x 1 , y 2 Β — Ρƒ 1 )

    Π’ Ρ‡Π΅ΠΌ Ρ€Π°Π·Π½ΠΈΡ†Π° ΠΌΠ΅ΠΆΠ΄Ρƒ Π²Π΅ΠΊΡ‚ΠΎΡ€ΠΎΠΌ полоТСния ΠΈ Π²Π΅ΠΊΡ‚ΠΎΡ€ΠΎΠΌ смСщСния?

    Π’Π΅ΠΊΡ‚ΠΎΡ€ полоТСния опрСдСляСтся ΠΊΠ°ΠΊ Π²Π΅ΠΊΡ‚ΠΎΡ€, ΡƒΠΊΠ°Π·Ρ‹Π²Π°ΡŽΡ‰ΠΈΠΉ Π»ΠΈΠ±ΠΎ ΠΏΠΎΠ»ΠΎΠΆΠ΅Π½ΠΈΠ΅, Π»ΠΈΠ±ΠΎ мСстополоТСниС любой Π·Π°Π΄Π°Π½Π½ΠΎΠΉ Ρ‚ΠΎΡ‡ΠΊΠΈ ΠΎΡ‚Π½ΠΎΡΠΈΡ‚Π΅Π»ΡŒΠ½ΠΎ любой ΠΏΡ€ΠΎΠΈΠ·Π²ΠΎΠ»ΡŒΠ½ΠΎΠΉ ΠΊΠΎΠ½Ρ‚Ρ€ΠΎΠ»ΡŒΠ½ΠΎΠΉ Ρ‚ΠΎΡ‡ΠΊΠΈ, Ρ‚Π°ΠΊΠΎΠΉ ΠΊΠ°ΠΊ Π½Π°Ρ‡Π°Π»ΠΎ ΠΊΠΎΠΎΡ€Π΄ΠΈΠ½Π°Ρ‚. ΠŸΡ€ΠΈΠ½ΠΈΠΌΠ°Ρ Π²ΠΎ Π²Π½ΠΈΠΌΠ°Π½ΠΈΠ΅, Ρ‡Ρ‚ΠΎ Π²Π΅ΠΊΡ‚ΠΎΡ€ смСщСния ΠΏΠΎΠΌΠΎΠ³Π°Π΅Ρ‚ Π½Π°ΠΌ Π½Π°ΠΉΡ‚ΠΈ ΠΈΠ·ΠΌΠ΅Π½Π΅Π½ΠΈΠ΅ Π²Π΅ΠΊΡ‚ΠΎΡ€Π° полоТСния Π΄Π°Π½Π½ΠΎΠ³ΠΎ ΠΎΠ±ΡŠΠ΅ΠΊΡ‚Π°.

    Π“Π΄Π΅ всСгда начинаСтся Π²Π΅ΠΊΡ‚ΠΎΡ€ полоТСния?

    Π’Π΅ΠΊΡ‚ΠΎΡ€ полоТСния начинаСтся Π² Π½Π°Ρ‡Π°Π»Π΅ ΠΊΠΎΠΎΡ€Π΄ΠΈΠ½Π°Ρ‚ ΠΈ заканчиваСтся Π² любой Π΄Ρ€ΡƒΠ³ΠΎΠΉ ΠΏΡ€ΠΎΠΈΠ·Π²ΠΎΠ»ΡŒΠ½ΠΎΠΉ Ρ‚ΠΎΡ‡ΠΊΠ΅. Они ΠΈΡΠΏΠΎΠ»ΡŒΠ·ΡƒΡŽΡ‚ΡΡ для опрСдСлСния полоТСния Ρ‚ΠΎΡ‡ΠΊΠΈ ΠΎΡ‚Π½ΠΎΡΠΈΡ‚Π΅Π»ΡŒΠ½ΠΎ Π½Π°Ρ‡Π°Π»Π° ΠΊΠΎΠΎΡ€Π΄ΠΈΠ½Π°Ρ‚.

    Π’ Ρ‡Π΅ΠΌ Ρ€Π°Π·Π½ΠΈΡ†Π° ΠΌΠ΅ΠΆΠ΄Ρƒ Π²Π΅ΠΊΡ‚ΠΎΡ€ΠΎΠΌ полоТСния ΠΈ Π΅Π΄ΠΈΠ½ΠΈΡ‡Π½Ρ‹ΠΌ Π²Π΅ΠΊΡ‚ΠΎΡ€ΠΎΠΌ?

    Π’Π΅ΠΊΡ‚ΠΎΡ€ считаСтся Π΅Π΄ΠΈΠ½ΠΈΡ‡Π½Ρ‹ΠΌ Π²Π΅ΠΊΡ‚ΠΎΡ€ΠΎΠΌ, Ссли ΠΎΠ½ ΠΈΡΠΏΠΎΠ»ΡŒΠ·ΡƒΠ΅Ρ‚ΡΡ для указания Ρ‚ΠΎΠ»ΡŒΠΊΠΎ направлСния ΠΈ ΠΈΠΌΠ΅Π΅Ρ‚ Π²Π΅Π»ΠΈΡ‡ΠΈΠ½Ρƒ, Ρ€Π°Π²Π½ΡƒΡŽ 1. Для направлСния Π½Π΅ трСбуСтся Π²Π΅Π»ΠΈΡ‡ΠΈΠ½Π°, поэтому Π²Π΅Π»ΠΈΡ‡ΠΈΠ½Π° Π΅Π΄ΠΈΠ½ΠΈΡ‡Π½ΠΎΠ³ΠΎ Π²Π΅ΠΊΡ‚ΠΎΡ€Π° всСгда Ρ€Π°Π²Π½Π° 1. A Π²Π΅ΠΊΡ‚ΠΎΡ€ полоТСния опрСдСляСтся ΠΊΠ°ΠΊ Π²Π΅ΠΊΡ‚ΠΎΡ€, ΠΊΠΎΡ‚ΠΎΡ€Ρ‹ΠΉ ΡƒΠΊΠ°Π·Ρ‹Π²Π°Π΅Ρ‚ Π»ΠΈΠ±ΠΎ ΠΏΠΎΠ»ΠΎΠΆΠ΅Π½ΠΈΠ΅, Π»ΠΈΠ±ΠΎ мСстополоТСниС любой Π·Π°Π΄Π°Π½Π½ΠΎΠΉ Ρ‚ΠΎΡ‡ΠΊΠΈ ΠΎΡ‚Π½ΠΎΡΠΈΡ‚Π΅Π»ΡŒΠ½ΠΎ любой ΠΏΡ€ΠΎΠΈΠ·Π²ΠΎΠ»ΡŒΠ½ΠΎΠΉ ΠΎΠΏΠΎΡ€Π½ΠΎΠΉ Ρ‚ΠΎΡ‡ΠΊΠΈ, Ρ‚Π°ΠΊΠΎΠΉ ΠΊΠ°ΠΊ Π½Π°Ρ‡Π°Π»ΠΎ ΠΊΠΎΠΎΡ€Π΄ΠΈΠ½Π°Ρ‚.

    Какая Ρ„ΠΎΡ€ΠΌΡƒΠ»Π° для Π²Π΅ΠΊΡ‚ΠΎΡ€Π° полоТСния?

    Если ΠΌΡ‹ Π·Π½Π°Π΅ΠΌ ΠΏΠΎΠ»ΠΎΠΆΠ΅Π½ΠΈΠ΅ любой Ρ‚ΠΎΡ‡ΠΊΠΈ Π² плоскости xy, Ρ‚ΠΎ ΠΌΡ‹ ΠΌΠΎΠΆΠ΅ΠΌ ΠΈΡΠΏΠΎΠ»ΡŒΠ·ΠΎΠ²Π°Ρ‚ΡŒ Ρ„ΠΎΡ€ΠΌΡƒΠ»Ρƒ для опрСдСлСния Π²Π΅ΠΊΡ‚ΠΎΡ€Π° полоТСния ΠΌΠ΅ΠΆΠ΄Ρƒ этими двумя Ρ‚ΠΎΡ‡ΠΊΠ°ΠΌΠΈ. НапримСр, рассмотрим Ρ‚ΠΎΡ‡ΠΊΡƒ A с ΠΊΠΎΠΎΡ€Π΄ΠΈΠ½Π°Ρ‚Π°ΠΌΠΈ (x k , y k ) Π² плоскости xy ΠΈ Π΄Ρ€ΡƒΠ³ΡƒΡŽ Ρ‚ΠΎΡ‡ΠΊΡƒ B с ΠΊΠΎΠΎΡ€Π΄ΠΈΠ½Π°Ρ‚Π°ΠΌΠΈ (x k+1 , y k+ 1 ). Π€ΠΎΡ€ΠΌΡƒΠ»Π° для опрСдСлСния Π²Π΅ΠΊΡ‚ΠΎΡ€Π° полоТСния ΠΎΡ‚ A Π΄ΠΎ B: AB = (x k+1 Β — x k , y ΠΊ+1 Β — Ρƒ ΠΊ ).

    ЯвляСтся Π»ΠΈ смСщСниС Π²Π΅ΠΊΡ‚ΠΎΡ€ΠΎΠΌ полоТСния?

    НСт, смСщСниС Π½Π΅ являСтся Π²Π΅ΠΊΡ‚ΠΎΡ€ΠΎΠΌ полоТСния. Π’Π΅ΠΊΡ‚ΠΎΡ€ полоТСния ΠΈΡΠΏΠΎΠ»ΡŒΠ·ΡƒΠ΅Ρ‚ΡΡ, Ρ‡Ρ‚ΠΎΠ±Ρ‹ ΠΏΠΎΠΌΠΎΡ‡ΡŒ Π½Π°ΠΌ Π½Π°ΠΉΡ‚ΠΈ мСстополоТСниС ΠΎΠ΄Π½ΠΎΠ³ΠΎ ΠΎΠ±ΡŠΠ΅ΠΊΡ‚Π° ΠΎΡ‚Π½ΠΎΡΠΈΡ‚Π΅Π»ΡŒΠ½ΠΎ Π΄Ρ€ΡƒΠ³ΠΎΠ³ΠΎ ΠΎΠ±ΡŠΠ΅ΠΊΡ‚Π°. Π’Π°ΠΆΠ½ΠΎ Π·Π½Π°Ρ‚ΡŒ ΠΏΠΎΠ»ΠΎΠΆΠ΅Π½ΠΈΠ΅ Ρ‚Π΅Π»Π°, ΠΊΠΎΠ³Π΄Π° ΠΌΡ‹ описываСм Π΅Π³ΠΎ Π΄Π²ΠΈΠΆΠ΅Π½ΠΈΠ΅. Но Ρ‚ΠΎΠ³Π΄Π° Π²Π΅ΠΊΡ‚ΠΎΡ€ смСщСния ΠΌΠΎΠΆΠ½ΠΎ ΠΎΠΏΡ€Π΅Π΄Π΅Π»ΠΈΡ‚ΡŒ ΠΊΠ°ΠΊ ΠΈΠ·ΠΌΠ΅Π½Π΅Π½ΠΈΠ΅ ΠΈΠ»ΠΈ Π²Π°Ρ€ΠΈΠ°Ρ†ΠΈΡŽ Π·Π°Π΄Π°Π½Π½ΠΎΠ³ΠΎ Π²Π΅ΠΊΡ‚ΠΎΡ€Π° полоТСния.

    ИзмСнСниС базиса β€” Π£Ρ‡Π΅Π±Π½Ρ‹Π΅ пособия ΠΏΠΎ расчСту

    ИзмСнСниС базиса β€” Π£Ρ‡Π΅Π±Π½ΠΎΠ΅ пособиС ΠΏΠΎ расчСту HMC

    ΠŸΡƒΡΡ‚ΡŒ $V$ β€” Π²Π΅ΠΊΡ‚ΠΎΡ€Π½ΠΎΠ΅ пространство, ΠΈ ΠΏΡƒΡΡ‚ΡŒ $S = \{{\bf v_1,v_2, \ldots, v_n}\}$ β€” мноТСство Π²Π΅ΠΊΡ‚ΠΎΡ€ΠΎΠ² Π² $V$. Напомним, Ρ‡Ρ‚ΠΎ $S$ ΠΎΠ±Ρ€Π°Π·ΡƒΠ΅Ρ‚ базис для $V$, Ссли Π²Ρ‹ΠΏΠΎΠ»Π½ΡΡŽΡ‚ΡΡ ΡΠ»Π΅Π΄ΡƒΡŽΡ‰ΠΈΠ΅ Π΄Π²Π° условия: ΠΏΡƒΡΡ‚ΡŒ $V$ β€” Π²Π΅ΠΊΡ‚ΠΎΡ€Π½ΠΎΠ΅ пространство ΠΈ ΠΏΡƒΡΡ‚ΡŒ $S = \{{\bf v_1,v_2, \ldots, v_n}\} $ β€” Π½Π°Π±ΠΎΡ€ Π²Π΅ΠΊΡ‚ΠΎΡ€ΠΎΠ² Π² $V$. Напомним, Ρ‡Ρ‚ΠΎ $S$ ΠΎΠ±Ρ€Π°Π·ΡƒΠ΅Ρ‚ базис для $V$, Ссли Π²Ρ‹ΠΏΠΎΠ»Π½ΡΡŽΡ‚ΡΡ ΡΠ»Π΅Π΄ΡƒΡŽΡ‰ΠΈΠ΅ Π΄Π²Π° условия:

    Если $S = \{{\bf v_1,v_2, \ldots, v_n}\}$ являСтся базисом для $V $, Ρ‚ΠΎ ΠΊΠ°ΠΆΠ΄Ρ‹ΠΉ Π²Π΅ΠΊΡ‚ΠΎΡ€ ${\bf v} \in V$ ΠΌΠΎΠΆΠ΅Ρ‚ Π±Ρ‹Ρ‚ΡŒ Π²Ρ‹Ρ€Π°ΠΆΠ΅Π½ ΠΎΠ΄Π½ΠΎΠ·Π½Π°Ρ‡Π½ΠΎ ΠΊΠ°ΠΊ линСйная комбинация ${\bf v_1,v_2, \ldots, v_n}$: $$ {\bf v} = c_1{\bf v_1} + c_2{\bf v_2} + \cdots + c_n{ \bf v_n}. $$ Π”ΡƒΠΌΠ°ΠΉΡ‚Π΅ ΠΎ $\left[\begin{array}{c} c_1\\ c_2\\ \vdots \\ c_n \end{array}\right]$ ΠΊΠ°ΠΊ ΠΎ ΠΊΠΎΠΎΡ€Π΄ΠΈΠ½Π°Ρ‚Π°Ρ… ΠΎΡ‚Π½ΠΎΡΠΈΡ‚Π΅Π»ΡŒΠ½ΠΎΠ³ΠΎ ${\bf v}$ ΠΊ Π±Π°Π·Π΅ $S$. Если $V$ ΠΈΠΌΠ΅Π΅Ρ‚ Ρ€Π°Π·ΠΌΠ΅Ρ€Π½ΠΎΡΡ‚ΡŒ , это количСство Π²Π΅ΠΊΡ‚ΠΎΡ€ΠΎΠ², Π½Π΅ΠΎΠ±Ρ…ΠΎΠ΄ΠΈΠΌΡ‹Ρ… для формирования основы. $n$, Ρ‚ΠΎ ΠΊΠ°ΠΆΠ΄Ρ‹ΠΉ Π½Π°Π±ΠΎΡ€ ΠΈΠ· $n$ Π»ΠΈΠ½Π΅ΠΉΠ½ΠΎ нСзависимых Π²Π΅ΠΊΡ‚ΠΎΡ€ΠΎΠ² Π² $V$ ΠΎΠ±Ρ€Π°Π·ΡƒΠ΅Ρ‚ базис для $V$. Π’ ΠΊΠ°ΠΆΠ΄ΠΎΠΌ ΠΏΡ€ΠΈΠ»ΠΎΠΆΠ΅Π½ΠΈΠΈ Ρƒ нас Π΅ΡΡ‚ΡŒ Π²Ρ‹Π±ΠΎΡ€, ΠΊΠ°ΠΊΡƒΡŽ основу ΠΌΡ‹ ΠΈΡΠΏΠΎΠ»ΡŒΠ·ΡƒΠ΅ΠΌ. Π’ этом ΡƒΡ€ΠΎΠΊΠ΅ ΠΌΡ‹ опишСм ΠΏΡ€Π΅ΠΎΠ±Ρ€Π°Π·ΠΎΠ²Π°Π½ΠΈΠ΅ ΠΊΠΎΠΎΡ€Π΄ΠΈΠ½Π°Ρ‚ Π²Π΅ΠΊΡ‚ΠΎΡ€ΠΎΠ² ΠΏΡ€ΠΈ смСнС базиса. 92$. Для Π²Π΅ΠΊΡ‚ΠΎΡ€Π° ${\bf v} \in V$ ΠΏΠΎ Π·Π°Π΄Π°Π½Π½Ρ‹ΠΌ Π΅Π³ΠΎ ΠΊΠΎΠΎΡ€Π΄ΠΈΠ½Π°Ρ‚Π°ΠΌ $[{\bf v}]_B$ Π² базисС $B$ ΠΌΡ‹ Ρ…ΠΎΡ‚Π΅Π»ΠΈ Π±Ρ‹ ΠΈΠΌΠ΅Ρ‚ΡŒ Π²ΠΎΠ·ΠΌΠΎΠΆΠ½ΠΎΡΡ‚ΡŒ Π²Ρ‹Ρ€Π°Π·ΠΈΡ‚ΡŒ ${\bf v}$ Π² Π΅Π³ΠΎ ΠΊΠΎΠΎΡ€Π΄ΠΈΠ½Π°Ρ‚ $[{\bf v}]_{B’}$ Π² базисС $B’$, ΠΈ Π½Π°ΠΎΠ±ΠΎΡ€ΠΎΡ‚.

    ΠŸΡ€Π΅Π΄ΠΏΠΎΠ»ΠΎΠΆΠΈΠΌ, Ρ‡Ρ‚ΠΎ базисныС Π²Π΅ΠΊΡ‚ΠΎΡ€Ρ‹ ${\bf u’}$ ΠΈ ${\bf w’}$ для $B’$ ΠΈΠΌΠ΅ΡŽΡ‚ ΡΠ»Π΅Π΄ΡƒΡŽΡ‰ΠΈΠ΅ ΠΊΠΎΠΎΡ€Π΄ΠΈΠ½Π°Ρ‚Ρ‹ ΠΎΡ‚Π½ΠΎΡΠΈΡ‚Π΅Π»ΡŒΠ½ΠΎ базиса $B$:

    \begin{Π²Ρ‹Ρ€Π°Π²Π½ΠΈΠ²Π°Π½ΠΈΠ΅*} ~[{\bf u’}]_B & = & \left[\begin{array}{c} a \\ b \end{массив}\right] \qquad \\ ~[{\bf w’}]_B & = & \left[\begin{array}{c} c \\ d \end{массив}\right]. \qquad \end{Π²Ρ‹Ρ€Π°Π²Π½ΠΈΠ²Π°Π½ΠΈΠ΅*}

    Π­Ρ‚ΠΎ ΠΎΠ·Π½Π°Ρ‡Π°Π΅Ρ‚, Ρ‡Ρ‚ΠΎ \begin{Π²Ρ‹Ρ€Π°Π²Π½ΠΈΠ²Π°Π½ΠΈΠ΅*} {\bf u’} & = & a{\bf u} + b{\bf w} \\ {\bf ш’} & = & c{\bf u} + d{\bf ш} \end{Π²Ρ‹Ρ€Π°Π²Π½ΠΈΠ²Π°Π½ΠΈΠ΅*}

    ΠΈΠ·ΠΌΠ΅Π½Π΅Π½ΠΈΠ΅ ΠΌΠ°Ρ‚Ρ€ΠΈΡ†Ρ‹ ΠΊΠΎΠΎΡ€Π΄ΠΈΠ½Π°Ρ‚ с $B’$ Π½Π° $B$ $$ P = \left[\begin{array}{cc} a & c \\ b & d \\ \end{array} \right ] $$ опрСдСляСт ΠΈΠ·ΠΌΠ΅Π½Π΅Π½ΠΈΠ΅ ΠΊΠΎΠΎΡ€Π΄ΠΈΠ½Π°Ρ‚ ${\bf v} \in V$ ΠΏΡ€ΠΈ Π·Π°ΠΌΠ΅Π½Π΅ базиса с $B’$ Π½Π° $B$. {-1} = \left[\begin{массив}{cc} \frac{1}{5} & \frac{2}{5} \\ -\frac{1}{5} ΠΈ \frac{3}{5} \ΠΊΠΎΠ½Π΅Ρ†{массив}\справа], $$ ΠΌΡ‹ ΠΌΠΎΠΆΠ΅ΠΌ ΡƒΠ±Π΅Π΄ΠΈΡ‚ΡŒΡΡ, Ρ‡Ρ‚ΠΎ $$ [{\bf v}]_{B’} = \left[\begin{массив}{cc} \frac{1}{5} & \frac{2}{5} \\ -\frac{1}{5} ΠΈ \frac{3}{5} \end{массив}\right]\left[\begin{массив}{c} 4\3 \end{массив}\right] = \left[\begin{массив}{c} 2\1 \ΠΊΠΎΠ½Π΅Ρ†{массив}\справа] $$ с Ρ‡Π΅Π³ΠΎ ΠΌΡ‹ Π½Π°Ρ‡Π°Π»ΠΈ.

    Π’ ΡΠ»Π΅Π΄ΡƒΡŽΡ‰Π΅ΠΌ ΠΏΡ€ΠΈΠΌΠ΅Ρ€Π΅ ΠΌΡ‹ Π²Π²ΠΎΠ΄ΠΈΠΌ Ρ‚Ρ€Π΅Ρ‚ΠΈΠΉ базис, Ρ‡Ρ‚ΠΎΠ±Ρ‹ ΠΏΠΎΡΠΌΠΎΡ‚Ρ€Π΅Ρ‚ΡŒ Π½Π° ΠΎΡ‚Π½ΠΎΡˆΠ΅Π½ΠΈΡ ΠΌΠ΅ΠΆΠ΄Ρƒ двумя нСстандартными Π±Π°Π·Π°ΠΌΠΈ .

    ΠŸΡ€ΠΈΠΌΠ΅Ρ€

    ΠŸΡƒΡΡ‚ΡŒ $B” = \left\{ \left[ {2 \top 1} \right],\left[ {1 \top 4} \right]\right\}$. Π§Ρ‚ΠΎΠ±Ρ‹ Π½Π°ΠΉΡ‚ΠΈ ΠΈΠ·ΠΌΠ΅Π½Π΅Π½ΠΈΠ΅ ΠΌΠ°Ρ‚Ρ€ΠΈΡ†Ρ‹ ΠΊΠΎΠΎΡ€Π΄ΠΈΠ½Π°Ρ‚ ΠΈΠ· базиса $B’$ ΠΈΠ· ΠΏΡ€Π΅Π΄Ρ‹Π΄ΡƒΡ‰Π΅Π³ΠΎ ΠΏΡ€ΠΈΠΌΠ΅Ρ€Π° Π² $B”$, сначала Π²Ρ‹Ρ€Π°Π·ΠΈΠΌ базисныС Π²Π΅ΠΊΡ‚ΠΎΡ€Ρ‹ $\left[ {3 \atop 1} \right]$ ΠΈ $\left[ { -2 \ΠΏΠΎΠ²Π΅Ρ€Ρ… 1} \справа]$ $B’$ ΠΊΠ°ΠΊ Π»ΠΈΠ½Π΅ΠΉΠ½Ρ‹Π΅ ΠΊΠΎΠΌΠ±ΠΈΠ½Π°Ρ†ΠΈΠΈ базисных Π²Π΅ΠΊΡ‚ΠΎΡ€ΠΎΠ² $\left[ {2 \ΠΏΠΎΠ²Π΅Ρ€Ρ… 1} \справа]$ ΠΈ $\left[ {1 \ΠΏΠΎΠ²Π΅Ρ€Ρ… 4} \справа] $ of $B”$: \begin{eqnarray*} \mbox{Set }\left[ \begin{array}{c} 3 \\ 1 \end{array}\right] & = & a\left[\begin {array}{c} 2 \\ 1 \end{массив}\right] + b\left[\begin{array}{c} 1 \\ 4 \end{массив}\right] \\ \left[\begin {array}{c} -2 \\ 1 \end{массив}\right] & = & c \left[\begin{array}{c} 2 \\ 1 \end{массив}\right] + d\left [\begin{array}{c} 1 \\ 4 \end{array}\right] \end{eqnarray*} ΠΈ Ρ€Π΅ΡˆΠΈΡ‚ΡŒ ΠΏΠΎΠ»ΡƒΡ‡Π΅Π½Π½Ρ‹Π΅ систСмы r $a,b,c,$ ΠΈ $d$: \begin{ eqnarray*} \left[ \begin{array}{c} 3 \\ 1 \end{массив}\right] & = & \frac{11}{7} \left[\begin{array}{c} 2 \ \ 1 \end{массив}\right] – \frac{1}{7}\left[\begin{array}{c} 1 \\ 4 \end{массив}\right] \\ \left[\begin{ array}{c} -2 \\ 1 \end{array}\right] & = & \frac{- 9}{7}\left[\begin{array}{c} 2 \\ 1 \end{массив}\right] + \frac{4}{7}\left[\begin{array}{c} 1 \\ 4 \end{array}\right] \end{eqnarray*} Π’Π°ΠΊΠΈΠΌ ΠΎΠ±Ρ€Π°Π·ΠΎΠΌ, матричная Ρ„ΠΎΡ€ΠΌΠ° ΠΏΠ΅Ρ€Π΅Ρ…ΠΎΠ΄Π° $B’$ Π² $B”$ Ρ€Π°Π²Π½Π° $$ \left[\begin{array}{cc} \frac{11}{ 7} & \frac{-9}{7} \\ \frac{-1}{7} & \frac{4}{7} \end{массив}\right]. $$ Π’Π΅ΠΊΡ‚ΠΎΡ€ ${\bf v}$ с ΠΊΠΎΠΎΡ€Π΄ΠΈΠ½Π°Ρ‚Π°ΠΌΠΈ $\left[ {2 \atop 1} \right]$ ΠΎΡ‚Π½ΠΎΡΠΈΡ‚Π΅Π»ΡŒΠ½ΠΎ базиса $B’$ ΠΈΠΌΠ΅Π΅Ρ‚ ΠΊΠΎΠΎΡ€Π΄ΠΈΠ½Π°Ρ‚Ρ‹ $$ \left[\begin{array}{cc} \frac{11}{7} & \frac{-9}{7} \\ \frac{-1}{9} & \frac{4}{7} \end{массив}\right]\left[\ begin{array}{c} 2 \\ 1 \end{array}\right] = \left[\begin{array}{c} \frac{13}{7} \\ \frac{2}{7} \ end{array}\right] $$ ΠΎΡ‚Π½ΠΎΡΠΈΡ‚Π΅Π»ΡŒΠ½ΠΎ базиса $B”$. ВСрнСмся ΠΊ стандартному базису: $$ [ {\bf v} ]_B = \frac{13}{7}\left[\begin{array}{c} 2 \\ 1 \end{array}\right ] + \frac{2}{7}\left[\begin{array}{c} 1 \\ 4 \end{array}\right] = \left[\begin{array}{c} 4 \\ 3 \ end{array}\right], $$, Ρ‡Ρ‚ΠΎ согласуСтся с Ρ€Π΅Π·ΡƒΠ»ΡŒΡ‚Π°Ρ‚Π°ΠΌΠΈ ΠΏΡ€Π΅Π΄Ρ‹Π΄ΡƒΡ‰Π΅Π³ΠΎ ΠΏΡ€ΠΈΠΌΠ΅Ρ€Π°.

    Π’Ρ€Π°Ρ‰Π΅Π½ΠΈΠ΅ осСй ΠΊΠΎΠΎΡ€Π΄ΠΈΠ½Π°Ρ‚

    ΠŸΡ€Π΅Π΄ΠΏΠΎΠ»ΠΎΠΆΠΈΠΌ, ΠΌΡ‹ ΠΏΠΎΠ»ΡƒΡ‡Π°Π΅ΠΌ Π½ΠΎΠ²ΡƒΡŽ систСму ΠΊΠΎΠΎΡ€Π΄ΠΈΠ½Π°Ρ‚ ΠΈΠ· стандартной ΠΏΡ€ΡΠΌΠΎΡƒΠ³ΠΎΠ»ΡŒΠ½ΠΎΠΉ систСмС ΠΊΠΎΠΎΡ€Π΄ΠΈΠ½Π°Ρ‚, вращая оси ΠΏΡ€ΠΎΡ‚ΠΈΠ² часовой стрСлки Π½Π° ΡƒΠ³ΠΎΠ» $\theta$. Новый базис $B’ = \left\{{\bf u’, v’}\right\}$ Π΅Π΄ΠΈΠ½ΠΈΡ‡Π½Ρ‹Ρ… Π²Π΅ΠΊΡ‚ΠΎΡ€ΠΎΠ² вдоль осСй $x’$ ΠΈ $y’$ соотвСтствСнно, ΠΈΠΌΠ΅Π΅Ρ‚ ΠΊΠΎΠΎΡ€Π΄ΠΈΠ½Π°Ρ‚Ρ‹ \begin{Π²Ρ‹Ρ€Π°Π²Π½ΠΈΠ²Π°Π½ΠΈΠ΅*} ~[{\bf u’}]_B & = & \left[\begin{массив}{c} \cos\Ρ‚Π΅Ρ‚Π° \\ \sin\Ρ‚Π΅Ρ‚Π° \ΠΊΠΎΠ½Π΅Ρ†{массив}\справа] \\ ~[{\bf v’}]_B & = & \left[\begin{массив}{c} -\sin\theta \\ \cos\theta \end{массив} \right] \end{Π²Ρ‹Ρ€Π°Π²Π½ΠΈΠ²Π°Π½ΠΈΠ΅*} Π² исходной систСмС ΠΊΠΎΠΎΡ€Π΄ΠΈΠ½Π°Ρ‚.

    Π”ΠΎΠ±Π°Π²ΠΈΡ‚ΡŒ ΠΊΠΎΠΌΠΌΠ΅Π½Ρ‚Π°Ρ€ΠΈΠΉ

    Π’Π°Ρˆ адрСс email Π½Π΅ Π±ΡƒΠ΄Π΅Ρ‚ ΠΎΠΏΡƒΠ±Π»ΠΈΠΊΠΎΠ²Π°Π½. ΠžΠ±ΡΠ·Π°Ρ‚Π΅Π»ΡŒΠ½Ρ‹Π΅ поля ΠΏΠΎΠΌΠ΅Ρ‡Π΅Π½Ρ‹ *